You are on page 1of 549

GATE IES PSU’S-2021

GATE IES PSU’s


PAST YEAR PAPER
SOLLASS
ECENOTES
ENGINEERING

GATE IES PSUs

SUB NAME-:
GATE ECE PAPER SOL
(1991-2010)

SHORT NOTES
PART-2003-2011
A-PDF Merger DEMO : PART-B 1991-2002
Purchase from www.A-PDF.com to remove the watermark

GATE EC
2003

Q. 1-30 Carry One Mark Each

MCQ 1.1 The minimum number of equations required to analyze the circuit shown in the
figure is

(A) 3 (B) 4
(C) 6 (D) 7
SOL 1.1 Hence (B) is correct option.
Number of loops = b − n + 1
= minimum number of equation
Number of branches = b = 8
Number of nodes = n = 5
Minimum number of equation
= 8−5+1 = 4
MCQ 1.2 A source of angular frequency 1 rad/sec has a source impedance consisting of 1 Ω
resistance in series with 1 H inductance. The load that will obtain the maximum
power transfer is
(A) 1 Ω resistance
(B) 1 Ω resistance in parallel with 1 H inductance
(C) 1 Ω resistance in series with 1 F capacitor
(D) 1 Ω resistance in parallel with 1 F capacitor
SOL 1.2 For maximum power transfer
ZL = ZS* = Rs − jXs

Brought to you by: Nodia and Company Visit us at: www.nodia.co.in


PUBLISHING FOR GATE
Page 2 GATE EC 2003 www.gatehelp.com

Thus ZL = 1 − 1j
Hence (C) is correct option.
MCQ 1.3 A series RLC circuit has a resonance frequency of 1 kHz and a quality factor
Q = 100 . If each of R, L and C is doubled from its original value, the new Q of the
circuit is
(A) 25 (B) 50
(C) 100 (D) 200
SOL 1.3 Hence (B) is correct option.
Q = 1 L
R C
When R, L and C are doubled,
Q' = 1 2L = 1 L =Q
2R 2C 2R C 2
Thus Q' = 100 = 50
2

MCQ 1.4 The Laplace transform of i (t) is given by


I (s) = 2
s (1 + s)
At t " 3, The value of i (t) tends to
(A) 0 (B) 1
(C) 2 (D) 3
SOL 1.4 From the Final value theorem we have
lim i (t) = lim sI (s) = lim s 2 = lim 2 =2
t"3 s"0 s " 0 s (1 + s) s " 0 (1 + s)

Hence (C) is correct answer


MCQ 1.5 The differential equation for the current i (t) in the circuit of the figure is

2 2
(A) 2 d 2i + 2 di + i (t) = sin t (B) d 2i + 2 di + 2i (t) = cos t
dt dt dt dt
2 2
(C) 2 d 2i + 2 di + i (t) = cos t (D) d 2i + 2 di + 2i (t) = sin t
dt dt dt dt
SOL 1.5 Applying KVL we get,
di (t) 1
sin t = Ri (t) + L
dt
+
C
# i (t) dt

Brought to you by: Nodia and Company Visit us at: www.nodia.co.in


PUBLISHING FOR GATE
Page 3 GATE EC 2003 www.gatehelp.com

di (t)
or sin t = 2i (t) + 2
dt
+ # i (t) dt
Differentiating with respect to t , we get
2di (t) 2d2 i (t)
cos t = + + i (t)
dt dt2
Hence (C) is correct option.
MCQ 1.6 n -type silicon is obtained by doping silicon with
(A) Germanium (B) Aluminium
(C) Boron (D) Phosphorus
SOL 1.6 Pentavalent make n −type semiconductor and phosphorous is pentavalent.
Hence option (D) is correct.
MCQ 1.7 The Bandgap of silicon at 300 K is
(A) 1.36 eV (B) 1.10 eV
(C) 0.80 eV (D) 0.67 eV
SOL 1.7 Hence option (B) is correct.
For silicon at 0 K Eg0 = 1.21 eV
At any temperature
EgT = Eg0 − 3.6 # 10 - 4 T
At T = 300 K,
Eg300 = 1.21 − 3.6 # 10 - 4 # 300 = 1.1 eV
This is standard value, that must be remembered.
MCQ 1.8 The intrinsic carrier concentration of silicon sample at 300 K is 1.5 # 1016 /m 3 . If
after doping, the number of majority carriers is 5 # 1020 /m 3 , the minority carrier
density is
(A) 4.50 # 1011/m 3 (B) 3.333 # 10 4 /m 3
(C) 5.00 # 1020 /m 3 (D) 3.00 # 10 - 5 /m 3
SOL 1.8 By Mass action law
np = ni2
2 16
p = ni = 1.5 # 10 # 120 .5 # 1016 = 4.5 # 1011
n 5 # 10
Hence option (A) is correct.
MCQ 1.9 Choose proper substitutes for X and Y to make the following statement correct
Tunnel diode and Avalanche photo diode are operated in X bias ad Y bias
respectively
(A) X: reverse, Y: reverse (B) X: reverse, Y: forward
(C) X: forward, Y: reverse (D) X: forward, Y: forward
SOL 1.9 Tunnel diode shows the negative characteristics in forward bias. It is used in forward
Brought to you by: Nodia and Company Visit us at: www.nodia.co.in
PUBLISHING FOR GATE
Page 4 GATE EC 2003 www.gatehelp.com

bias.
Avalanche photo diode is used in reverse bias.
Hence option (C) is correct.
MCQ 1.10 For an n − channel enhancement type MOSFET, if the source is connected at a
higher potential than that of the bulk (i.e. VSB > 0 ), the threshold voltage VT of the
MOSFET will
(A) remain unchanged (B) decrease
(C) change polarity (D) increase
SOL 1.10 Hence option (D) is correct.
MCQ 1.11 Choose the correct match for input resistance of various amplifier configurations
shown below :
Configuration Input resistance
CB : Common Base LO : Low
CC : Common Collector MO : Moderate
CE : Common Emitter HI : High
(A) CB − LO, CC − MO, CE − HI
(B) CB − LO, CC − HI, CE − MO
(C) CB − MO, CC − HI, CE − LO
(D) CB − HI, CC − LO, CE − MO
SOL 1.11 For the different combinations the table is as follows

CE CE CC CB
Ai High High Unity
Av High Unity High
Ri Medium High Low
Ro Medium Low High
Hence (B) is correct option.
MCQ 1.12 The circuit shown in the figure is best described as a

(A) bridge rectifier (B) ring modulator


(C) frequency discriminator (D) voltage double
SOL 1.12 This circuit having two diode and capacitor pair in parallel, works as voltage

Brought to you by: Nodia and Company Visit us at: www.nodia.co.in


PUBLISHING FOR GATE
Page 5 GATE EC 2003 www.gatehelp.com

doubler.
Hence (D) is correct option.
MCQ 1.13 If the input to the ideal comparators shown in the figure is a sinusoidal signal of 8
V (peak to peak) without any DC component, then the output of the comparators
has a duty cycle of

(A) 1/2 (B) 1/3


(C) 1/6 (D) 1/2
SOL 1.13 If the input is sinusoidal signal of 8 V (peak to peak) then
Vi = 4 sin ωt
The output of comparator will be high when input is higher than Vref = 2 V and
will be low when input is lower than Vref = 2 V. Thus the waveform for input is
shown below

From fig, first crossover is at ωt1 and second crossover is at ωt2 where
4 sin ωt1 = 2V
Thus ωt1 = sin - 1 1 = π
2 6
ωt2 = π − π = 5π
6 6
5π π
−6
Duty Cycle = 6 =1
2π 3
Thus the output of comparators has a duty cycle of 1 .
3
Hence (B) is correct option.
MCQ 1.14 If the differential voltage gain and the common mode voltage gain of a differential
amplifier are 48 dB and 2 dB respectively, then common mode rejection ratio is
(A) 23 dB (B) 25 dB

Brought to you by: Nodia and Company Visit us at: www.nodia.co.in


PUBLISHING FOR GATE
Page 6 GATE EC 2003 www.gatehelp.com

(C) 46 dB (D) 50 dB
SOL 1.14 Hence (C) is correct option.
CMMR = Ad
Ac
or 20 log CMMR = 20 log Ad − 20 log Ac
= 48 − 2 = 46 dB
Where Ad "Differential Voltage Gain
and AC " Common Mode Voltage Gain
MCQ 1.15 Generally, the gain of a transistor amplifier falls at high frequencies due to the
(A) internal capacitances of the device
(B) coupling capacitor at the input
(C) skin effect
(D) coupling capacitor at the output
SOL 1.15 The gain of amplifier is
− gm
Ai =
gb + jωC
Thus the gain of a transistor amplifier falls at high frequencies due to the internal
capacitance that are diffusion capacitance and transition capacitance.
Hence (B) is correct option.
MCQ 1.16 The number of distinct Boolean expressions of 4 variables is
(A) 16 (B) 256
(C) 1023 (D) 65536
SOL 1.16 The number of distinct boolean expression of n variable is 22n . Thus
22 = 216 = 65536
4

Hence (D) is correct answer.


MCQ 1.17 The minimum number of comparators required to build an 8-bits flash ADC is
(A) 8 (B) 63
(C) 255 (D) 256
SOL 1.17 In the flash analog to digital converter, the no. of comparators is equal to 2n - 1,
where n is no. of bit.s
So, 2n - 1 = 28 − 1 = 255
Hence (C) is correct answer.
MCQ 1.18 The output of the 74 series of GATE of TTL gates is taken from a BJT in
(A) totem pole and common collector configuration
(B) either totem pole or open collector configuration
(C) common base configuration

Brought to you by: Nodia and Company Visit us at: www.nodia.co.in


PUBLISHING FOR GATE
Page 7 GATE EC 2003 www.gatehelp.com

(D) common collector configuration


SOL 1.18 When output of the 74 series gate of TTL gates is taken from BJT then the
configuration is either totem pole or open collector configuration .
Hence (B) is correct answer.
MCQ 1.19 Without any additional circuitry, an 8:1 MUX can be used to obtain
(A) some but not all Boolean functions of 3 variables
(B) all functions of 3 variables but non of 4 variables
(C) all functions of 3 variables and some but not all of 4 variables
(D) all functions of 4 variables
SOL 1.19 A 2n: 1 MUX can implement all logic functions of (n + 1) variable without andy
additional circuitry. Here n = 3 . Thus a 8 : 1 MUX can implement all logic functions
of 4 variable.
Here (D) is correct answer.
MCQ 1.20 A 0 to 6 counter consists of 3 flip flops and a combination circuit of 2 input gate
(s). The common circuit consists of
(A) one AND gate (B) one OR gate
(C) one AND gate and one OR gate (D) two AND gates
SOL 1.20 Counter must be reset when it count 111. This can be implemented by following
circuitry

Hence (D) is correct answer.


MCQ 1.21 The Fourier series expansion of a real periodic signal with fundamental frequency
f0 is given by gp (t) =/ cn e j2πf t . It is given that c3 = 3 + j5 . Then c−3 is
0

(A) 5 + j3 n =- 3
(B) − 3 − j5
(C) − 5 + j3 (D) 3 − j5
SOL 1.21 Hence (D) is correct answer.
HereC3 = 3 + j5
For real periodic signal
C−k = Ck*
Thus C−3 = Ck = 3 − j5
MCQ 1.22 Let x (t) be the input to a linear, time-invariant system. The required output is
4π (t − 2). The transfer function of the system should be
(A) 4e j4πf (B) 2e−j8πf
(C) 4e−j4πf (D) 2e j8πf

Brought to you by: Nodia and Company Visit us at: www.nodia.co.in


PUBLISHING FOR GATE
Page 8 GATE EC 2003 www.gatehelp.com

SOL 1.22 Hence (C) is correct answer.


y (t) = 4x (t − 2)
Taking Fourier transform we get
Y (e j2πf ) = 4e−j2πf2 X (e j2πf ) Time Shifting property

Y (e j2πf )
or j 2π f
= 4e−4jπf
X (e )

Thus H (e j2πf ) = 4e−4jπf


MCQ 1.23 A sequence x (n) with the z −transform X (z) = z 4 + z2 − 2z + 2 − 3z−4 is applied as an
input to a linear, time-invariant system with the impulse response h (n) = 2δ (n − 3)
where
1, n = 0
δ (n) = )
0, otherwise
The output at n = 4 is
(A) − 6 (B) zero
(C) 2 (D) − 4
SOL 1.23 Hence (B) is correct answer.
We have h (n) = 3δ (n − 3)
or H (z) = 2z−3 Taking z transform
−4
X (z) = z + z − 2z + 2 − 3z
4 2

Now Y (z) = H (z) X (z)


= 2z−3 (z 4 + z2 − 2z + 2 − 3z−4)
= 2 (z + z−1 − 2z−2 + 2z−3 − 3z−7)
Taking inverse z transform we have
y (n) = 2[ δ (n + 1) + δ (n − 1) − 2δ (n − 2)+ 2δ (n − 3) − 3δ (n − 7)]
At n = 4 ,y (4) = 0
MCQ 1.24 Fig. shows the Nyquist plot of the open-loop transfer function G (s) H (s) of a system.
If G (s) H (s) has one right-hand pole, the closed-loop system is

(A) always stable


(B) unstable with one closed-loop right hand pole
(C) unstable with two closed-loop right hand poles
(D) unstable with three closed-loop right hand poles

Brought to you by: Nodia and Company Visit us at: www.nodia.co.in


PUBLISHING FOR GATE
Page 9 GATE EC 2003 www.gatehelp.com

SOL 1.24 Hence (A) is correct option.


Z = P−N
N " Net encirclement of (− 1 + j0) by Nyquist plot,
P " Number of open loop poles in right hand side of s − plane
Z " Number of closed loop poles in right hand side of s − plane
Here N = 1 and P = 1
Thus Z =0
Hence there are no roots on RH of s −plane and system is always stable.
MCQ 1.25 A PD controller is used to compensate a system. Compared to the uncompensated
system, the compensated system has
(A) a higher type number (B) reduced damping
(C) higher noise amplification (D) larger transient overshoot
SOL 1.25 PD Controller may accentuate noise at higher frequency. It does not effect the type
of system and it increases the damping. It also reduce the maximum overshoot.
Hence (C) is correct option.
MCQ 1.26 The input to a coherent detector is DSB-SC signal plus noise. The noise at the
detector output is
(A) the in-phase component (B) the quadrature - component
(C) zero (D) the envelope
SOL 1.26 The input is a coherent detector is DSB - SC signal plus noise. The noise at the
detector output is the in-phase component as the quadrature component nq (t) of
the noise n (t) is completely rejected by the detector.
Hence (A) is correct option.
MCQ 1.27 The noise at the input to an ideal frequency detector is white. The detector is
operating above threshold. The power spectral density of the noise at the output is
(A) raised - cosine (B) flat
(C) parabolic (D) Gaussian
SOL 1.27 The noise at the input to an ideal frequency detector is white. The PSD of noise at
the output is parabolic
Hence (C) is correct option.
MCQ 1.28 At a given probability of error, binary coherent FSK is inferior to binary coherent
PSK by.
(A) 6 dB (B) 3 dB
(C) 2 dB (D) 0 dB
SOL 1.28 Hence (B) is correct option.
We have Pe = 1 erfc c Ed m
2 2η

Brought to you by: Nodia and Company Visit us at: www.nodia.co.in


PUBLISHING FOR GATE
Page 10 GATE EC 2003 www.gatehelp.com

Since Pe of Binary FSK is 3 dB inferior to binary PSK


MCQ 1.29 The unit of 4# H is
(A) Ampere (B) Ampere/meter
(C) Ampere/meter 2 (D) Ampere-meter
SOL 1.29 By Maxwells equations
4# H = 2D + J
2t
Thus 4# H has unit of current density J that is A/m2
Hence (C) is correct option
MCQ 1.30 The depth of penetration of electromagnetic wave in a medium having conductivity
σ at a frequency of 1 MHz is 25 cm. The depth of penetration at a frequency of 4
MHz will be
(A) 6.25 dm (B) 12.50 cm
(C) 50.00 cm (D) 100.00 cm
SOL 1.30 Hence (B) is correct option.
We know that δ \ 1
f
Thus δ2 = f1
δ1 f2
δ2 = 1
25 4
or δ2 = 1 # 25 = 12.5 cm
4

Q.31-90 Carry Two Marks Each

MCQ 1.31 Twelve 1 Ω resistance are used as edges to form a cube. The resistance between two
diagonally opposite corners of the cube is
(A) 5 Ω (B) 1 Ω
6

(C) 6 Ω (D) 3 Ω
5 2
SOL 1.31 For current i there is 3 similar path. So current will be divide in three path

Brought to you by: Nodia and Company Visit us at: www.nodia.co.in


PUBLISHING FOR GATE
Page 11 GATE EC 2003 www.gatehelp.com

so, we get
Vab − b i # 1l − b i # 1l − b 1 # 1l = 0
3 6 3
Vab = R = 1 + 1 + 1 = 5 Ω
eq
i 3 6 3 6
Hence (A) is correct option.
MCQ 1.32 The current flowing through the resistance R in the circuit in the figure has the
form P cos 4t where P is

(A) (0.18 + j0.72) (B) (0.46 + j1.90)


(C) − (0.18 + j1.90) (D) − (0.192 + j0.144)
SOL 1.32 Data are missing in question as L1 &L2 are not given

The circuit for Q. 33 & 34 is given below.


Assume that the switch S is in position 1 for a long time and thrown to position
2 at t = 0 .

MCQ 1.33 At t = 0+ , the current i1 is


(A) − V (B) − V
2R R
Brought to you by: Nodia and Company Visit us at: www.nodia.co.in
PUBLISHING FOR GATE
Page 12 GATE EC 2003 www.gatehelp.com

(C) − V (D) zero


4R
SOL 1.33 Data are missing in question as L1 &L2 are not given
MCQ 1.34 I1 (s) and I2 (s) are the Laplace transforms of i1 (t) and i2 (t) respectively. The
equations for the loop currents I1 (s) and I2 (s) for the circuit shown in the figure,
after the switch is brought from position 1 to position 2 at t = 0 , are
R + Ls + Cs1 − Ls I1 (s) V
(A) > H= G = G
s
=
− Ls R + Cs I2 (s)
1
0
R + Ls + Cs1 − Ls I1 (s) − Vs
(B) > 1 H= G == G
− Ls R + Cs I2 (s) 0
R + Ls + Cs1 − Ls − Vs
(C) >
R + Ls + Cs1 H=I2 (s)G = 0 G
I1 (s)
=
− Ls
R + Ls + Cs1 − Cs V
(D) >
R + Ls + Cs1 H=I2 (s) = 0 G
I1 (s)
G
s
=
− Ls
SOL 1.34 At t = 0 - circuit is in steady state. So inductor act as short circuit and capacitor
act as open circuit.

At t = 0 - , i1 (0 -) = i2 (0 -) = 0
vc (0 -) = V
At t = 0+ the circuit is as shown in fig. The voltage across capacitor and current in
inductor can’t be changed instantaneously. Thus

At t = 0+ , i1 = i2 =− V
2R
Hence (A) is correct option.

Brought to you by: Nodia and Company Visit us at: www.nodia.co.in


PUBLISHING FOR GATE
Page 13 GATE EC 2003 www.gatehelp.com

MCQ 1.35 An input voltage v (t) = 10 2 cos (t + 10c) + 10 5 cos (2t + 10c) V is applied to a
series combination of resistance R = 1 Ω and an inductance L = 1 H. The resulting
steady-state current i (t) in ampere is
(A) 10 cos (t + 55c) + 10 cos (2t + 10c + tan−1 2)
(B) 10 cos (t + 55c) + 10 3
2 cos (2t + 55c)
(C) 10 cos (t − 35c) + 10 cos (2t + 10c − tan−1 2)
(D) 10 cos (t − 35c) + 3
2 cos (2t − 35c)
SOL 1.35 Hence (C) is correct option
v (t) = 10 2 cos (t + 10c) + 10 5 cos (2t + 10c)
1 4444 2 4444 3 1 4444 4 2 4444 43
v1 v2

Thus we get ω1 = 1 and ω2 = 2


Now Z1 = R + jω1 L = 1 + j1
Z2 = R + jω2 L = 1 + j2
v (t) v (t)
i (t) = 1 + 2
Z1 Z2
10 2 cos (t + 10c) 10 5 cos (2t + 10c)
= +
1+j 1 + j2
10 2 cos (t + 10c) 10 5 cos (2t + 10c)
= +
12 + 22 + tan−1 1 12 + 22 tan−1 2
10 2 cos (t + 10c) 10 5 cos (2t + 10c)
= +
2 + tan−1 45c 5 tan−1 2
i (t) = 10 cos (t − 35c) + 10 cos (2t + 10c − tan−1 2)
MCQ 1.36 The driving point impedance Z (s) of a network has the pole-zero locations as
shown in the figure. If Z (0) = 3 , then Z (s) is

3 (s + 3) 2 (s + 3)
(A) (B)
s + 2s + 3
2
s + 2s + 2
2

3 (s + 3) 2 (s − 3)
(C) 2 (D) 2
s + 2s + 2 s − 2s − 3

SOL 1.36 Hence (B) is correct option.


Zeros =− 3
Pole1 =− 1 + j
Pole 2 =− 1 − j

Brought to you by: Nodia and Company Visit us at: www.nodia.co.in


PUBLISHING FOR GATE
Page 14 GATE EC 2003 www.gatehelp.com

K (s + 3)
Z (s) =
(s + 1 + j)( s + 1 − j)
K (s + 3) K (s + 3)
= =
(s + 1) − j
2 2
(s + 1) 2 + 1
From problem statement Z (0) ω = 0 = 3
Thus 3K = 3 and we get K = 2
2
2 (s + 3)
Z (s) = 2
s + 2s + 2
MCQ 1.37 The impedance parameters z11 and z12 of the two-port network in the figure are

(A) z11 = 2.75 Ω and z12 = 0.25 Ω (B) z11 = 3 Ω and z12 = 0.5 Ω
(C) z11 = 3 Ω and z12 = 0.25 Ω (D) z11 = 2.25 Ω and z12 = 0.5 Ω
SOL 1.37 Using 3− Y conversion

R1 =2 # 1 = 2 = 0. 5
2+1+1 4
R2 = 1 # 1 = 1 = 0.25
2+1+1 4
R3 = 2 # 1 = 0.5
2+1+1
Now the circuit is as shown in figure below.

Now z11 = V1 = 2 + 0.5 + 0.25 = 2.75


I1 I2 = 0

z12 = R3 = 0.25

Brought to you by: Nodia and Company Visit us at: www.nodia.co.in


PUBLISHING FOR GATE
Page 15 GATE EC 2003 www.gatehelp.com

Hence (A) is correct option.


MCQ 1.38 An n −type silicon bar 0.1 cm long and 100 μm2 i cross-sectional area has a majority
carrier concentration of 5 # 1020 /m 2 and the carrier mobility is 0.13 m2 /V-s at 300
K. If the charge of an electron is 1.5 # 10 - 19 coulomb, then the resistance of the bar
is
(A) 106 Ohm (B) 10 4 Ohm
(C) 10 - 1 Ohm (D) 10 - 4 Ohm
SOL 1.38 Hence option (A) is correct.
ρl
We that R = , ρ = 1 and α = nqun
A σ
From above relation we have
R = 1
nqμn A
= 0.1 # 10 - 2 = 106 Ω
20 - 19 - 12
5 # 10 # 1.6 # 10 # 0.13 # 100 # 10
MCQ 1.39 The electron concentration in a sample of uniformly doped n -type silicon at 300
K varies linearly from 1017 /cm 3 at x = 0 to 6 # 1016 /cm 3 at x = 2μm . Assume a
situation that electrons are supplied to keep this concentration gradient constant
with time. If electronic charge is 1.6 # 10 - 19 coulomb and the diffusion constant
Dn = 35 cm 2 /s, the current density in the silicon, if no electric field is present, is
(A) zero (B) -112 A/cm 2
(C) +1120 A/cm 2 (D) -1120 A/cm 2
SOL 1.39 Hence option (D) is correct.
dn = 6 # 1016 − 1017
dx 2 # 10 - 4 − 0
=− 2 # 1020
Now Jn = nqμe E + Dn q dn
dx
Since no electric field is present, E = 0 and we get
So, Jn = qDn dn
dx
= 1.6 # 10 - 19 # 35 # (− 2 # 1020) =− 1120 A/cm 2
MCQ 1.40 Match items in Group 1 with items in Group 2, most suitably.
Group 1 Group 2
P. LED 1. Heavy doping
Q. Avalanche photo diode 2. Coherent radiation
R. Tunnel diode 3. Spontaneous emission
S. LASER 4. Current gain

Brought to you by: Nodia and Company Visit us at: www.nodia.co.in


PUBLISHING FOR GATE
Page 16 GATE EC 2003 www.gatehelp.com

(A) P - 1, Q - 2, R - 4, S - 3
(B) P - 2, Q - 3, R - 1, S - 4
(C) P - 3 Q - 4, R - 1, S - 2
(D) P - 2, Q - 1, R - 4, S - 3
SOL 1.40 LED works on the principal of spontaneous emission.
In the avalanche photo diode due to the avalanche effect there is large current gain.
Tunnel diode has very large doping.
LASER diode are used for coherent radiation.
Hence option (C) is correct.
MCQ 1.41 At 300 K, for a diode current of 1 mA, a certain germanium diode requires a
forward bias of 0.1435 V, whereas a certain silicon diode requires a forward bias of
0.718 V. Under the conditions state above, the closest approximation of the ratio
of reverse saturation current in germanium diode to that in silicon diode is
(A) 1 (B) 5
(C) 4 # 103 (D) 8 # 103
SOL 1.41 Hence option (C) is correct.
I = Io `e η V − 1j
V D1
We know that si T

where η = 1 for germanium and η = 2 silicon. As per question


Io `e e − 1j = Io `e ηV − 1j
V
Dsi V DGe
hVT
n Ge T

VDsi 0.718
= eVηV − 1 = e 2 #026.1435
# 10 − 1
Io
= 4 # 103
-3
or si T

Io e 26 # 10 − 1
e ηV − 1
DGe
si -3
T

MCQ 1.42 A particular green LED emits light of wavelength 5490 Ac. The energy bandgap of
the semiconductor material used there is
(Plank’s constant = 6.626 # 10 - 34 J − s )
(A) 2.26 eV (B) 1.98 eV
(C) 1.17 eV (D) 0.74 eV
SOL 1.42 Hence option (A) is correct
−34 8
Eg = hc = 6.626 # 10 # −310# 10 = 3.62 J
λ 54900 # 10
E (J) −19
In eV Eg (eV) = g = 3.62 # 10−19 = 2.26 eV
e 1.6 # 10
Alternatively
Eg = 1.24 eV = 1.24 = 2.26 eV
λ (μm) 5490 # 10−4 μm

MCQ 1.43 When the gate-to-source voltage (VGs) of a MOSFET with threshold voltage of 400
mV, working in saturation is 900 mV, the drain current is observed to be 1 mA.

Brought to you by: Nodia and Company Visit us at: www.nodia.co.in


PUBLISHING FOR GATE
Page 17 GATE EC 2003 www.gatehelp.com

Neglecting the channel width modulation effect and assuming that the MOSFET is
operating at saturation, the drain current for an applied VGS of 1400 mV is
(A) 0.5 mA (B) 2.0 mA
(C) 3.5 mA (D) 4.0 mA
SOL 1.43 We know that
ID = K (VGS − VT ) 2
I (VGS2 − VT ) 2
Thus D2
=
ID1 (VGS1 − VT ) 2
Substituting the values we have
ID2 = (1.4 − 0.4) = 4
2

ID1 (0.9 − 0.4) 2


or ID2 = 4IDI = 4 mA
Hence option (D) is correct.
MCQ 1.44 If P is Passivation, Q is n −well implant, R is metallization and S is source/drain
diffusion, then the order in which they are carried out in a standard n −well CMOS
fabrication process, is
(A) P − Q − R − S (B) Q − S − R − P
(C) R − P − S − Q (D) S − R − Q − P
SOL 1.44 In n −well CMOS fabrication following are the steps :
(i) n − well implant (Q)
(ii) Source drain diffusion (S)
(iii) Metalization (R)
(iv) Passivation (P)
Hence option (B) is correct.
MCQ 1.45 An amplifier without feedback has a voltage gain of 50, input resistance of 1 k Ω
and output resistance of 2.5 kΩ. The input resistance of the current-shunt negative
feedback amplifier using the above amplifier with a feedback factor of 0.2, is
(A) 1 kΩ (B) 1 kΩ
11 5
(C) 5 kΩ (D) 11 kΩ
SOL 1.45 Hence (A) is correct option.
We have Ri = 1kΩ, β = 0.2, A = 50

Ri
Thus, Rif = = 1 kΩ
(1 + Aβ) 11

MCQ 1.46 In the amplifier circuit shown in the figure, the values of R1 and R2 are such that
the transistor is operating at VCE = 3 V and IC = 1.5 mA when its β is 150. For a
transistor with β of 200, the operating point (VCE , IC ) is

Brought to you by: Nodia and Company Visit us at: www.nodia.co.in


PUBLISHING FOR GATE
Page 18 GATE EC 2003 www.gatehelp.com

(A) (2 V, 2 mA) (B) (3 V, 2 mA)


(C) (4 V, 2 mA) (D) (4 V, 1 mA)
SOL 1.46 The DC equivalent circuit is shown as below. This is fixed bias circuit operating in
active region.

In first case
VCC − IC1 R2 − VCE1 = 0
or 6 − 1.5mR2 − 3 = 0
or R2 = 2kΩ
IB1 = IC1 = 1.5m = 0.01 mA
β1 150
In second case IB2 will we equal to IB1 as there is no in R1.
Thus IC2 = β2 IB2 = 200 # 0.01 = 2 mA
VCE2 = VCC − IC2 R2 = 6 − 2m # 2 kΩ = 2 V
Hence (A) is correct option.
MCQ 1.47 The oscillator circuit shown in the figure has an ideal inverting amplifier. Its
frequency of oscillation (in Hz) is

(A) 1 (B) 1
(2π 6 RC) (2πRC)

(C) 1 (D) 6
( 6 RC) (2πRC)

Brought to you by: Nodia and Company Visit us at: www.nodia.co.in


PUBLISHING FOR GATE
Page 19 GATE EC 2003 www.gatehelp.com

SOL 1.47 The given circuit is a R − C phase shift oscillator and frequency of its oscillation is
f = 1
2π 6 RC
Hence (A) is correct option.
MCQ 1.48 The output voltage of the regulated power supply shown in the figure is

(A) 3 V (B) 6 V
(C) 9 V (D) 12 V
SOL 1.48 If we see th figure we find that the voltage at non-inverting terminal is 3 V by the
zener diode and voltage at inverting terminal will be 3 V. Thus Vo can be get by
applying voltage division rule, i.e.
20 V = 3
20 + 40 o
or V0 = 9 V
Hence (C) is correct option.
MCQ 1.49 The action of JFET in its equivalent circuit can best be represented as a
(A) Current controlled current source
(B) Current controlled voltage source
(C) Voltage controlled voltage source
(D) Voltage controlled current source
SOL 1.49 For a JFET in active region we have
IDS = IDSS c1 − VGS m
2

VP
From above equation it is clear that the action of a JFET is voltage controlled
current source.
Hence option (D) is correct.
MCQ 1.50 If the op-amp in the figure is ideal, the output voltage Vout will be equal to

Brought to you by: Nodia and Company Visit us at: www.nodia.co.in


PUBLISHING FOR GATE
Page 20 GATE EC 2003 www.gatehelp.com

(A) 1 V (B) 6 V
(C) 14 V (D) 17 V
SOL 1.50 The circuit is as shown below

V+ = 8 (3) = 8 kΩ
1+8 3
V+ = V- = 8 V
3
Now applying KCL at inverting terminal we get
V- − 2 + V- − Vo = 0
1 5
or Vo = 6V- − 10
= 6 # 8 − 10 = 6 V
3
Hence (B) is correct option.
MCQ 1.51 Three identical amplifiers with each one having a voltage gain of 50, input resistance
of 1 kΩ and output resistance of 250 Ω are cascaded. The opened circuit voltages
gain of the combined amplifier is
(A) 49 dB (B) 51 dB
(C) 98 dB (D) 102 dB
SOL 1.51 The equivalent circuit of 3 cascade stage is as shown in fig.

Brought to you by: Nodia and Company Visit us at: www.nodia.co.in


PUBLISHING FOR GATE
Page 21 GATE EC 2003 www.gatehelp.com

V2 = 1k 50V1 = 40V1
1k + 0.25k
Similarly V3 = 1k 50V2 = 40V2
1k + 0.25k
or V3 = 40 # 40V1
Vo = 50V3 = 50 # 40 # 40V1
or AV = Vo = 50 # 40 # 40 = 8000
V1
or 20 log AV = 20 log 8000 = 98 dB
Hence (C) is correct option.
MCQ 1.52 An ideal sawtooth voltages waveform of frequency of 500 Hz and amplitude 3 V
is generated by charging a capacitor of 2 μF in every cycle. The charging requires
(A) Constant voltage source of 3 V for 1 ms
(B) Constant voltage source of 3 V for 2 ms
(C) Constant voltage source of 1 mA for 1 ms
(D) Constant voltage source of 3 mA for 2 ms
SOL 1.52 If a constant current is made to flow in a capacitor, the output voltage is integration
of input current and that is sawtooth waveform as below :
#
t
VC = 1 idt
C 0
The time period of wave form is
T = 1 = 1 = 2 m sec
f 500
-3

#
20 # 10
Thus 3= 1 idt
2 # 106 0
or i (2 # 10 - 3 − 0) = 6 # 10 - 6
or i = 3 mA
Thus the charging require 3 mA current source for 2 msec.
Hence (D) is correct option
MCQ 1.53 The circuit in the figure has 4 boxes each described by inputs P, Q, R and outputs
Y, Z with Y = P 5 Q 5 R and Z = RQ + PR + QP
The circuit acts as a

Brought to you by: Nodia and Company Visit us at: www.nodia.co.in


PUBLISHING FOR GATE
Page 22 GATE EC 2003 www.gatehelp.com

(A) 4 bit adder giving P + Q


(B) 4 bit subtractor giving P − Q
(C) 4 bit subtractor giving Q-P
(D) 4 bit adder giving P + Q + R
SOL 1.53 Hence (B) is correct answer.
We have Y = P 5 Q 5 R
Z = RQ + PR + QP
Here every block is a full subtractor giving P − Q − R where R is borrow. Thus
circuit acts as a 4 bit subtractor giving P − Q .
MCQ 1.54 If the function W, X, Y and Z are as follows
W = R + PQ + RS X = PQRS + PQRS + PQRS
Y = RS + PR + PQ + P .Q Z = R + S + PQ + P .Q .R + PQ .S
Then,
(A) W = Z, X = Z (B) W = Z, X = Y
(C) W = Y (D) W = Y = Z
SOL 1.54 Hence (A) is correct answer.
W = R + PQ + RS
X = PQRS + PQRS + PQRS
Y = RS + PR + PQ + PQ
= RS + PR $ PQ $ PQ
= RS + (P + R )( P + Q)( P + Q)
= RS + (P + PQ + PR + QR )( P + Q)
= RS + PQ + QR (P + P ) + QR
= RS + PQ + QR
Z = R + S + PQ + PQR + PQS
= R + S + PQ $ PQR $ PQS
= R + S + (P + Q )( P + Q + R)( P + Q + S)
= R + S + PQ + PQ + PQS + PR + PQR
+ PRS + PQ + PQS + PQR + QRS
= R + S + PQ + PQS + PR + PQR + PRS

Brought to you by: Nodia and Company Visit us at: www.nodia.co.in


PUBLISHING FOR GATE
Page 23 GATE EC 2003 www.gatehelp.com

+ PQS + PQR + QRS


= R + S + PQ (1 + S) + PR (1 + P ) + PRS
+ PQS + PQR + QRS
= R + S + PQ + PR + PRS + PQS
+ PQR + QRS
= R + S + PQ + PR (1 + Q ) + PQS + QRS
= R + S + PQ + PR + PQS + QRS
Thus W = Z and X = Z
MCQ 1.55 A 4 bit ripple counter and a bit synchronous counter are made using flip flops
having a propagation delay of 10 ns each. If the worst case delay in the ripple
counter and the synchronous counter be R and S respectively, then
(A) R = 10 ns, S = 40 ns (B) R = 40 ns, S = 10 ns
(C) R = 10 ns S = 30 ns (D) R = 30 ns, S = 10 ns
SOL 1.55 Propagation delay of flip flop is
tpd = 10 nsec
Propagation delay of 4 bit ripple counter
R = 4tpd = 40 ns
and in synchronous counter all flip-flop are given clock simultaneously, so
S = tpd = 10 ns
Hence (B) is correct answer.
MCQ 1.56 The DTL, TTL, ECL and CMOS famil GATE of digital ICs are compared in the
following 4 columns

(P) (Q) (R) (S)


Fanout is minimum DTL DTL TTL CMOS
Power consumption is TTL CMOS ECL DTL
minimum
Propagation delay is CMOS ECL TTL TTL
minimum
The correct column is
(A) P (B) Q
(C) R (D) S
SOL 1.56 The DTL has minimum fan out and CMOS has minimum power consumption.
Propagation delay is minimum in ECL.
Hence (B) is correct answer.
MCQ 1.57 The circuit shown in the figure is a 4 bit DAC

Brought to you by: Nodia and Company Visit us at: www.nodia.co.in


PUBLISHING FOR GATE
Page 24 GATE EC 2003 www.gatehelp.com

The input bits 0 and 1 are represented by 0 and 5 V respectively. The OP AMP
is ideal, but all the resistance and the 5 v inputs have a tolerance of ! 10%. The
specification (rounded to nearest multiple of 5%) for the tolerance of the DAC is
(A) ! 35% (B) ! 20%
(C) ! 10% (D) ! 5%
SOL 1.57 Hence (A) is correct answer.
Vo =− V1 :R bo + R b1 + R b2 + R b 3D
R 2R 4R 4R
Exact value when V1 = 5 , for maximum output
VoExact =− 5 :1 + 1 + 1 + 1 D =− 9.375
2 4 8
Maximum Vout due to tolerance
Vo max =− 5.5 :110 + 110 + 110 + 110 D
90 2 # 90 4 # 90 8 # 90
=− 12.604
Tolerance = 34.44% = 35%
MCQ 1.58 The circuit shown in figure converts

(A) BCD to binary code (B) Binary to excess - 3 code


(C) Excess -3 to gray code (D) Gray to Binary code
SOL 1.58 Hence (D) is correct answer.
Let input be 1010; output will be 1101
Let input be 0110; output will be 0100

Brought to you by: Nodia and Company Visit us at: www.nodia.co.in


PUBLISHING FOR GATE
Page 25 GATE EC 2003 www.gatehelp.com

Thus it convert gray to Binary code.


MCQ 1.59 In the circuit shown in the figure, A is parallel-in, parallel-out 4 bit register, which
loads at the rising edge of the clock C . The input lines are connected to a 4 bit bus,
W . Its output acts at input to a 16 # 4 ROM whose output is floating when the
input to a partial table of the contents of the ROM is as follows

Data 0011 1111 0100 1010 1011 1000 0010 1000

Address 0 2 4 6 8 10 11 14

The clock to the register is shown, and the data on the W bus at time t1 is 0110.
The data on the bus at time t2 is

(A) 1111 (B) 1011


(C) 1000 (D) 0010
SOL 1.59 After t = t1, at first rising edge of clock, the output of shift register is 0110, which
in input to address line of ROM. At 0110 is applied to register. So at this time data
stroed in ROM at 1010 (10), 1000 will be on bus.
When W has the data 0110 and it is 6 in decimal, and it’s data value at that add
is 1010

Brought to you by: Nodia and Company Visit us at: www.nodia.co.in


PUBLISHING FOR GATE
Page 26 GATE EC 2003 www.gatehelp.com

then 1010 i.e. 10 is acting as odd, at time t2 and data at that movement is 1000.
Hence (C) is correct answer.
MCQ 1.60 In an 8085 microprocessor, the instruction CMP B has been executed while the
content of the accumulator is less than that of register B . As a result
(A) Carry flag will be set but Zero flag will be reset
(B) Carry flag will be rest but Zero flag will be set
(C) Both Carry flag and Zero flag will be rest
(D) Both Carry flag and Zero flag will be set
SOL 1.60 CMP B & Compare the accumulator content with context of Register B
If A < R CY is set and zero flag will be reset.
Hence (A) is correct answer.
MCQ 1.61 Let X and Y be two statistically independent random variables uniformly
distributed in the ranges (− 1, 1) and (− 2, 1) respectively. Let Z = X + Y . Then
the probability that (z #− 1) is
(A) zero (B) 1
6

(C) 1 (D) 1
3 12
SOL 1.61 The pdf of Z will be convolution of pdf of X and pdf of Y as shown below.
#- 3 fZ (z) dz
z
Now p [Z # z] =

#- 3fZ (z) dz
-2
p [Z #− 2] =

= Area [z #− 2]
= 1 # 1 #1 = 1
2 6 12

Brought to you by: Nodia and Company Visit us at: www.nodia.co.in


PUBLISHING FOR GATE
Page 27 GATE EC 2003 www.gatehelp.com

Hence (D) is correct option.


MCQ 1.62 Let P be linearity, Q be time-invariance, R be causality and S be stability. A
discrete time system has the input-output relationship,
x (n) n$1
y (n) = *0, n=0
x (n + 1) n # − 1
where x (n) is the input and y (n) is the output. The above system has the properties
(A) P, S but not Q, R (B) P, Q, S but not R
(C) P, Q, R, S (D) Q, R, S but not P
SOL 1.62 System is non causal because output depends on future value
For n # 1 y (− 1) = x (− 1 + 1) = x (0)
y (n − n0) = x (n − n0 + 1) Time varying
y (n) = x (n + 1) Depends on Future
i.e. y (1) = x (2) None causal
For bounded input, system has bounded output. So it is stable.
y (n) = x (n) for n $ 1
= 0 for n = 0
= x (x + 1) for n #− 1
So system is linear.
Hence (A) is correct answer.

Common data for Q 63 & 64 :

Brought to you by: Nodia and Company Visit us at: www.nodia.co.in


PUBLISHING FOR GATE
Page 28 GATE EC 2003 www.gatehelp.com

The system under consideration is an RC low-pass filter (RC-LPF) with


R = 1 k Ω and C = 1.0 μ F.

MCQ 1.63 Let H (f) denote the frequency response of the RC-LPF. Let f1 be the highest
H (f1)
frequency such that 0 # f # f1 $ 0.95 . Then f1 (in Hz) is
H (0)
(A) 324.8 (B) 163.9
(C) 52.2 (D) 104.4
SOL 1.63 The frequency response of RC-LPF is
H (f) = 1
1 + j2πfRC
Now H (0) = 1
H (f1) 1
= $ 0.95
H (0) 1 + 4π2 f12 R2 C2
or 1 + 4π2 f12 R2 C2 # 1.108
or 4π2 f12 R2 C2 # 0.108
or 2πf1 RC # 0.329
or f1 # 0.329
2πRC
or f1 # 0.329
2πRC
or f1 # 0.329
2π1k # 1μ
or f1 # 52.2 Hz
Thus f1 max = 52.2 Hz
Hence (C) is correct answer.
MCQ 1.64 Let tg (f) be the group delay function of the given RC-LPF and f2 = 100 Hz. Then
tg (f2) in ms, is
(A) 0.717 (B) 7.17
(C) 71.7 (D) 4.505
SOL 1.64 Hence (A) is correct answer
H (ω) = 1
1 + jωRC
θ (ω) =− tan−1 ωRC
dθ (ω) RC
tg =− =
dω 1 + ω2 R2 C2
= 10−3 = 0.717 ms
1 + 4π # 10 4 # 10−6
2

Common Data for Questions 65 & 66 :

Brought to you by: Nodia and Company Visit us at: www.nodia.co.in


PUBLISHING FOR GATE
Page 29 GATE EC 2003 www.gatehelp.com

X (t) is a random process with a constant mean value of 2 and the auto
correlation function Rxx (τ) = 4 (e - 0.2 τ + 1).

MCQ 1.65 Let X be the Gaussian random variable obtained by sampling the process at t = ti
and let
#
3 x2

Q (α) = − 1 e dy 2

α 2π
The probability that 6x # 1@ is
(A) 1 − Q (0.5) (B) Q (0.5)

(C) Q c 1 m (D) 1 − Q c 1 m
2 2 2 2
SOL 1.65 Hence (D) is correct option.
We have RXX (τ) = 4 (e - 0.2 τ + 1)
RXX (0) = 4 (e - 0.2 0 + 1) = 8 = σ2
or σ = 2 2 Given
mean μ =0
Now P (x # 1) = Fx (1)
X−μ
= 1 − Qc
σ m
at x = 1

= 1 − Qc 1 − 0 m = 1 − Qc 1 m
2 2 2 2

MCQ 1.66 Let Y and Z be the random variable obtained by sampling X (t) at t = 2 and t = 4
respectively. Let W = Y − Z . The variance of W is
(A) 13.36 (B) 9.36
(C) 2.64 (D) 8.00
SOL 1.66 Hence (C) is correct option.
W = Y−Z
E [W2] = E [Y − Z] 2
= E [Y2] + E [Z2] − 2E [YZ]
= σw2
We haveE [X2 (t)] = Rx (10)
= 4 [e - 0.2 0 + 1] = 4 [1 + 1] = 8
E [Y2] = E [X2 (2)] = 8
E [Z2] = E [X2 (4)] = 8
E [YZ] = RXX (2) = 4 [e−0.2 (4 − 2) + 1] = 6.68
E [W2] = σw2 = 8 + 8 − 2 # 6.68 = 2.64
MCQ 1.67 Let x (t) = 2 cos (800π) + cos (1400πt). x (t) is sampled with the rectangular pulse
train shown in the figure. The only spectral components (in kHz) present in the
sampled signal in the frequency range 2.5 kHz to 3.5 kHz are

Brought to you by: Nodia and Company Visit us at: www.nodia.co.in


PUBLISHING FOR GATE
Page 30 GATE EC 2003 www.gatehelp.com

(A) 2.7, 3.4 (B) 3.3, 3.6


(C) 2.6, 2.7, 3.3, 3.4, 3.6 (D) 2.7, 3.3
SOL 1.67 Hence (D) is correct option.
The frequency of pulse train is
f 1- 3 = 1 k Hz
10
The Fourier Series coefficient of given pulse train is
−T /2
Cn = 1 #
o

Ae−jnω t dt o

To −T /2 o

−To /6
= 1
To #
−To /6
Ae−jηω t dt
o

= A [e−jω t] −−TT //66


o o

To (− jηωo) o

= A (e−jω t − e jηω T /6)


o o o

(− j2πn)
= A (e jηπ/3 − e−jηπ/3)
j2πn
or Cn = A sin ` nπ j
πn 3
From Cn it may be easily seen that 1, 2, 4, 5, 7, harmonics are present and 0, 3, 6, 9,..
are absent. Thus p (t) has 1 kHz, 2 kHz, 4 kHz, 5 kHz, 7 kHz,... frequency component
and 3 kHz, 6 kHz.. are absent.
The signal x (t) has the frequency components 0.4 kHz and 0.7 kHz. The sampled
signal of x (t) i.e. x (t)* p (t) will have
1 ! 0.4 and 1 ! 0.7 kHz
2 ! 0.4 and 2 ! 0.7 kHz
4 ! 0.4 and 4 ! 0.7 kHz
Thus in range of 2.5 kHz to 3.5 kHz the frequency present is
2 + 0.7 = 2.7 kHz
4 − 0.7 = 3.3 kHz
MCQ 1.68 The signal flow graph of a system is shown in Fig. below. The transfer function
C (s)/ R (s) of the system is

Brought to you by: Nodia and Company Visit us at: www.nodia.co.in


PUBLISHING FOR GATE
Page 31 GATE EC 2003 www.gatehelp.com

(A) 6 (B) 6s
s + 29s + 6
2
s + 29s + 6
2

s (s + 2) s (s + 27)
(C) 2 (D) 2
s + 29s + 6 s + 29s + 6

SOL 1.68 Mason Gain Formula


Σpk 3 k
T (s) =
3
In given SFG there is only forward path and 3 possible loop.
p1 = 1
31 = 1 + 3 + 24 = s + 27
s s s
L1 = − 2 , L2 = − 24 and L3 = − 3
s s s
where L1 and L3 are non-touching
C (s) p1 3 1
This =
R (s) 1 − (loop gain) + pair of non − touching loops
^ s +s27 h ^ s h
s + 27

= =
1 − ^ −s3 − 24s − s2 h + −s2 . −s3 1 + 29s + s62

s (s + 27)
= 2
s + 29s + 6
Hence (D) is correct option.

MCQ 1.69 The root locus of system G (s) H (s) = K has the break-away point
s (s + 2)( s + 3)
located at
(A) (− 0.5, 0) (B) (− 2.548, 0)
(C) (− 4, 0) (D) (− 0.784, 0)
SOL 1.69 We have
1 + G (s) H (s) = 0
or 1+ K =0
s (s + 2)( s + 3)
or K =− s (s2 + 5s2 + 6s)
dK =− (3s2 + 10s + 6) = 0
ds
which gives s = − 10 ! 100 − 72 =− 0.784, − 2.548
6
The location of poles on s − plane is

Brought to you by: Nodia and Company Visit us at: www.nodia.co.in


PUBLISHING FOR GATE
Page 32 GATE EC 2003 www.gatehelp.com

Since breakpoint must lie on root locus so s =− 0.748 is possible.


Hence (D) is correct option.
MCQ 1.70 The approximate Bode magnitude plot of a minimum phase system is shown in Fig.
below. The transfer function of the system is

(s + 0.1) 3 (s + 0.1) 3
(A) 108 (B) 107
(s + 10) 2 (s + 100) (s + 10)( s + 100)
(s + 0.1) 2 (s + 0.1) 3
(C) (D)
(s + 10) 2 (s + 100) (s + 10)( s + 100) 2

SOL 1.70 The given bode plot is shown below

At ω = 0.1 change in slope is + 60 dB " 3 zeroes at ω = 0.1


At ω = 10 change in slope is − 40 dB " 2 poles at ω = 10
At ω = 100 change in slope is − 20 dB " 1 poles at ω = 100
K ( 0s.1 + 1) 3
Thus T (s) = s
( 10 + 1) 2 ( 100
s
+ 1)
Now 20 log10 K = 20
or K = 10
10 ( 0s.1 + 1) 3 108 (s + 0.1) 3
Thus T (s) = s =
( 10 + 1) 2 ( 100
s
+ 1) (s + 10) 2 (s + 100)
Hence (A) is correct option.

Brought to you by: Nodia and Company Visit us at: www.nodia.co.in


PUBLISHING FOR GATE
Page 33 GATE EC 2003 www.gatehelp.com

MCQ 1.71 A second-order system has the transfer function


C (s)
= 2 4
R (s) s + 4s + 4
With r (t) as the unit-step function, the response c (t) of the system is represented
by

SOL 1.71 The characteristics equation is


s2 + 4s + 4 = 0
Comparing with
s2 + 2ξωn + ωn2 = 0
we get 2ξωn = 4 and ωn2 = 4
Thus ξ =1 Critically damped
ts = 4 = 4 = 2
ξωn 1#2
Hence (B) is correct option.
MCQ 1.72 The gain margin and the phase margin of feedback system with
G (s) H (s) = 8 are
(s + 100) 3
(A) dB, 0c (B) 3, 3
(C) 3, 0c (D) 88.5 dB, 3
SOL 1.72 Hence (B) is correct option.
MCQ 1.73 The zero-input response of a system given by the state-space equation
xo1 1 0 x1 x1 (0) 1
=xo G = =1 1G=x G and =x (0)G = = 0 G is
2 2 2

Brought to you by: Nodia and Company Visit us at: www.nodia.co.in


PUBLISHING FOR GATE
Page 34 GATE EC 2003 www.gatehelp.com

tet et
(A) = G (B) = G
t t
et t
(C) = t G (D) = t G
te te
SOL 1.73 We have
xo1 1 0 x1 x1 (0) 1
=xo G = =1 G=
1 x2 G and =
x2 (0) G == G
0
2
1 0
A ==
1 1G
s 0 1 0 s−1 0
(sI − A) = = G −= G = =
0 s 1 1 − 1 s − 1G
(s − 1) 0 1
0
1 > H = > +1 H
−1 s−1
(sI − A) =
(s − 1) 2 + 1 (s − 1)
1
(s − 1) 2 s−1
t
L−1 [(sI − A) −1] = eAt = = t t G
e 0
te e
et 0 1 et
x (t) = e # [x (t0)] = = t t G= G = = t G
At
te e 0 te
Hence (C) is correct option.
MCQ 1.74 A DSB-SC signal is to be generated with a carrier frequency fc = 1 MHz using a
non-linear device with the input-output characteristic V0 = a0 vi + a1 vi3 where a0
and a1 are constants. The output of the non-linear device can be filtered by an
appropriate band-pass filter.
Let Vi = Aci cos (2πfi ct) + m (t) is the message signal. Then the value of fci (in MHz)
is
(A) 1.0 (B) 0.333
(B) 0.5 (D) 3.0
SOL 1.74 Hence (C) is correct option.
vi = Ac1 cos (2πfc t) + m (t)
v0 = ao vi + avi3
v0 = a0 [Ac' cos (2πfc' t) + m (t)] + a1 [Ac' cos (2πfc' t) + m (t)] 3
= a0 Ac' cos (2πfc' t) + a0 m (t) + a1 [(Ac' cos 2πfc' t) 3
+ (Ac' cos (2πfc') t) 2 m (t) + 3Ac' cos (2πfc' t) m2 (t) + m3 (t)]
1 + cos (4πfc' t)
= a0 Ac' cos (2πfc' t) + a0 m (t) + a1 (Ac' cos 2fc' t) 3 + 3a1 Ac'2 ; E m (t)
2
= 3a1 Ac' cos (2πfc' t) m2 (t) + m3 (t)

The term 3a1 Ac' ( cos 42πf t ) m (t) is a DSB-SC signal having carrier frequency 1. MHz.
'
c

Thus 2fc' = 1 MHz or fc' = 0.5 MHz

Brought to you by: Nodia and Company Visit us at: www.nodia.co.in


PUBLISHING FOR GATE
Page 35 GATE EC 2003 www.gatehelp.com

Common Data for Question 75 & 76 :


Let m (t) = cos [(4π # 103) t] be the message signal &
c (t) = 5 cos [(2π # 106 t)] be the carrier.

MCQ 1.75 c (t) and m (t) are used to generate an AM signal. The modulation index of the
Total sideband power
generated AM signal is 0.5. Then the quantity is
Carrier power
(A) 1 (B) 1
2 4

(C) 1 (D) 1
3 8
SOL 1.75 Hence (D) is correct option.
PT = Pc c1 + α m
2

2
P (0.5) 2
Psb = Pc α = c
2

2 2
or Psb = 1
Pc 8

MCQ 1.76 c (t) and m (t) are used to generated an FM signal. If the peak frequency deviation
of the generated FM signal is three times the transmission bandwidth of the AM
signal, then the coefficient of the term cos [2π (1008 # 103 t)] in the FM signal (in
terms of the Bessel coefficients) is
(A) 5J4 (3) (B) 5 J8 (3)
2

(C) 5 J8 (4) (D) 5J4 (6)


2
SOL 1.76 Hence (D) is correct option.
AM Band width = 2fm
Peak frequency deviation = 3 (2fm) = 6fm
6f
Modulation index β = m = 6
fm
The FM signal is represented in terms of Bessel function as
3
xFM (t) = Ac /Jn (β) cos (ωc − nωn) t
n =- 3
ωc + nωm = 2π (1008 # 103)
2π106 + n4π # 103 = 2π (1008 # 103), n = 4
Thus coefficient = 5J4 (6)
MCQ 1.77 Choose the correct one from among the alternative A, B, C, D after matching an
item in Group 1 with most appropriate item in Group 2.
Group 1 Group 2
P. Ring modulator 1. Clock recovery

Brought to you by: Nodia and Company Visit us at: www.nodia.co.in


PUBLISHING FOR GATE
Page 36 GATE EC 2003 www.gatehelp.com

Q. VCO 2. Demodulation of FM
R. Foster-Seely discriminator 3. Frequency conversion
S. Mixer 4. Summing the two inputs
5. Generation of FM
6. Generation of DSB-Sc
(A) P − 1; Q − 3; R − 2; S − 4 (B) P − 6; Q = 5; R − 2; S − 3
(C) P − 6; Q − 1; R − 3; S − 2 (D) P − 5; Q − 6; R − 1; S − 3
SOL 1.77 Hence (B) is correct option.
Ring modulation $ Generation of DSB - SC
VCO $ Generation of FM
Foster seely discriminator $ Demodulation of fm
mixer $ frequency conversion
MCQ 1.78 A superheterodyne receiver is to operate in the frequency range 550 kHz - 1650
kHz, with the intermediate frequency of 450 kHz. Let R = Cmax /Cmin denote the
required capacitance ratio of the local oscillator and I denote the image frequency
(in kHz) of the incoming signal. If the receiver is tuned to 700 kHz, then
(A) R = 4.41, I = 1600 (B) R = 2.10, I − 1150
(C) R = 3.0, I = 600 (D) R = 9.0, I = 1150
SOL 1.78 Hence (A) is correct option.
fmax = 1650 + 450 = 2100 kHz
fmin = 550 + 450 = 1000 kHz
or f = 1
2π LC
frequency is minimum, capacitance will be maximum
f2
R = Cmax = max 2
= (2.1) 2
Cmin fmin
or R = 4.41
fi = fc + 2fIF = 700 + 2 (455) = 1600 kHz
MCQ 1.79 A sinusoidal signal with peak-to-peak amplitude of 1.536 V is quantized into 128
levels using a mid-rise uniform quantizer. The quantization-noise power is
(A) 0.768 V (B) 48 # 10 - 6 V2
(B) 12 # 10 - 6 V2 (D) 3.072 V
SOL 1.79 Hence (C) is correct option.
2mp
Step size δ = = 1.536 = 0.012 V
L 128
(0.012) 2
Quantization Noise power = δ =
2

12 12
= 12 # 10−6 V2

Brought to you by: Nodia and Company Visit us at: www.nodia.co.in


PUBLISHING FOR GATE
Page 37 GATE EC 2003 www.gatehelp.com

MCQ 1.80 If Eb , the energy per bit of a binary digital signal, is 10 - 5 watt-sec and the one-sided
power spectral density of the white noise, N0 = 10 - 6 W/Hz, then the output SNR
of the matched filter is
(A) 26 dB (B) 10 dB
(C) 20 dB (D) 13 dB
SOL 1.80 Hence (D) is correct option.
Eb = 10 - 6 watt-sec
No = 10 - 5 W/Hz
6
(SNR) matched filler = EN
o
= 10 - 5 = .05
2
o
2 # 10
(SNR)dB = 10 log 10 (0.05) = 13 dB
MCQ 1.81 The input to a linear delta modulator having a step-size 3= 0.628 is a sine wave
with frequency fm and peak amplitude Em . If the sampling frequency fx = 40 kHz,
the combination of the sine-wave frequency and the peak amplitude, where slope
overload will take place is
Em fm
(A) 0.3 V 8 kHz
(B) 1.5 V 4 kHz
(C) 1.5 V 2 kHz
(D) 3.0 V 1 kHz
SOL 1.81 Hence (B) is correct option.
3 fs
For slopeoverload to take place Em $
2πfm
This is satisfied with Em = 1.5 V and fm = 4 kHz
MCQ 1.82 If S represents the carrier synchronization at the receiver and ρ represents the
bandwidth efficiency, then the correct statement for the coherent binary PSK is
(A) ρ = 0.5, S is required (B) ρ = 1.0, S is required
(C) ρ = 0.5, S is not required (D) ρ = 1.0, S is not required
SOL 1.82 Hence (A) is correct option.
If s " carrier synchronization at receiver
ρ " represents bandwidth efficiency
then for coherent binary PSK ρ = 0.5 and s is required.
MCQ 1.83 A signal is sampled at 8 kHz and is quantized using 8 - bit uniform quantizer.
Assuming SNRq for a sinusoidal signal, the correct statement for PCM signal with
a bit rate of R is
(A) R = 32 kbps, SNRq = 25.8 dB (B) R = 64 kbps, SNRq = 49.8 dB
(C) R = 64 kbps, SNRq = 55.8 dB (D) R = 32 kbps, SNRq = 49.8 dB
SOL 1.83 Hence (B) is correct option.

Brought to you by: Nodia and Company Visit us at: www.nodia.co.in


PUBLISHING FOR GATE
Page 38 GATE EC 2003 www.gatehelp.com

Bit Rate = 8k # 8 = 64 kbps


(SNR)q = 1.76 + 6.02n dB
= 1.76 + 6.02 # 8 = 49.8 dB
MCQ 1.84 Medium 1 has the electrical permittivity ε1 = 1.5ε0 farad/m and occupies the
region to the left of x = 0 plane. Medium 2 has the electrical permittivity ε2 = 2.5ε0
farad/m and occupies the region to the right of x = 0 plane. If E1 in medium 1 is
E1 = (2ux − 3uy + 1uz ) volt/m, then E2 in medium 2 is
(A) (2.0ux − 7.5uy + 2.5uz ) volt/m (B) (2.0ux − 2.0uy + 0.6uz ) volt/m
(C) (2.0ux − 3.0uy + 1.0uz ) volt/m (D) (2.0ux − 2.0uy + 0.6uz ) volt/m
SOL 1.84 Hence (C) is correct option.
We have E1 = 2ux − 3uy + 1uz
E1t = − 3uy + uy and E1n = 2ux
Since for dielectric material at the boundary, tangential component of electric field
are equal
E1t =− 3uy + uy = E2t (x = 0 plane)
E1n = 2ux
At the boundary the for normal component of electric field are
D1n = D2n
or ε1 E1n = ε2 E2n
or 1.5εo 2ux = 2.5εo E2n
or E2n = 3 ux = 1.2ux
2.5
Thus E2 = E2t + E2n =− 3uy + uz + 1.2ux
MCQ 1.85 If the electric field intensity is given by E = (xux + yuy + zuz ) volt/m, the potential
difference between X (2, 0, 0) and Y (1, 2, 3) is
(A) + 1 volt (B) − 1 volt
(C) + 5 volt (D) + 6 volt
SOL 1.85 Hence (C) is correct option.
We have E = xux + yuy + zuz
dl = utx dx + uty dy + utz dz
#XE.dl
Y
VXY =−

#1 #2 ydyutz + #3 zdzuzt
2 0 0
= xdxutx +
0
2 2 y2 2 0
=−= x + +z G
2 1 2 2 2 3
=− 1 [22 − 12 + 02 − 22 + 02 − 32] = 5
2

MCQ 1.86 A uniform plane wave traveling in air is incident on the plane boundary between

Brought to you by: Nodia and Company Visit us at: www.nodia.co.in


PUBLISHING FOR GATE
Page 39 GATE EC 2003 www.gatehelp.com

air and another dielectric medium with εr = 4 . The reflection coefficient for the
normal incidence, is
(A) zero (B) 0.5+180c
(B) 0.333+0c (D) 0.333+180c
SOL 1.86 Hence (D) is correct option.
μ
η =
ε
Reflection coefficient
η − η1
τ = 2
η2 + η1
Substituting values for η1 and η2 we have
μ
− με
= 1 − εr = 1 −
o 0

τ = εμε o r o 4 since εr = 4
μ
εε +
o
o r ε
o
o
1 + εr 1+ 4
= − 1 = 0.333+180c
3

MCQ 1.87 If the electric field intensity associated with a uniform plane electromagnetic wave
traveling in a perfect dielectric medium is given by E (z, t) = 10 cos (2π107 t − 0.1πz)
V/m, then the velocity of the traveling wave is
(A) 3.00 # 108 m/sec (B) 2.00 # 108 m/sec
(C) 6.28 # 107 m/sec (D) 2.00 # 107 m/sec
SOL 1.87 Hence (B) is correct option.
We have E (z, t) = 10 cos (2π # 107 t − 0.1πz)
where ω = 2π # 107 t
β = 0.1π
u = ω = 2π # 10 = 2 # 108 m/s
7
Phase Velocity
β 0.1π

MCQ 1.88 A short - circuited stub is shunt connected to a transmission line as shown in
fig. If Z0 = 50 ohm, the admittance Y seen at the junction of the stub and the
transmission line is

Brought to you by: Nodia and Company Visit us at: www.nodia.co.in


PUBLISHING FOR GATE
Page 40 GATE EC 2003 www.gatehelp.com

(A) (0.01 − j0.02) mho (B) (0.02 − j0.01) mho


(C) (0.04 − j0.02) mho (D) (0.02 + j0) mho
SOL 1.88 The fig of transmission line is as shown below .
[Z + jZo tan βl]
We know that Zin = Zo L
[Zo + jZL tan βl]
For line 1, l = λ and β = 2π , ZL1 = 100Ω
2 λ
[Z + jZo tan π]
Thus Zin1 = Zo L = ZL = 100Ω
[Zo + jZL tan π]
For line 2, l = λ and β = 2π , ZL2 = 0 (short circuit)
8 λ
[0 + jZo tan π4 ]
Thus Zin2 = Zo = jZo = j50Ω
[Zo + 0]
Y = 1 + 1 = 1 + 1 = 0.01 − j0.02
Zin1 Zin2 100 j50

Hence (A) is correct option.

Brought to you by: Nodia and Company Visit us at: www.nodia.co.in


PUBLISHING FOR GATE
Page 41 GATE EC 2003 www.gatehelp.com

MCQ 1.89 A rectangular metal wave guide filled with a dielectric material of relative permittivity
εr = 4 has the inside dimensions 3.0 cm # 1.2 cm. The cut-off frequency for the
dominant mode is
(A) 2.5 GHz (B) 5.0 GHz
(C) 10.0 GHz (D) 12.5 GHz
SOL 1.89 Hence (A) is correct option.
8
u = c = 3 # 10 = 1.5 # 108
ε0 2
In rectangular waveguide the dominant mode is TE10 and
fC = v ` m j2 + ` n j2
2 a b
8
= 1.5 # 10 1 2 + 0 2 = 1.5 # 108 = 2.5 GHz
` 0.03 j ` b j
2 0.06

MCQ 1.90 Two identical antennas are placed in the θ = π/2 plane as shown in Fig. The
elements have equal amplitude excitation with 180c polarity difference, operating
at wavelength λ. The correct value of the magnitude of the far-zone resultant
electric field strength normalized with that of a single element, both computed for
φ = 0 , is

(A) 2 cos b 2πs l (B) 2 sin b 2πs l


λ λ

(C) 2 cos a πs k (D) 2 sin a πs k


λ λ

SOL 1.90 Hence (D) is correct option.


ψ
Normalized array factor = 2 cos
2
ψ = βd sin θ cos φ + δ
θ = 90c,
d = 2 s,
φ = 45c,
δ = 180c
ψ βd sin θ cos φ + δ
Now 2 cos = 2 cos ; E
2 2
= 2 cos 8 2π 2 s cos 45c + 180 B
λ. 2 2

Brought to you by: Nodia and Company Visit us at: www.nodia.co.in


PUBLISHING FOR GATE
Page 42 GATE EC 2003 www.gatehelp.com

= 2 cos 8 πs + 90cB = 2 sin ` πs j


λ λ

Answer Sheet
1. (B) 19. (D) 37. (A) 55. (B) 73. (C)
2. (C) 20. (D) 38. (A) 56. (B) 74. (C)
3. (B) 21. (D) 39. (D) 57. (A) 75. (D)
4. (C) 22. (C) 40. (C) 58. (D) 76. (D)
5. (C) 23. (B) 41. (C) 59. (C) 77. (B)
6. (D) 24. (A) 42. (A) 60. (A) 78. (A)
7. (B) 25. (C) 43. (D) 61. (D) 79. (C)
8. (A) 26. (A) 44. (B) 62. (A) 80. (D)
9. (C) 27. (C) 45. (A) 63. (C) 81. (B)
10. (D) 28. (B) 46. (A) 64. (A) 82. (A)
11. (B) 29. (C) 47. (A) 65. (D) 83. (B)
12. (D) 30. (B) 48. (C) 66. (C) 84. (C)
13. (B) 31. (A) 49. (B) 67. (D) 85. (C)
14. (C) 32. (*) 50. (B) 68. (D) 86. (D)
15. (B) 33. (*) 51. (C) 69. (D) 87. (B)
16. (D) 34. (A) 52. (D) 70 (A) 88. (A)
17. (C) 35. (C) 53. (B) 71 (B) 89. (A)
18. (B) 36. (B) 54. (A) 72 (B) 90. (D)

Brought to you by: Nodia and Company Visit us at: www.nodia.co.in


PUBLISHING FOR GATE
GATE EC
2004

Q.1 - 30 Carry One Mark Each

MCQ 1.1 Consider the network graph shown in the figure. Which one of the following is NOT
a ‘tree’ of this graph ?

(A) a (B) b
(C) c (D) d
SOL 1.1 For a tree there must not be any loop. So a, c, and d don’t have any loop. Only b
has loop.
Hence (B) is correct option.
MCQ 1.2 The equivalent inductance measured between the terminals 1 and 2 for the circuit
shown in the figure is

Brought to you by: Nodia and Company Visit us at: www.nodia.co.in


PUBLISHING FOR GATE
Page 2 GATE EC 2004 www.gatehelp.com

(A) L1 + L2 + M (B) L1 + L2 − M
(C) L1 + L2 + 2M (D)L1 + L2 − 2M
SOL 1.2 The sign of M is as per sign of L If current enters or exit the dotted terminals of
both coil. The sign of M is opposite of L If current enters in dotted terminal of a
coil and exit from the dotted terminal of other coil.
Thus Leq = L1 + L2 − 2M
Hence (D) is correct option.
MCQ 1.3 The circuit shown in the figure, with R = 13 Ω, L = 14 H and C = 3 F has input
voltage v (t) = sin 2t . The resulting current i (t) is

(A) 5 sin (2t + 53.1c) (B) 5 sin (2t − 53.1c)


(C) 25 sin (2t + 53.1c) (D) 25 sin (2t − 53.1c)
SOL 1.3 Here ω = 2 and V = 1+0c
Y = 1 + jω C + 1
R jω L
= 3 + j2 # 3 + 1 1 = 3 + j4
j2 # 4
= 5+ tan - 1 4 = 5+53.11c
3
I = V * Y = (1+0c)( 5+53.1c) = 5+53.1c
Thus i (t) = 5 sin (2t + 53.1c)
Hence (A) is correct option.
MCQ 1.4 For the circuit shown in the figure, the time constant RC = 1 ms. The input voltage
is vi (t) = 2 sin 103 t . The output voltage vo (t) is equal to

Brought to you by: Nodia and Company Visit us at: www.nodia.co.in


PUBLISHING FOR GATE
Page 3 GATE EC 2004 www.gatehelp.com

(A) sin (103 t − 45c) (B) sin (103 t + 45c)


(C) sin (103 t − 53c) (D) sin (103 t + 53c)
SOL 1.4 Hence (A) is correct option.
vi (t) = 2 sin 103 t
Here ω = 103 rad and Vi = 2 +0c
1
jω C 1
Now V0 = .Vt = V
R+ 1 1 + j ωCR i
jω C
= 1 2 +0c
1 + j # 103 # 10 - 3
= 1 − 45c
v0 (t) = sin (103 t − 45c)
MCQ 1.5 For the R − L circuit shown in the figure, the input voltage vi (t) = u (t). The
current i (t) is

SOL 1.5 Hence (C) is correct option.


Input voltage vi (t) = u (t)
Taking laplace transform Vi (s) = 1
s
Brought to you by: Nodia and Company Visit us at: www.nodia.co.in
PUBLISHING FOR GATE
Page 4 GATE EC 2004 www.gatehelp.com

Impedance Z (s) = s + 2
V (s) 1
I (s) = i =
s + 2 s (s + 2)
or I (s) = 1 ; 1 − 1 E
2 s s+2
Taking inverse laplace transform
i (t) = 1 (1 − e−2t) u (t)
2
At t = 0 , i (t) = 0
At t = 12 , i (t) = 0.31
At t = 3 , i (t) = 0.5
Graph (C) satisfies all these conditions.
MCQ 1.6 The impurity commonly used for realizing the base region of a silicon n − p − n
transistor is
(A) Gallium (B) Indium
(C) Boron (D) Phosphorus
SOL 1.6 Trivalent impurities are used for making p type semiconductor. Boron is trivalent.
Hence option (C) is correct
MCQ 1.7 If for a silicon npn transistor, the base-to-emitter voltage (VBE ) is 0.7 V and the
collector-to-base voltage (VCB) is 0.2 V, then the transistor is operating in the
(A) normal active mode (B) saturation mode
(C) inverse active mode (D) cutoff mode
SOL 1.7 Here emitter base junction is forward biased and base collector junction is reversed
biased. Thus transistor is operating in normal active region.
Hence option (A) is correct.
MCQ 1.8 Consider the following statements S1 and S2.
S1 : The β of a bipolar transistor reduces if the base width is increased.
S2 : The β of a bipolar transistor increases if the dopoing concentration in the base
is increased.
Which remarks of the following is correct ?
(A) S1 is FALSE and S2 is TRUE
(B) Both S1 and S2 are TRUE
(C) Both S1 and S2 are FALSE
(D) S1 is TRUE and S2 is FALSE
SOL 1.8 Hence option (D) is correct.
We have β = α
1−α

Brought to you by: Nodia and Company Visit us at: www.nodia.co.in


PUBLISHING FOR GATE
GATE IES PSU’S-2021
GATE ACADEMY
Leading inStitUte for eSe/gate/PSU’S

ECE ENGINEERING
NOTED-: These Are Only Sample Notes,
If You Want Complete Full GATE
ACADEMY GATE IES PSU ECE Branch
ALL SUBJECT FULL NOTES BUY Click
below Link Buy

Noted-: Download File Zip format so use zip rar app extract zip
file easily open folder available all subject pdf.
Page 5 GATE EC 2004 www.gatehelp.com

Thus α -" β -
α ." β .
If the base width increases, recombination of carrier in base region increases and
α decreases & hence β decreases. If doping in base region increases, recombination
of carrier in base increases and α decreases thereby decreasing β . Thus S1 is true
and S2 is false.
MCQ 1.9 An ideal op-amp is an ideal
(A) voltage controlled current source (B) voltage controlled voltage source
(C) current controlled current source (D) current controlled voltage source
SOL 1.9 An ideal OPAMP is an ideal voltage controlled voltage source.
Hence (B) is correct option.
MCQ 1.10 Voltage series feedback (also called series-shunt feedback) results in
(A) increase in both input and output impedances
(B) decrease in both input and output impedances
(C) increase in input impedance and decrease in output impedance
(D) decrease in input impedance and increase in output impedance
SOL 1.10 In voltage series feed back amplifier, input impedance increases by factor (1 + Aβ)
and output impedance decreases by the factor (1 + Aβ).
Rif = Ri (1 + Aβ)
Rof = Ro
(1 + Aβ)
Hence (C) is correct option.
MCQ 1.11 The circuit in the figure is a

(A) low-pass filter (B) high-pass filter


(C) band-pass filter (D) band-reject filter
SOL 1.11 This is a Low pass filter, because
At ω = 3 V0 = 0
Vin
and at ω = 0 V0 = 1
Vin
Hence (A) is correct option.

Brought to you by: Nodia and Company Visit us at: www.nodia.co.in


PUBLISHING FOR GATE
Page 6 GATE EC 2004 www.gatehelp.com

MCQ 1.12 Assuming VCEsat = 0.2 V and β = 50 , the minimum base current (IB) required to
drive the transistor in the figure to saturation is

(A) 56 μA (B) 140 mA


(C) 60 mA (D) 3 mA
SOL 1.12 Applying KVL we get
VCC − IC RC − VCE = 0
or IC = VCC − VCE = 3 − 0.2 = 2.8 mA
RC 1k

Now IB = IC = 2.8m = 56 μA
β 50
Hence option (A) is correct.
MCQ 1.13 A master - slave flip flop has the characteristic that
(A) change in the output immediately reflected in the output
(B) change in the output occurs when the state of the master is affected
(C) change in the output occurs when the state of the slave is affected
(D) both the master and the slave states are affected at the same time
SOL 1.13 A master slave D-flip flop is shown in the figure.

In the circuit we can see that output of flip-flop call be triggered only by transition
of clock from 1 to 0 or when state of slave latch is affected.
Hence (C) is correct answer.
MCQ 1.14 The range of signed decimal numbers that can be represented by 6-bits 1’s
complement number is
(A) -31 to +31 (B) -63 to +63
(C) -64 to +63 (D) -32 to +31

Brought to you by: Nodia and Company Visit us at: www.nodia.co.in


PUBLISHING FOR GATE
Page 7 GATE EC 2004 www.gatehelp.com

SOL 1.14 The range of signed decimal numbers that can be represented by n − bits 1’s
complement number is − (2n - 1 − 1) to + (2n - 1 − 1).
Thus for n = 6 we have
Range =− (26 - 1 − 1) to + (26 - 1 − 1)
=− 31 to + 31
Hence (A) is correct answer.
MCQ 1.15 A digital system is required to amplify a binary-encoded audio signal. The user
should be able to control the gain of the amplifier from minimum to a maximum
in 100 increments. The minimum number of bits required to encode, in straight
binary, is
(A) 8 (B) 6
(C) 5 (D) 7
SOL 1.15 The minimum number of bit require to encode 100 increment is
2n $ 100
or n $7
Hence (D) is correct answer.
MCQ 1.16 Choose the correct one from among the alternatives A, B, C, D after matching an
item from Group 1 most appropriate item in Group 2.
Group 1 Group 2
P. Shift register 1. Frequency division
Q. Counter 2. Addressing in memory chips
R. Decoder 3. Serial to parallel data conversion
(A) P − 3, Q − 2, R − 1 (B) P − 3, Q − 1, R − 2
(C) P − 2, Q − 1, R − 3 (D) P − 1, Q − 2, R − 2
SOL 1.16 Shift Register " Serial to parallel data conversion
Counter " Frequency division
Decoder " Addressing in memory chips.
Hence (B) is correct answer.
MCQ 1.17 The figure the internal schematic of a TTL AND-OR-OR-Invert (AOI) gate. For
the inputs shown in the figure, the output Y is

(A) 0 (B) 1
(C) AB (D) AB
SOL 1.17 For the TTL family if terminal is floating, then it is at logic 1.
Thus Y = (AB + 1) = AB .0 = 0

Brought to you by: Nodia and Company Visit us at: www.nodia.co.in


PUBLISHING FOR GATE
Page 8 GATE EC 2004 www.gatehelp.com

Hence (A) is correct answer.


MCQ 1.18 Given figure is the voltage transfer characteristic of

(A) an NOMS inverter with enhancement mode transistor as load


(B) an NMOS inverter with depletion mode transistor as load
(C) a CMOS inverter
(D) a BJT inverter
SOL 1.18 Hence option (C) is correct
MCQ 1.19 The impulse response h [n] of a linear time-invariant system is given by
h [n] = u [n + 3] + u [n − 2) − 2n [n − 7] where u [n] is the unit step sequence. The
above system is
(A) stable but not causal (B) stable and causal
(C) causal but unstable (D) unstable and not causal
3
SOL 1.19 A system is stable if / h (n) < 3. The plot of given h (n) is
n =− 3

3 6
Thus / h (n) = /
n =− 3
h (n)
n =− 3

= 1+1+1+1+2+2+2+2+2
= 15 < 3
Hence system is stable but h (n) ! 0 for n < 0 . Thus it is not causal.
Hence (A) is correct answer.
MCQ 1.20 The distribution function Fx (x) of a random variable x is shown in the figure. The
probability that X = 1 is

Brought to you by: Nodia and Company Visit us at: www.nodia.co.in


PUBLISHING FOR GATE
Page 9 GATE EC 2004 www.gatehelp.com

(A) zero (B) 0.25


(C) 0.55 (D) 0.30
SOL 1.20 Hence (D) is correct option.
F (x1 # X < x2) = p (X = x2) − P (X = x1)
or P (X = 1) = P (X = 1+) − P (X = 1 -)
= 0.55 − 0.25 = 0.30
MCQ 1.21 The z -transform of a system is H (z) = z −z0.2 . If the ROC is z < 0.2 , then the
impulse response of the system is
(A) (0.2) n u [n] (B) (0.2) n u [− n − 1]
(C) − (0.2) n u [n] (D) − (0.2) n u [− n − 1]
SOL 1.21 Hence (D) is correct answer.
H (z) = z z < 0.2
z − 0.2
We know that
− an u [− n − 1] * 1 z <a
1 − az−1
Thus h [n] =− (0.2) n u [− n − 1]
MCQ 1.22 The Fourier transform of a conjugate symmetric function is always
(A) imaginary (B) conjugate anti-symmetric
(C) real (D) conjugate symmetric
SOL 1.22 The Fourier transform of a conjugate symmetrical function is always real.
Hence (C) is correct answer.
MCQ 1.23 The gain margin for the system with open-loop transfer function
2 (1 + s)
G (s) H (s) = , is
s2
(A) 3 (B) 0
(C) 1 (D) − 3
SOL 1.23 The open loop transfer function is
2 (1 + s)
G (s) H (s) =
s2
Substituting s = jω we have
2 (1 + jω)
G (jω) H (jω) = ...(1)
− ω2

Brought to you by: Nodia and Company Visit us at: www.nodia.co.in


PUBLISHING FOR GATE
Page 10 GATE EC 2004 www.gatehelp.com

+G (jω) H (jω) =− 180c + tan−1 ω


The frequency at which phase becomes − 180c, is called phase crossover frequency.
Thus − 180 =− 180c + tan−1 ωφ
or tan−1 ωφ = 0
or ωφ = 0
The gain at ωφ = 0 is
G (jω) H (jω) = 2 1 + ω2 = 3
ω2
Thus gain margin is = 1 = 0 and in dB this is − 3 .
3
Hence (D) is correct option

MCQ 1.24 Given G (s) H (s) = K .The point of intersection of the asymptotes of
s (s + 1)( s + 3)
the root loci with the real axis is
(A) − 4 (B) 1.33
(C) − 1.33 (D) 4
SOL 1.24 Centroid is the point where all asymptotes intersects.
ΣReal of Open Loop Pole − ΣReal Part of Open Loop Pole
σ =
ΣNo.of Open Loop Pole − ΣNo.of Open Loop zero
= − 1 − 3 =− 1.33
3
Hence (C) is correct option.
MCQ 1.25 In a PCM system, if the code word length is increased from 6 to 8 bits, the signal
to quantization noise ratio improves by the factor
(A) 8 (B) 12
6
(C) 16 (D) 8
SOL 1.25 When word length is 6
` N jN = 6 = 2 = 2
S 2#6 12

When word length is 8


` N jN = 8 = 2 = 2
S 2#8 16

^ N hN = 8
S 16
Now = 212 = 2 4 = 16
^ N hN = 6
S
2
Thus it improves by a factor of 16.
Hence (C) is correct option.
MCQ 1.26 An AM signal is detected using an envelop detector. The carrier frequency and
modulating signal frequency are 1 MHz and 2 kHz respectively. An appropriate
value for the time constant of the envelop detector is
(A) 500μ sec (B) 20μ sec

Brought to you by: Nodia and Company Visit us at: www.nodia.co.in


PUBLISHING FOR GATE
Page 11 GATE EC 2004 www.gatehelp.com

(C) 0.2μ sec (D) 1μ sec


SOL 1.26 Hence (B) is correct option.
Carrier frequency fc = 1 # 106 Hz
Modulating frequency
fm = 2 # 103 Hz
For an envelope detector
2πfc > 1 > 2πfm
Rc
1 < RC < 1
2πfc 2πfm
1 < RC < 1
2πfc 2πfm
1 < RC < 1
2π10 6
2 # 103
1.59 # 10 - 7 < RC < 7.96 # 10 - 5
so, 20 μsec sec best lies in this interval.
MCQ 1.27 An AM signal and a narrow-band FM signal with identical carriers, modulating
signals and modulation indices of 0.1 are added together. The resultant signal can
be closely approximated by
(A) broadband FM (B) SSB with carrier
(C) DSB-SC (D) SSB without carrier
SOL 1.27 Hence (B) is correct option.
SAM (t) = Ac [1 + 0.1 cos ωm t] cos ωm t
sNBFM (t) = Ac cos [ωc t + 0.1 sin ωm t]
s (t) = SAM (t) + SNB fm (t)
= Ac [1 + 0.1 cos ωm t] cos ωc t + Ac cos (ωc t + 0.1 sin ωm t)
= Ac cos ωc t + Ac 0.1 cos ωm t cos ωc t
+ Ac cos ωc t cos (0.1 sin ωm t) − Ac sin ωc t. sin (0.1 sin ωm t)
As 0.1 sin ωm t ,+ 0.1 to − 0.1
so cos (0.1 sin ωm t) . 1
As when θ is small cos θ . 1 and sin θ , θ, thus
sin (0.1 sin ωm t) = 0.1 sincos ωc t cos ωm t + Ac cos ωc t − Ac 0.1 sin ωm t sin ωc t
= 2Ac cos ωc t + 0.1Ac cos (ωc + ωm) t
1 44 2 44 3 1 4444 4 2 4444 43
cosec USB
Thus it is SSB with carrier.
MCQ 1.28 In the output of a DM speech encoder, the consecutive pulses are of opposite
polarity during time interval t1 # t # t2 . This indicates that during this interval
(A) the input to the modulator is essentially constant
(B) the modulator is going through slope overload

Brought to you by: Nodia and Company Visit us at: www.nodia.co.in


PUBLISHING FOR GATE
Page 12 GATE EC 2004 www.gatehelp.com

(C) the accumulator is in saturation


(D) the speech signal is being sampled at the Nyquist rate
SOL 1.28 Consecutive pulses are of same polarity when modulator is in slope overload.
Consecutive pulses are of opposite polarity when the input is constant.
Hence (A) is correct option.
MCQ 1.29 The phase velocity of an electromagnetic wave propagating in a hollow metallic
rectangular waveguide in the TE10 mode is
(A) equal to its group velocity
(B) less than the velocity of light in free space
(C) equal to the velocity of light in free space
(D) greater than the velocity of light in free space
SOL 1.29 We know that vp > c > vg .
Hence (D) is correct option.
MCQ 1.30 Consider a lossless antenna with a directive gain of + 6 dB. If 1 mW of power is fed
to it the total power radiated by the antenna will be
(A) 4 mW (B) 1 mW
(C) 7 mW (D) 1/4 mW
SOL 1.30 Hence (A) is correct option.
4πU (θ, φ)
We have GD (θ, φ) =
Prad
For lossless antenna
Prad = Pin
Here we have Prad = Pin = 1 mW
and 10 log GD (θ, φ) = 6 dB
or GD (θ, φ) = 3.98
Thus the total power radiated by antenna is
4πU (θ, φ) = Prad GD (θ, φ) = 1 m # 3.98 = 3.98 mW

Q.31 - 90 Carry Two Marks Each

MCQ 1.31 For the lattice shown in the figure, Za = j2 Ω and Zb = 2 Ω . The values of the open
z11 z12
circuit impedance parameters 6 z @ = =
z21 z22 G
are

Brought to you by: Nodia and Company Visit us at: www.nodia.co.in


PUBLISHING FOR GATE
Page 13 GATE EC 2004 www.gatehelp.com

1−j 1+j 1−j 1+j


(A) =
1 + j 1 + jG
(B) =
−1 + j 1 − j G
1+j 1+j 1 + j −1 + j
(C) =
1 − j 1 − jG
(D) =
−1 + j 1 + j G
SOL 1.31 We know that
V1 = z11 I1 + z12 I2
V2 = z11 I1 + z22 I2
where z11 = V1
I1 I = 0
2

z21 = V2
I1 I = 0
1

Consider the given lattice network, when I2 = 0 . There is two similar path in the
circuit for the current I1. So I = 1 I1
2

For z11 applying KVL at input port we get


V1 = I (Za + Zb)
Thus V1 = 1 I1 (Za + Zb)
2
z11 = 1 (Za + Zb)
2
For Z21 applying KVL at output port we get
V2 = Za I1 − Zb I1
2 2
Thus V2 = 1 I1 (Za − Zb)
2
z21 = 1 (Za − Zb)
2
For this circuit z11 = z22 and z12 = z21. Thus
R V
S Za + Zb Za − Zb W
z11 z12
=z z G = SS Za − Zb Za + Zb WW
2 2
21 22
S 2 2 W
T X
Here Za = 2j and Zb = 2Ω
z11 z12 1+j j−1
Thus = G ==
z21 z22 j − 1 1 + jG
Hence (D) is correct option.

Brought to you by: Nodia and Company Visit us at: www.nodia.co.in


PUBLISHING FOR GATE
Page 14 GATE EC 2004 www.gatehelp.com

MCQ 1.32 The circuit shown in the figure has initial current iL (0−) = 1 A through the inductor
and an initial voltage vC (0−) =− 1 V across the capacitor. For input v (t) = u (t),
the Laplace transform of the current i (t) for t $ 0 is

(A) s (B) s+2


s +s+1
2
s +s+1 2

(C) 2 s − 2 (D) 2 1
s +s+1 s +s+1
SOL 1.32 Applying KVL,
Ldi (t) 1
#0
3
v (t) = Ri (t) + + i (t) dt
dt C
Taking L.T. on both sides,
I (s) vc (0+)
V (s) = RI (s) + LsI (s) − Li (0+) + +
sC sC
v (t) = u (t) thus V (s) = 1
s
Hence 1 = I (s) + sI (s) − 1 + I (s) − 1
s s s
2 + 1 = I (s) 6s2 + s + 1@
s s
or I (s) = 2 s + 2
s +s+1
Hence (B) is correct option.
MCQ 1.33 Consider the Bode magnitude plot shown in the fig. The transfer function H (s) is

(s + 10) 10 (s + 1)
(A) (B)
(s + 1)( s + 100) (s + 10)( s + 100)
102 (s + 1) 103 (s + 100)
(C) (D)
(s + 10)( s + 100) (s + 1)( s + 10)

Brought to you by: Nodia and Company Visit us at: www.nodia.co.in


PUBLISHING FOR GATE
Page 15 GATE EC 2004 www.gatehelp.com

SOL 1.33 The given bode plot is shown below

At ω = 1 change in slope is +20 dB " 1 zero at ω = 1


At ω = 10 change in slope is − 20 dB " 1 poles at ω = 10
At ω = 100 change in slope is − 20 dB " 1 poles at ω = 100
K (s + 1)
Thus T (s) = s
( 10 + 1)( 100
s
+ 1)
Now 20 log10 K =− 20 " K = 0.1
0.1 (s + 1) 100 (s + 1)
Thus T (s) = s =
( 10 + 1)( 100 + 1) (s + 10)( s + 100)
s

Hence (C) is correct option.


Vo (s)
MCQ 1.34 The transfer function H (s) = of an RLC circuit is given by
Vi (s)
H (s) = 106
s + 20s + 106
2

The Quality factor (Q-factor) of this circuit is


(A) 25 (B) 50
(C) 100 (D) 5000
SOL 1.34 Characteristics equation is
s2 + 20s + 106 = 0
Comparing with s2 + 2ξωn s + ωn2 = 0 we have
ωn = 106 = 103
2ξω = 20
Thus 2ξ = 203 = 0.02
10
Now Q = 1 = 1 = 50
2ξ 0.02
Hence (B) is correct option.
MCQ 1.35 For the circuit shown in the figure, the initial conditions are zero. Its transfer
V (s)
function H (s) = c is
Vi (s)

Brought to you by: Nodia and Company Visit us at: www.nodia.co.in


PUBLISHING FOR GATE
Page 16 GATE EC 2004 www.gatehelp.com

(A) 1 (B) 106


s + 106 s + 106
2
s2 + 103 s + 106

(C) 103 (D) 106


s2 + 103 s + 106 s2 + 106 s + 106

SOL 1.35 Hence (D) is correct option.


V (s)
H (s) = 0
Vi (s)
1
= sC = 2 1
R + sL + 1 s LC + sCR + 1
sC
= 2 −2 1
s (10 # 10 ) + s (10−4 # 10 4) + 1
−4

= −6 2 1 = 2 106
10 s + s + 1 s + 106 s + 106

d 2y dy
MCQ 1.36 A system described by the following differential equation 2 + 3 + 2y = x (t) is
dt dt
initially at rest. For input x (t) = 2u (t), the output y (t) is
(A) (1 − 2e−t + e−2t) u (t) (B) (1 + 2e−t − 2e−2t) u (t)
(C) (0.5 + e−t + 1.5e−2t) u (t) (D) (0.5 + 2e−t + 2e−2t) u (t)
SOL 1.36 Hence Correct Option is (A)

2 + 3 dt + 2y = x ^ t h
d2y dy
Given,
dt
Taking Laplace Transformation both sides, we have
6s + 3s + 2@Y ^s h = X ^s h = s
2 2

Y ^s h = 2 =1− 2 + 1
s ^s + 1h^s + 2h s s + 1 s + 2
or

Increasing Laplace transformation gives,


y ^ t h = ^1 − 2e−t + e−2t h u ^ t h
MCQ 1.37 Consider the following statements S1 and S2
S1 : At the resonant frequency the impedance of a series RLC circuit is zero.
S2 : In a parallel GLC circuit, increasing the conductance G results in increase in
its Q factor.

Brought to you by: Nodia and Company Visit us at: www.nodia.co.in


PUBLISHING FOR GATE
Page 17 GATE EC 2004 www.gatehelp.com

Which one of the following is correct?


(A) S1 is FALSE and S2 is TRUE
(B) Both S1 and S2 are TRUE
(C) S1 is TRUE and S2 is FALSE
(D) Both S1 and S2 are FALSE
SOL 1.37 Impedance of series RLC circuit at resonant frequency is minimum, not zero.
Actually imaginary part is zero.
Z = R + j ` ωL − 1 j
ωC
At resonance ωL − 1 = 0 and Z = R that is purely resistive. Thus S1 is false
ωC
Now quality factor Q =R C
L
Since G = 1 , Q = 1 C
R G L
If G - then Q . provided C and L are constant. Thus S2 is also false.
Hence (D) is correct option.
MCQ 1.38 In an abrupt p − n junction, the doping concentrations on the p −side and n -side
are NA = 9 # 1016 /cm 3 respectively. The p − n junction is reverse biased and the
total depletion width is 3 μm. The depletion width on the p −side is
(A) 2.7 μm (B) 0.3 μm
(C) 2.25 μm (D) 0.75 μm
SOL 1.38 We know that
Wp NA = Wn ND
3 μ # 1016
or Wp = Wn # ND = = 0.3 μ m
NA 9 # 1016
Hence option (B) is correct.
MCQ 1.39 The resistivity of a uniformly doped n −type silicon sample is 0.5Ω - mc. If the
electron mobility (μn) is 1250 cm 2 /V-sec and the charge of an electron is 1.6 # 10 - 19
Coulomb, the donor impurity concentration (ND) in the sample is
(A) 2 # 1016 /cm 3 (B) 1 # 1016 /cm 3
(C) 2.5 # 1015 /cm 3 (D) 5 # 1015 /cm 3
SOL 1.39 Hence option (B) is correct.
Conductivity σ = nqun
or resistivity ρ = 1 = 1
σ nqμn
Thus n = 1 = 1 = 1016 /cm 3
qρμn - 19
1.6 # 10 # 0.5 # 1250
For n type semiconductor n = ND

Brought to you by: Nodia and Company Visit us at: www.nodia.co.in


PUBLISHING FOR GATE
Page 18 GATE EC 2004 www.gatehelp.com

MCQ 1.40 Consider an abrupt p − n junction. Let Vbi be the built-in potential of this junction
and VR be the applied reverse bias. If the junction capacitance (Cj ) is 1 pF for
Vbi + VR = 1 V, then for Vbi + VR = 4 V, Cj will be
(A) 4 pF (B) 2 pF
(C) 0.25 pF (D) 0.5 pF
SOL 1.40 We know that
eεS NA ND
1

Cj = ;
2 (Vbi + VR)( NA + ND) E
2

Thus Cj \ 1
(Vbi + VR)
C j2 (Vbi + VR) 1 1 =1
Now = =
C j1 (Vbi + VR) 2 4 2
Cj1
or Cj2 = = 1 = 0.5 pF
2 2
Hence option (D) is correct.
MCQ 1.41 Consider the following statements Sq and S2.
S1 : The threshold voltage (VT ) of MOS capacitor decreases with increase in gate
oxide thickness.
S2 : The threshold voltage (VT ) of a MOS capacitor decreases with increase in
substrate doping concentration.
Which Marks of the following is correct ?
(A) S1 is FALSE and S2 is TRUE
(B) Both S1 and S2 are TRUE
(C) Both S1 and S2 are FALSE
(D) S1 is TRUE and S2 is FALSE
SOL 1.41 Increase in gate oxide thickness makes difficult to induce charges in channel. Thus
VT increases if we increases gate oxide thickness. Hence S1 is false.
Increase in substrate doping concentration require more gate voltage because
initially induce charges will get combine in substrate. Thus VT increases if we
increase substrate doping concentration. Hence S2 is false.
Hence option (C) is correct.
MCQ 1.42 The drain of an n-channel MOSFET is shorted to the gate so that VGS = VDS . The
threshold voltage (VT ) of the MOSFET is 1 V. If the drain current (ID) is 1 mA for
VGS = 2 V, then for VGS = 3 V, ID is
(A) 2 mA (B) 3 mA
(C) 9 mA (D) 4 mA

Brought to you by: Nodia and Company Visit us at: www.nodia.co.in


PUBLISHING FOR GATE
Page 19 GATE EC 2004 www.gatehelp.com

SOL 1.42 We know that


ID = K (VGS − VT ) 2

IDS = (VGS2 − VT )
2
Thus
IDI (VGS1 − VT ) 2
Substituting the values we have
ID2 = (3 − 1) = 4
2

ID1 (2 − 1) 2
or ID2 = 4IDI = 4 mA
Hence option (D) is correct.
MCQ 1.43 The longest wavelength that can be absorbed by silicon, which has the bandgap
of 1.12 eV, is 1.1 μm. If the longest wavelength that can be absorbed by another
material is 0.87 μm, then bandgap of this material is
(A) 1.416 A/cm 2 (B) 0.886 eV
(C) 0.854 eV (D) 0.706 eV
SOL 1.43 Hence option (A) is correct.
Eg \ 1
λ
= λ1 = 1.1
Eg2
Thus
Eg1 λ2 0.87
or Eg2 = 1.1 # 1.12 = 1.416 eV
0.87

MCQ 1.44 The neutral base width of a bipolar transistor, biased in the active region, is 0.5 μ
m. The maximum electron concentration and the diffusion constant in the base are
1014 / cm 3 and Dn = 25 cm 2 /sec respectively. Assuming negligible recombination in
the base, the collector current density is (the electron charge is 1.6 # 10 - 19 Coulomb)
(A) 800 A/cm 2 (B) 8 A/cm 2
(C) 200 A/cm 2 (D) 2 A/cm 2
SOL 1.44 Concentration gradient
dn = 1014 = 2 # 1018
dx -4
0.5 # 10
q = 1.6 # 10 - 19 C
Dn = 25
dn = 1014
dx 0.5 # 10 - 4
JC = qDn dn
dx
= 1.6 # 10 - 19 # 25 # 2 # 1018 = 8 A/cm 2

Hence option (B) is correct.

Brought to you by: Nodia and Company Visit us at: www.nodia.co.in


PUBLISHING FOR GATE
Page 20 GATE EC 2004 www.gatehelp.com

MCQ 1.45 Assume that the β of transistor is extremely large and VBE = 0.7V, IC and VCE in
the circuit shown in the figure

(A) IC = 1 mA, VCE = 4.7 V (B) IC = 0.5 mA, VCE = 3.75 V


(C) IC = 1 mA, VCE = 2.5 V (D) IC = 0.5 mA, VCE = 3.9 V
SOL 1.45 The thevenin equivalent is shown below

VT = R1 V = 1
#5 = 1 V
R1 + R2 C 4+1
Since β is large is large, IC . IE , IB . 0 and
IE = VT − VBE = 1 − 0.7 = 3 mA
RE 300
Now VCE = 5 − 2.2kIC − 300IE
= 5 − 2.2k # 1m − 300 # 1m
= 2.5 V
Hence (C) is correct option
MCQ 1.46 A bipolar transistor is operating in the active region with a collector current of 1
mA. Assuming that the β of the transistor is 100 and the thermal voltage (VT ) is
25 mV, the transconductance (gm) and the input resistance (rπ) of the transistor in
the common emitter configuration, are
(A) gm = 25 mA/V and rπ = 15.625 kΩ
(B) gm = 40 mA/V and rπ = 4.0 kΩ
(C) gm = 25 mA/V and rπ = 2.5 k Ω
(D) gm = 40 mA/V and rπ = 2.5 kΩ

Brought to you by: Nodia and Company Visit us at: www.nodia.co.in


PUBLISHING FOR GATE
Page 21 GATE EC 2004 www.gatehelp.com

SOL 1.46 When IC >> ICO


IC
gm = = 1mA = 0.04 = 40 mA/V
VT 25mV
β
rπ = = 100 - 3 = 2.5 kΩ
gm 40 # 10
Hence (D) is correct option.
MCQ 1.47 The value of C required for sinusoidal oscillations of frequency 1 kHz in the circuit
of the figure is

(A) 1 μF (B) 2π μF

(C) 1 μF (D) 2π 6 μF]


2π 6
SOL 1.47 The given circuit is wein bridge oscillator. The frequency of oscillation is
2πf = 1
RC
or C = 1 = 1 = 1 μ
2πRf 2π # 10 # 10
3 3 2π
Hence (A) is correct option.
MCQ 1.48 In the op-amp circuit given in the figure, the load current iL is

(A) − Vs (B) Vs
R2 R2

Brought to you by: Nodia and Company Visit us at: www.nodia.co.in


PUBLISHING FOR GATE
Page 22 GATE EC 2004 www.gatehelp.com

(C) − Vs (D) Vs
RL R1
SOL 1.48 The circuit is as shown below

We know that for ideal OPAMP


V- = V+
Applying KCL at inverting terminal
V- − Vs + V- − V0 = 0
R1 R1
or 2V- − Vo = Vs ...(1)
Applying KCL at non-inverting terminal
V+ V − Vo
+ IL + + =0
R2 R2
or 2V+ − Vo + IL R2 = 0 ...(2)
Since V- = V+ , from (1) and (2) we have
Vs + IL R2 = 0
or IL =− Vs
R2
Hence (A) is correct option.
MCQ 1.49 In the voltage regulator shown in the figure, the load current can vary from 100
mA to 500 mA. Assuming that the Zener diode is ideal (i.e., the Zener knee current
is negligibly small and Zener resistance is zero in the breakdown region), the value
of R is

(A) 7 Ω (B) 70 Ω
(C) 70 Ω (D) 14 Ω
3
SOL 1.49 If IZ is negligible the load current is

Brought to you by: Nodia and Company Visit us at: www.nodia.co.in


PUBLISHING FOR GATE
Page 23 GATE EC 2004 www.gatehelp.com

12 − Vz = I
L
R
as per given condition
100 mA # 12 − VZ # 500 mA
R
At IL = 100 mA 12 − 5 = 100 mA VZ = 5 V
R
or R = 70Ω
At IL = 500 mA 12 − 5 = 500 mA VZ = 5 V
R
or R = 14 Ω
Thus taking minimum we get
R = 14 Ω
Hence (D) is correct option.
MCQ 1.50 In a full-wave rectifier using two ideal diodes, Vdc and Vm are the dc and peak values
of the voltage respectively across a resistive load. If PIV is the peak inverse voltage
of the diode, then the appropriate relationships for this rectifier are
(A) Vdc = Vm , PIV = 2Vm (B) Idc = 2 Vm , PIV = 2Vm
π π

(C) Vdc = 2 Vm , PIV = Vm (D) Vdc Vm , PIV = Vm


π π
SOL 1.50 Hence (B) is correct option.
MCQ 1.51 The minimum number of 2- to -1 multiplexers required to realize a 4- to -1
multiplexers is
(A) 1 (B) 2
(C) 3 (D) 4

SOL 1.51 Number of MUX is 4 = 2 and 2 = 1. Thus the total number 3 multiplexers is
3 2
required.
Hence (C) is correct answer.
MCQ 1.52 The Boolean expression AC + BC is equivalent to
(A) AC + BC + AC (B) BC + AC + BC + ACB
(C) AC + BC + BC + ABC (D) ABC + ABC + ABC + ABC
SOL 1.52 Hence (D) is correct answer.
AC + BC = AC1 + BC 1
= AC (B + B ) + BC (A + A)
= ACB + ACB + BC A + BC A
MCQ 1.53 11001, 1001, 111001 correspond to the 2’s complement representation of which one
of the following sets of number
(A) 25,9, and 57 respectively (B) -6, -6, and -6 respectively
Brought to you by: Nodia and Company Visit us at: www.nodia.co.in
PUBLISHING FOR GATE
Page 24 GATE EC 2004 www.gatehelp.com

(C) -7, -7 and -7 respectively (D) -25, -9 and -57 respectively


SOL 1.53 Hence (C) is correct answer.
11001 1001 111001
00110 0110 000110
+1 +1 +1
00111 0111 000111
7 7 7
Thus 2’s complement of 11001, 1001 and 111001 is 7. So the number given in the
question are 2’s complement correspond to -7.

MCQ 1.54 The 8255 Programmable Peripheral Interface is used as described below.
(i) An A/D converter is interface to a microprocessor through an 8255.
The conversion is initiated by a signal from the 8255 on Port C. A signal on Port
C causes data to be stobed into Port A.
(ii) Two computers exchange data using a pair of 8255s. Port A works as a
bidirectional data port supported by appropriate handshaking signals.
The appropriate modes of operation of the 8255 for (i) and (ii) would be
(A) Mode 0 for (i) and Mode 1 for (ii)
(B) Mode 1 for (i) and Mode 2 for (ii)
(C) Mode for (i) and Mode 0 for (ii)
(D) Mode 2 for (i) and Mode 1 for (ii)
SOL 1.54 For 8255, various modes are described as following.
Mode 1 : Input or output with hand shake
In this mode following actions are executed
1. Two port (A & B) function as 8 - bit input output ports.
2. Each port uses three lines from C as a hand shake signal
3. Input & output data are latched.
Form (ii) the mode is 1.
Mode 2 : Bi-directional data transfer
This mode is used to transfer data between two computer. In this mode port A can
be configured as bidirectional port. Port A uses five signal from port C as hand
shake signal.
For (1), mode is 2
Hence (D) is correct answer.
MCQ 1.55 The number of memory cycles required to execute the following 8085 instructions
(i) LDA 3000 H
(ii) LXI D, FOF1H
would be

Brought to you by: Nodia and Company Visit us at: www.nodia.co.in


PUBLISHING FOR GATE
Page 25 GATE EC 2004 www.gatehelp.com

(A) 2 for (i) and 2 for (ii) (B) 4 for (i) and 3 for (ii)
(C) 3 for (i) and 3 for (ii) (D) 3 for (i) and 4 for (ii)
SOL 1.55 LDA 16 bit & Load accumulator directly this instruction copies data byte from
memory location (specified within the instruction) the accumulator.
It takes 4 memory cycle-as following.
1. in instruction fetch
2. in reading 16 bit address
1. in copying data from memory to accumulator
LXI D, (F0F1) 4 & It copies 16 bit data into register pair D and E.
It takes 3 memory cycles.
Hence (B) is correct answer.
MCQ 1.56 In the modulo-6 ripple counter shown in figure, the output of the 2- input gate is
used to clear the J-K flip-flop
The 2-input gate is

(A) a NAND gate (B) a NOR gate


(C) an OR gate (D) a AND gare
SOL 1.56 In the modulo - 6 ripple counter at the end of sixth pulse (i.e. after 101 or at 110)
all states must be cleared. Thus when CB is 11 the all states must be cleared. The
input to 2-input gate is C and B and the desired output should be low since the
CLEAR is active low
Thus when C and B are 0, 0, then output must be 0. In all other case the output
must be 1. OR gate can implement this functions.
Hence (C) is correct answer.
MCQ 1.57 Consider the sequence of 8085 instructions given below
LXI H, 9258
MOV A, M
CMA
MOV M, A
Which one of the following is performed by this sequence ?
(A) Contents of location 9258 are moved to the accumulator

Brought to you by: Nodia and Company Visit us at: www.nodia.co.in


PUBLISHING FOR GATE
Page 26 GATE EC 2004 www.gatehelp.com

(B) Contents of location 9258 are compared with the contents of the accumulator
(C) Contents of location 8529 are complemented and stored in location 8529
(D) Contents of location 5892 are complemented and stored in location 5892
SOL 1.57 Hence (A) is correct answer.
LXI H, 9258H ; 9258H " HL
MOV A, M ; (9258H) " A
CMa ; A"A
MOV M, A ; A"M
This program complement the data of memory location 9258H.
MCQ 1.58 A Boolean function f of two variables x and y is defined as follows :
f (0, 0) = f (0, 1) = f (1, 1) = 1; f (1, 0) = 0
Assuming complements of x and y are not available, a minimum cost solution for
realizing f using only 2-input NOR gates and 2- input OR gates (each having unit
cost) would have a total cost of
(A) 1 unit (B) 4 unit
(C) 3 unit (D) 2 unit
SOL 1.58 Hence (D) is correct answer.
We have f (x, y) = xy + xy + xy = x (y + y) + xy = x + xy
or f (x, y) = x + y
Here compliments are not available, so to get x we use NOR gate. Thus desired
circuit require 1 unit OR and 1 unit NOR gate giving total cost 2 unit.
MCQ 1.59 It is desired to multiply the numbers 0AH by 0BH and store the result in the
accumulator. The numbers are available in registers B and C respectively. A part
of the 8085 program for this purpose is given below :
MVI A, 00H
LOOP ------
------
-----
HLT
END
The sequence of instructions to complete the program would be
(A) JNX LOOP, ADD B, DCR C
(B) ADD B, JNZ LOOP, DCR C
(C) DCR C, JNZ LOOP, ADD B
(D) ADD B, DCR C, JNZ LOOP
SOL 1.59 Hence (D) is correct answer.
MVI A, 00H ; Clear accumulator
LOOP ADD B ; Add the contents of B to A

Brought to you by: Nodia and Company Visit us at: www.nodia.co.in


PUBLISHING FOR GATE
Page 27 GATE EC 2004 www.gatehelp.com

DCR C ; Decrement C
JNZ LOOP ; If C is not zero jump to loop
HLT
END
This instruction set add the contents of B to accumulator to contents of C times.
Hence (D) is correct answer.
MCQ 1.60 A 1 kHz sinusoidal signal is ideally sampled at 1500 samples/sec and the sampled
signal is passed through an ideal low-pass filter with cut-off frequency 800 Hz. The
output signal has the frequency.
(A) zero Hz (B) 0.75 kHz
(C) 0.5 kHz (D) 0.25 kHz
SOL 1.60 Hence Correct Option is (C)
Here fs = 1500 samples/sec, fm = kHz
The sampled frequency are 2.5 kHz, 0.5 kHz, Since LPF has cut-off frequency
800 Hz, then only output signal of frequency 0.5 kHz would pass through it
MCQ 1.61 A rectangular pulse train s (t) as shown in the figure is convolved with the signal
cos2 (4p # 103 t). The convolved signal will be a

(A) DC (B) 12 kHz sinusoid


(C) 8 kHz sinusoid (D) 14 kHz sinusoid
SOL 1.61 Hence Correct Option is (D)
S ^ t h = 1 61 + 2 cos ωs t + 2 cos 2ωs t + .....................@
Ts
^1 + cos 8π # 103 t h
cos 4π # 10 t =
2 3
2
ωs = 2π = 2π # 10 # 103
0.1 # 10−3
3

S^t h * x^t h = # S^τ h # ^τ − t hdτ


−3
3

= # 10 # 10 61 + 2 cos ω t + 2 cos 2ω t + ........@dt


3
s s
−3

61 + cos 8π # 103 t@
# 2
Brought to you by: Nodia and Company Visit us at: www.nodia.co.in
PUBLISHING FOR GATE
Page 28 GATE EC 2004 www.gatehelp.com

So, frequencies present will be fs ! fm, 2fs ! 3fs ! fm; fs = 10 kHz

fm = 8π # 10 = 4 kHz
3


Hence 14 kHz sinusoidal signal will be present
MCQ 1.62 Consider the sequence x [n] = [− 4 − j51 + j25]. The conjugate anti-symmetric part
-
of the sequence is
(A) [− 4 − j2.5, j2, 4 − j2.5] (B) [− j2.5, 1, j2.5]
(C) [− j2.5, j2, 0] (D) [− 4, 1, 4]
SOL 1.62 Hence (A) is correct answer.
We have x (n) = [− 4 − j5, 1 + 2j, 4]
-

x *( n) = [− 4 + j5, 1 − 2j, 4]
-

x *( − n) = [4, 1 − 2j, − 4 + j5]


-

x (n) − x* (− n)
xcas (n) =
2
= [− 4 − j 25 , 2j 4 − j 25 ]
-

MCQ 1.63 A causal LTI system is described by the difference equation


2y [n] = αy [n − 2] − 2x [n] + βx [n − 1]
The system is stable only if
(A) α = 2 , β < 2 (B) α > 2, β > 2
(C) α < 2 , any value of β (D) β < 2 , any value of α
SOL 1.63 Hence (C) is correct answer.
We have 2y (n) = αy (n − 2) − 2x (n) + βx (n − 1)
Taking z transform we get
2Y (z) = αY (z) z−2 − 2X (z) + βX (z) z−1
βz−1 − 2
=c m
Y (z)
or ...(i)
X (z) 2 − αz−2
z ( β2 − z)
or H (z) = 2 α
(z − 2 )
It has poles at ! α/2 and zero at 0 and β/2 . For a stable system poles must lie
inside the unit circle of z plane. Thus
α <1
2
or α <2
But zero can lie anywhere in plane. Thus, β can be of any value.

Brought to you by: Nodia and Company Visit us at: www.nodia.co.in


PUBLISHING FOR GATE
Page 29 GATE EC 2004 www.gatehelp.com

MCQ 1.64 A causal system having the transfer function H (s) = 1/ (s + 2) is excited with
10u (t). The time at which the output reaches 99% of its steady state value is
(A) 2.7 sec (B) 2.5 sec
(C) 2.3 sec (D) 2.1 sec
SOL 1.64 Hence (C) is correct option.
We have r (t) = 10u (t)
or R (s) = 10
s
Now H (s) = 1
s+2
C (s) = H (s) $ R (s) = 1 $ 10 10
s + 2 s s (s + 2)
or C (s) = 5 − 5
s s+2
c (t) = 5 [1 − e−2t]
The steady state value of c (t) is 5. It will reach 99% of steady state value reaches
at t , where
5 [1 − e−2t] = 0.99 # 5
or 1 − e−2t = 0.99
e−2t = 0.1
or − 2t = ln 0.1
or t = 2.3 sec
MCQ 1.65 The impulse response h [n] of a linear time invariant system is given as
− 2 2 n = 1, − 1
h [ n] = * 4 2 n = 2, − 2
0 otherwise
If the input to the above system is the sequence e jπn/4 , then the output is
(A) 4 2 e jπn/4 (B) 4 2 e−jπn/4
(C) 4e jπn/4 (D) − 4e jπn/4
SOL 1.65 Hence (D) is correct answer.
We have x (n) = e jπn/4
and h (n) = 4 2 δ (n + 2) − 2 2 δ (n + 1) − 2 2 δ (n − 1)
+ 4 2 δ (n − 2)
Now y (n) = x (n)* h (n)
3 2
= / x (n − k) h (k) = / x (n − k) h (k)
k =− 3 k =− 2

or y (n) = x (n + 2) h (− 2) + x (n + 1) h (− 1)
+ x (n − 1) h (1) + x (n − 2) h (2)
π π π π
j (n + 2) j (n + 1) j (n − 1)
= 4 2e 4
−2 2e 4
−2 2e 4
+ 4 2 e j (n − 2)
4

Brought to you by: Nodia and Company Visit us at: www.nodia.co.in


PUBLISHING FOR GATE
Page 30 GATE EC 2004 www.gatehelp.com

= 4 2 6e j (n + 2) + e j (n − 2)@ − 2 2 6e j (n + 1) + e j @
π π π π
4 4 4 4
(n − 1)

= 4 2 e j n 6e j + e−j @ − 2 2 e j n 6e j + e−j @
π π π π π π
4 2 2 2 4 4

π π
= 4 2 e j n [0] − 2 2 e j n [2 cos π4 ]
4 4

y (n) =− 4e j n
r

or 4

MCQ 1.66 Let x (t) and y (t) with Fourier transforms F (f) and Y (f) respectively be related as
shown in Fig. Then Y (f) is

(A) − 1 X (f/2) e−jπf (B) − 1 X (f/2) e j2πf


2 2
(C) − X (f/2) e j2πf (D) − X (f/2) e−j2πf
SOL 1.66 From given graph the relation in x (t) and y (t) is
y (t) =− x [2 (t + 1)]
F
x (t) X (f)
Using scaling we have
1 X f
a ca m
F
x (at)
1X f
2 c2m
F
Thus x (2t)
Using shifting property we ge
x (t − t0) = e−j2πft X (f) 0

j2πf
e−j2πf (− 1) 1 X b l = e X b l
F f f
Thus x [2 (t + 1)]
2 2 2 2
j 2π f
f
−e Xc m
F
− x [2 (t + 1)]
2 2
Hence (B) is correct answer.
MCQ 1.67 A system has poles at 0.1 Hz, 1 Hz and 80 Hz; zeros at 5 Hz, 100 Hz and 200 Hz.
The approximate phase of the system response at 20 Hz is
(A) − 90c (B) 0c
(C) 90c (D) − 180c
SOL 1.67 Approximate (comparable to 90c) phase shift are
Due to pole at 0.01 Hz " − 90c
Due to pole at 80 Hz " − 90c
Due to pole at 80 Hz " 0
Due to zero at 5 Hz " 90c

Brought to you by: Nodia and Company Visit us at: www.nodia.co.in


PUBLISHING FOR GATE
Page 31 GATE EC 2004 www.gatehelp.com

Due to zero at 100 Hz " 0


Due to zero at 200 Hz " 0
Thus approximate total − 90c phase shift is provided.
Hence (A) is correct option.

MCQ 1.68 Consider the signal flow graph shown in Fig. The gain x5 is
x1

1 − (be + cf + dg) bedg


(A) (B)
abcd 1 − (be + cf + dg)
abcd 1 − (be + cf + dg) + bedg
(C) (D)
1 − (be + cf + dg) + bedg abcd

SOL 1.68 Mason Gain Formula


Σpk 3 k
T (s) =
3
In given SFG there is only one forward path and 3 possible loop.
p1 = abcd
31 = 1
3= 1 − (sum of indivudual loops) - (Sum of two non touching loops)
= 1 − (L1 + L2 + L3) + (L1 L3)
Non touching loop are L1 and L3 where
L1 L2 = bedg

C (s) p1 3 1
Thus =
R (s) 1 − (be + cf + dg) + bedg
= abcd
1 − (be + cf + dg) + bedg
Hence (C) is correct option
−2 2
If A = =
1 − 3G
MCQ 1.69 , then sin At is

sin (− 4t) + 2 sin (− t) − 2 sin (− 4t) + 2 sin (− t)


(A) 1 = G
3 − sin (− 4t) + sin (− t) 2 sin (− 4t) + sin (− t)
sin (− 2t) sin (2t)
(B) =
sin (t) sin (− 3t)G
sin (4t) + 2 sin (t) 2 sin (− 4t) − 2 sin (− t)
(C) 1 =
3 − sin (− 4t) + sin (t) 2 sin (4t) + sin (t) G
cos (− t) + 2 cos (t) 2 cos (− 4t) + 2 cos (− t)
(D) 1 = G
3 − cos (− 4t) + cos (− t) − 2 cos (− 4t) + cos (t)

Brought to you by: Nodia and Company Visit us at: www.nodia.co.in


PUBLISHING FOR GATE
Page 32 GATE EC 2004 www.gatehelp.com

SOL 1.69 Hence (A) is correct option


−2 2
A ==
1 − 3G
We have

Characteristic equation is
[λI − A] = 0
λ + 2 −2
or =0
−1 λ + 3
or (λ + 2)( λ + 3) − 2 = 0
or λ2 + 5λ + 4 = 0
Thus λ1 =− 4 and λ2 =− 1
Eigen values are − 4 and − 1.
Eigen vectors for λ1 =− 4
(λ1 I − A) X1 = 0
λ1 + 2 − 2 x11
or = 1 λ + 3G=x G = 0
1 21

− 2 − 2 x11
=− 1 − 1G=x G = 0
21

or − 2x11 − 2x21 = 0
or x11 + x21 = 0
We have only one independent equation x11 =− x21.
Let x21 = K , then x11 =− K , the Eigen vector will be
x11 −K −1
=x G = = K G = K = 1 G
21

Now Eigen vector for λ2 =− 1


(λ2 I − A) X2 = 0
λ2 + 2 − 2 x12
or = − 1 λ + 3G=x G = 0
2 22

1 − 2 x12
or =− 1 2 G=x G = 0
22

We have only one independent equation x12 = 2x22


Let x22 = K , then x12 = 2K . Thus Eigen vector will be
x12 2K 2
=x G = = K G = K = 1 G
22

Digonalizing matrix
x11 x12 −1 2
M == G ==
x21 x22 1 1G
1 −2
M−1 = ` − 1 j=
1 − 1G
Now
3 −
Brought to you by: Nodia and Company Visit us at: www.nodia.co.in
PUBLISHING FOR GATE
Page 33 GATE EC 2004 www.gatehelp.com

Now Diagonal matrix of sin At is D where


sin (λ1 t) 0 sin (− 4t) 0
D == G ==
0 sin (λ2 t) 0 sin (λ2 t)G
Now matrixB = sin At = MDM−1
− 1 2 sin (− 4t) 0 1 −2
=−` 1 j= G=
3 1 1 0 sin (− t) − 1 − 1G
G=

− sin (− 4t) − 2 sin (− t) 2 sin (− 4t) − 2 sin (− t)


=−` 1 j=
3 sin (− 4t) + 2 sin (t) − 2 sin (− 4t) − sin (− t)G
− sin (− 4t) − 2 sin (− t) 2 sin (− 4t) − 2 sin (− t)
=−` 1 j=
3 sin (− 4t) − sin (− t) − 2 sin (− 4t) + 2 sin (− t)G
sin (− 4t) + 2 sin (− t) − 2 sin (− 4t) + 2 sin (− t)
= ` 1 j= Gs
3 − sin (− 4t + sin (− t) 2 sin (− 4t) + sin (− t)

MCQ 1.70 The open-loop transfer function of a unity feedback system is


G (s) = K
s (s + s + 2)( s + 3)
2

The range of K for which the system is stable is


(A) 21 > K > 0 (B) 13 > K > 0
4

(C) 21 < K < 3 (D) − 6 < K < 3


4
SOL 1.70 For ufb system the characteristic equation is
1 + G (s) = 0

1+ K1 + G (s) =0
s (s + 2s + 2)( s + 3)
2

s 4 + 4s3 + 5s2 + 6s + K = 0
The routh table is shown below. For system to be stable,
(21 − 4K)
0 < K and 0 <
2/7
This gives 0 < K < 21
4

s4 1 5 K
s3 4 6 0
s2 7
2 K
21 − 4K
s1 7/2
0
s0 K

Brought to you by: Nodia and Company Visit us at: www.nodia.co.in


PUBLISHING FOR GATE
Page 34 GATE EC 2004 www.gatehelp.com

Hence (A) is correct option


MCQ 1.71 For the polynomial P (s) = s2 + s 4 + 2s3 + 2s2 + 3s + 15 the number of roots which
lie in the right half of the s −plane is
(A) 4 (B) 2
(C) 3 (D) 1
SOL 1.71 Hence (B) is correct option.
We have P (s) = s5 + s 4 + 2s3 + 3s + 15
The routh table is shown below.
If ε " 0+ then 2ε +ε 12 is positive and −15ε2−ε +2412ε − 144 is negative. Thus there are two sign
2

change in first column. Hence system has 2 root on RHS of plane.

s5 1 2 3
s4 1 2 15
s3 ε − 12 0
2ε + 12
s2 ε 15 0
−15ε − 24ε − 144
2
s1 2ε + 12

s0 0

MCQ 1.72 The state variable equations of a system are : xo1 =− 3x1 − x2 = u, xo2 = 2x1 and
y = x1 + u . The system is
(A) controllable but not observable
(B) observable but not controllable
(C) neither controllable nor observable
(D) controllable and observable
SOL 1.72 Hence (D) is correct option.
x1 − 3 − 1 x1 1
We have = G = = G= G = 0G
+ u
x2 2 0 x2
x1 1
and Y = [1 0]= G + = Gu
x2 2
−3 −1 1
Here A == G , B = = G and C = [1 0]
2 0 0
The controllability matrix is
QC = [B AB ]
1 −3
==
0 2G
det QC ! 0 Thus controllable
The observability matrix is

Brought to you by: Nodia and Company Visit us at: www.nodia.co.in


PUBLISHING FOR GATE
Page 35 GATE EC 2004 www.gatehelp.com

Q0 = [CT AT CT ]
1 −3
==
0 − 1G
!0

det Q0 ! 0 Thus observable

1 0
Given A = =
0 1G
MCQ 1.73 , the state transition matrix eAt is given by

0 e−t et 0
(A) > −t H (B) = G
e 0 0 et
e−t 0 0 et
(C) > H (D) = G
0 e−t et 0
SOL 1.73 Hence (B) is correct option.
s 0 1 0 s−1 0
(sI − A) = = G −= G ==
0 s 0 1 0 s − 1G
(s − 1) 0 1
0
1 = 0 (s − 1)G > 0 H
s−1
(sI − A) −1 = = 1
(s − 1) 2 s−1

eAt = L−1 [(sI − A)] −1

et 0
== G
0 et

MCQ 1.74 Consider the signal x (t) shown in Fig. Let h (t) denote the impulse response of the
filter matched to x (t), with h (t) being non-zero only in the interval 0 to 4 sec. The
slope of h (t) in the interval 3 < t < 4 sec is

(A) 1 sec - 1 (B) − 1 sec - 1


2

(C) − 1 sec - 1 (D) 1 sec - 1


2
SOL 1.74 The impulse response of matched filter is
h (t) = x (T − t)
Since here T = 4 , thus
h (t) = x (4 − t)
The graph of h (t) is as shown below.

Brought to you by: Nodia and Company Visit us at: www.nodia.co.in


PUBLISHING FOR GATE
Page 36 GATE EC 2004 www.gatehelp.com

From graph it may be easily seen that slope between 3 < t < 4 is − 1.
Hence (B) is correct option.
MCQ 1.75 A 1 mW video signal having a bandwidth of 100 MHz is transmitted to a receiver
through cable that has 40 dB loss. If the effective one-side noise spectral density at
the receiver is 10 - 20 Watt/Hz, then the signal-to-noise ratio at the receiver is
(A) 50 dB (B) 30 dB
(C) 40 dB (D) 60 dB
SOL 1.75 The SNR at transmitter is
SNRtr = Ptr
NB
10 - 3 = 109
10 - 20 # 100 # 106
In dB SNRtr = 10 log 109 = 90 dB
Cable Loss = 40 db
At receiver after cable loss we have
SNRRc = 90 − 40 = 50 dB
Hence (A) is correct option.
MCQ 1.76 A 100 MHz carrier of 1 V amplitude and a 1 MHz modulating signal of 1 V
amplitude are fed to a balanced modulator. The ourput of the modulator is passed
through an ideal high-pass filter with cut-off frequency of 100 MHz. The output
of the filter is added with 100 MHz signal of 1 V amplitude and 90c phase shift as
shown in the figure. The envelope of the resultant signal is

(A) constant (B) 1 + sin (2π # 106 t)

(C) 5 − sin (2π − 106 t) (D) 5 + cos (2π # 106 t)


4 4
SOL 1.76 Hence (C) is correct option.
We have fc = 100 MHz = 100 # 106 and fm = 1 MHz
= 1 # 106

Brought to you by: Nodia and Company Visit us at: www.nodia.co.in


PUBLISHING FOR GATE
Page 37 GATE EC 2004 www.gatehelp.com

The output of balanced modulator is


VBM (t) = [cos ωc t][ cos ωc t]
= 1 [cos (ωc + ωm) t + cos (ωc − ωm) t]
2
If VBM (t) is passed through HPF of cut off frequency fH = 100 # 106 , then only
(ωc + ωm) passes and output of HPF is
VHP (t) = 1 cos (ωc + ωm) t
2
Now V0 (t) = VHP (t) + sin (2π # 100 # 106) t

= 1 cos [2π100 # 106 + 2π # 1 # 106 t] + sin (2π # 100 # 106) t


2
= 1 cos [2π108 + 2π106 t] + sin (2π108) t
2
= 1 [cos (2π108 t) t cos (2π106 t)] − sin [2π108 t sin (2π106 t) + sin 2π108 t]
2
= 1 cos (2π106 t) cos 2π108 t + `1 − 1 sin 2π106 t j sin 2π108 t
2 2
This signal is in form
= A cos 2π108 t + B sin 2π108 t
The envelope of this signal is
= A2 + B2
2 2
= ` 1 cos (2π106 t)j + `1 − 1 sin (2π106 t j
2 2
= 1 cos2 (2π106 t) + 1 + 1 sin2 (2π106 t) − sin (2π106 t)
4 4
= 1 + 1 − sin (2π106 t)
4
= 5 − sin (2π106 t)
4

MCQ 1.77 Two sinusoidal signals of same amplitude and frequencies 10 kHz and 10.1 kHz are
added together. The combined signal is given to an ideal frequency detector. The
output of the detector is
(A) 0.1 kHz sinusoid (B) 20.1 kHz sinusoid
(C) a linear function of time (D) a constant
SOL 1.77 Hence (A) is correct option.
s (t) = A cos [2π10 # 103 t] + A cos [2π10.1 # 103 t]
Here T1 = 1 = 100μ sec
10 # 103
and T2 = 1 = 99μ sec
10.1 # 103
Period of added signal will be LCM [T1, T2]

Brought to you by: Nodia and Company Visit us at: www.nodia.co.in


PUBLISHING FOR GATE
Page 38 GATE EC 2004 www.gatehelp.com

Thus T = LCM [100, 99] = 9900μ sec


Thus frequency f = 1 = 0.1 kHz
9900μ

MCQ 1.78 Consider a binary digital communication system with equally likely 0’s and 1’s.
When binary 0 is transmitted the detector input can lie between the levels − 0.25
V and + 0.25 V with equl probability : when binary 1 is transmitted, the voltage
at the detector can have any value between 0 and 1 V with equal probability. If the
detector has a threshold of 0.2 V (i.e., if the received signal is greater than 0.2 V,
the bit is taken as 1), the average bit error probability is
(A) 0.15 (B) 0.2
(C) 0.05 (D) 0.5
SOL 1.78 The pdf of transmission of 0 and 1 will be as shown below :

Probability of error of 1
P (0 # X # 0.2) = 0.2
Probability of error of 0 :
P (0.2 # X # 0.25) = 0.05 # 2 = 0.1

P (0 # X # 0.2) + P (0.2 # X # 0.25)


Average error =
2
= 0.2 + 0.1 = 0.15
0
Hence (A) is correct option.
MCQ 1.79 A random variable X with uniform density in the interval 0 to 1 is quantized as
follows :
If 0 # X # 0.3 , xq = 0
If 0.3 < X # 1, xq = 0.7
where xq is the quantized value of X.
The root-mean square value of the quantization noise is
(A) 0.573 (B) 0.198
(C) 2.205 (D) 0.266
SOL 1.79 Hence (B) is correct option.
The square mean value is
#- 3 (x − xq) 2 f (x) dx
3
σ2 =

Brought to you by: Nodia and Company Visit us at: www.nodia.co.in


PUBLISHING FOR GATE
Page 39 GATE EC 2004 www.gatehelp.com

#0 (x − xq) 2 f (x) dx
1
=

#0 #0.3 (x − 0.7) 2 f (x) dx


0. 3 0. 1
= (x − 0) 2 f (x) dx +
3 0. 3 3 2 1
= ; x E + ; x + 0.49x − 14 x E
3 0 3 2 0. 3
or σ2 = 0.039
RMS = σ2 = 0.039 = 0.198
MCQ 1.80 Choose the current one from among the alternative A, B, C, D after matching an
item from Group 1 with the most appropriate item in Group 2.
Group 1 Group 2
1. FM P. Slope overload
2. DM Q. μ-law
3. PSK R. Envelope detector
4. PCM S. Hilbert transform
T. Hilbert transform
U. Matched filter
(A) 1 - T, 2 - P, 3 - U, 4 - S (B) 1 - S, 2 - U, 3 - P, 4 - T
(C) 1 - S, 2 - P, 3 - U, 4 - Q (D) 1 - U, 2 - R, 3 - S, 4 - Q
SOL 1.80 Hence (C) is correct option.
FM $ Capture effect
DM $ Slope over load
PSK $ Matched filter
PCM $ μ − law
MCQ 1.81 Three analog signals, having bandwidths 1200 Hz, 600 Hz and 600 Hz, are sampled
at their respective Nyquist rates, encoded with 12 bit words, and time division
multiplexed. The bit rate for the multiplexed. The bit rate for the multiplexed
signal is
(A) 115.2 kbps (B) 28.8 kbps
(C) 57.6 kbps (D) 38.4 kbps
SOL 1.81 Since fs = 2fm , the signal frequency and sampling frequency are as follows
fm1 = 1200 Hz $ 2400 samples per sec
fm2 = 600 Hz $ 1200 samples per sec
fm3 = 600 Hz $ 1200 samples per sec
Thus by time division multiplexing total 4800 samples per second will be sent.
Since each sample require 12 bit, total 4800 # 12 bits per second will be sent
Thus bit rate Rb = 4800 # 12 = 57.6 kbps
Hence (C) is correct option.
MCQ 1.82 Consider a system shown in the figure. Let X (f) and Y (f) and denote the Fourier

Brought to you by: Nodia and Company Visit us at: www.nodia.co.in


PUBLISHING FOR GATE
Page 40 GATE EC 2004 www.gatehelp.com

transforms of x (t) and y (t) respectively. The ideal HPF has the cutoff frequency
10 kHz.

The positive frequencies where Y (f) has spectral peaks are


(A) 1 kHz and 24 kHz (B) 2 kHz and 244 kHz
(C) 1 kHz and 14 kHz (D) 2 kHz and 14 kHz
SOL 1.82 The input signal X (f) has the peak at 1 kHz and − 1 kHz. After balanced modulator
the output will have peak at fc ! 1 kHz i.e. :
10 ! 1 $ 11 and 9 kHz
10 ! (− 1) $ 9 and 11 kHz
9 kHz will be filtered out by HPF of 10 kHz. Thus 11 kHz will remain. After
passing through 13 kHz balanced modulator signal will have 13 ! 11 kHz signal i.e.
2 and 24 kHz.
Thus peak of Y (f) are at 2 kHz and 24 kHz.
Hence (B) is correct option.
MCQ 1.83 A parallel plate air-filled capacitor has plate area of 10 - 4 m 2 and plate separation
of 10 - 3 m. It is connect - ed to a 0.5 V, 3.6 GHz source. The magnitude of the
displacement current is ( ε = 361π 10 - 9 F/m)
(A) 10 mA (B) 100 mA
(C) 10 A (D) 1.59 mA
SOL 1.83 The capacitance is
C = εo A = 8.85 # 10 - 3 # 10 = 8.85 # 10 - 13
- 12 -4

d 10
The charge on capacitor is
Q = CV = 8.85 # 10 - 13 = 4.427 # 10 - 13
Displacement current in one cycle
Q
I = = fQ = 4.427 # 10 - 13 # 3.6 # 109 = 1.59 mA
T
Hence (D) is correct option.

Brought to you by: Nodia and Company Visit us at: www.nodia.co.in


PUBLISHING FOR GATE
Page 41 GATE EC 2004 www.gatehelp.com

MCQ 1.84 A source produces binary data at the rate of 10 kbps. The binary symbols are
represented as shown in the figure given below.

The source output is transmitted using two modulation schemes, namely Binary
PSK (BPSK) and Quadrature PSK (QPSK). Let B1 and B2 be the bandwidth
requirements of BPSK and QPSK respectively. Assume that the bandwidth of he
above rectangular pulses is 10 kHz, B1 and B2 are
(A) B1 = 20 kHz, B2 = 20 kHz (B) B1 = 10 kHz, B2 = 20 kHz
(C) B1 = 20 kHz, B2 = 10 kHz (D) B1 = 20 kHz, B2 = 10 kHz
SOL 1.84 The required bandwidth of M array PSK is
BW = 2Rb
n
where 2n = M and Rb is bit rate
For BPSK, M = 2 = 2n $ n = 1

Thus B1 = 2Rb = 2 # 10 = 20 kHz


1
For QPSK, M = 4 = 2n $ n = 2

Thus B2 = 2Rb = 10 kHz


2
Hence (C) is correct option.
MCQ 1.85 Consider a 300 Ω, quarter - wave long (at 1 GHz) transmission line as shown in
Fig. It is connected to a 10 V, 50 Ω source at one end and is left open circuited at
the other end. The magnitude of the voltage at the open circuit end of the line is

(A) 10 V (B) 5 V
(C) 60 V (D) 60/7 V
SOL 1.85 Hence (C) is correct option.
VL = ZO
Vin Zin
Brought to you by: Nodia and Company Visit us at: www.nodia.co.in
PUBLISHING FOR GATE
Page 42 GATE EC 2004 www.gatehelp.com

or VL = ZO Vin = 10 # 300 = 60 V
Zin 50

MCQ 1.86 In a microwave test bench, why is the microwave signal amplitude modulated at 1
kHz
(A) To increase the sensitivity of measurement
(B) To transmit the signal to a far-off place
(C) To study amplitude modulations
(D) Because crystal detector fails at microwave frequencies
SOL 1.86 Hence (D) is correct option.
MCQ 1.87 If E = (atx + jaty) e jkz - kωt and H = (k/ωμ) (aty + katx ) e jkz - jωt , the time-averaged Poynting
vector is
(A) null vector (B) (k/ωμ) atz
(C) (2k/ωμ) atz (D) (k/2ωμ) atz
SOL 1.87 Hence (A) is correct option.
Ravg = 1 Re [E # H*]
2
E # H* = (atx + jaty) e jkz − jωt # k (− jatx + aty) e−jkz + jωt
ωμ
= atz ; k − (− j) (j) k E = 0
ωμ ωμ
Thus Ravg = 1 Re [E # H*] = 0
2

MCQ 1.88 Consider an impedance Z = R + jX marked with point P in an impedance Smith


chart as shown in Fig. The movement from point P along a constant resistance
circle in the clockwise direction by an angle 45c is equivalent to

(A) adding an inductance in series with Z


(B) adding a capacitance in series with Z
(C) adding an inductance in shunt across Z
(D) adding a capacitance in shunt across Z

Brought to you by: Nodia and Company Visit us at: www.nodia.co.in


PUBLISHING FOR GATE
Page 43 GATE EC 2004 www.gatehelp.com

SOL 1.88 Suppose at point P impedance is


Z = r + j (− 1)
If we move in constant resistance circle from point P in clockwise direction by an
angle 45c, the reactance magnitude increase. Let us consider a point Q at 45c from
point P in clockwise direction. It’s impedance is
Z1 = r − 0.5j
or Z1 = Z + 0.5j
Thus movement on constant r - circle by an +45c in CW direction is the addition
of inductance in series with Z .
Hence (A) is correct option.
MCQ 1.89 A plane electromagnetic wave propagating in free space is incident normally on a
large slab of loss-less, non-magnetic, dielectric material with ε > ε0 . Maxima and
minima are observed when the electric field is measured in front of the slab. The
maximum electric field is found to be 5 times the minimum field. The intrinsic
impedance of the medium should be
(A) 120π Ω (B) 60π Ω
(C) 600π Ω (D) 24π Ω
SOL 1.89 Hence (D) is correct option.
1− Γ
We have VSWR = Emax = 5 =
Emin 1+ Γ
or Γ =2
3
Thus Γ =− 2
3
η − η1
Now Γ= 2
η2 + η1
η − 120π
or −2 = 2
3 η2 + 120π
or η2 = 24π
MCQ 1.90 A lossless transmission line is terminated in a load which reflects a part of the
incident power. The measured VSWR is 2. The percentage of the power that is
reflected back is
(A) 57.73 (B) 33.33
(C) 0.11 (D) 11.11
SOL 1.90 Hence (D) is correct option.
1− Γ
The VSWR 2=
1+ Γ
or Γ =1
3

Brought to you by: Nodia and Company Visit us at: www.nodia.co.in


PUBLISHING FOR GATE
Page 44 GATE EC 2004 www.gatehelp.com

Pref
Thus = Γ2= 1
Pinc 9
or Pref = Pinc
9
i.e. 11.11% of incident power is reflected.

Answer Sheet
1. (B) 19. (A) 37. (D) 55. (B) 73. (B)
2. (D) 20. (D) 38. (B) 56. (C) 74. (B)
3. (A) 21. (D) 39. (B) 57. (A) 75. (A)
4. (A) 22. (C) 40. (D) 58. (D) 76. (C)
5. (C) 23. (D) 41. (C) 59. (D) 77. (A)
6. (C) 24. (C) 42. (D) 60. (C) 78. (A)
7. (A) 25. (C) 43. (A) 61. (D) 79. (B)
8. (D) 26. (B) 44. (B) 62. (A) 80. (C)
9. (B) 27. (B) 45. (C) 63. (C) 81. (C)
10. (C) 28. (A) 46. (D) 64. (C) 82. (B)
11. (A) 29. (D) 47. (A) 65. (D) 83. (D)
12. (A) 30. (A) 48. (A) 66. (B) 84. (C)
13. (C) 31. (D) 49. (D) 67. (A) 85. (C)
14. (A) 32. (B) 50. (B) 68. (C) 86. (D)
15. (D) 33. (C) 51. (C) 69. (A) 87. (A)
16. (B) 34. (B) 52. (D) 70 (A) 88. (A)
17. (A) 35. (D) 53. (C) 71 (B) 89. (D)
18. (C) 36. (A) 54. (D) 72 (D) 90. (D)

Brought to you by: Nodia and Company Visit us at: www.nodia.co.in


PUBLISHING FOR GATE
GATE EC
2005

Question 1 - 30 Carry One Mark Each

MCQ 1.1 The following differential equation has


d2 y dy 3
3 c 2 m + 4 c m + y2 + 2 = x
dt dt
(A) degree = 2 , order = 1 (B) degree = 1, order = 2
(C) degree = 4 , order = 3 (D) degree = 2 , order = 3
SOL 1.1 Order is the highest derivative term present in the equation and degree is the power
of highest derivative term.
Order = 2 , degree = 1
Hence (B) is correct answer.
MCQ 1.2 Choose the function f (t); − 3 < t < 3 for which a Fourier series cannot be defined.
(A) 3 sin (25t) (B) 4 cos (20t + 3) + 2 sin (710t)
(C) exp (− t ) sin (25t) (D) 1
SOL 1.2 Fourier series is defined for periodic function and constant.
3 sin (25t) is a periodic function.
4 cos (20t + 3) + 2 sin (710t) is sum of two periodic function and also a periodic
function.
e− t sin (25t) is not a periodic function, so FS can’t be defined for it.
1 is constant
Hence (C) is correct option.
MCQ 1.3 A fair dice is rolled twice. The probability that an odd number will follow an even
number is
(A) 1/2 (B) 1/6
(C) 1/3 (D) 1/4
SOL 1.3 Probability of coming odd number is 12 and the probability of coming even number
is 12 . Both the events are independent to each other, thus probability of coming odd
number after an even number is 12 # 12 = 14 .
Hence (D) is correct answer.

Brought to you by: Nodia and Company Visit us at: www.nodia.co.in


PUBLISHING FOR GATE
Page 2 GATE EC 2005 www.gatehelp.com

d2 y dy
MCQ 1.4 A solution of the following differential equation is given by 2
−5 + 6y = 0
dx dx
(A) y = e2x + e−3x (B) y = e2x + e3x
(C) y = e−2x + 33x (D) y = e−2x + e−3x
SOL 1.4 Hence (B) is correct answer.
d2 y dy
We have 2
−5 + 6y = 0
dx dx
The A.E. is m2 − 5m + 6 = 0
m = 3, 2
The CF is yc = C1 e3x + C2 e2x
Since Q = 0 , thus y = C1 e3x + C2 e2x
Thus only (B) may be correct.
MCQ 1.5 The function x (t) is shown in the figure. Even and odd parts of a unit step function
u (t) are respectively,

(A) 1 , 1 x (t) (B) − 1 , 1 x (t)


2 2 2 2

(C) 1 , − 1 x (t) (D) − 1 , − 1 x (t)


2 2 2 2
SOL 1.5 Hence (A) is correct answer.
g (t) + g (− t)
Ev{g (t)} =
2
g (t) − g (− t)
odd{g (t)} =
2
Here g (t) = u (t)
u (t) + u (− t) 1
Thus ue (t) = =
2 2
u (t) − u (− t) x (t)
uo (t) = =
2 2

MCQ 1.6 The region of convergence of z − transform of the sequence


b 6 l u (n) − b 5 l u (− n − 1) must be
5 n 6 n

(A) z < 5 (B) z > 5


6 6

Brought to you by: Nodia and Company Visit us at: www.nodia.co.in


PUBLISHING FOR GATE
Page 3 GATE EC 2005 www.gatehelp.com

(C) 5 < z < 6 (D) 6 < z < 3


6 5 5
SOL 1.6 Hence (C) is correct answer.
Here x1 (n) = ` 5 jn u (n)
6
X1 (z) = 1 ROC : R1 " z > 5
1 − ^ 65 z−1h 6
x2 (n) =−` 6 jn u (− n − 1)
5
X1 (z) = 1 − 1 ROC : R2 " z < 6
1 − ^ 65 z−1h 5
Thus ROC of x1 (n) + x2 (n) is R1 + R2 which is 5 < z < 6
6 5

MCQ 1.7 The condition on R, L and C such that the step response y (t) in the figure has no
oscillations, is

(A) R $ 1 L (B) R $ L
2 C C

(C) R $ 2 L (D) R = 1
C LC
SOL 1.7 Transfer function is
1 1
Y (s) sC 1 LC
= = 2 =
U (s) R + sL + 1 s LC + scR + 1 2 R
s + s+ 1
sC L LC
Comparing with s + 2ξωn s + ωn = 0 we have
2 2

Here 2ξωn = R ,
L
and ωn = 1
LC
Thus ξ = R LC = R C
2L 2 L
For no oscillations, ξ $ 1
Thus R C $ 1
2 L
or R $2 L
C
Hence (C) is correct option.

Brought to you by: Nodia and Company Visit us at: www.nodia.co.in


PUBLISHING FOR GATE
Page 4 GATE EC 2005 www.gatehelp.com

MCQ 1.8 The ABCD parameters of an ideal n: 1 transformer shown in the figure are
n 0
>0 x H

The value of x will be


(A) n (B) 1
n

(C) n2 (D) 12
n
SOL 1.8 For given transformer
I2 = V1 = n
I1 V2 1
or I1 = I2 and V1 = nV2
n
Comparing with standard equation
V1 = AV2 + BI2
I1 = CV2 + DI2
A B n 0
=C D G = = 0 1 G
n

Thus x = 1
n
Hence (B) is correct option.

MCQ 1.9 In a series RLC circuit, R = 2 kΩ , L = 1 H, and C = 1 μF The resonant frequency


400
is
(A) 2 # 10 4 Hz (B) 1 # 10 4 Hz
π
(C) 10 4 Hz (D) 2π # 10 4 Hz
SOL 1.9 Hence (B) is correct option.
We have L = 1H and C = 1 # 10−6
400
Resonant frequency
f0 = 1 == 1
2π LC 2π 1 # 1 # 10 - 6
400
3 4
= 10 # 20 = 10 Hz
2π π

Brought to you by: Nodia and Company Visit us at: www.nodia.co.in


PUBLISHING FOR GATE
Page 5 GATE EC 2005 www.gatehelp.com

MCQ 1.10 The maximum power that can be transferred to the load resistor RL from the
voltage source in the figure is

(A) 1 W (B) 10 W
(C) 0.25 W (D) 0.5 W
SOL 1.10 Maximum power will be transferred when RL = Rs = 100Ω
In this case voltage across RL is 5 V, therefore
2
Pmax = V = 5 # 5 = 0.25 W
R 100
Hence (C) is correct option.
MCQ 1.11 The bandgap of Silicon at room temperature is
(A) 1.3 eV (B) 0.7 eV
(C) 1.1 eV (D) 1.4 eV
SOL 1.11 For silicon at 0 K,
Eg0 = 1.21 eV
At any temperature
EgT = Eg0 − 3.6 # 10 - 4 T
At T = 300 K,
Eg300 = 1.21 − 3.6 # 10 - 4 # 300 = 1.1 eV
This is standard value, that must be remembered.
Hence option (C) is correct.
MCQ 1.12 A Silicon PN junction at a temperature of 20c C has a reverse saturation current
of 10 pico - Ameres (pA). The reserve saturation current at 40cC for the same bias
is approximately
(A) 30 pA (B) 40 pA
(C) 50 pA (D) 60 pA
SOL 1.12 The reverse saturation current doubles for every 10cC rise in temperature as follows
I0 (T) = I 01 # 2(T − T )/10
1

Thus at 40c C, I0 = 40 pA
Hence option (B) is correct.
MCQ 1.13 The primary reason for the widespread use of Silicon in semiconductor device
technology is

Brought to you by: Nodia and Company Visit us at: www.nodia.co.in


PUBLISHING FOR GATE
Page 6 GATE EC 2005 www.gatehelp.com

(A) abundance of Silicon on the surface of the Earth.


(B) larger bandgap of Silicon in comparison to Germanium.
(C) favorable properties of Silicon - dioxide (SiO2)
(D) lower melting point
SOL 1.13 Silicon is abundant on the surface of earth in the from of SiO2 .
Hence option (A) is correct.
MCQ 1.14 The effect of current shunt feedback in an amplifier is to
(A) increase the input resistance and decrease the output resistance
(B) increases both input and output resistance
(C) decrease both input and output resistance
(D) decrease the input resistance and increase the output resistance
SOL 1.14 The effect of current shunt feedback in an amplifier is to decrease the input
resistance and increase the output resistance as :
Rif = Ri
1 + Aβ
Rof = R0 (1 + Aβ)
where Ri " Input resistance without feedback
Rif " Input resistance with feedback.
Hence (D) is correct option.
MCQ 1.15 The input resistance Ri of the amplifier shown in the figure is

(A) 30 kΩ (B) 10 kΩ
4
(C) 40 kΩ (D) infinite
SOL 1.15 Since the inverting terminal is at virtual ground, the current flowing through the
voltage source is
Is = Vs
10k
or Vs = 10 kΩ = R
in
Is
Hence (B) is correct option.

Brought to you by: Nodia and Company Visit us at: www.nodia.co.in


PUBLISHING FOR GATE
Page 7 GATE EC 2005 www.gatehelp.com

MCQ 1.16 The first and the last critical frequency of an RC -driving point impedance function
must respectively be
(A) a zero and a pole (B) a zero and a zero
(C) a pole and a pole (D) a pole and a zero
SOL 1.16 For stability poles and zero interlace on real axis. In RC series network the driving
point impedance is
Zins = R + 1 = 1 + sRC
Cs sC
Here pole is at origin and zero is at s =− 1/RC , therefore first critical frequency is
a pole and last critical frequency is a zero.
For RC parallel network the driving point impedance is
R 1
Zinp = Cs = R
R+ 1 1 + sRC
Cs
Here pole is s =− 1/RC and zero is at 3, therefore first critical frequency is a pole
and last critical frequency is a zero.
Hence (C) is correct option.
MCQ 1.17 The cascade amplifier is a multistage configuration of
(A) CC − CB (B) CE − CB
(C) CB − CC (D) CE − CC
SOL 1.17 The CE configuration has high voltage gain as well as high current gain. It performs
basic function of amplifications. The CB configuration has lowest Ri and highest Ro
. It is used as last step to match a very low impedance source and to drain a high
impedance load
Thus cascade amplifier is a multistage configuration of CE-CB
Hence (B) is correct option
MCQ 1.18 Decimal 43 in Hexadecimal and BCD number system is respectively
(A) B2, 0100 011 (B) 2B, 0100 0011
(C) 2B, 0011 0100 (D) B2, 0100 0100
SOL 1.18 Dividing 43 by 16 we get

g
2
16 43
32
11
11 in decimal is equivalent is B in hexamal.
Thus 4310 * 2B16
Now 410 * 01002
310 * 00112
Thus 4310 * 01000011BCD

Brought to you by: Nodia and Company Visit us at: www.nodia.co.in


PUBLISHING FOR GATE
Page 8 GATE EC 2005 www.gatehelp.com

Hence (B) is correct answer.


MCQ 1.19 The Boolean function f implemented in the figure using two input multiplexes is

(A) ABC + ABC (B) ABC + ABC


(C) ABC + ABC (D) ABC + ABC
SOL 1.19 The diagram is as shown in fig

f' = BC + BC
f = f' A + f ' 0
= f'A = ABC + ABC
Hence (A) is correct answer.
MCQ 1.20 Which of the following can be impulse response of a causal system ?

SOL 1.20 For causal system h (t) = 0 for t # 0 . Only (D) satisfy this condition.
Hence (D) is correct answer.

Brought to you by: Nodia and Company Visit us at: www.nodia.co.in


PUBLISHING FOR GATE
Page 9 GATE EC 2005 www.gatehelp.com

MCQ 1.21 Let x (n) = ( 12 ) n u (n), y (n) = x2 (n) and Y (e jω) be the Fourier transform of y (n) then
Y (e j0)
(A) 1 (B) 2
4

(C) 4 (D) 4
3
SOL 1.21 Hence (D) is correct answer.
x (n) = b 1 l u (n)
n

2
y (n) = x2 (n) = b 1 l u2 (n)
2n

2
2 n
y (n) = ;b 1 l E u (n) = b 1 l u (n)
n
or ...(1)
2 4
n=3 n=3
/ b 14 l e−jωn
n
Y (e jω) = / y (n) e−jωn =
n =− 3 n=0
n=3 n
/ ` 14 j = 1 +b1l +b1l+b1l +b1l
1 3 4
or Y (e j0) =
n=0
4 4 4 4
or Y (e j0) = 1 =4
1− 1
4
3
Alternative :
Taking z transform of (1) we get
Y (z) = 1
1 − 14 z−1
Substituting z = e jω we have
Y (e jω) = 1
1 − 14 e−jω
Y (e j0) = 1 1 = 4
1− 4 3

MCQ 1.22 Find the correct match between group 1 and group 2.
Group 1 Group 2
P. {1 + km (t) A sin (ωc t)} W. Phase modulation
Q. km (t) A sin (ωc t) X. Frequency modulation
R. A sin {ωc t + km (t)} Y. Amplitude modulation
#- 3
t
S. A sin ; ωc t + k m (t) dt E Z. DSB-SC modulation

(A) P − Z, Q − Y, R − X, S − W
(B) P − W, Q − X, R − Y, S − Z
(C) P − X, Q − W, R − Z, S − Y
(D) P − Y, Q − Z, R − W, S − X
SOL 1.22 Hence (D) is correct option.
{1 + km (t)} A sin (ωc t) $ Amplitude modulation

Brought to you by: Nodia and Company Visit us at: www.nodia.co.in


PUBLISHING FOR GATE
Page 10 GATE EC 2005 www.gatehelp.com

dm (t) Asin (ωc t) $ DSB-SC modulation


A sin {cos t + km (t)} $ Phase Modulation
A sin [ωct + k] t- 3 m (t) dt $ Frequency Modulation
MCQ 1.23 The power in the signal s (t) = 8 cos (20π − π2 ) + 4 sin (15πt) is
(A) 40 (B) 41
(C) 42 (D) 82
SOL 1.23 Hence (A) is correct answer.
s (t) = 8 cos ` π − 20πt j + 4 sin 15πt
2
= 8 sin 20πt + 4 sin 15πt
Here A1 = 8 and A2 = 4 . Thus power is
2 2 2 2
P = A1 + A2 = 8 + 4 = 40
2 2 2 2

MCQ 1.24 Which of the following analog modulation scheme requires the minimum transmitted
power and minimum channel bandwidth ?
(A) VSB (B) DSB-SC
(C) SSB (D) AM
SOL 1.24 Hence (C) is correct option.
VSB $ fm + fc
DSB - SC $ 2fm
SSB $ fm
AM $ 2fm
Thus SSB has minimum bandwidth and it require minimum power.
MCQ 1.25 A linear system is equivalently represented by two sets of state equations :
Xo = AX + BU and Wo = CW + DU
The eigenvalues of the representations are also computed as [λ] and [μ]. Which one
of the following statements is true ?
(A) [λ] = [μ] and X = W (B) [λ] = [μ] and X ! W
(C) [λ] ! [μ] and X = W (D) [λ] = [μ] and X ! W
SOL 1.25 Hence (C) is correct option
We have Xo = AX + BU where λ is set of Eigen values
and o
W = CW + DU where μ is set of Eigen values
If a liner system is equivalently represented by two sets of state equations, then for
both sets, states will be same but their sets of Eigne values will not be same i.e.
X = W but λ ! μ
MCQ 1.26 Which one of the following polar diagrams corresponds to a lag network ?

Brought to you by: Nodia and Company Visit us at: www.nodia.co.in


PUBLISHING FOR GATE
Page 11 GATE EC 2005 www.gatehelp.com

SOL 1.26 The transfer function of a lag network is


T (s) = 1 + sT β > 1; T > 0
1 + sβT

T (jω) = 1 + ω2 T2
1 + ω2 β2 T2
and +T (jω) = tan−1 (ωT) − tan−1 (ωβT)
At ω = 0 , T (jω) =1
At ω = 0 ,+T (jω) =− tan−1 0 = 0
At ω = 3 , T (jω) = 1
β
At ω = 3 ,+T (jω) =0
Hence (D) is correct option.
MCQ 1.27 Despite the presence of negative feedback, control systems still have problems of
instability because the
(A) Components used have non- linearities
(B) Dynamic equations of the subsystem are not known exactly.
(C) Mathematical analysis involves approximations.
(D) System has large negative phase angle at high frequencies.
SOL 1.27 Despite the presence of negative feedback, control systems still have problems
of instability because components used have nonlinearity. There are always some
variation as compared to ideal characteristics.
Hence (A) is correct option.
MCQ 1.28 The magnetic field intensity vector of a plane wave is given by
H (x, y, z, t) = 10 sin (50000t + 0.004x + 30) aty
t
where ay , denotes the unit vector in y direction. The wave is propagating with a
phase velocity.
(A) 5 # 10 4 m/s (B) − 3 # 108 m/s
(C) − 1.25 # 107 m/s (D) 3 # 108 m/s]

Brought to you by: Nodia and Company Visit us at: www.nodia.co.in


PUBLISHING FOR GATE
Page 12 GATE EC 2005 www.gatehelp.com

SOL 1.28 Hence (C) is correct option.


ω = 50, 000 and β =− 0.004
Phase Velocity is vP = ω = 5 # 10 - 3 = 1.25 # 107 m/s
4

β − 4 # 10
MCQ 1.29 Many circles are drawn in a Smith Chart used for transmission line calculations.
The circles shown in the figure represent

(A) Unit circles


(B) Constant resistance circles
(C) Constant reactance circles
(D) Constant reflection coefficient circles.
SOL 1.29 The given figure represent constant reactance circle.
Hence (C) is correct option.
MCQ 1.30 Refractive index of glass is 1.5. Find the wavelength of a beam of light with frequency
of 1014 Hz in glass. Assume velocity of light is 3 # 108 m/s in vacuum
(A) 3 μm (B) 3 mm
(C) 2 μm (D) 1 mm
SOL 1.30 Refractive index of glass μ = 1.5
Frequency f = 1014 Hz
c = 3 # 108 m/sec
8
λ = c = 3 # 10
14
= 3 # 10 - 6
f 10
wavelength in glass is
λg = α = 3 # 10 = 2 # 10 - 6 m
-6

μ 1.5
Hence (C) is correct option.

Question 31 - 80 Carry Two Marks Each

MCQ 1.31 In what range should Re (s) remain so that the Laplace transform of the function
e(a + 2) t + 5 exits.
(A) Re (s) > a + 2 (B) Re (s) > a + 7

Brought to you by: Nodia and Company Visit us at: www.nodia.co.in


PUBLISHING FOR GATE
Page 13 GATE EC 2005 www.gatehelp.com

(C) Re (s) < 2 (D) Re (s) > a + 5


SOL 1.31 Hence (A) is correct answer.
We have f (t) = e(a + 2) t + 5 = e5 .e(a + 2) t
Taking laplace transform we get
F (s) = e5 ;
s − (a + 2) E
1 Thus Re (s) > (a + 2)

−4 2
Given the matrix =
4 3G
MCQ 1.32 , the eigenvector is
3 4
(A) = G (B) = G
2 3
2 −1
(C) = G (D) = G
−1 2
SOL 1.32 Hence (C) is correct answer.
−4 2
A ==
4 3G
We have

Characteristic equation is
A − λI = 0
4−λ 2
or =0
4 3−λ
or (− 4 − λ)(3 − λ) − 8 =0
or − 12 + λ + λ2 − 8 =0
or λ2 + λ − 20 =0
or λ =− 5, 4 Eigen values
Eigen vector for λ =− 5
(A − λI) Xi =0
1 − (− 5) 2 x1 0
= 4 8 − 4G=x2 G
== G
0
1 2 x1 0
=0 0G=x G == G
0
R2 − 4R1
2

x1 + 2x2 = 0
Let − x1 = 2 & x2 =− 1,
2
X ==
−1G
Thus Eigen vector

2 − 0.1 1
a
MCQ 1.33 Let, A = = G and A - 1 = = 2 G. Then (a + b) =
0 3 0 b
(A) 7/20 (B) 3/20
(C) 19/60 (D) 11/20

Brought to you by: Nodia and Company Visit us at: www.nodia.co.in


PUBLISHING FOR GATE
Page 14 GATE EC 2005 www.gatehelp.com

SOL 1.33 We have


2 − 0.1 1
a
A == G and A - 1 = = 2 G
0 3 0 b
Now AA - 1 = I
2 − 0.1 12 a 1 0
or =0 3 G= G = =0 1 G
0 b
1 2a − 0.1b 1 0
or =0 3b G ==
0 1G
or 2a − 0.1 = 0 and 3b = 1
Thus solving above we have b = 1 and a = 1
3 60
Therefore a+b = 1 + 1 = 7
3 60 20
Hence (A) is correct option.
2
#0 exp c− x m dx is
1 3
MCQ 1.34 The value of the integral I =
2π 8
(A) 1 (B) π
(C) 2 (D) 2π
SOL 1.34 Gaussian PDF is
#
1 3 − (x − μ)2
f (x) = e 2σ2 dx for − 3 # x # 3
2π σ −3

# f (x) dx
3
and =1
−3

Substituting μ = 0 and σ = 2 in above we get


#
1 3 − x2
e dx = 18

2π 2 − 3
or
2π 2 0
#
1 2 3e− dx = 1 x2
8

#
1 3 − x2
or e dx = 18

2π 0
Hence (A) is correct option.
MCQ 1.35 The derivative of the symmetric function drawn in given figure will look like

Brought to you by: Nodia and Company Visit us at: www.nodia.co.in


PUBLISHING FOR GATE
Page 15 GATE EC 2005 www.gatehelp.com

SOL 1.35 For x > 0 the slope of given curve is negative. Only (C) satisfy this condition.
Hence (C) is correct answer.
MCQ 1.36 Match the following and choose the correct combination:
Group I Group 2
E. Newton-Raphson method 1. Solving nonlinear equations
F. Runge-kutta method 2. Solving linear simultaneous
equations
G. Simpson’s Rule 3. Solving ordinary differential
equations
H. Gauss elimination 4. Numerical integration
5. Interpolation
6. Calculation of Eigenvalues
(A) E − 6, F − 1, G − 5, H − 3 (B) E − 1, F − 6, G − 4, H − 3
(C) E − 1, F − 3, G − 4, H − 2 (D) E − 5, F − 3, G − 4, H − 1
SOL 1.36 Hence (C) is correct option.
Newton - Raphson " Method-Solving nonlinear eq.
Runge - kutta Method " Solving ordinary differential eq.
Simpson’s Rule " Numerical Integration
Gauss elimination " Solving linear simultaneous eq.
MCQ 1.37 Given an orthogonal matrix
R1 1 1 1 V
S W
S1 1 − 1 − 1W
A =S
1 − 1 0 0W
S W
S0 0 1 1 W
6AA @- 1 is T X
T

R1 V R1 V
S 4 0 0 0W S 2 0 0 0W
S 0 14 0 0 W S 0 12 0 0 W
(A) S 1 W (B) S 1 W
S0 0 2 0W S0 0 2 0 W
S 0 0 0 12 W S 0 0 0 12 W
Brought to you by: Nodia
T and Company
X T Visit
X us at: www.nodia.co.in
PUBLISHING FOR GATE
Page 16 GATE EC 2005 www.gatehelp.com

R1 R1 V
S 0 0 0 VW S 4 0 0 0W
S0 1 0 0W S0 1 0 0 W
(C) S (D) S 4 1 W
0 0 1 0W S0 0 4 0 W
S W
S0 0 0 1W S 0 0 0 14 W
T X T X

SOL 1.37 Hence (C) is correct option.


From orthogonal matrix
[AAT ] = I
Since the inverse of I is I , thus
[AAT ] −1 = I−1 = I
MCQ 1.38 For the circuit shown in the figure, the instantaneous current i1 (t) is

(A) 10 3 90c A (B) 10 3 − 90c A


2 2
(C) 5 60c A (D) 5 − 60c A
SOL 1.38 Applying KCL we get
i1 (t) + 5+0c = 10+60c
or i1 (t) = 10+60c − 5+0c = 5 + 5 3j − 5
or i1 (t) = 5 3 +90c = 10 3 +90c
2
Hence (A) is correct option.
MCQ 1.39 Impedance Z as shown in the given figure is

(A) j29 Ω (B) j9 Ω


(C) j19 Ω (D) j39 Ω

Brought to you by: Nodia and Company Visit us at: www.nodia.co.in


PUBLISHING FOR GATE
Page 17 GATE EC 2005 www.gatehelp.com

SOL 1.39 If L1 = j5Ω and L3 = j2Ω the mutual induction is subtractive because current enters
from dotted terminal of j2Ω coil and exit from dotted terminal of j5Ω. If L2 = j2Ω
and L3 = j2Ω the mutual induction is additive because current enters from dotted
terminal of both coil.
Thus Z = L1 − M13 + L2 + M23 + L3 − M31 + M32
= j5 + j10 + j2 + j10 + j2 − j10 + j10 = j9
Hence (B) is correct option.
MCQ 1.40 For the circuit shown in the figure, Thevenin’s voltage and Thevenin’s equivalent
resistance at terminals a − b is

(A) 5 V and 2 Ω (B) 7.5 V and 2.5 Ω


(C) 4 V and 2 Ω (D) 3 V and 2.5 Ω
SOL 1.40 Open circuit at terminal ab is shown below

Applying KCL at node we get


Vab + Vab − 10 = 1
5 5
or Vab = 7.5 = Vth
Short circuit at terminal ab is shown below

Short circuit current from terminal ab is


Isc = 1 + 10 = 3 A
5
Thus Rth = Vth = 7.5 = 2.5 Ω
Isc 3

Brought to you by: Nodia and Company Visit us at: www.nodia.co.in


PUBLISHING FOR GATE
Page 18 GATE EC 2005 www.gatehelp.com

Here current source being in series with dependent voltage source make it ineffective.
Hence (B) is correct option.
MCQ 1.41 If R1 = R2 = R4 = R and R3 = 1.1R in the bridge circuit shown in the figure, then
the reading in the ideal voltmeter connected between a and b is

(A) 0.238 V (B) 0.138 V


(C) − 0.238 V (D) 1 V
SOL 1.41 Here Va = 5 V because R1 = R2 and total voltage drop is 10 V.
Now Vb = R3 # 10 = 1.1 # 10 = 5.238 V
R3 + R4 2.1
V = Va − Vb = 5 − 5.238 =− 0.238 V
Hence (C) is correct option.
MCQ 1.42 The h parameters of the circuit shown in the figure are

0. 1 0. 1 10 − 1
(A) =
− 0. 1 0. 3 G
(B) =
1 0.05G
30 20 10 1
(C) = G (D) =
20 20 − 1 0.05G
SOL 1.42 For h parameters we have to write V1 and I2 in terms of I1 and V2 .
V1 = h11 I1 + h12 V2
I2 = h21 I1 + h22 V2
Applying KVL at input port
V1 = 10I1 + V2
Applying KCL at output port
V2 = I + I
1 2
20
or I2 =− I1 + V2
20

Brought to you by: Nodia and Company Visit us at: www.nodia.co.in


PUBLISHING FOR GATE
Page 19 GATE EC 2005 www.gatehelp.com

Thus from above equation we get


h11 h12 10 1
=h h G = =− 1 0.05G
12 22

Hence (D) is correct option.


MCQ 1.43 A square pulse of 3 volts amplitude is applied to C − R circuit shown in the figure.
The capacitor is initially uncharged. The output voltage V2 at time t = 2 sec is

(A) 3 V (B) − 3 V
(C) 4 V (D) − 4 V
SOL 1.43 Hence (B) is correct option.
Time constant RC = 0.1 # 10 - 6 # 103 = 10 - 4 sec
Since time constant RC is very small, so steady state will be reached in 2 sec. At
t = 2 sec the circuit is as shown in fig.

Vc = 3 V
V2 =− Vc =− 3 V
MCQ 1.44 A Silicon sample A is doped with 1018 atoms/cm 3 of boron. Another sample B
of identical dimension is doped with 1018 atoms/cm 3 phosphorus. The ratio of
electron to hole mobility is 3. The ratio of conductivity of the sample A to B is
(A) 3 (B) 1
3

(C) 2 (D) 3
3 2
SOL 1.44 Hence option (B) is correct.
σn = nqμn
σp = pqμp (n = p)
σp μ
= p =1
σn μn 3

Brought to you by: Nodia and Company Visit us at: www.nodia.co.in


PUBLISHING FOR GATE
Page 20 GATE EC 2005 www.gatehelp.com

MCQ 1.45 A Silicon PN junction diode under reverse bias has depletion region of width 10 μ
m. The relative permittivity of Silicon, εr = 11.7 and the permittivity of free space
ε0 = 8.85 # 10 - 12 F/m. The depletion capacitance of the diode per square meter is
(A) 100 μF (B) 10 μF
(C) 1 μF (D) 20 μF
SOL 1.45 Hence option (B) is correct.
C = ε0 εr A
d
or C = ε0 εr = 8.85 # 10−12 # 11.7 = 10.35 μ F
A d 10 # 10−6

MCQ 1.46 For an npn transistor connected as shown in figure VBE = 0.7 volts. Given that
reverse saturation current of the junction at room temperature 300 K is 10 - 13 A,
the emitter current is

(A) 30 mA (B) 39 mA
(C) 49 mA (D) 20 mA
SOL 1.46 Hence (C) is correct option.
IE = Is `e nV − 1j = 10 - 13 c
V BE

T
0.7 − 1m = 49 mA
-3
e1 # 26 # 10
MCQ 1.47 The voltage e0 is indicated in the figure has been measured by an ideal voltmeter.
Which of the following can be calculated ?

(A) Bias current of the inverting input only


(B) Bias current of the inverting and non-inverting inputs only
(C) Input offset current only
(D) Both the bias currents and the input offset current

Brought to you by: Nodia and Company Visit us at: www.nodia.co.in


PUBLISHING FOR GATE
Page 21 GATE EC 2005 www.gatehelp.com

SOL 1.47 The circuit is as shown below

Writing equation for I− have


e 0 − V− = I
-
1M
or e0 = I− (1M) + V− ...(1)
Writing equation for I+ we have
0 − V+
= I+
1M
or V+ = − I+ (1M) ...(2)
Since for ideal OPAMP V+ = V- , from (1) and (2) we have
e0 = I− (1M) − I + (1M)
= (I− − I+) (1M) = IOS (1M)
Thus if e0 has been measured, we can calculate input offset current IOS only.
Hence (C) is correct option.
MCQ 1.48 The Op-amp circuit shown in the figure is filter. The type of filter and its cut. Off
frequency are respectively

(A) high pass, 1000 rad/sec. (B) Low pass, 1000 rad/sec
(C) high pass, 1000 rad/sec (D) low pass, 10000 rad/sec
SOL 1.48 At low frequency capacitor is open circuit and voltage acr s non-inverting terminal
is zero. At high frequency capacitor act as short circuit and all input voltage appear
at non-inverting terminal. Thus, this is high pass circuit.
The frequency is given by
ω = 1 = 1 = 1000 rad/sec
RC 1 # 103 # 1 # 10 - 6
Hence (C) is correct option.

Brought to you by: Nodia and Company Visit us at: www.nodia.co.in


PUBLISHING FOR GATE
Page 22 GATE EC 2005 www.gatehelp.com

MCQ 1.49 In an ideal differential amplifier shown in the figure, a large value of (RE ).
(A) increase both the differential and common - mode gains.
(B) increases the common mode gain only.
(C) decreases the differential mode gain only.
(D) decreases the common mode gain only.
SOL 1.49 Common mode gain
ACM =− RC
2RE
And differential mode gain
ADM =− gm RC
Thus only common mode gain depends on RE and for large value of RE it decreases.
Hence (D) is correct option.
MCQ 1.50 For an n -channel MOSFET and its transfer curve shown in the figure, the threshold
voltage is

(A) 1 V and the device is in active region


(B) − 1 V and the device is in saturation region
(C) 1 V and the device is in saturation region
(D) − 1 V and the device is an active region
SOL 1.50 From the graph it can be easily seen that Vth = 1 V
Now VGS = 3 − 1 = 2 V
and VDS = 5 − 1 = 4 V
Since VDS > VGS $ VDS > VGS − Vth
Thus MOSFET is in saturation region.
Hence option (C) is correct.
MCQ 1.51 The circuit using a BJT with β = 50 and VBE = 0.7V is shown in the figure. The
base current IB and collector voltage by VC and respectively

Brought to you by: Nodia and Company Visit us at: www.nodia.co.in


PUBLISHING FOR GATE
Page 23 GATE EC 2005 www.gatehelp.com

(A) 43 μA and 11.4 Volts (B) 40 μA and 16 Volts


(C) 45 μA and 11 Volts (D) 50 μA and 10 Volts
SOL 1.51 The circuit under DC condition is shown in fig below

Applying KVL we have


VCC − RB IB − VBE − RE IE = 0
orVCC − RB IB − VBE − RE (β + 1) IB = 0 Since IE = IB + βIB
or IB = VCC − VBE
RB + (β + 1) RE
= 20 − 0.7 = 40μ A
430k + (50 + 1)1 k
Now IC = βIB = 50 # 40μ = 2 mA
VC = VCC − RC IC = 20 − 2m # 2k = 16 V
Hence (B) is correct option.
MCQ 1.52 The Zener diode in the regulator circuit shown in the figure has a Zener voltage of
5.8 volts and a zener knee current of 0.5 mA. The maximum load current drawn
from this current ensuring proper functioning over the input voltage range between
20 and 30 volts, is

Brought to you by: Nodia and Company Visit us at: www.nodia.co.in


PUBLISHING FOR GATE
Page 24 GATE EC 2005 www.gatehelp.com

(A) 23.7 mA (B) 14.2 mA


(C) 13.7 mA (D) 24.2 mA
SOL 1.52 The maximum load current will be at maximum input voltage i.e.
Vmax = 30 V i.e.
Vmax − VZ = I + I
L Z
1k
or 30 − 5.8 = IL = 0.5 m
1k
or IL = 24.2 − 0.5 = 23.7 mA
Hence (A) is correct option.
MCQ 1.53 The transistors used in a portion of the TTL gate show in the figure have β = 100 .
The base emitter voltage of is 0.7 V for a transistor in active region and 0.75 V for
a transistor in saturation. If the sink current I = 1 A and the output is at logic 0,
then the current IR will be equal to

(A) 0.65 mA (B) 0.70 mA


(C) 0.75 mA (D) 1.00 mA
SOL 1.53 The circuit is as shown below

If output is at logic 0, the we have V0 = 0 which signifies BJT Q3 is in saturation


and applying KVL we have
VBE3 = IR # 1k
or 0.75 = IR # 1k
or IR = 0.75 mA

Brought to you by: Nodia and Company Visit us at: www.nodia.co.in


PUBLISHING FOR GATE
Page 25 GATE EC 2005 www.gatehelp.com

Hence (C) is correct answer.


MCQ 1.54 The Boolean expression for the truth table shown is

(A) B (A + C)( A + C ) (B) B (A + C )( A + C)


(C) B (A + C )( A + C) (D) B (A + C)( A + C )
SOL 1.54 Hence (A) is correct answer.
We have f = ABC + ABC
= B (AC + AC ) = B (A + C)( A + C )
MCQ 1.55 Both transistors T1 and T2 show in the figure, have a β = 100 , threshold voltage of
1 Volts. The device parameters K1 and K2 of T1 and T2 are, respectively, 36 μA/V2
and 9 μA/V 2 . The output voltage Vo i s

(A) 1 V (B) 2 V
(C) 3 V (D) 4 V
SOL 1.55 Hence (D) is correct option.
MCQ 1.56 The present output Qn of an edge triggered JK flip-flop is logic 0. If J = 1, then
Qn + 1
(A) Cannot be determined (B) Will be logic 0
(C) will be logic 1 (D) will rave around
SOL 1.56 Characteristic equation for a jk flip-flop is written as
Qn + 1 = JQ n + K Qn
Where Qn is the present output
Qn + 1 is next output
So, Qn + 1 = 10 + K : 0 Qn = 0
Brought to you by: Nodia and Company Visit us at: www.nodia.co.in
PUBLISHING FOR GATE
Page 26 GATE EC 2005 www.gatehelp.com

Qn + 1 = 1
Hence (C) is correct answer.
MCQ 1.57 The given figure shows a ripple counter using positive edge triggered flip-flops. If
the present state of the counter is Q2 Q1 Q0 = 001 then is next state Q2 Q1 Q will be

(A) 010 (B) 111


(C) 100 (D) 101
SOL 1.57 Since T2 T1 T0 is at 111, at every clock Q2 Q1 Q0 will be changes. Ir present state is
011, the next state will be 100.
Hence (C) is correct answer.
MCQ 1.58 What memory address range is NOT represents by chip # 1 and chip # 2 in the
figure A0 to A15 in this figure are the address lines and CS means chip select.

(A) 0100 - 02FF (B) 1500 - 16FF


(C) F900 - FAFF (D) F800 - F9FF
SOL 1.58 Hence (D) is correct answer.
MCQ 1.59 The output y (t) of a linear time invariant system is related to its input x (t) by the
following equations
y (t)= 0.5x (t − td + T) + x (t − td ) + 0.5x (t − td + T)

Brought to you by: Nodia and Company Visit us at: www.nodia.co.in


PUBLISHING FOR GATE
Page 27 GATE EC 2005 www.gatehelp.com

The filter transfer function H (ω) of such a system is given by


(A) (1 + cos ωT) e−jωt d
(B) (1 + 0.5 cos ωT) e−jωt d

(C) (1 − cos ωT) e−jωt d


(D) (1 − 0.5 cos ωT) e−jωt
d

SOL 1.59 Hence (A) is correct answer.


y (t) = 0.5x (t − td + T) + x (t − td ) + 0.5x (t − td − T)
Taking Fourier transform we have
Y (ω) = 0.5e−jω (− t + T) X (ω) + e−jωt X (ω) + 0.5e−jω (− t − T) X (ω)
d d d

Y (ω)
or = e−jωt [0.5e jωT + 1 + 0.5e−jωT ]
d

X (ω)
= e−jωt [0.5 (e jωT + e−jωT ) + 1] = e−jωt [cos ωT + 1]
d d

Y (ω)
or H (ω) = = e−jωt (cos ωT + 1)
d

X (ω)

MCQ 1.60 Match the following and choose the correct combination.
Group 1
E. Continuous and aperiodic signal
F. Continuous and periodic signal
G. Discrete and aperiodic signal
H. Discrete and periodic signal
Group 2
1. Fourier representation is continuous and aperiodic
2. Fourier representation is discrete and aperiodic
3. Fourier representation is continuous and periodic
4. Fourier representation is discrete and periodic

(A) E − 3, F − 2, G − 4, H − 1
(B) E − 1, F − 3, G − 2, H − 4
(C) E − 1, F − 2, G − 3, H − 4
(D) E − 2, F − 1, G − 4, H − 3
SOL 1.60 For continuous and aperiodic signal Fourier representation is continuous and
aperiodic.
For continuous and periodic signal Fourier representation is discrete and aperiodic.
For discrete and aperiodic signal Fourier representation is continuous and periodic.
For discrete and periodic signal Fourier representation is discrete and periodic.
Hence (C) is correct answer.
MCQ 1.61 A signal x (n) = sin (ω0 n + φ) is the input to a linear time- invariant system having
a frequency response H (e jω). If the output of the system Ax (n − n0) then the most
general form of +H (e jω) will be

Brought to you by: Nodia and Company Visit us at: www.nodia.co.in


PUBLISHING FOR GATE
Page 28 GATE EC 2005 www.gatehelp.com

(A) − n0 ω0 + β for any arbitrary real


(B) − n0 ω0 + 2πk for any arbitrary integer k
(C) n0 ω0 + 2πk for any arbitrary integer k
(D) − n0 ω0 φ
SOL 1.61 Hence (B) is correct answer.
y (n) = Ax (n − no)
Taking Fourier transform
Y (e jω) = Ae−jω n X (e jω)
o o

Y (e jω) −jω n
or H (e jω) = jω = Ae
o o

X (e )

Thus +H (e ) =− ωo no
For LTI discrete time system phase and frequency of H (e jω) are periodic with
period 2π. So in general form
θ (ω) =− no ωo + 2πk
MCQ 1.62 For a signal x (t) the Fourier transform is X (f). Then the inverse Fourier transform
of X (3f + 2) is given by
j4πt
(A) 1 x` t j e j3πt (B) 1 x` t j e - 3
2 2 3 3
(C) 3x (3t) e−j4πt (D) x (3t + 2)
SOL 1.62 Hence (B) is correct answer.
F
x (t) X (f)
Using scaling we have
1 X f
a ca m
F
x (at)

Thus x b 1 f l
F
3X (3f)
3
Using shifting property we get
e−j2πf t x (t) = X (f + f0)
0

Thus 1 e−j 3 πt x b 1 t l
4 F
X (3f + 2)
3 3
e−j2π t x b 1 t l
F
3X (3 (f + 23 ))
2
3

b3 l
1 e−jπ t x 1 t
4
3
F
X [3 (f + 23 )]
3

MCQ 1.63 The polar diagram of a conditionally stable system for open loop gain K = 1 is
shown in the figure. The open loop transfer function of the system is known to be
stable. The closed loop system is stable for

Brought to you by: Nodia and Company Visit us at: www.nodia.co.in


PUBLISHING FOR GATE
Page 29 GATE EC 2005 www.gatehelp.com

(A) K < 5 and 1 < K < 1 (B) K < 1 and 1 < K < 5
2 8 8 2

(C) K < 1 and 5 < K (D) K > 1 and 5 > K


8 8
SOL 1.63 Hence (B) is correct option
MCQ 1.64 In the derivation of expression for peak percent overshoot
− πξ
Mp = exp e o # 100%
1 − ξ2
Which one of the following conditions is NOT required ?
(A) System is linear and time invariant
(B) The system transfer function has a pair of complex conjugate poles and no
zeroes.
(C) There is no transportation delay in the system.
(D) The system has zero initial conditions.
SOL 1.64 The peak percent overshoot is determined for LTI second order closed loop system
with zero initial condition. It’s transfer function is
T (s) = 2 ωn2
s + 2ξωn s + ωn2
Transfer function has a pair of complex conjugate poles and zeroes.
Hence (C) is correct option.
MCQ 1.65 Given the ideal operational amplifier circuit shown in the figure indicate the correct
transfer characteristics assuming ideal diodes with zero cut-in voltage.

Brought to you by: Nodia and Company Visit us at: www.nodia.co.in


PUBLISHING FOR GATE
Page 30 GATE EC 2005 www.gatehelp.com

SOL 1.65 Only one diode will be in ON conditions


When lower diode is in ON condition, then
Vu = 2k Vsat = 2 10 = 8 V
2.5k 2.5
when upper diode is in ON condition
Vu = 2k Vsat = 2 (− 10) =− 5 V
2.5k 4
Hence (B) is correct option.
MCQ 1.66 A ramp input applied to an unity feedback system results in 5% steady state error.
The type number and zero frequency gain of the system are respectively
(A) 1 and 20 (B) 0 and 20
(C) 0 and 1 (D) 1 and 1
20 20
SOL 1.66 For ramp input we have R (s) = 21
s
Now ess = lim sE (s)
s"0

Brought to you by: Nodia and Company Visit us at: www.nodia.co.in


PUBLISHING FOR GATE
Page 31 GATE EC 2005 www.gatehelp.com

R (s) 1
= lim s = lim
s"0 1 + G (s) s " 0 s + sG (s)
or ess = lim 1 = 5% = 1 Finite
s " 0 sG (s) 20
But kv = 1 = lim sG (s) = 20
ess s"0

kv is finite for type 1 system having ramp input.


Hence (A) is correct option.
MCQ 1.67 A double integrator plant G (s) = K/s2, H (s) = 1 is to be compensated to achieve
the damping ratio ζ = 0.5 and an undamped natural frequency, ωn = 5 rad/sec
which one of the following compensator Ge (s) will be suitable ?
(A) s + 3 (B) s + 99
s + 99 s+3

(C) s−6 (D) s − 6


s + 8.33 s
SOL 1.67 Hence (A) is correct option.
K (1 − s)
MCQ 1.68 An unity feedback system is given as G (s) = .
s (s + 3)
Indicate the correct root locus diagram.

SOL 1.68 Hence (A) is correct option.


MCQ 1.69 A MOS capacitor made using p type substrate is in the accumulation mode. The
dominant charge in the channel is due to the presence of
(A) holes (B) electrons
(C) positively charged icons (D) negatively charged ions
SOL 1.69 In accumulation mode for NMOS having p −substrate, when positive voltage is
applied at the gate, this will induce negative charge near p − type surface beneath
the gate. When VGS is made sufficiently large, an inversion of electrons is formed
and this in effect forms and n − channel.
Hence option (B) is correct.

Brought to you by: Nodia and Company Visit us at: www.nodia.co.in


PUBLISHING FOR GATE
Page 32 GATE EC 2005 www.gatehelp.com

MCQ 1.70 A device with input X (t) and output y (t) is characterized by: Y (t) = x2 (t). An FM
signal with frequency deviation of 90 kHz and modulating signal bandwidth of 5
kHz is applied to this device. The bandwidth of the output signal is
(A) 370 kHz (B) 190 kHz
(C) 380 kHz (D) 95 kHz
SOL 1.70 Let x (t) be the input signal where
x (t) = cos (cos t + β1 cos ωm t)
cos (2ωc t + 2β1 cos ωm t)
y (t) = x2 (t) = 1 +
2 2
3f
Here β = 2β1 and β1 = = 90 = 18
fm 5
BW = 2 (β + 1) fm = 2 (2 # 18 + 1) # 5 = 370 kHz
Hence (A) is correct option.
MCQ 1.71 A signal as shown in the figure is applied to a matched filter. Which of the following
does represent the output of this matched filter ?

SOL 1.71 The transfer function of matched filter is


h (t) = x (t − t) = x (2 − t)
The output of matched filter is the convolution of x (t) and h (t) as shown below

Brought to you by: Nodia and Company Visit us at: www.nodia.co.in


PUBLISHING FOR GATE
Page 33 GATE EC 2005 www.gatehelp.com

Hence (C) is correct option.


MCQ 1.72 Noise with uniform power spectral density of N0 W/Hz is passed though a filter
H (ω) = 2 exp (− jωtd ) followed by an ideal pass filter of bandwidth B Hz. The
output noise power in Watts is
(A) 2N0 B (B) 4N0 B
(C) 8N0 B (D) 16N0 B
SOL 1.72 Hence (B) is correct option.
We have H (f) = 2e - jωt
d

H (f) = 2
G0 (f) = H (f) 2 Gi (f)
= 4No W/Hz
The noise power is = 4No # B

MCQ 1.73 A carrier is phase modulated (PM) with frequency deviation of 10 kHz by a single
tone frequency of 1 kHz. If the single tone frequency is increased to 2 kHz, assuming
that phase deviation remains unchanged, the bandwidth of the PM signal is
(A) 21 kHz (B) 22 kHz
(C) 42 kHz (D) 44 kHz
SOL 1.73 The phase deviation is
3f
β = = 10 = 10
fm 1
If phase deviation remain same and modulating frequency is changed
BW = 2 (β + 1) fm' = 2 (10 + 1) 2 = 44 kHz
Hence (D) is correct option.
MCQ 1.74 An output of a communication channel is a random variable v with the probability
density function as shown in the figure. The mean square value of v is

Brought to you by: Nodia and Company Visit us at: www.nodia.co.in


PUBLISHING FOR GATE
Page 34 GATE EC 2005 www.gatehelp.com

(A) 4 (B) 6
(C) 8 (D) 9
SOL 1.74 As the area under pdf curve must be unity
1 (4 # k) = 1 $ k = 1
2 2
Now mean square value is
#- 3
+3
σv2 = v2 p (v) dv

#0
4
= v2 ` v j dv as p (v) = 1 v
8 8
v3
#0
4
= c 8 m dv = 8

Hence (C) is correct option.


MCQ 1.75 Which one of the following does represent the electric field lines for the mode in the
cross-section of a hollow rectangular metallic waveguide ?

SOL 1.75 Hence (D) is correct option.


MCQ 1.76 Characteristic impedance of a transmission line is 50 Ω. Input impedance of the
open-circuited line when the transmission line a short circuited, then value of the
input impedance will be.
(A) 50 Ω (B) 100 + j150Ω
(C) 7.69 + j11.54Ω (D) 7.69 − j11.54Ω
SOL 1.76 Hence (D) is correct option
Zo2 = ZOC .ZSC
2
ZZC = Zo = 50 # 50 = 50
ZOC 100 + j150 2 + 3j

Brought to you by: Nodia and Company Visit us at: www.nodia.co.in


PUBLISHING FOR GATE
Page 35 GATE EC 2005 www.gatehelp.com

50 (2 − 3j)
= = 7.69 − 11.54j
13
Hence (D) is correct option

MCQ 1.77 Two identical and parallel dipole antennas are kept apart by a distance of λ in the
4
H - plane. They are fed with equal currents but the right most antenna has a phase
shift of + 90c. The radiation pattern is given as.

SOL 1.77 The array factor is


βd sin θ + α
A = cos b l
2
Here β = 2π , d = λ and α = 90c
λ 4
2π λ
sin θ + π2 π π
Thus A = cos c λ 4 m = cos ` sin θ + j
2 4 2
The option (A) satisfy this equation.

Common Data Questions 78, 79 and 80 :


Given, rd = 20kΩ , IDSS = 10 mA, Vp =− 8 V

Brought to you by: Nodia and Company Visit us at: www.nodia.co.in


PUBLISHING FOR GATE
Page 36 GATE EC 2005 www.gatehelp.com

MCQ 1.78 Zi and Z0 of the circuit are respectively


(A) 2 MΩ and 2 kΩ (B) 2 MΩ and 20 kΩ
11

(C) infinity and 2 MΩ (D) infinity and 20 kΩ


11
SOL 1.78 The small signal model is as shown below

From the figure we have


Zin = 2 MΩ
and Z0 = rd RD = 20k 2k = 20 kΩ
11
Hence (B) is correct option.
MCQ 1.79 ID and VDS under DC conditions are respectively
(A) 5.625 mA and 8.75 V (B) 1.875 mA and 5.00 V
(C) 4.500 mA and 11.00 V (D) 6.250 mA and 7.50 V
SOL 1.79 The circuit in DC condition is shown below

Since the FET has high input resistance, gate current can be neglect and we get
VGS =− 2 V
Since VP < VGS < 0 , FET is operating in active region
(− 2) 2
ID = IDSS c1 − VGS m = 10 c1 −
(− 8) m
2
Now = 5.625 mA
VP
Now VDS = VDD − ID RD = 20 − 5.625 m # 2 k = 8.75 V
Hence (A) is correct option.
MCQ 1.80 Transconductance in milli-Siemens (mS) and voltage gain of the amplifier are

Brought to you by: Nodia and Company Visit us at: www.nodia.co.in


PUBLISHING FOR GATE
Page 37 GATE EC 2005 www.gatehelp.com

respectively
(A) 1.875 mS and 3.41 (B) 1.875 ms and -3.41
(C) 3.3 mS and -6 (D) 3.3 mS and 6
SOL 1.80 The transconductance is
gm = 2
VP ID IDSS
or, = 2 5.625mA # 10mA = 1.875 mS
8
The gain is A =− gm (rd RD)
So, = 1.875ms # 20 K =− 3.41
11
Hence (B) is correct option.

Linked Answer Questions : Q. 81 to 90 Carry Two Marks Each


Consider an 8085 microprocessor system.
MCQ 1.81 The following program starts at location 0100H.
LXI SP, OOFF
LXI H, 0701
MVI A, 20H
SUB M
The content of accumulator when the program counter reaches 0109 H is
(A) 20 H (B) 02 H
(C) 00 H (D) FF H
SOL 1.81 0100H LXI SP, 00FF ; Load SP with 00FFG
0103H LXI H, 0701 ; Load HL with 0107H
0106H MVI A, 20H ; Move A with 20 H
0108 H SUB M ; Subtract the contents of memory
; location whose address is stored in HL
; from the A and store in A
0109H ORI 40H ; 40H OR [A] and store in A
010BH ADD M ; Add the contents of memeory location
; whose address is stored in HL to A
; and store in A
HL contains 0107H and contents of 0107H is 20H
Thus after execution of SUB the data of A is 20H - 20H = 00
Hence (C) is correct answer.
MCQ 1.82 If in addition following code exists from 019H onwards,
ORI 40 H
ADD M

Brought to you by: Nodia and Company Visit us at: www.nodia.co.in


PUBLISHING FOR GATE
Page 38 GATE EC 2005 www.gatehelp.com

What will be the result in the accumulator after the last instruction is executed ?
(A) 40 H (B) 20 H
(C) 60 H (D) 42 H
SOL 1.82 Before ORI instruction the contents of A is 00H. On execution the ORI 40H the
contents of A will be 40H
00H = 00000000
40H = 01000000
ORI 01000000
After ADD instruction the contents of memory location whose address is stored in
HL will be added to and will be stored in A
40H + 20 H = 60 H
Hence (C) is correct answer.

Statement for Linked Answer Question 83 and 84 :


The open loop transfer function of a unity feedback system is given by
−2s
G (s) = 3e
s (s + 2)
MCQ 1.83 The gain and phase crossover frequencies in rad/sec are, respectively
(A) 0.632 and 1.26 (B) 0.632 and 0.485
(C) 0.485 and 0.632 (D) 1.26 and 0.632
SOL 1.83 Hence (D) is correct option
−2s
G (s) = 3e
s (s + 2)
or G (jω) = 3e−2jω
jω (jω + 2)
G (jω) = 3
ω ω2 + 4
Let at frequency ωg the gain is 1. Thus
3 =1
ωg (ωg2 + 4)
or ωg4 + 4ωg2 − 9 = 0
or ωg2 = 1.606
or ωg = 1.26 rad/sec
Now +G (jω) =− 2ω − π − tan−1 ω
2 2
Let at frequency ωφ we have +GH =− 180c
ω
− π =− 2ωφ − π − tan−1 φ
2 2
ω
or 2ωφ + tan−1 φ = π
2 2

Brought to you by: Nodia and Company Visit us at: www.nodia.co.in


PUBLISHING FOR GATE
Page 39 GATE EC 2005 www.gatehelp.com

ωφ 1 ωφ 3
or 2ωφ + c − ` jm = π
2 3 2 2
5ωφ ωφ3
or − =π
2 24 2

5ωφ
2 2
or ωφ = 0.63 rad/sec
MCQ 1.84 Based on the above results, the gain and phase margins of the system will be
(A) −7.09 dB and 87.5c (B) 7.09 dB and 87.5c
(C) 7.09 dB and − 87.5c (D) − 7.09 and − 87.5c
SOL 1.84 The gain at phase crossover frequency ωφ is
G (jωg) = 3 = 3
1

ωφ (ωφ + 4)
2
0.63 (0.632 + 4) 2

or G (jωg) = 2.27
G.M. =− 20 log G (jωg)
− 20 log 2.26 =− 7.08 dB
Since G.M. is negative system is unstable.
The phase at gain corss over frequency is
ω
+G (jωg) =− 2ωg − π − tan−1 g
2 2
=− 2 # 1.26 − π − tan−1 1.26
2 2
or =− 4.65 rad or − 266.5c
PM = 180c + +G (jωg) = 180c − 266.5c =− 86.5c
Hence (D) is correct option.

Common Data for Question 85 and 86 :


Asymmetric three-level midtread quantizer is to be designed assuming equiprobable
occurrence of all quantization levels.

MCQ 1.85 If the probability density function is divide into three regions as shown in the
figure, the value of a in the figure is
(A) 1 (B) 2
3 3
Brought to you by: Nodia and Company Visit us at: www.nodia.co.in
PUBLISHING FOR GATE
Page 40 GATE EC 2005 www.gatehelp.com

(C) 1 (D) 1
2 4
SOL 1.85 As the area under pdf curve must be unity and all three region are equivaprobable.
Thus are under each region must be 13 .
2a # 1 = 1 $ a = 2
4 3 3
Hence (B) is correct option.
MCQ 1.86 The quantization noise power for the quantization region between − a and + a in
the figure is
(A) 4 (B) 1
81 9

(C) 5 (D) 2
81 81
SOL 1.86 Hence (A) is correct option.
3 a 3
#- a #0
+a
x $ 1 dx = 1 ; x E = a
a 2
Nq = x2 p (x) dx = 2
4 2 3 0 6
Substituting a = 2 we have
3
Nq = 4
81

Statement of Linked Answer Questions 87 & 88 :


Voltage standing wave pattern in a lossless transmission line with characteristic
impedance 50 and a resistive load is shown in the figure.

MCQ 1.87 The value of the load resistance is


(A) 50 Ω (B) 200 Ω
(C) 12.5 Ω (D) 0
SOL 1.87 From the diagram, VSWR is
s = Vmax = 4 = 4
Vmin 1
When minima is at load ZO = s.ZL
or ZL = Zo = 50 = 12.5Ω
s 4

Brought to you by: Nodia and Company Visit us at: www.nodia.co.in


PUBLISHING FOR GATE
Page 41 GATE EC 2005 www.gatehelp.com

Hence (C) is correct option.


MCQ 1.88 reflection coefficient is given by
(A)− 0.6 (B) − 1
(C) 0.6 (D) 0
SOL 1.88 The reflection coefficient is
Γ = ZL − ZO = 12.5 − 50 =− 0.6
ZL + ZO 125. + 50
Hence (A) is correct option.

Statement of linked answer question 89 and89:


A sequence x (n) has non-zero values as shown in the figure.

x ( n2 − 1), For n even


MCQ 1.89 The sequence y (n) = * will be
0, For n odd

Brought to you by: Nodia and Company Visit us at: www.nodia.co.in


PUBLISHING FOR GATE
Page 42 GATE EC 2005 www.gatehelp.com

SOL 1.89 Thus (A) is correct option.


From x (n) = [ 12 , 1, 2, 1, 1, 12 ]
y (n) = x ^ n2 − 1h, n even
= 0 , for n odd
n =− 2 , y (− 2) = x ( −22 − 1) = x (− 2) = 12
n =− 1, y (− 1) = 0
n = 0, y (0) = x ( 20 − 1) = x (− 1) = 1
n = 1, y (1) = 0
n=2 y (2) = x ( 22 − 1) = x (0) = 2
n = 3, y (3) = 0
n=4 y (4) = x ( 24 − 1) = x (1) = 1
n = 5, y (5) = 0
n=6 y (6) = x ( 26 − 1) = x (2) = 12
Hence y (n) = 1 δ (n + 2) + δ (n) + 2δ (n − 2) + δ (n − 4)+ 1 δ (n − 6)
2 2
MCQ 1.90 The Fourier transform of y (2n) will be
(A) e−j2ω [cos 4ω + 2 cos 2ω + 2] (B) cos 2ω + 2 cos ω + 2

Brought to you by: Nodia and Company Visit us at: www.nodia.co.in


PUBLISHING FOR GATE
Page 43 GATE EC 2005 www.gatehelp.com

(C) e−jω [cos 2ω + 2 cos ω + 2] (D) e−j2ω [cos 2ω + 2 cos + 2]


SOL 1.90 Here y (n) is scaled and shifted version of x (n) and again y (2n) is scaled version of
y (n) giving
z (n) = y (2n) = x (n − 1)
= 1 δ (n + 1) + δ (n) + 2δ (n − 1) + δ (n − 2) + 1 δ (n − 3)
2 2
Taking Fourier transform.
Z (e jω) = 1 e jω + 1 + 2e−jω + e−2jω + 1 e−3jω
2 2
= e−jω b 1 e2jω + e jω + 2 + e−jω + 1 e−2jω l
2 2
2jω
= e−jω b e + e−2jω + e jω + 2 + e−jω
2 l

or Z (e jω) = e−jω [cos 2ω + 2 cos ω + 2]


Hence (C) is correct answer

Answer Sheet
1. (B) 19. (A) 37. (C) 55. (D) 73. (D)
2. (C) 20. (D) 38. (A) 56. (C) 74. (C)
3. (D) 21. (D) 39. (B) 57. (C) 75. (D)
4. (B) 22. (D) 40. (B) 58. (D) 76. (D)
5. (A) 23. (A) 41. (C) 59. (A) 77. (A)
6. (C) 24. (C) 42. (D) 60. (C) 78. (B)
7. (C) 25. (C) 43. (B) 61. (B) 79. (A)
8. (B) 26. (D) 44. (B) 62. (B) 80. (B)
9. (B) 27. (A) 45. (B) 63. (B) 81. (C)
10. (C) 28. (C) 46. (C) 64. (C) 82. (C)
11. (C) 29. (C) 47. (C) 65. (B) 83. (D)
12. (B) 30. (C) 48. (C) 66. (A) 84. (D)
13. (A) 31. (A) 49. (D) 67. (A) 85. (B)
14. (D) 32. (C) 50. (C) 68. (A) 86. (A)
15. (B) 33. (A) 51. (B) 69. (B) 87. (C)
16. (C) 34. (A) 52. (A) 70 (A) 88. (A)
17. (B) 35. (C) 53. (C) 71 (C) 89. (A)
18. (B) 36. (C) 54. (A) 72 (B) 90. (C)

Brought to you by: Nodia and Company Visit us at: www.nodia.co.in


PUBLISHING FOR GATE
GATE EC
2007

Q.1 to Q.20 carry one mark each

MCQ 1.1 If E denotes expectation, the variance of a random variable X is given by


(A) E [X2] − E2 [X] (B) E [X2] + E2 [X]
(C) E [X2] (D) E2 [X]
SOL 1.1 The variance of a random variable x is given by
E [X2] − E2 [X]
Hence (A) is correct option.
MCQ 1.2 The following plot shows a function which varies linearly with x . The value of the
#1 ydx
2
integral I = is

(A) 1.0 (B) 2.5


(C) 4.0 (D) 5.0
SOL 1.2 The given plot is straight line whose equation is
x +y =1
−1 1
or y = x+1
#1 ydx #1 (x + 1) dx
2 2
Now I = =
(x + 1) 2 2
=; E = − = 2.5
9 4
2 2 2
Hence (B) is correct answer.
MCQ 1.3 For x << 1, coth (x) can be approximated as

Brought to you by: Nodia and Company Visit us at: www.nodia.co.in


PUBLISHING FOR GATE
Page 2 GATE EC 2007 www.gatehelp.com

(A) x (B) x2
(C) 1 (D) 12
x x
SOL 1.3 Hence (C) is correct answer.
coth x = cosh x
sinh x
as x << 1, cosh x . 1 and sinh x . x
Thus coth x . 1
x

sin b θ l
2
MCQ 1.4 lim is
θ"0 θ
(A) 0.5 (B) 1
(C) 2 (D) not defined
SOL 1.4 Hence (A) is correct answer.
sin ^ θ2 h sin ^ θ2 h 1 lim sin ^ 2 h = 1 = 0.5
θ
lim = lim =
θ"0 θ θ " 0 2^ θ h 2 θ " 0 ^ θ2 h 2
2

MCQ 1.5 Which one of following functions is strictly bounded?


(A) 1/x2 (B) ex
2

(C) x2 (D) e - x
SOL 1.5 Hence (D) is correct answer.
We have, lim 12 = 3
x"0 x

lim x2 = 3
x"3

lim e - x = 3
x"3

lim e - x = 0
2

x"3

lim e - x = 1
2 2

Thus e - x is strictly bounded.


x"0

MCQ 1.6 For the function e - x , the linear approximation around x = 2 is


(A) (3 − x) e - 2 (B) 1 − x
(C) 63 + 3 2 − (1 − 2 ) x @e - 2 (D) e - 2
SOL 1.6 Hence (A) is correct answer.
We have f (x) = e - x = e - (x - 2) - 2 = e - (x - 2) e - 2
(x − 2) 2
= ;1 − (x − 2) + ...E e - 2
2!
= 61 − (x − 2)@ e - 2 Neglecting higher powers

Brought to you by: Nodia and Company Visit us at: www.nodia.co.in


PUBLISHING FOR GATE
Page 3 GATE EC 2007 www.gatehelp.com

= (3 − x) e - 2
MCQ 1.7 An independent voltage source in series with an impedance Zs = Rs + jXs delivers
a maximum average power to a load impedance ZL when
(A) ZL = Rs + jXs (B) ZL = Rs
(C) ZL = jXs (D) ZL = Rs − jXs
SOL 1.7 According to maximum Power Transform Theorem
ZL = Zs* = (Rs − jXs)
Hence (D) is correct option.
MCQ 1.8 The RC circuit shown in the figure is

(A) a low-pass filter (B) a high-pass filter


(C) a band-pass filter (D) a band-reject filter
SOL 1.8 At ω " 3 , capacitor acts as short circuited and circuit acts as shown in fig below

Here we get V0 = 0
Vi

At ω " 0 , capacitor acts as open circuited and circuit look like as shown in fig
below

Here we get also V0 = 0


Vi
So frequency response of the circuit is as shown in fig and circuit is a Band pass
filter.

Brought to you by: Nodia and Company Visit us at: www.nodia.co.in


PUBLISHING FOR GATE
Page 4 GATE EC 2007 www.gatehelp.com

Hence (C) is correct option.


MCQ 1.9 The electron and hole concentrations in an intrinsic semiconductor are ni per cm3
at 300 K. Now, if acceptor impurities are introduced with a concentration of NA per
cm3 (where NA >> ni , the electron concentration per cm3 at 300 K will be)
(A) ni (B) ni + NA
2
(C) NA − ni (D) ni
NA
SOL 1.9 As per mass action law
np = ni2
If acceptor impurities are introduces
p = NA
Thus nNA = ni2
2
or n = ni
NA
Hence option (D) is correct.
MCQ 1.10 In a p+ n junction diode under reverse biased the magnitude of electric field is
maximum at
(A) the edge of the depletion region on the p-side
(B) the edge of the depletion region on the n -side
(C) the p+ n junction
(D) the centre of the depletion region on the n -side
SOL 1.10 The electric field has the maximum value at the junction of p+ n .
Hence option (C) is correct.
MCQ 1.11 The correct full wave rectifier circuit is

Brought to you by: Nodia and Company Visit us at: www.nodia.co.in


PUBLISHING FOR GATE
Page 5 GATE EC 2007 www.gatehelp.com

SOL 1.11 The circuit shown in (C) is correct full wave rectifier circuit.

Hence (C) is correct option.


MCQ 1.12 In a transconductance amplifier, it is desirable to have
(A) a large input resistance and a large output resistance
(B) a large input resistance and a small output resistance
(C) a small input resistance and a large output resistance
(D) a small input resistance and a small output resistance
SOL 1.12 In the transconductance amplifier it is desirable to have large input resistance and
large output resistance.
Hence (A) is correct option.
MCQ 1.13 X = 01110 and Y = 11001 are two 5-bit binary numbers represented in two’s
complement format. The sum of X and Y represented in two’s complement format
using 6 bits is
(A) 100111 (B) 0010000
(C) 000111 (D) 101001
SOL 1.13 MSB of Y is 1, thus it is negative number and X is positive number
Now we have X = 01110 = (14) 10
and Y = 11001 = (− 7) 10
X + Y = (14) + (− 7) = 7
In signed two’s complements from 7 is
(7) 10 = 000111
Brought to you by: Nodia and Company Visit us at: www.nodia.co.in
PUBLISHING FOR GATE
Page 6 GATE EC 2007 www.gatehelp.com

Hence (C) is correct answer.


MCQ 1.14 The Boolean function Y = AB + CD is to be realized using only 2 - input NAND
gates. The minimum number of gates required is
(A) 2 (B) 3
(C) 4 (D) 5
SOL 1.14 Hence (B) is correct answer.
Y = AB + CD = AB .CD
This is SOP form and we require only 3 NAND gate

MCQ 1.15 If the closed-loop transfer function of a control system is given as T (s) s−5
(s + 2)( s + 3)
, then It is
(A) an unstable system (B) an uncontrollable system
(C) a minimum phase system (D) a non-minimum phase system
SOL 1.15 In a minimum phase system, all the poles as well as zeros are on the left half of the
s −plane. In given system as there is right half zero (s = 5), the system is a non-
minimum phase system.
Hence (D) is correct option.

MCQ 1.16 If the Laplace transform of a signal Y (s) = 1 , then its final value is
s (s − 1)
(A) − 1 (B) 0
(C) 1 (D) Unbounded
SOL 1.16 Hence (D) is correct answer.
Y (s) = 1
s (s − 1)
Final value theorem is applicable only when all poles of system lies in left half of S
-plane. Here s = 1 is right s −plane pole. Thus it is unbounded.
MCQ 1.17 If R (τ) is the auto correlation function of a real, wide-sense stationary random
process, then which of the following is NOT true
(A) R (τ) = R (− τ)
(B) R (τ) # R (0)
(C) R (τ) =− R (− τ)
(D) The mean square value of the process is R (0)
SOL 1.17 Autocorrelation is even function.
Hence (C) is correct option
MCQ 1.18 If S (f) is the power spectral density of a real, wide-sense stationary random process,
then which of the following is ALWAYS true?
(A) S (0) # S (f) (B) S (f) $ 0

Brought to you by: Nodia and Company Visit us at: www.nodia.co.in


PUBLISHING FOR GATE
Page 7 GATE EC 2007 www.gatehelp.com

#- 3 S (f) df = 0
3
(C) S (− f) =− S (f) (D)
SOL 1.18 Power spectral density is non negative. Thus it is always zero or greater than zero.
Hence (B) is correct option.
MCQ 1.19 A plane wave of wavelength λ is traveling in a direction making an angle 30c with
positive x − axis and 90c with positive y − axis. The E field of the plane wave can
be represented as (E0 is constant)
3 π x− π z π 3πz
t 0 e j c ωt −
(A) E = yE λ λ m t 0 e jc ωt − λ x −
(B) E = yE λ m

3 π x+ π z π 3πz
t 0 e jc ωt +
(C) E = yE λ λ m t 0 e jc ωt − λ x +
(D) E = yE λ m

SOL 1.19 Hence (A) is correct option.


γ = β cos 30cx ! β sin 30cy
= 2π 3 x ! 2π 1 y
λ 2 λ 2
= π 3 x! πy
λ λ
π 3 x! π y
E = ay E0 e j (ωt - γ) = ay E0 e j;ωt - c λ λ mE

MCQ 1.20 If C is code curve enclosing a surface S , then magnetic field intensity H , the
current density j and the electric flux density D are related by
(A) ##
S
H $ ds =
c
##
c j + 2t m $ d t
2D (B) H $ d l =
S S
#
c j + 2t m $ dS
2D
##
(C) ##S H $ dS = #C c j + 22Dt m $ d t (D) #C H $ d l # = ##S c j + 22Dt m $ ds
c

SOL 1.20 Hence (D) is correct option.


4# H = J + 2D Maxwell Equations
2t
## 4# H $ ds = ##
`J + 2t j .ds
2D Integral form
s s
#
H $ dl = ##
`J + 2t j .ds
2D Stokes Theorem
s

Q.21 to Q.75 carry two marks each.

MCQ 1.21 It is given that X1, X2 ...XM at M non-zero, orthogonal vectors. The dimension of
the vector space spanned by the 2M vectors X1, X2,... XM , − X1, − X2,... − XM is
(A) 2M (B) M + 1
(C) M
(D) dependent on the choice of X1, X2,... XM
SOL 1.21 For two orthogonal vectors, we require two dimensions to define them and similarly

Brought to you by: Nodia and Company Visit us at: www.nodia.co.in


PUBLISHING FOR GATE
Page 8 GATE EC 2007 www.gatehelp.com

for three orthogonal vector we require three dimensions to define them. 2M vectors
are basically M orthogonal vector and we require M dimensions to define them.
Hence (C) is correct answer.
MCQ 1.22 Consider the function f (x) = x2 − x − 2 . The maximum value of f (x) in the closed
interval [− 4, 4] is
(A) 18 (B) 10
(C) − 225 (D) indeterminate
SOL 1.22 We have
f (x) = x2 − x + 2
f'( x) = 2x − 1 = 0 " x = 1
2
f"( x) = 2
Since f"( x) = 2 > 0 , thus x = 1 is minimum point. The maximum value in closed
2
interval 6− 4, 4@ will be at x =− 4 or x = 4
Now maximum value
= max [f (− 4), f (4)]
= max (18, 10)
= 18
Hence (A) is correct answer.
MCQ 1.23 An examination consists of two papers, Paper 1 and Paper 2. The probability of
failing in Paper 1 is 0.3 and that in Paper 2 is 0.2. Given that a student has failed
in Paper 2, the probability of failing in Paper 1 is 0.6. The probability of a student
failing in both the papers is
(A) 0.5 (B) 0.18
(C) 0.12 (D) 0.06
SOL 1.23 Hence (C) is correct answer.
Probability of failing in paper 1 is P (A) = 0.3
Possibility of failing in Paper 2 is P (B) = 0.2
Probability of failing in paper 1, when
student has failed in paper 2 is P ^ BA h = 0.6
We know that
(P + B)
Pb A l =
B P (B)
or P (A + B) = P (B) P b A l = 0.6 # 0.2 = 0.12
B

d2 y
MCQ 1.24 The solution of the differential equation k2 2 = y − y2 under the boundary
dx
conditions
(i) y = y1 at x = 0 and

Brought to you by: Nodia and Company Visit us at: www.nodia.co.in


PUBLISHING FOR GATE
Page 9 GATE EC 2007 www.gatehelp.com

(ii) y = y2 at x = 3 , where k, y1 and y2 are constants, is

(A) y = (y1 − y2) exp a− x2 k + y2 (B) y = (y2 − y1) exp a− x k + y1


k k

(C) y = ^y1 − y2h sinh a x k + y1 (D) y = ^y1 − y2h exp a− x k + y2


k k
SOL 1.24 Hence (D) is correct answer.
d2 y
We have k2 2 = y − y2
dx
2
d y y y
or 2
− 2 =− 22
dx k k
A.E. D − 2 =0
2 1
k
or D =! 1
k
x x

C.F. = C1 e - + C2 e
k k

− y22
c 2 m = y2
P.I. = 1
D2 − 12 k
Thus solution is k
x x

y = C1 e - + C2 e + y2
k k

From y (0) = y1 we get


C1 + C2 = y1 − y2
From y (3) = y2 we get that C1 must be zero.
Thus C2 = y1 − y2
x

y = (y1 − y2) e - + y2
k

MCQ 1.25 The equation x3 − x2 + 4x − 4 = 0 is to be solved using the Newton - Raphson


method. If x = 2 is taken as the initial approximation of the solution, then next
approximation using this method will be
(A) 2/3 (B) 4/3
(C) 1 (D) 3/2
SOL 1.25 We have
f (x) = x3 − x2 + 4x − 4
f'( x) = 3x2 − 2x + 4
Taking x0 = 2 in Newton-Raphosn method
f (x0) 23 − 22 + 4 (2) − 4
x1 = x0 − = 2− =4
f'( x0) 3 (2) − 2 (2) + 4
2 3
Hence (B) is correct answer.
MCQ 1.26 Three functions f1 (t), f2 (t) and f3 (t) which are zero outside the interval [0, T] are
shown in the figure. Which of the following statements is correct?

Brought to you by: Nodia and Company Visit us at: www.nodia.co.in


PUBLISHING FOR GATE
Page 10 GATE EC 2007 www.gatehelp.com

(A) f1 (t) and f2 (t) are orthogonal (B) f1 (t) and f3 (t) are orthogonal
(C) f2 (t) and f3 (t) are orthogonal (D) f1 (t) and f2 (t) are orthonormal
SOL 1.26 For two orthogonal signal f (x) and g (x)
#- 3
+3
f (x) g (x) dx =0

i.e. common area between f (x) and g (x) is zero.


Hence (C) is correct options.
MCQ 1.27 If the semi-circular control D of radius 2 is as shown in the figure, then the value
of the integral 1
(s − 1)
2 #
ds is
D

(A) jπ (B) − jπ
(C) − π (D) π
SOL 1.27 We know that

s
#
2
1 ds = 2πj
− 1
[sum of residues]
D
Singular points are at s = ! 1 but only s =+ 1 lies inside the given contour, Thus
Residue at s =+ 1 is

Brought to you by: Nodia and Company Visit us at: www.nodia.co.in


PUBLISHING FOR GATE
Page 11 GATE EC 2007 www.gatehelp.com

lim (s − 1) f (s) = lim (s − 1) 1 =1


s"1 s"1 s2 − 1 2
# s2 −1 1 ds = 2πj` 12 j = πj
D
Hence (A) is correct answer.
MCQ 1.28 Two series resonant filters are as shown in the figure. Let the 3-dB bandwidth of
Filter 1 be B1 and that of Filter 2 be B2 . the value B1 is
B2

(A) 4 (B) 1
(C) 1/2 (D) 1/4

SOL 1.28 We know that bandwidth of series RLC circuit is R . Therefore


L
Bandwidth of filter 1 is B1 = R
L1
Bandwidth of filter 2 is B2 = R = R = 4R
L2 L1 /4 L1
Dividing above equation B1 = 1
B2 4
Hence (D) is correct option.
MCQ 1.29 For the circuit shown in the figure, the Thevenin voltage and resistance looking
into X − Y are

(A) 4
3 V, 2 Ω (B) 4 V, 23 Ω
(C) 4
3 V, 23 Ω (D) 4 V, 2 Ω
SOL 1.29 Here Vth is voltage across node also. Applying nodal analysis we get

Brought to you by: Nodia and Company Visit us at: www.nodia.co.in


PUBLISHING FOR GATE
Page 12 GATE EC 2007 www.gatehelp.com

Vth + Vth + Vth − 2i = 2


2 1 1
But from circuit i = Vth = Vth
1
Therefore
Vth + Vth + Vth − 2Vth = 2
2 1 1
or Vth = 4 volt
From the figure shown below it may be easily seen that the short circuit current at
terminal XY is isc = 2 A because i = 0 due to short circuit of 1 Ω resistor and all
current will pass through short circuit.

Therefore Rth = Vth = 4 = 2 Ω


isc 2
Hence (D) is correct option.
MCQ 1.30 In the circuit shown, vC is 0 volts at t = 0 sec. For t > 0 , the capacitor current iC (t)
, where t is in seconds is given by

(A) 0.50 exp (− 25t) mA (B) 0.25 exp (− 25t) mA


(C) 0.50 exp (− 12.5t) mA (D) 0.25 exp (− 6.25t) mA
SOL 1.30 The voltage across capacitor is
At t = 0+ , Vc (0+) = 0
At t = 3 , VC (3) = 5 V
The equivalent resistance seen by capacitor as shown in fig is
Req = 20 20 = 10kΩ

Brought to you by: Nodia and Company Visit us at: www.nodia.co.in


PUBLISHING FOR GATE
GATE IES PSU’S-2021
MADE EASY
Leading inStitUte for eSe/gate/PSU’S

ECE ENGINEERING
NOTED-: These Are Only Sample Notes,
If You Want Complete Full MADE EASY
GATE IES PSU ECE Branch ALL SUBJECT
FULL NOTES BUY Click below Link Buy

Noted-: Download File Zip format so use zip rar app extract zip
file easily open folder available all subject pdf.
Page 13 GATE EC 2007 www.gatehelp.com

Time constant of the circuit is


τ = Req C = 10k # 4μ = 0.04 s
Using direct formula
Vc (t) = VC (3) − [Vc (3) − Vc (0)] e−t/τ
= VC (3) (1 − e−t/τ) + VC (0) e−t/τ = 5 (1 − e−t/0.04)
or Vc (t) = 5 (1 − e−25t)
dV (t)
Now IC (t) = C C
dt
= 4 # 10−6 # (− 5 # 25e−25t) = 0.5e−25t mA
Hence (A) is correct option.
MCQ 1.31 In the ac network shown in the figure, the phasor voltage VAB (in Volts) is

(A) 0 (B) 5+30c


(C) 12.5+30c (D) 17+30c
SOL 1.31 Hence (D) is correct option.
(5 − 3j) # (5 + 3j)
Impedance = (5 − 3j) (5 + 3j) =
5 − 3j + 5 + 3j
(5) 2 − (3j) 2
= = 25 + 9 = 3.4
10 10
VAB = Current # Impedance = 5+30c # 34 = 17+30c
MCQ 1.32 A p+ n junction has a built-in potential of 0.8 V. The depletion layer width a reverse
bias of 1.2 V is 2 μm. For a reverse bias of 7.2 V, the depletion layer width will be
(A) 4 μm (B) 4.9 μm
(C) 8 μm (D) 12 μm
SOL 1.32 Hence option (A) is correct.
W = K V + VR
Now 2μ = K 0.8 + 1.2
From above two equation we get

Brought to you by: Nodia and Company Visit us at: www.nodia.co.in


PUBLISHING FOR GATE
Page 14 GATE EC 2007 www.gatehelp.com

W = 0.8 + 7.2 = 8 =2
2μ 0.8 + 1.2 2
or W2 = 4 μ m
MCQ 1.33 Group I lists four types of p − n junction diodes. Match each device in Group I
with one of the option in Group II to indicate the bias condition of the device in
its normal mode of operation.
Group - I Group-II
(P) Zener Diode (1) Forward bias
(Q) Solar cell (2) Reverse bias
(R) LASER diode
(S) Avalanche Photodiode

(A) P - 1, Q - 2, R - 1, S - 2 (B) P - 2, Q - 1, R - 1, S - 2
(C) P - 2, Q - 2, R - 1, S- -2 (D) P - 2, Q - 1, R - 2, S - 2
SOL 1.33 Zener diode and Avalanche diode works in the reverse bias and laser diode works
in forward bias.
In solar cell diode works in forward bias but photo current is in reverse direction.
Thus
Zener diode : Reverse Bias
Solar Cell : Forward Bias
Laser Diode : Forward Bias
Avalanche Photo diode : Reverse Bias
Hence option (B) is correct.
MCQ 1.34 The DC current gain (β) of a BJT is 50. Assuming that the emitter injection
efficiency is 0.995, the base transport factor is
(A) 0.980 (B) 0.985
(C) 0.990 (D) 0.995
SOL 1.34 Hence option (B) is correct.
β
α= = 50 = 50
β + 1 50 + 1 51
Current Gain = Base Transport Factor # Emitter injection Efficiency
α = β1 # β2
or β1 = α = 50 = 0.985
β2 51 # 0.995

MCQ 1.35 Group I lists four different semiconductor devices. match each device in Group I
with its charactecteristic property in Group II
Group-I Group-II
(P) BJT (1) Population iniversion
(Q) MOS capacitor (2) Pinch-off voltage

Brought to you by: Nodia and Company Visit us at: www.nodia.co.in


PUBLISHING FOR GATE
Page 15 GATE EC 2007 www.gatehelp.com

(R) LASER diode (3) Early effect


(S) JFET (4) Flat-band voltage

(A) P - 3, Q - 1, R - 4, S - 2 (B) P - 1, Q - 4, R - 3, S - 2
(C) P - 3, Q - 4, R - 1, S - 2 (D) P - 3, Q - 2, R - 1, S - 4
SOL 1.35 In BJT as the B-C reverse bias voltage increases, the B-C space charge region width
increases which xB (i.e. neutral base width) > A change in neutral base width will
change the collector current. A reduction in base width will causes the gradient in
minority carrier concentration to increase, which in turn causes an increased in the
diffusion current. This effect si known as base modulation as early effect.
In JFET the gate to source voltage that must be applied to achieve pinch off
voltage is described as pinch off voltage and is also called as turn voltage or
threshold voltage.
In LASER population inversion occurs on the condition when concentration of
electrons in one energy state is greater than that in lower energy state, i.e. a non
equilibrium condition.
In MOS capacitor, flat band voltage is the gate voltage that must be applied to
create flat ban condition in which there is no space charge region in semiconductor
under oxide.
Therefore
BJT : Early effect
MOS capacitor: Flat-band voltage
LASER diode : Population inversion
JFET : Pinch-off voltage
Hence option (C) is correct.
MCQ 1.36 For the Op-Amp circuit shown in the figure, V0 is

(A) -2 V (B) -1 V
(C) -0.5 V (D) 0.5 V
SOL 1.36 We redraw the circuit as shown in fig.

Brought to you by: Nodia and Company Visit us at: www.nodia.co.in


PUBLISHING FOR GATE
Page 16 GATE EC 2007 www.gatehelp.com

Applying voltage division rule


v+ = 0.5 V
We know that v+ = v-
Thus v- = 0.5 V
Now i = 1 − 0.5 = 0.5 mA
1k
and i = 0.5 − v0 = 0.5 mA
2k
or v0 = 0.5 − 1 =− 0.5 V
Hence (C) is correct option.
MCQ 1.37 For the BJT circuit shown, assume that the β of the transistor is very large and
VBE = 0.7 V. The mode of operation of the BJT is

(A) cut-off (B) saturation


(C) normal active (D) reverse active
SOL 1.37 If we assume β very large, then IB = 0 and IE = IC ; VBE = 0.7 V. We assume that
BJT is in active, so applying KVL in Base-emitter loop
IE = 2 − VBE = 2 − 0.7 = 1.3 mA
RE 1k
Since β is very large, we have IE = IC , thus
IC = 1.3 mA
Now applying KVL in collector-emitter loop
10 − 10IC − VCE − IC =0
or VCE =− 4.3 V
Now VBC = VBE − VCE
= 0.7 − (− 4.3) = 5 V
Since VBC > 0.7 V, thus transistor in saturation.
Hence (B) is correct option

Brought to you by: Nodia and Company Visit us at: www.nodia.co.in


PUBLISHING FOR GATE
Page 17 GATE EC 2007 www.gatehelp.com

MCQ 1.38 In the Op-Amp circuit shown, assume that the diode current follows the equation
I = Is exp (V/VT ). For Vi = 2V, V0 = V01, and for Vi = 4V, V0 = V02 .
The relationship between V01 and V02 is

(A) V02 = 2 Vo1 (B) Vo2 = e2 Vo1


(C) Vo2 = Vo1 1n2 (D) Vo1 − Vo2 = VT 1n2
SOL 1.38 Here the inverting terminal is at virtual ground and the current in resistor and
diode current is equal i.e.
IR = ID
or Vi = I eV /V
D T
s
R
or VD = VT 1n Vi
Is R
For the first condition
VD = 0 − Vo1 = VT 1n 2
Is R
For the first condition
VD = 0 − Vo1 = VT 1n 4
Is R
Subtracting above equation
Vo1 − Vo2 = VT 1n 4 − VT 1n 2
Is R Is R
or Vo1 − Vo2 = VT 1n 4 = VT 1n2
2
Hence (D) is correct option.
MCQ 1.39 In the CMOS inverter circuit shown, if the trans conductance parameters of the
NMOS and PMOS transistors are
W
kn = kp = μn Cox Wn = μCox p = 40μA/V2
Ln Lp
and their threshold voltages ae VTHn = VTHp = 1 V the current I is

Brought to you by: Nodia and Company Visit us at: www.nodia.co.in


PUBLISHING FOR GATE
Page 18 GATE EC 2007 www.gatehelp.com

(A) 0 A (B) 25 μA
(C) 45 μA (D) 90 μA
SOL 1.39 Hence (D) is correct option
We have Vthp = Vthp = 1 V
W W
and P
= N = 40μA/V2
LP LN
From figure it may be easily seen that Vas for each NMOS and PMOS is 2.5 V
μA
Thus ID = K (Vas − VT ) 2 = 40 2 (2.5 − 1) 2 = 90 μ A
V
MCQ 1.40 For the Zener diode shown in the figure, the Zener voltage at knee is 7 V, the knee
current is negligible and the Zener dynamic resistance is 10 Ω. If the input voltage
(Vi) range is from 10 to 16 V, the output voltage (V0) ranges from

(A) 7.00 to 7.29 V (B) 7.14 to 7.29 V


(C) 7.14 to 7.43 V (D) 7.29 to 7.43 V
SOL 1.40 We have VZ = 7 volt, VK = 0, RZ = 10Ω
Circuit can be modeled as shown in fig below

Since Vi is lies between 10 to 16 V, the range of voltage across 200 kΩ


V200 = Vi − VZ = 3 to 9 volt
The range of current through 200 kΩ is
3 = 15 mA to 9 = 45 mA
200k 200k
The range of variation in output voltage
15m # RZ = 0.15 V to 45m # RZ = 0.45
Thus the range of output voltage is 7.15 Volt to 7.45 Volt
Hence (C) is correct option.
MCQ 1.41 The Boolean expression Y = ABC D + ABCD + ABC D + ABC D can be minimized
to

Brought to you by: Nodia and Company Visit us at: www.nodia.co.in


PUBLISHING FOR GATE
Page 19 GATE EC 2007 www.gatehelp.com

(A) Y = ABC D + ABC + AC D (B) Y = ABC D + BCD + ABC D


(C) Y = ABCD + BC D + ABC D (D) Y = ABCD + BC D + ABC D
SOL 1.41 Hence (D) is correct answer.
Y = ABCD + ABCD + ABC D + ABC D
= ABCD + ABC D + ABC D + ABC D
= ABCD + ABC D + BC D (A + A)
= ABCD + ABC D + BC D A+A = 1
MCQ 1.42 The circuit diagram of a standard TTL NOT gate is shown in the figure. Vi = 25
V, the modes of operation of the transistors will be

(A) Q1: revere active; Q2: normal active; Q3: saturation; Q4: cut-off
(B) Q1: revere active; Q2: saturation; Q3: saturation; Q4: cut-off
(C) Q1: normal active; Q2: cut-off; Q3: cut-off; Q4: saturation
(D) Q1: saturation; Q2: saturation; Q3: saturation; Q4: normal active
SOL 1.42 In given TTL NOT gate when Vi = 2.5 (HIGH), then
Q1 " Reverse active
Q2 " Saturation
Q3 " Saturation
Q4 " cut - off region
Hence (B) is correct answer.
MCQ 1.43 In the following circuit, X is given by

Brought to you by: Nodia and Company Visit us at: www.nodia.co.in


PUBLISHING FOR GATE
Page 20 GATE EC 2007 www.gatehelp.com

(A) X = ABC + ABC + ABC + ABC


(B) X = ABC + ABC + ABC + ABC
(C) X = AB + BC + AC
(D) X = AB + BC + AC
SOL 1.43 The circuit is as shown below

Y = AB + AB
and X = YC + YC
= (AB + AB ) C + (AB + AB ) C
= (AB + AB) C + (AB + AB ) C
= ABC + ABC + ABC + ABC
Hence (A) is correct answer.
MCQ 1.44 The following binary values were applied to the X and Y inputs of NAND latch
shown in the figure in the sequence indicated below :
X = 0,Y = 1; X = 0, Y = 0; X = 1; Y = 1
The corresponding stable P, Q output will be.

(A) P = 1, Q = 0; P = 1, Q = 0; P = 1, Q = 0 or P = 0, Q = 1
(B) P = 1, Q = 0; P = 0, Q = 1; or P = 0, Q = 1; P = 0, Q = 1
(C) P = 1, Q = 0; P = 1, Q = 1; P = 1, Q = 0 or P = 0, Q = 1
(D) P = 1, Q = 0; P = 1, Q = 1; P = 1, Q = 1
SOL 1.44 Hence (C) is correct answer.
For X = 0, Y = 1 P = 1, Q = 0
For X = 0, Y = 0 P = 1, Q = 1
For X = 1, Y = 1 P = 1, Q = 0 or P = 0, Q = 1

MCQ 1.45 For the circuit shown, the counter state (Q1 Q0) follows the sequence

Brought to you by: Nodia and Company Visit us at: www.nodia.co.in


PUBLISHING FOR GATE
Page 21 GATE EC 2007 www.gatehelp.com

(A) 00, 01, 10, 11, 00 (B) 00, 01, 10, 00, 01
(C) 00, 01, 11, 00, 01 (D) 00, 10, 11, 00, 10
SOL 1.45 For this circuit the counter state (Q1, Q0) follows the sequence 00, 01, 10, 00 ... as
shown below

Clock D1 D0 Q1 Q0 Q1 NOR Q0
00 1
1st 01 10 0
2nd 10 01 0
3rd 00 00 0

Hence (A) is correct answer.


MCQ 1.46 An 8255 chip is interfaced to an 8085 microprocessor system as an I/O mapped I/O
as show in the figure. The address lines A0 and A1 of the 8085 are used by the 8255
chip to decode internally its thee ports and the Control register. The address lines
A3 to A7 as well as the IO/M signal are used for address decoding. The range of
addresses for which the 8255 chip would get selected is

(A) F8H - FBH (B) F8GH - FCH


(C) F8H - FFH (D) F0H - F7H
SOL 1.46 Chip 8255 will be selected if bits A3 to A7 are 1. Bit A0 to A2 can be 0 or.
1. Thus address range is

Brought to you by: Nodia and Company Visit us at: www.nodia.co.in


PUBLISHING FOR GATE
Page 22 GATE EC 2007 www.gatehelp.com

11111000 F8H
11111111 FFH
Hence (C) is correct answer.
MCQ 1.47 The 3-dB bandwidth of the low-pass signal e−t u (t), where u (t) is the unit step
function, is given by
(A) 1 Hz (B) 1 2 − 1 Hz
2π 2π
(C) 3 (D) 1 Hz
SOL 1.47 Hence (A) is correct answer.
x (t) = e−t u (t)
Taking Fourier transform
X (jω) = 1
1 + jω
X (jω) = 1 2
1+ω
Magnitude at 3dB frequency is 1
2
Thus 1 = 1
2 1 + ω2
or ω = 1 rad
or f = 1 Hz

MCQ 1.48 A Hilbert transformer is a


(A) non-linear system (B) non-causal system
(C) time-varying system (D) low-pass system
SOL 1.48 A Hilbert transformer is a non-linear system.
Hence (A) is correct answer.
MCQ 1.49 The frequency response of a linear, time-invariant system is given by
H (f) = 1 + j510πf . The step response of the system is
(B) 5 61 − e− 5@ u (t)
t
(A) 5 (1 − e−5t) u (t)
(C) 1 (1 − e−5t) u (t) (D) 1 ^1 − e− 5 h u (t)
t

2 5
SOL 1.49 Hence (B) is correct answer.
H (f) = 5
1 + j10πf
H (s) = 5 = 5 = 1
1 + 5s 5^s + 15 h s + 15
Step response Y (s) = 1 a 1
s ^s + 5 h
or Y (s) = 1 1 1 = 5 − 5 1
s ^s + 5 h s s+ 5
Brought to you by: Nodia and Company Visit us at: www.nodia.co.in
PUBLISHING FOR GATE
Page 23 GATE EC 2007 www.gatehelp.com

or y (t) = 5 (1 − e−t/5) u (t)

MCQ 1.50 A 5-point sequence x [n] is given as x [− 3] = 1, x [− 2] = 1, x [− 1] = 0, x [0] = 5 and


x [1] = 1. Let X (eiω) denoted the discrete-time Fourier transform of x [n]. The value
π
of #
−π
X (e jω) dω is

(A) 5 (B) 10π


(C) 16π (D) 5 + j10π
SOL 1.50 For discrete time Fourier transform (DTFT) when N " 3
π
x [n] = 1
2π − π #
X (e jω) e jωn dω

Putting n = 0 we get
π π
x [0] = 1
2π #
−π
X (e jω) e jω0 dω = 1
2π #
−π
X (e jω) dω
π
or #
−π
X (e jω) dω = 2πx [0] = 2π # 5 = 10π

Hence (B) is correct answer.


MCQ 1.51 The z −transform X (z) of a sequence x [n] is given by X [z] = 1 −0.25z . It is given that −1

the region of convergence of X (z) includes the unit circle. The value of x [0] is
(A) − 0.5 (B) 0
(C) 0.25 (D) 05
SOL 1.51 Hence (B) is correct answer.
X (z) = 0.5 −1
1 − 2z
Since ROC includes unit circle, it is left handed system
x (n) =− (0.5) (2) −n u (− n − 1)
x (0) = 0
If we apply initial value theorem
x (0) = lim X (z) = lim 0.5 −1 = 0.5
z"3 z " 31 − 2z

That is wrong because here initial value theorem is not applicable because signal
x (n) is defined for n < 0 .
MCQ 1.52 A control system with PD controller is shown in the figure. If the velocity error
constant KV = 1000 and the damping ratio ζ = 0.5 , then the value of KP and KD
are

Brought to you by: Nodia and Company Visit us at: www.nodia.co.in


PUBLISHING FOR GATE
Page 24 GATE EC 2007 www.gatehelp.com

(A) KP = 100, KD = 0.09 (B) KP = 100, KD = 0.9


(C) KP = 10, KD = 0.09 (D) KP = 10, KD = 0.9
SOL 1.52 Hence (B) is correct option
We have Kv = lim sG (s) H (s)
s"0

(Kp + KD s) 100
or 1000 = lim s = Kp
s"0 s (s + 100)
Now characteristics equations is
1 + G (s) H (s) = 0
(K + KD s) 100
1000 = lims " 0 s p = Kp
s (s + 100)
Now characteristics equation is
1 + G (s) H (s) = 0

(100 + KD s) 100
or 1+ =0 Kp = 100
s (s + 10)
or s2 + (10 + 100KD) s + 10 4 = 0

Comparing with s2 + 2ξωn + ωn2 = 0 we get


2ξωn = 10 + 100KD
or KD = 0.9
MCQ 1.53 The transfer function of a plant is
T (s) = 5
(s + 5)( s2 + s + 1)
The second-order approximation of T (s) using dominant pole concept is
(A) 1 (B) 5
(s + 5)( s + 1) (s + 5)( s + 1)

(C) 5 (D) 1
s2 + s + 1 s2 + s + 1
SOL 1.53 Hence (D) is correct option.
We have T (s) = 5
(s + 5)( s2 + s + 1)
= 5 = 2 1
5`1 + j (s + s + 1)
s 2
s +s+1
5
In given transfer function denominator is (s + 5)[( s + 0.5) 2 + 43 ]. We can see easily
that pole at s =− 0.5 ! j 23 is dominant then pole at s =− 5 . Thus we have
approximated it.
MCQ 1.54 The open-loop transfer function of a plant is given as G (s) = s 1- 1 . If the plant is
2

operated in a unity feedback configuration, then the lead compensator that an


stabilize this control system is

Brought to you by: Nodia and Company Visit us at: www.nodia.co.in


PUBLISHING FOR GATE
Page 25 GATE EC 2007 www.gatehelp.com

10 (s − 1) 10 (s + 4)
(A) (B)
s+2 s+2
10 (s + 2) 2 (s + 2)
(C) (D)
s + 10 s + 10
SOL 1.54 Hence (A) is correct option.
G (s) = 2 1 = 1
s −1 (s + 1)( s − 1)
The lead compensator C (s) should first stabilize the plant i.e. remove 1 term.
(s − 1)
From only options (A), C (s) can remove this term

1 10 (s − 1)
ThusG (s) C (s) = #
(s + 1)( s − 1) (s + 2)
= 10 Only option (A) satisfies.
(s + 1)( s + 2)

MCQ 1.55 A unity feedback control system has an open-loop transfer function
G (s) = K
s (s + 7s + 12)
2

The gain K for which s = 1 + j1 will lie on the root locus of this system is
(A) 4 (B) 5.5
(C) 6.5 (D) 10
SOL 1.55 For ufb system the characteristics equation is
1 + G (s) = 0
or 1+ K =0
s (s2 + 7s + 12)
or s (s2 + 7s + 12) + K = 0
Point s =− 1 + j lie on root locus if it satisfy above equation i.e
(− 1 + j)[( − 1 + j) 2 + 7 (− 1 + j) + 12) + K] = 0
or K =+ 10
Hence (D) is correct option.
MCQ 1.56 The asymptotic Bode plot of a transfer function is as shown in the figure. The
transfer function G (s) corresponding to this Bode plot is

(A) 1 (B) 1
(s + 1)( s + 20) s (s + 1)( s + 20)
Brought to you by: Nodia and Company Visit us at: www.nodia.co.in
PUBLISHING FOR GATE
Page 26 GATE EC 2007 www.gatehelp.com

(C) 100 (D) 100


s (s + 1)( s + 20) s (s + 1)( 1 + 0.05s)
SOL 1.56 At every corner frequency there is change of -20 db/decade in slope which indicate
pole at every corner frequency. Thus
G (s) = K
s (1 + s)`1 + s j
20
Bode plot is in (1 + sT) form
20 log K = 60 dB = 1000
ω ω = 0. 1
Thus K =5
Hence G (s) = 100
s (s + 1)( 1 + .05s)
Hence (D) is correct option.
MCQ 1.57 The state space representation of a separately excited DC servo motor dynamics
is given as

−1 1 ω
> di H = =− 1 − 10G=ia G + =10Gu
dt 0
o
dt

where ω is the speed of the motor, ia is the armature current and u is the armature
ω (s)
voltage. The transfer function of the motor is
U (s)
(A) 2 10 (B) 2 1
s + 11s + 11 s + 11s + 11
(C) 2 10s + 10 (D) 2 1
s + 11s + 11 s + s + 11

SOL 1.57 Hence (A) is correct option.



−1 1 ω
We have > didt H = =
0
a G=
− 1 − 10 in G + =10G
u
dt

or dω =− ω + i ...(1)
n
dt
and dia =− ω − 10i + 10u ...(2)
a
dt
Taking laplace transform (i) we get
sω (s) =− ω (s) = Ia (s)
or (s + 1) ω (s) = Ia (s) ...(3)
Taking laplace transform (ii) we get
sIa (s) =− ω (s) − 10Ia (s) + 10U (s)
or ω (s) = (− 10 − s) Ia (s) + 10U (s)
= (− 10 − s)( s + 1) ω (s) + 10U (s) From (3)
or ω (s) =− [s2 + 11s + 10] ω (s) + 10U (s)

Brought to you by: Nodia and Company Visit us at: www.nodia.co.in


PUBLISHING FOR GATE
Page 27 GATE EC 2007 www.gatehelp.com

or(s2 + 11s + 11) ω (s) = 10U (s)


ω (s)
or = 2 10
U (s) (s + 11s + 11)
MCQ 1.58 In delta modulation, the slope overload distortion can be reduced by
(A) decreasing the step size (B) decreasing the granular noise
(C) decreasing the sampling rate (D) increasing the step size
SOL 1.58 Slope overload distortion can be reduced by increasing the step size
3 $ slope of x (t)
Ts
Hence (D) is correct option.
MCQ 1.59 The raised cosine pulse p (t) is used for zero ISI in digital communications. The
expression for p (t) with unity roll-off factor is given by
p (t) = sin 4πWt
4πWt (1 − 16W2 t2)
The value of p (t) at t = 1 is
4W
(A) − 0.5 (B) 0
(C) 0.5 (D) 3
SOL 1.59 Hence (C) is correct option.
sin (4πWt)
We have p (t) =
4πWt (1 − 16W2 t2)
at t = 1 it is 0 form. Thus applying L' Hospital rule
4W 0
4πW cos (4πWt)
p( ) =
1
4W

4πW [1 − 48W2 t2]


cos (4πWt)
= = cos π = 0.5
1 − 48W t
2 2 1−3

MCQ 1.60 In the following scheme, if the spectrum M (f) of m (t) is as shown, then the spectrum
Y (f) of y (t) will be

Brought to you by: Nodia and Company Visit us at: www.nodia.co.in


PUBLISHING FOR GATE
Page 28 GATE EC 2007 www.gatehelp.com

SOL 1.60 The block diagram is as shown below

Here M1 (f) = Mt (f)


j 2π B
Y1 (f) = M (f) c e + e -j2πB
2 m
j 2π B
Y2 (f) = M1 (f) c e − e -j2πB
2 m

Y (f) = Y1 (f) + Y2 (f)


All waveform is shown below

Brought to you by: Nodia and Company Visit us at: www.nodia.co.in


PUBLISHING FOR GATE
Page 29 GATE EC 2007 www.gatehelp.com

Hence (B) is correct option.


MCQ 1.61 During transmission over a certain binary communication channel, bit errors occur
independently with probability p. The probability of AT MOST one bit in error
in a block of n bits is given by
(A) pn (B) 1 − pn
(C) np (1 − p) n - 1 + (1 + p) n (D) 1 − (1 − p) n
SOL 1.61 By Binomial distribution the probability of error is
pe = n Cr pr (1 − p) n - r
Probability of at most one error
= Probability of no error + Probability of one error
= n C0 p0 (1 − p) n - 0 + n C1 p1 (1 − p) n - 1

= (1 − p) n + np (1 − p) n - 1
Hence (C) is correct option.
MCQ 1.62 In a GSM system, 8 channels can co-exist in 200 kHz bandwidth using TDMA.
A GSM based cellular operator is allocated 5 MHz bandwidth. Assuming a
frequency reuse factor of 1 , i.e. a five-cell repeat pattern, the maximum number of
5
simultaneous channels that can exist in one cell is
(A) 200 (B) 40
(C) 25 (D) 5
SOL 1.62 Bandwidth allocated for 1 Channel = 5 M Hz
Average bandwidth for 1 Channel 5 = 1 MHz
5
Total Number of Simultaneously Channel = 1M # 8 = 40 Channel
200k
Hence (B) is correct option.
MCQ 1.63 In a Direct Sequence CDMA system the chip rate is 1.2288 # 106 chips per second.
If the processing gain is desired to be AT LEAST 100, the data rate
(A) must be less than or equal to 12.288 # 103 bits per sec
(B) must be greater than 12.288 # 103 bits per sec
(C) must be exactly equal to 12.288 # 103 bits per sec
(D) can take any value less than 122.88 # 103 bits per sec
SOL 1.63 Hence (A) is correct option.
Chip Rate RC = 1.2288 # 106 chips/sec

Brought to you by: Nodia and Company Visit us at: www.nodia.co.in


PUBLISHING FOR GATE
Page 30 GATE EC 2007 www.gatehelp.com

Data Rate Rb = RC
G
Since the processing gain G must be at least 100, thus for Gmin we get
6
Rb max = RC = 1.2288 # 10 = 12.288 # 103 bps
Gmin 100

MCQ 1.64 An air-filled rectangular waveguide has inner dimensions of 3 cm # 2 cm. The wave
impedance of the TE20 mode of propagation in the waveguide at a frequency of 30
GHz is (free space impedance η0 = 377 Ω )
(A) 308 Ω (B) 355 Ω
(C) 400 Ω (D) 461 Ω
SOL 1.64 The cut-off frequency is
fc = c ` m j2 + ` n j2
2 a b
Since the mode is TE20, m = 2 and n = 0
8
fc = c m = 3 # 10 # 2 = 10 GHz
2 2 2 # 0.03
ηo 377
η' = fc 2
= 10 = 400Ω
1−c m
2
1 − c 10 10 m
f 3 # 10
Hence (C) is correct option.
MCQ 1.65 The H field (in A/m) of a plane wave propagating in free space is given by
H = xt 5 3 cos (ωt − βz) + yt`ωt − βz + π j .
η0 2
The time average power flow density in Watts is
η
(A) 0 (B) 100
100 η0

(C) 50η20 (D) 50


η0
SOL 1.65 Hence (D) is correct option.
2
= Hx2 + Hy2 = c 5 3 m + c 5 m = c 10 m
2 2
We have H 2
ηo ηo ηo
E 2 ηo H 2
η 2
For free space P = = = o c 10 m = 50 watts
2ηo 2 2 ηo ηo

MCQ 1.66 The E field in a rectangular waveguide of inner dimension a # b is given by


ωμ
E = 2 ` λ j H0 sin ` 2πx j sin (ωt − βz) yt
2

h 2 a
Where H0 is a constant, and a and b are the dimensions along the x − axis and the
y − axis respectively. The mode of propagation in the waveguide is
(A) TE20 (B) TM11
(C) TM20 (D) TE10

Brought to you by: Nodia and Company Visit us at: www.nodia.co.in


PUBLISHING FOR GATE
Page 31 GATE EC 2007 www.gatehelp.com

SOL 1.66 Hence (A) is correct option.


ωμ
E = 2 ` π j H0 sin ` 2πx j sin (ωt − βz) yt
2

h 2 a
This is TE mode and we know that
mπy
Ey \ sin ` mπx j cos `
a b j
Thus m = 2 and n = 0 and mode is TE20
MCQ 1.67 A load of 50 Ω is connected in shunt in a 2-wire transmission line of Z0 = 50Ω as
shown in the figure. The 2-port scattering parameter matrix (s-matrix) of the shunt
element is

−1 1
(A) > 12 1H
0 1
(B) =
1 0G
2
2 − 2

− 13 2 1
− 43
(C) > 3
H (D) > 4
1H
2
3 − 1
3 −3
4 4
SOL 1.67 The 2-port scattering parameter matrix is
S11 S12
S ==
S21 S22 G
(Z Z ) − Zo (50 50) − 50
S11 = L 0 = =− 1
(ZL Z0) + Zo (50 50) + 50 3
2 (ZL Zo) 2 (50 50)
S12 = S21 = = =2
(ZL Zo) + Zo (50 50) + 50 3
(Z Z ) − Zo (50 50) − 50
S22 = L o = =− 1
(ZL Zo) + Zo (50 50) + 50 3
Hence (C) is correct option.
MCQ 1.68 The parallel branches of a 2-wire transmission line re terminated in 100Ω and
200Ω resistors as shown in the figure. The characteristic impedance of the line is
Z0 = 50Ω and each section has a length of λ . The voltage reflection coefficient Γ
4
at the input is

Brought to you by: Nodia and Company Visit us at: www.nodia.co.in


PUBLISHING FOR GATE
Page 32 GATE EC 2007 www.gatehelp.com

(A) − j 7 (B) − 5
5 7

(C) j 5 (D) 5
7 7
SOL 1.68 The input impedance is
if l = λ
2
Zin = Zo ;
ZL 4
2 2
Zin1 = Zo1 = 50 = 25
ZL1 100
2 2
Zin2 = Zo2 = 50 = 12.5
ZL2 200
Now ZL = Zin1 Zin2
25 12.5 = 25
3
(50) 2
Zs = = 300
25/3
Γ = ZS − Zo = 300 − 50 = 5
ZS + Zo 300 + 50 7
Hence (D) is correct option.

MCQ 1.69 A λ dipole is kept horizontally at a height of λ0 above a perfectly conducting


2 2
infinite ground plane. The radiation pattern in the lane of the dipole (E plane)
looks approximately as

Brought to you by: Nodia and Company Visit us at: www.nodia.co.in


PUBLISHING FOR GATE
Page 33 GATE EC 2007 www.gatehelp.com

SOL 1.69 Using the method of images, the configuration is as shown below

Here d = λ, α = π, thus βd = 2π

Array factor is
βd cos ψ + α
= cos ; E
2
2π cos ψ + π
= cos ; E = sin (π cos ψ)
2
Hence (B) is correct option.
MCQ 1.70 A right circularly polarized (RCP) plane wave is incident at an angle 60c to the
normal, on an air-dielectric interface. If the reflected wave is linearly polarized, the
relative dielectric constant ξr2 is.

(A) 2 (B) 3
(C) 2 (D) 3
SOL 1.70 The Brewster angle is
tan θn = εr2
εr1
tan 60c = εr2
1
or εr2 = 3

Brought to you by: Nodia and Company Visit us at: www.nodia.co.in


PUBLISHING FOR GATE
Page 34 GATE EC 2007 www.gatehelp.com

Hence (D) is correct option.

Common Data Questions

Common Data for Questions 71, 72, 73 :


The figure shows the high-frequency capacitance - voltage characteristics of Metal/
Sio 2 /silicon (MOS) capacitor having an area of 1 # 10 - 4 cm 2 . Assume that the
permittivities (ε0 εr ) of silicon and Sio2 are 1 # 10 - 12 F/cm and 3.5 # 10 - 13 F/cm
respectively.

MCQ 1.71 The gate oxide thickness in the MOS capacitor is


(A) 50 nm (B) 143 nm
(C) 350 nm (D) 1 μm
SOL 1.71 At low voltage when there is no depletion region and capacitance is decide by SiO2
thickness only,
C = ε0 εr1 A
D
−13
or D = ε0 εr1 A = 3.5 # 10 −# 10−4 = 50 nm
C 7 # 10 12
Hence option (A) is correct
MCQ 1.72 The maximum depletion layer width in silicon is
(A) 0.143 μm (B) 0.857 μm
(C) 1 μm (D) 1.143 μm
SOL 1.72 The construction of given capacitor is shown in fig below

Brought to you by: Nodia and Company Visit us at: www.nodia.co.in


PUBLISHING FOR GATE
Page 35 GATE EC 2007 www.gatehelp.com

When applied voltage is 0 volts, there will be no depletion region and we get
C1 = 7 pF
When applied voltage is V , a depletion region will be formed as shown in fig an
total capacitance is 1 pF. Thus
CT = 1 pF
or CT = C1 C2 = 1 pF
C1 + C2
or 1 = 1 + 1
CT C1 C2
Substituting values of CT and C1 we get
C2 = 7 pF
6
D2 = ε0 εr2 A = 1 # 710 #- 1210 = 6 # 10 - 4 cm
- 12 -4
Now
C2 6 # 10
7
= 0.857 μm
Hence option (B) is correct.
MCQ 1.73 Consider the following statements about the C − V characteristics plot :
S1 : The MOS capacitor has as n -type substrate
S2 : If positive charges are introduced in the oxide, the C − V polt will shift to the
left.
Then which of the following is true?
(A) Both S1 and S2 are true
(B) S1 is true and S2 is false
(C) S1 is false and S2 is true
(D) Both S1 and S2 are false
SOL 1.73 Depletion region will not be formed if the MOS capacitor has n type substrate but
from C-V characteristics, C reduces if V is increased. Thus depletion region must
be formed. Hence S1 is false
If positive charges is introduced in the oxide layer, then to equalize the effect the
applied voltage V must be reduced. Thus the C − V plot moves to the left. Hence
S2 is true.
Hence option (C) is correct.

Brought to you by: Nodia and Company Visit us at: www.nodia.co.in


PUBLISHING FOR GATE
Page 36 GATE EC 2007 www.gatehelp.com

Common Data for Questions 74 & 75 :


Two 4-array signal constellations are shown. It is given that φ1 and φ2 constitute
an orthonormal basis for the two constellation. Assume that the four symbols in
both the constellations are equiprobable. Let N0 denote the power spectral density
2
of white Gaussian noise.

MCQ 1.74 The if ratio or the average energy of Constellation 1 to the average energy of
Constellation 2 is
(A) 4a2 (B) 4
(C) 2 (D) 8
SOL 1.74 Energy of constellation 1 is
Eg1 = (0) 2 + (− 2 a) 2 + (− 2 a) 2 + ( 2 a) 2 + (− 2 2 a) 2

= 2a2 + 2a2 + 2a2 + 8a2 = 16a2


Energy of constellation 2 is
Eg2 = a2 + a2 + a2 + a2 = 4a2

Eg1 16a2
Ratio = = =4
Eg2 4a2
Hence (B) is correct option.
MCQ 1.75 If these constellations are used for digital communications over an AWGN channel,
then which of the following statements is true ?
(A) Probability of symbol error for Constellation 1 is lower
(B) Probability of symbol error for Constellation 1 is higher
(C) Probability of symbol error is equal for both the constellations
(D) The value of N0 will determine which of the constellations has a lower probability
of symbol error

SOL 1.75 Noise Power is same for both which is N0 .


2
Thus probability of error will be lower for the constellation 1 as it has higher signal
energy.
Hence (A) is correct option.

Brought to you by: Nodia and Company Visit us at: www.nodia.co.in


PUBLISHING FOR GATE
Page 37 GATE EC 2007 www.gatehelp.com

Linked Answer Questions : Q.76 to Q.85 carry two marks each.

Statement for Linked Answer Questions 76 & 77:


Consider the Op-Amp circuit shown in the figure.

MCQ 1.76 The transfer function V0 (s)/ Vi (s) is


(A) 1 − sRC (B) 1 + sRC
1 + sRC 1 − sRC

(C) 1 (D) 1
1 − sRC 1 + sRC
SOL 1.76 The voltage at non-inverting terminal is
1
V+ = sC 1 Vi = 1 V
R + sC 1 + sCR i
Now V- = V+ = 1 V
1 + sCR i
Applying voltage division rule
(V + Vi)
V+ = R1 (V0 + Vi) = o
R1 + R1 2
1 (V + Vi)
or Vi = o
1 + sCR 2
or Vo =− 1 + 2
Vi 1 + sRC
V0 = 1 − sRC
Vi 1 + sRC
Hence (A) is correct option.

MCQ 1.77 If Vi = V1 sin (ωt) and V0 = V2 sin (ωt + φ), then the minimum and maximum values
of φ (in radians) are respectively
(A) − π and π (B) 0 and π
2 2 2

(C) − π and 0 (D) − π and 0


2
SOL 1.77 Hence (C) is correct option.
V0 = H (s) = 1 − sRC
Vi 1 + sRC
Brought to you by: Nodia and Company Visit us at: www.nodia.co.in
PUBLISHING FOR GATE
Page 38 GATE EC 2007 www.gatehelp.com

1 − jωRC
H (jω) =
1 + jωRC
+H (jω) = φ =− tan - 1 ωRC − tan - 1 ωRC
=− 2 tan - 2 ωRC
Minimum value, φmin = − π (at ω " 3)
Maximum value, φmax = 0( at ω = 0)

Statement for Linked Answer Questions 78 & 79 :


An 8085 assembly language program is given below.
Line 1: MVI A, B5H
2: MVI B, OEH
3: XRI 69H
4: ADD B
5: ANI 9BH
6: CPI 9FH
7: STA 3010H
8: HLT
MCQ 1.78 The contents of the accumulator just execution of the ADD instruction in line 4
will be
(A) C3H (B) EAH
(C) DCH (D) 69H
SOL 1.78 Line 1 : MVI A, B5H ; Move B5H to A
2 : MVI B, 0EH ; Move 0EH to B
3 : XRI 69H ; [A] XOR 69H and store in A
; Contents of A is CDH
4 : ADDB ; Add the contents of A to contents of B and
; store in A, contents of A is EAH
5 : ANI 9BH ; [a] AND 9BH, and store in A,
; Contents of A is 8 AH
6 : CPI 9FH ; Compare 9FH with the contents of A
; Since 8 AH < 9BH, CY = 1
7 : STA 3010 H ; Store the contents of A to location 3010 H
8 : HLT ; Stop
Thus the contents of accumulator after execution of ADD instruction is EAH.
Hence (B) is correct answer.
MCQ 1.79 After execution of line 7 of the program, the status of the CY and Z flags will be
(A) CY = 0, Z = 0 (B) CY = 0, Z = 1

Brought to you by: Nodia and Company Visit us at: www.nodia.co.in


PUBLISHING FOR GATE
Page 39 GATE EC 2007 www.gatehelp.com

(C) CY = 1, Z = 0 (D) CY = 1, Z = 1
SOL 1.79 The CY = 1 and Z = 0
Hence (C) is correct answer.

Statement for linked Answer Question 80 & 81 :


Consider a linear system whose state space representation is x (t) = Ax (t). If
1
the initial state vector of the system is x (0) = = G, then the system response is
−2
e−2x 1
−2tH . If the itial state vector of the system changes to x (0) = =
− 2G
x (t) = > , then
− 2e −t
e
the system response becomes x (t) = > −tH
−e
MCQ 1.80 The eigenvalue and eigenvector pairs (λi vi) for the system are
1 1 1 1
(A) e− 1 = Go and e− 2 = Go (B) e− 1, = Go and e2, = Go
−1 −2 −1 −2
1 1 1 1
(C) e− 1, = Go and e− 2, = Go (D) e− 2 = Go and e1, = Go
−1 −2 −1 −2

SOL 1.80 Hence (A) is correct option.


We have xo (t) = Ax (t)
p q
A ==
r sG
Let

1 e−2t
For initial state vector x (0) = = G the system response is x (t) = > H
−2 − 2e−2t
d −2t
e p q 1
==
r s G=− 2G
Thus > d dt −2t H
dt (− 2e )
t=0
−2 (0)
− 2e p q 1
or > 4e−2 (0) H = =r s G=− 2G
−2 p − 2q
= 4 G = = r − 2s G

We get p − 2q =− 2 and r − 2s = 4 ...(i)


1 e−t
For initial state vector x (0) = = G the system response is x (t) = > −tH
−1 −e
d
e−t p q 1
==
> d (− e−t)H r s G=− 1G
dt
Thus
dt
t=0
− (0)
−e p q 1
> e− (0) H = =r s G=− 1G
−1 p−q
= 1G = = r − s G

Brought to you by: Nodia and Company Visit us at: www.nodia.co.in


PUBLISHING FOR GATE
Page 40 GATE EC 2007 www.gatehelp.com

We get p − q =− 1 and r − s = 1 ...(2)


Solving (1) and (2) set of equations we get
p q 0 1
=r s G = =− 2 − 3G

The characteristic equation


λI − A = 0
λ −1
=0
2 λ+3
or λ (λ + 3) + 2 = 0
or λ =− 1, − 2
Thus Eigen values are − 1 and − 2
Eigen vectors for λ1 =− 1
(λ1 I − A) X1 = 0
λ1 − 1 x11
or = 2 λ + 3G=x G = 0
1 21

− 1 − 1 x11
= 2 2 G=x G = 0
21

or − x11 − x21 = 0
or x11 + x21 = 0
We have only one independent equation x11 =− x21.Let x11 = K , then x21 =− K , the
Eigen vector will be
x11 K 1
=x G = =− K G = K =− 1G
21

Now Eigen vector for λ2 =− 2


(λ2 I − A) X2 = 0
λ2 − 1 x12
or = 2 λ + 3G=x G = 0
2 22

− 2 − 1 x11
or = 2 1 G=x G = 0
21

or − x11 − x21 = 0
or x11 + x21 = 0
We have only one independent equation x11 =− x21.
Let x11 = K, then x21 =− K , the Eigen vector will be
x12 K 1
=x G = =− 2K G = K =− 2G
22

MCQ 1.81 The system matrix A is


0 1 1 1
(A) =
− 1 1G
(B) =
− 1 − 2G

Brought to you by: Nodia and Company Visit us at: www.nodia.co.in


PUBLISHING FOR GATE
Page 41 GATE EC 2007 www.gatehelp.com

2 1 0 1
(C) = G (D) =
−1 −1 − 2 − 3G
SOL 1.81 As shown in previous solution the system matrix is
0 1
A ==
− 2 − 3G
Hence (D) is correct option.

Statement for Linked Answer Question 82 & 83 :


An input to a 6-level quantizer has the probability density function f (x) as shown
in the figure. Decision boundaries of the quantizer are chosen so as to maximize the
entropy of the quantizer output. It is given that 3 consecutive decision boundaries
are’ − 1'.'0' and '1' .

MCQ 1.82 The values of a and b are


(A) a = 1 and b = 1 (B) a = 1 and b = 3
6 12 5 40

(C) a = 1 and b = 1 (D) a = 1 and b = 1


4 16 3 24
SOL 1.82 Area under the pdf curve must be unity
Thus 2a + 4a + 4b =1
2a + 8b = 1 ...(1)
For maximum entropy three region must by equivaprobable thus
2a = 4b = 4b ...(2)
From (1) and (2) we get
b = 1 and a = 1
12 6
Hence (A) is correct option.
MCQ 1.83 Assuming that the reconstruction levels of the quantizer are the mid-points of the
decision boundaries, the ratio of signal power to quantization noise power is
(A) 152 (B) 64
9 3

(C) 76 (D) 28
3
SOL 1.83 Hence correct option is ( )

Brought to you by: Nodia and Company Visit us at: www.nodia.co.in


PUBLISHING FOR GATE
Page 42 GATE EC 2007 www.gatehelp.com

Statement for Linked Answer Question 84 and 85 :


In the Digital-to-Analog converter circuit shown in the figure below,
VR = 10V and R = 10kΩ

MCQ 1.84 The current i is


(A) 31.25μA (B) 62.5μA
(C) 125μA (D) 250μA
SOL 1.84 Since the inverting terminal is at virtual ground the resistor network can be reduced
as follows

The current from voltage source is


I = VR = 10 = 1 mA
R 10k
This current will be divide as shown below

Brought to you by: Nodia and Company Visit us at: www.nodia.co.in


PUBLISHING FOR GATE
Page 43 GATE EC 2007 www.gatehelp.com

-3
Now i = I = 1 # 10 = 62.5 μ A
16 16
Hence (B) is correct answer.
MCQ 1.85 The voltage V0 is
(A) − 0.781 V (B) − 1.562 V
(C) − 3.125 V (D) − 6.250 V
SOL 1.85 The net current in inverting terminal of OP - amp is
I- = 1 + 1 = 5I
4 16 16
So that V0 =− R # 5I =− 3.125
16
Hence (C) is correct answer.

Answer Sheet
1. (A) 19. (A) 37. (B) 55. (D) 73. (C)
2. (B) 20. (D) 38. (D) 56. (D) 74. (B)
3. (C) 21. (C) 39. (D) 57. (A) 75. (A)
4. (A) 22. (A) 40. (C) 58. (D) 76. (A)
5. (D) 23. (C) 41. (D) 59. (C) 77. (C)
6. (A) 24. (D) 42. (B) 60. (B) 78. (B)
7. (D) 25. (B) 43. (A) 61. (C) 79. (C)
8. (C) 26. (C) 44. (C) 62. (B) 80. (A)
9. (D) 27. (A) 45. (A) 63. (A) 81. (D)
10. (C) 28. (D) 46. (C) 64. (C) 82. (A)
11. (C) 29. (D) 47. (A) 65. (D) 83. (*)
12. (A) 30. (A) 48. (A) 66. (A) 84. (B)
13. (C) 31. (D) 49. (B) 67. (C) 85. (C)

Brought to you by: Nodia and Company Visit us at: www.nodia.co.in


PUBLISHING FOR GATE
Page 44 GATE EC 2007 www.gatehelp.com

14. (B) 32. (A) 50. (B) 68. (D)


15. (D) 33. (B) 51. (B) 69. (B)
16. (D) 34. (B) 52. (B) 70 (D)
17. (C) 35. (C) 53. (D) 71 (A)
18. (B) 36. (C) 54. (A) 72 (B)
**********

Brought to you by: Nodia and Company Visit us at: www.nodia.co.in


PUBLISHING FOR GATE
GATE EC
2007

Q.1 to Q.20 carry one mark each

MCQ 1.1 If E denotes expectation, the variance of a random variable X is given by


(A) E [X2] − E2 [X] (B) E [X2] + E2 [X]
(C) E [X2] (D) E2 [X]
SOL 1.1 The variance of a random variable x is given by
E [X2] − E2 [X]
Hence (A) is correct option.
MCQ 1.2 The following plot shows a function which varies linearly with x . The value of the
#1 ydx
2
integral I = is

(A) 1.0 (B) 2.5


(C) 4.0 (D) 5.0
SOL 1.2 The given plot is straight line whose equation is
x +y =1
−1 1
or y = x+1
#1 ydx #1 (x + 1) dx
2 2
Now I = =
(x + 1) 2 2
=; E = − = 2.5
9 4
2 2 2
Hence (B) is correct answer.
MCQ 1.3 For x << 1, coth (x) can be approximated as

Brought to you by: Nodia and Company Visit us at: www.nodia.co.in


PUBLISHING FOR GATE
Page 2 GATE EC 2007 www.gatehelp.com

(A) x (B) x2
(C) 1 (D) 12
x x
SOL 1.3 Hence (C) is correct answer.
coth x = cosh x
sinh x
as x << 1, cosh x . 1 and sinh x . x
Thus coth x . 1
x

sin b θ l
2
MCQ 1.4 lim is
θ"0 θ
(A) 0.5 (B) 1
(C) 2 (D) not defined
SOL 1.4 Hence (A) is correct answer.
sin ^ θ2 h sin ^ θ2 h 1 lim sin ^ 2 h = 1 = 0.5
θ
lim = lim =
θ"0 θ θ " 0 2^ θ h 2 θ " 0 ^ θ2 h 2
2

MCQ 1.5 Which one of following functions is strictly bounded?


(A) 1/x2 (B) ex
2

(C) x2 (D) e - x
SOL 1.5 Hence (D) is correct answer.
We have, lim 12 = 3
x"0 x

lim x2 = 3
x"3

lim e - x = 3
x"3

lim e - x = 0
2

x"3

lim e - x = 1
2 2

Thus e - x is strictly bounded.


x"0

MCQ 1.6 For the function e - x , the linear approximation around x = 2 is


(A) (3 − x) e - 2 (B) 1 − x
(C) 63 + 3 2 − (1 − 2 ) x @e - 2 (D) e - 2
SOL 1.6 Hence (A) is correct answer.
We have f (x) = e - x = e - (x - 2) - 2 = e - (x - 2) e - 2
(x − 2) 2
= ;1 − (x − 2) + ...E e - 2
2!
= 61 − (x − 2)@ e - 2 Neglecting higher powers

Brought to you by: Nodia and Company Visit us at: www.nodia.co.in


PUBLISHING FOR GATE
Page 3 GATE EC 2007 www.gatehelp.com

= (3 − x) e - 2
MCQ 1.7 An independent voltage source in series with an impedance Zs = Rs + jXs delivers
a maximum average power to a load impedance ZL when
(A) ZL = Rs + jXs (B) ZL = Rs
(C) ZL = jXs (D) ZL = Rs − jXs
SOL 1.7 According to maximum Power Transform Theorem
ZL = Zs* = (Rs − jXs)
Hence (D) is correct option.
MCQ 1.8 The RC circuit shown in the figure is

(A) a low-pass filter (B) a high-pass filter


(C) a band-pass filter (D) a band-reject filter
SOL 1.8 At ω " 3 , capacitor acts as short circuited and circuit acts as shown in fig below

Here we get V0 = 0
Vi

At ω " 0 , capacitor acts as open circuited and circuit look like as shown in fig
below

Here we get also V0 = 0


Vi
So frequency response of the circuit is as shown in fig and circuit is a Band pass
filter.

Brought to you by: Nodia and Company Visit us at: www.nodia.co.in


PUBLISHING FOR GATE
Page 4 GATE EC 2007 www.gatehelp.com

Hence (C) is correct option.


MCQ 1.9 The electron and hole concentrations in an intrinsic semiconductor are ni per cm3
at 300 K. Now, if acceptor impurities are introduced with a concentration of NA per
cm3 (where NA >> ni , the electron concentration per cm3 at 300 K will be)
(A) ni (B) ni + NA
2
(C) NA − ni (D) ni
NA
SOL 1.9 As per mass action law
np = ni2
If acceptor impurities are introduces
p = NA
Thus nNA = ni2
2
or n = ni
NA
Hence option (D) is correct.
MCQ 1.10 In a p+ n junction diode under reverse biased the magnitude of electric field is
maximum at
(A) the edge of the depletion region on the p-side
(B) the edge of the depletion region on the n -side
(C) the p+ n junction
(D) the centre of the depletion region on the n -side
SOL 1.10 The electric field has the maximum value at the junction of p+ n .
Hence option (C) is correct.
MCQ 1.11 The correct full wave rectifier circuit is

Brought to you by: Nodia and Company Visit us at: www.nodia.co.in


PUBLISHING FOR GATE
Page 5 GATE EC 2007 www.gatehelp.com

SOL 1.11 The circuit shown in (C) is correct full wave rectifier circuit.

Hence (C) is correct option.


MCQ 1.12 In a transconductance amplifier, it is desirable to have
(A) a large input resistance and a large output resistance
(B) a large input resistance and a small output resistance
(C) a small input resistance and a large output resistance
(D) a small input resistance and a small output resistance
SOL 1.12 In the transconductance amplifier it is desirable to have large input resistance and
large output resistance.
Hence (A) is correct option.
MCQ 1.13 X = 01110 and Y = 11001 are two 5-bit binary numbers represented in two’s
complement format. The sum of X and Y represented in two’s complement format
using 6 bits is
(A) 100111 (B) 0010000
(C) 000111 (D) 101001
SOL 1.13 MSB of Y is 1, thus it is negative number and X is positive number
Now we have X = 01110 = (14) 10
and Y = 11001 = (− 7) 10
X + Y = (14) + (− 7) = 7
In signed two’s complements from 7 is
(7) 10 = 000111
Brought to you by: Nodia and Company Visit us at: www.nodia.co.in
PUBLISHING FOR GATE
Page 6 GATE EC 2007 www.gatehelp.com

Hence (C) is correct answer.


MCQ 1.14 The Boolean function Y = AB + CD is to be realized using only 2 - input NAND
gates. The minimum number of gates required is
(A) 2 (B) 3
(C) 4 (D) 5
SOL 1.14 Hence (B) is correct answer.
Y = AB + CD = AB .CD
This is SOP form and we require only 3 NAND gate

MCQ 1.15 If the closed-loop transfer function of a control system is given as T (s) s−5
(s + 2)( s + 3)
, then It is
(A) an unstable system (B) an uncontrollable system
(C) a minimum phase system (D) a non-minimum phase system
SOL 1.15 In a minimum phase system, all the poles as well as zeros are on the left half of the
s −plane. In given system as there is right half zero (s = 5), the system is a non-
minimum phase system.
Hence (D) is correct option.

MCQ 1.16 If the Laplace transform of a signal Y (s) = 1 , then its final value is
s (s − 1)
(A) − 1 (B) 0
(C) 1 (D) Unbounded
SOL 1.16 Hence (D) is correct answer.
Y (s) = 1
s (s − 1)
Final value theorem is applicable only when all poles of system lies in left half of S
-plane. Here s = 1 is right s −plane pole. Thus it is unbounded.
MCQ 1.17 If R (τ) is the auto correlation function of a real, wide-sense stationary random
process, then which of the following is NOT true
(A) R (τ) = R (− τ)
(B) R (τ) # R (0)
(C) R (τ) =− R (− τ)
(D) The mean square value of the process is R (0)
SOL 1.17 Autocorrelation is even function.
Hence (C) is correct option
MCQ 1.18 If S (f) is the power spectral density of a real, wide-sense stationary random process,
then which of the following is ALWAYS true?
(A) S (0) # S (f) (B) S (f) $ 0

Brought to you by: Nodia and Company Visit us at: www.nodia.co.in


PUBLISHING FOR GATE
Page 7 GATE EC 2007 www.gatehelp.com

#- 3 S (f) df = 0
3
(C) S (− f) =− S (f) (D)
SOL 1.18 Power spectral density is non negative. Thus it is always zero or greater than zero.
Hence (B) is correct option.
MCQ 1.19 A plane wave of wavelength λ is traveling in a direction making an angle 30c with
positive x − axis and 90c with positive y − axis. The E field of the plane wave can
be represented as (E0 is constant)
3 π x− π z π 3πz
t 0 e j c ωt −
(A) E = yE λ λ m t 0 e jc ωt − λ x −
(B) E = yE λ m

3 π x+ π z π 3πz
t 0 e jc ωt +
(C) E = yE λ λ m t 0 e jc ωt − λ x +
(D) E = yE λ m

SOL 1.19 Hence (A) is correct option.


γ = β cos 30cx ! β sin 30cy
= 2π 3 x ! 2π 1 y
λ 2 λ 2
= π 3 x! πy
λ λ
π 3 x! π y
E = ay E0 e j (ωt - γ) = ay E0 e j;ωt - c λ λ mE

MCQ 1.20 If C is code curve enclosing a surface S , then magnetic field intensity H , the
current density j and the electric flux density D are related by
(A) ##
S
H $ ds =
c
##
c j + 2t m $ d t
2D (B) H $ d l =
S S
#
c j + 2t m $ dS
2D
##
(C) ##S H $ dS = #C c j + 22Dt m $ d t (D) #C H $ d l # = ##S c j + 22Dt m $ ds
c

SOL 1.20 Hence (D) is correct option.


4# H = J + 2D Maxwell Equations
2t
## 4# H $ ds = ##
`J + 2t j .ds
2D Integral form
s s
#
H $ dl = ##
`J + 2t j .ds
2D Stokes Theorem
s

Q.21 to Q.75 carry two marks each.

MCQ 1.21 It is given that X1, X2 ...XM at M non-zero, orthogonal vectors. The dimension of
the vector space spanned by the 2M vectors X1, X2,... XM , − X1, − X2,... − XM is
(A) 2M (B) M + 1
(C) M
(D) dependent on the choice of X1, X2,... XM
SOL 1.21 For two orthogonal vectors, we require two dimensions to define them and similarly

Brought to you by: Nodia and Company Visit us at: www.nodia.co.in


PUBLISHING FOR GATE
Page 8 GATE EC 2007 www.gatehelp.com

for three orthogonal vector we require three dimensions to define them. 2M vectors
are basically M orthogonal vector and we require M dimensions to define them.
Hence (C) is correct answer.
MCQ 1.22 Consider the function f (x) = x2 − x − 2 . The maximum value of f (x) in the closed
interval [− 4, 4] is
(A) 18 (B) 10
(C) − 225 (D) indeterminate
SOL 1.22 We have
f (x) = x2 − x + 2
f'( x) = 2x − 1 = 0 " x = 1
2
f"( x) = 2
Since f"( x) = 2 > 0 , thus x = 1 is minimum point. The maximum value in closed
2
interval 6− 4, 4@ will be at x =− 4 or x = 4
Now maximum value
= max [f (− 4), f (4)]
= max (18, 10)
= 18
Hence (A) is correct answer.
MCQ 1.23 An examination consists of two papers, Paper 1 and Paper 2. The probability of
failing in Paper 1 is 0.3 and that in Paper 2 is 0.2. Given that a student has failed
in Paper 2, the probability of failing in Paper 1 is 0.6. The probability of a student
failing in both the papers is
(A) 0.5 (B) 0.18
(C) 0.12 (D) 0.06
SOL 1.23 Hence (C) is correct answer.
Probability of failing in paper 1 is P (A) = 0.3
Possibility of failing in Paper 2 is P (B) = 0.2
Probability of failing in paper 1, when
student has failed in paper 2 is P ^ BA h = 0.6
We know that
(P + B)
Pb A l =
B P (B)
or P (A + B) = P (B) P b A l = 0.6 # 0.2 = 0.12
B

d2 y
MCQ 1.24 The solution of the differential equation k2 2 = y − y2 under the boundary
dx
conditions
(i) y = y1 at x = 0 and

Brought to you by: Nodia and Company Visit us at: www.nodia.co.in


PUBLISHING FOR GATE
Page 9 GATE EC 2007 www.gatehelp.com

(ii) y = y2 at x = 3 , where k, y1 and y2 are constants, is

(A) y = (y1 − y2) exp a− x2 k + y2 (B) y = (y2 − y1) exp a− x k + y1


k k

(C) y = ^y1 − y2h sinh a x k + y1 (D) y = ^y1 − y2h exp a− x k + y2


k k
SOL 1.24 Hence (D) is correct answer.
d2 y
We have k2 2 = y − y2
dx
2
d y y y
or 2
− 2 =− 22
dx k k
A.E. D − 2 =0
2 1
k
or D =! 1
k
x x

C.F. = C1 e - + C2 e
k k

− y22
c 2 m = y2
P.I. = 1
D2 − 12 k
Thus solution is k
x x

y = C1 e - + C2 e + y2
k k

From y (0) = y1 we get


C1 + C2 = y1 − y2
From y (3) = y2 we get that C1 must be zero.
Thus C2 = y1 − y2
x

y = (y1 − y2) e - + y2
k

MCQ 1.25 The equation x3 − x2 + 4x − 4 = 0 is to be solved using the Newton - Raphson


method. If x = 2 is taken as the initial approximation of the solution, then next
approximation using this method will be
(A) 2/3 (B) 4/3
(C) 1 (D) 3/2
SOL 1.25 We have
f (x) = x3 − x2 + 4x − 4
f'( x) = 3x2 − 2x + 4
Taking x0 = 2 in Newton-Raphosn method
f (x0) 23 − 22 + 4 (2) − 4
x1 = x0 − = 2− =4
f'( x0) 3 (2) − 2 (2) + 4
2 3
Hence (B) is correct answer.
MCQ 1.26 Three functions f1 (t), f2 (t) and f3 (t) which are zero outside the interval [0, T] are
shown in the figure. Which of the following statements is correct?

Brought to you by: Nodia and Company Visit us at: www.nodia.co.in


PUBLISHING FOR GATE
Page 10 GATE EC 2007 www.gatehelp.com

(A) f1 (t) and f2 (t) are orthogonal (B) f1 (t) and f3 (t) are orthogonal
(C) f2 (t) and f3 (t) are orthogonal (D) f1 (t) and f2 (t) are orthonormal
SOL 1.26 For two orthogonal signal f (x) and g (x)
#- 3
+3
f (x) g (x) dx =0

i.e. common area between f (x) and g (x) is zero.


Hence (C) is correct options.
MCQ 1.27 If the semi-circular control D of radius 2 is as shown in the figure, then the value
of the integral 1
(s − 1)
2 #
ds is
D

(A) jπ (B) − jπ
(C) − π (D) π
SOL 1.27 We know that

s
#
2
1 ds = 2πj
− 1
[sum of residues]
D
Singular points are at s = ! 1 but only s =+ 1 lies inside the given contour, Thus
Residue at s =+ 1 is

Brought to you by: Nodia and Company Visit us at: www.nodia.co.in


PUBLISHING FOR GATE
Page 11 GATE EC 2007 www.gatehelp.com

lim (s − 1) f (s) = lim (s − 1) 1 =1


s"1 s"1 s2 − 1 2
# s2 −1 1 ds = 2πj` 12 j = πj
D
Hence (A) is correct answer.
MCQ 1.28 Two series resonant filters are as shown in the figure. Let the 3-dB bandwidth of
Filter 1 be B1 and that of Filter 2 be B2 . the value B1 is
B2

(A) 4 (B) 1
(C) 1/2 (D) 1/4

SOL 1.28 We know that bandwidth of series RLC circuit is R . Therefore


L
Bandwidth of filter 1 is B1 = R
L1
Bandwidth of filter 2 is B2 = R = R = 4R
L2 L1 /4 L1
Dividing above equation B1 = 1
B2 4
Hence (D) is correct option.
MCQ 1.29 For the circuit shown in the figure, the Thevenin voltage and resistance looking
into X − Y are

(A) 4
3 V, 2 Ω (B) 4 V, 23 Ω
(C) 4
3 V, 23 Ω (D) 4 V, 2 Ω
SOL 1.29 Here Vth is voltage across node also. Applying nodal analysis we get

Brought to you by: Nodia and Company Visit us at: www.nodia.co.in


PUBLISHING FOR GATE
Page 12 GATE EC 2007 www.gatehelp.com

Vth + Vth + Vth − 2i = 2


2 1 1
But from circuit i = Vth = Vth
1
Therefore
Vth + Vth + Vth − 2Vth = 2
2 1 1
or Vth = 4 volt
From the figure shown below it may be easily seen that the short circuit current at
terminal XY is isc = 2 A because i = 0 due to short circuit of 1 Ω resistor and all
current will pass through short circuit.

Therefore Rth = Vth = 4 = 2 Ω


isc 2
Hence (D) is correct option.
MCQ 1.30 In the circuit shown, vC is 0 volts at t = 0 sec. For t > 0 , the capacitor current iC (t)
, where t is in seconds is given by

(A) 0.50 exp (− 25t) mA (B) 0.25 exp (− 25t) mA


(C) 0.50 exp (− 12.5t) mA (D) 0.25 exp (− 6.25t) mA
SOL 1.30 The voltage across capacitor is
At t = 0+ , Vc (0+) = 0
At t = 3 , VC (3) = 5 V
The equivalent resistance seen by capacitor as shown in fig is
Req = 20 20 = 10kΩ

Brought to you by: Nodia and Company Visit us at: www.nodia.co.in


PUBLISHING FOR GATE
Page 13 GATE EC 2007 www.gatehelp.com

Time constant of the circuit is


τ = Req C = 10k # 4μ = 0.04 s
Using direct formula
Vc (t) = VC (3) − [Vc (3) − Vc (0)] e−t/τ
= VC (3) (1 − e−t/τ) + VC (0) e−t/τ = 5 (1 − e−t/0.04)
or Vc (t) = 5 (1 − e−25t)
dV (t)
Now IC (t) = C C
dt
= 4 # 10−6 # (− 5 # 25e−25t) = 0.5e−25t mA
Hence (A) is correct option.
MCQ 1.31 In the ac network shown in the figure, the phasor voltage VAB (in Volts) is

(A) 0 (B) 5+30c


(C) 12.5+30c (D) 17+30c
SOL 1.31 Hence (D) is correct option.
(5 − 3j) # (5 + 3j)
Impedance = (5 − 3j) (5 + 3j) =
5 − 3j + 5 + 3j
(5) 2 − (3j) 2
= = 25 + 9 = 3.4
10 10
VAB = Current # Impedance = 5+30c # 34 = 17+30c
MCQ 1.32 A p+ n junction has a built-in potential of 0.8 V. The depletion layer width a reverse
bias of 1.2 V is 2 μm. For a reverse bias of 7.2 V, the depletion layer width will be
(A) 4 μm (B) 4.9 μm
(C) 8 μm (D) 12 μm
SOL 1.32 Hence option (A) is correct.
W = K V + VR
Now 2μ = K 0.8 + 1.2
From above two equation we get

Brought to you by: Nodia and Company Visit us at: www.nodia.co.in


PUBLISHING FOR GATE
Page 14 GATE EC 2007 www.gatehelp.com

W = 0.8 + 7.2 = 8 =2
2μ 0.8 + 1.2 2
or W2 = 4 μ m
MCQ 1.33 Group I lists four types of p − n junction diodes. Match each device in Group I
with one of the option in Group II to indicate the bias condition of the device in
its normal mode of operation.
Group - I Group-II
(P) Zener Diode (1) Forward bias
(Q) Solar cell (2) Reverse bias
(R) LASER diode
(S) Avalanche Photodiode

(A) P - 1, Q - 2, R - 1, S - 2 (B) P - 2, Q - 1, R - 1, S - 2
(C) P - 2, Q - 2, R - 1, S- -2 (D) P - 2, Q - 1, R - 2, S - 2
SOL 1.33 Zener diode and Avalanche diode works in the reverse bias and laser diode works
in forward bias.
In solar cell diode works in forward bias but photo current is in reverse direction.
Thus
Zener diode : Reverse Bias
Solar Cell : Forward Bias
Laser Diode : Forward Bias
Avalanche Photo diode : Reverse Bias
Hence option (B) is correct.
MCQ 1.34 The DC current gain (β) of a BJT is 50. Assuming that the emitter injection
efficiency is 0.995, the base transport factor is
(A) 0.980 (B) 0.985
(C) 0.990 (D) 0.995
SOL 1.34 Hence option (B) is correct.
β
α= = 50 = 50
β + 1 50 + 1 51
Current Gain = Base Transport Factor # Emitter injection Efficiency
α = β1 # β2
or β1 = α = 50 = 0.985
β2 51 # 0.995

MCQ 1.35 Group I lists four different semiconductor devices. match each device in Group I
with its charactecteristic property in Group II
Group-I Group-II
(P) BJT (1) Population iniversion
(Q) MOS capacitor (2) Pinch-off voltage

Brought to you by: Nodia and Company Visit us at: www.nodia.co.in


PUBLISHING FOR GATE
Page 15 GATE EC 2007 www.gatehelp.com

(R) LASER diode (3) Early effect


(S) JFET (4) Flat-band voltage

(A) P - 3, Q - 1, R - 4, S - 2 (B) P - 1, Q - 4, R - 3, S - 2
(C) P - 3, Q - 4, R - 1, S - 2 (D) P - 3, Q - 2, R - 1, S - 4
SOL 1.35 In BJT as the B-C reverse bias voltage increases, the B-C space charge region width
increases which xB (i.e. neutral base width) > A change in neutral base width will
change the collector current. A reduction in base width will causes the gradient in
minority carrier concentration to increase, which in turn causes an increased in the
diffusion current. This effect si known as base modulation as early effect.
In JFET the gate to source voltage that must be applied to achieve pinch off
voltage is described as pinch off voltage and is also called as turn voltage or
threshold voltage.
In LASER population inversion occurs on the condition when concentration of
electrons in one energy state is greater than that in lower energy state, i.e. a non
equilibrium condition.
In MOS capacitor, flat band voltage is the gate voltage that must be applied to
create flat ban condition in which there is no space charge region in semiconductor
under oxide.
Therefore
BJT : Early effect
MOS capacitor: Flat-band voltage
LASER diode : Population inversion
JFET : Pinch-off voltage
Hence option (C) is correct.
MCQ 1.36 For the Op-Amp circuit shown in the figure, V0 is

(A) -2 V (B) -1 V
(C) -0.5 V (D) 0.5 V
SOL 1.36 We redraw the circuit as shown in fig.

Brought to you by: Nodia and Company Visit us at: www.nodia.co.in


PUBLISHING FOR GATE
Page 16 GATE EC 2007 www.gatehelp.com

Applying voltage division rule


v+ = 0.5 V
We know that v+ = v-
Thus v- = 0.5 V
Now i = 1 − 0.5 = 0.5 mA
1k
and i = 0.5 − v0 = 0.5 mA
2k
or v0 = 0.5 − 1 =− 0.5 V
Hence (C) is correct option.
MCQ 1.37 For the BJT circuit shown, assume that the β of the transistor is very large and
VBE = 0.7 V. The mode of operation of the BJT is

(A) cut-off (B) saturation


(C) normal active (D) reverse active
SOL 1.37 If we assume β very large, then IB = 0 and IE = IC ; VBE = 0.7 V. We assume that
BJT is in active, so applying KVL in Base-emitter loop
IE = 2 − VBE = 2 − 0.7 = 1.3 mA
RE 1k
Since β is very large, we have IE = IC , thus
IC = 1.3 mA
Now applying KVL in collector-emitter loop
10 − 10IC − VCE − IC =0
or VCE =− 4.3 V
Now VBC = VBE − VCE
= 0.7 − (− 4.3) = 5 V
Since VBC > 0.7 V, thus transistor in saturation.
Hence (B) is correct option

Brought to you by: Nodia and Company Visit us at: www.nodia.co.in


PUBLISHING FOR GATE
Page 17 GATE EC 2007 www.gatehelp.com

MCQ 1.38 In the Op-Amp circuit shown, assume that the diode current follows the equation
I = Is exp (V/VT ). For Vi = 2V, V0 = V01, and for Vi = 4V, V0 = V02 .
The relationship between V01 and V02 is

(A) V02 = 2 Vo1 (B) Vo2 = e2 Vo1


(C) Vo2 = Vo1 1n2 (D) Vo1 − Vo2 = VT 1n2
SOL 1.38 Here the inverting terminal is at virtual ground and the current in resistor and
diode current is equal i.e.
IR = ID
or Vi = I eV /V
D T
s
R
or VD = VT 1n Vi
Is R
For the first condition
VD = 0 − Vo1 = VT 1n 2
Is R
For the first condition
VD = 0 − Vo1 = VT 1n 4
Is R
Subtracting above equation
Vo1 − Vo2 = VT 1n 4 − VT 1n 2
Is R Is R
or Vo1 − Vo2 = VT 1n 4 = VT 1n2
2
Hence (D) is correct option.
MCQ 1.39 In the CMOS inverter circuit shown, if the trans conductance parameters of the
NMOS and PMOS transistors are
W
kn = kp = μn Cox Wn = μCox p = 40μA/V2
Ln Lp
and their threshold voltages ae VTHn = VTHp = 1 V the current I is

Brought to you by: Nodia and Company Visit us at: www.nodia.co.in


PUBLISHING FOR GATE
Page 18 GATE EC 2007 www.gatehelp.com

(A) 0 A (B) 25 μA
(C) 45 μA (D) 90 μA
SOL 1.39 Hence (D) is correct option
We have Vthp = Vthp = 1 V
W W
and P
= N = 40μA/V2
LP LN
From figure it may be easily seen that Vas for each NMOS and PMOS is 2.5 V
μA
Thus ID = K (Vas − VT ) 2 = 40 2 (2.5 − 1) 2 = 90 μ A
V
MCQ 1.40 For the Zener diode shown in the figure, the Zener voltage at knee is 7 V, the knee
current is negligible and the Zener dynamic resistance is 10 Ω. If the input voltage
(Vi) range is from 10 to 16 V, the output voltage (V0) ranges from

(A) 7.00 to 7.29 V (B) 7.14 to 7.29 V


(C) 7.14 to 7.43 V (D) 7.29 to 7.43 V
SOL 1.40 We have VZ = 7 volt, VK = 0, RZ = 10Ω
Circuit can be modeled as shown in fig below

Since Vi is lies between 10 to 16 V, the range of voltage across 200 kΩ


V200 = Vi − VZ = 3 to 9 volt
The range of current through 200 kΩ is
3 = 15 mA to 9 = 45 mA
200k 200k
The range of variation in output voltage
15m # RZ = 0.15 V to 45m # RZ = 0.45
Thus the range of output voltage is 7.15 Volt to 7.45 Volt
Hence (C) is correct option.
MCQ 1.41 The Boolean expression Y = ABC D + ABCD + ABC D + ABC D can be minimized
to

Brought to you by: Nodia and Company Visit us at: www.nodia.co.in


PUBLISHING FOR GATE
Page 19 GATE EC 2007 www.gatehelp.com

(A) Y = ABC D + ABC + AC D (B) Y = ABC D + BCD + ABC D


(C) Y = ABCD + BC D + ABC D (D) Y = ABCD + BC D + ABC D
SOL 1.41 Hence (D) is correct answer.
Y = ABCD + ABCD + ABC D + ABC D
= ABCD + ABC D + ABC D + ABC D
= ABCD + ABC D + BC D (A + A)
= ABCD + ABC D + BC D A+A = 1
MCQ 1.42 The circuit diagram of a standard TTL NOT gate is shown in the figure. Vi = 25
V, the modes of operation of the transistors will be

(A) Q1: revere active; Q2: normal active; Q3: saturation; Q4: cut-off
(B) Q1: revere active; Q2: saturation; Q3: saturation; Q4: cut-off
(C) Q1: normal active; Q2: cut-off; Q3: cut-off; Q4: saturation
(D) Q1: saturation; Q2: saturation; Q3: saturation; Q4: normal active
SOL 1.42 In given TTL NOT gate when Vi = 2.5 (HIGH), then
Q1 " Reverse active
Q2 " Saturation
Q3 " Saturation
Q4 " cut - off region
Hence (B) is correct answer.
MCQ 1.43 In the following circuit, X is given by

Brought to you by: Nodia and Company Visit us at: www.nodia.co.in


PUBLISHING FOR GATE
Page 20 GATE EC 2007 www.gatehelp.com

(A) X = ABC + ABC + ABC + ABC


(B) X = ABC + ABC + ABC + ABC
(C) X = AB + BC + AC
(D) X = AB + BC + AC
SOL 1.43 The circuit is as shown below

Y = AB + AB
and X = YC + YC
= (AB + AB ) C + (AB + AB ) C
= (AB + AB) C + (AB + AB ) C
= ABC + ABC + ABC + ABC
Hence (A) is correct answer.
MCQ 1.44 The following binary values were applied to the X and Y inputs of NAND latch
shown in the figure in the sequence indicated below :
X = 0,Y = 1; X = 0, Y = 0; X = 1; Y = 1
The corresponding stable P, Q output will be.

(A) P = 1, Q = 0; P = 1, Q = 0; P = 1, Q = 0 or P = 0, Q = 1
(B) P = 1, Q = 0; P = 0, Q = 1; or P = 0, Q = 1; P = 0, Q = 1
(C) P = 1, Q = 0; P = 1, Q = 1; P = 1, Q = 0 or P = 0, Q = 1
(D) P = 1, Q = 0; P = 1, Q = 1; P = 1, Q = 1
SOL 1.44 Hence (C) is correct answer.
For X = 0, Y = 1 P = 1, Q = 0
For X = 0, Y = 0 P = 1, Q = 1
For X = 1, Y = 1 P = 1, Q = 0 or P = 0, Q = 1

MCQ 1.45 For the circuit shown, the counter state (Q1 Q0) follows the sequence

Brought to you by: Nodia and Company Visit us at: www.nodia.co.in


PUBLISHING FOR GATE
Page 21 GATE EC 2007 www.gatehelp.com

(A) 00, 01, 10, 11, 00 (B) 00, 01, 10, 00, 01
(C) 00, 01, 11, 00, 01 (D) 00, 10, 11, 00, 10
SOL 1.45 For this circuit the counter state (Q1, Q0) follows the sequence 00, 01, 10, 00 ... as
shown below

Clock D1 D0 Q1 Q0 Q1 NOR Q0
00 1
1st 01 10 0
2nd 10 01 0
3rd 00 00 0

Hence (A) is correct answer.


MCQ 1.46 An 8255 chip is interfaced to an 8085 microprocessor system as an I/O mapped I/O
as show in the figure. The address lines A0 and A1 of the 8085 are used by the 8255
chip to decode internally its thee ports and the Control register. The address lines
A3 to A7 as well as the IO/M signal are used for address decoding. The range of
addresses for which the 8255 chip would get selected is

(A) F8H - FBH (B) F8GH - FCH


(C) F8H - FFH (D) F0H - F7H
SOL 1.46 Chip 8255 will be selected if bits A3 to A7 are 1. Bit A0 to A2 can be 0 or.
1. Thus address range is

Brought to you by: Nodia and Company Visit us at: www.nodia.co.in


PUBLISHING FOR GATE
Page 22 GATE EC 2007 www.gatehelp.com

11111000 F8H
11111111 FFH
Hence (C) is correct answer.
MCQ 1.47 The 3-dB bandwidth of the low-pass signal e−t u (t), where u (t) is the unit step
function, is given by
(A) 1 Hz (B) 1 2 − 1 Hz
2π 2π
(C) 3 (D) 1 Hz
SOL 1.47 Hence (A) is correct answer.
x (t) = e−t u (t)
Taking Fourier transform
X (jω) = 1
1 + jω
X (jω) = 1 2
1+ω
Magnitude at 3dB frequency is 1
2
Thus 1 = 1
2 1 + ω2
or ω = 1 rad
or f = 1 Hz

MCQ 1.48 A Hilbert transformer is a


(A) non-linear system (B) non-causal system
(C) time-varying system (D) low-pass system
SOL 1.48 A Hilbert transformer is a non-linear system.
Hence (A) is correct answer.
MCQ 1.49 The frequency response of a linear, time-invariant system is given by
H (f) = 1 + j510πf . The step response of the system is
(B) 5 61 − e− 5@ u (t)
t
(A) 5 (1 − e−5t) u (t)
(C) 1 (1 − e−5t) u (t) (D) 1 ^1 − e− 5 h u (t)
t

2 5
SOL 1.49 Hence (B) is correct answer.
H (f) = 5
1 + j10πf
H (s) = 5 = 5 = 1
1 + 5s 5^s + 15 h s + 15
Step response Y (s) = 1 a 1
s ^s + 5 h
or Y (s) = 1 1 1 = 5 − 5 1
s ^s + 5 h s s+ 5
Brought to you by: Nodia and Company Visit us at: www.nodia.co.in
PUBLISHING FOR GATE
Page 23 GATE EC 2007 www.gatehelp.com

or y (t) = 5 (1 − e−t/5) u (t)

MCQ 1.50 A 5-point sequence x [n] is given as x [− 3] = 1, x [− 2] = 1, x [− 1] = 0, x [0] = 5 and


x [1] = 1. Let X (eiω) denoted the discrete-time Fourier transform of x [n]. The value
π
of #
−π
X (e jω) dω is

(A) 5 (B) 10π


(C) 16π (D) 5 + j10π
SOL 1.50 For discrete time Fourier transform (DTFT) when N " 3
π
x [n] = 1
2π − π #
X (e jω) e jωn dω

Putting n = 0 we get
π π
x [0] = 1
2π #
−π
X (e jω) e jω0 dω = 1
2π #
−π
X (e jω) dω
π
or #
−π
X (e jω) dω = 2πx [0] = 2π # 5 = 10π

Hence (B) is correct answer.


MCQ 1.51 The z −transform X (z) of a sequence x [n] is given by X [z] = 1 −0.25z . It is given that −1

the region of convergence of X (z) includes the unit circle. The value of x [0] is
(A) − 0.5 (B) 0
(C) 0.25 (D) 05
SOL 1.51 Hence (B) is correct answer.
X (z) = 0.5 −1
1 − 2z
Since ROC includes unit circle, it is left handed system
x (n) =− (0.5) (2) −n u (− n − 1)
x (0) = 0
If we apply initial value theorem
x (0) = lim X (z) = lim 0.5 −1 = 0.5
z"3 z " 31 − 2z

That is wrong because here initial value theorem is not applicable because signal
x (n) is defined for n < 0 .
MCQ 1.52 A control system with PD controller is shown in the figure. If the velocity error
constant KV = 1000 and the damping ratio ζ = 0.5 , then the value of KP and KD
are

Brought to you by: Nodia and Company Visit us at: www.nodia.co.in


PUBLISHING FOR GATE
Page 24 GATE EC 2007 www.gatehelp.com

(A) KP = 100, KD = 0.09 (B) KP = 100, KD = 0.9


(C) KP = 10, KD = 0.09 (D) KP = 10, KD = 0.9
SOL 1.52 Hence (B) is correct option
We have Kv = lim sG (s) H (s)
s"0

(Kp + KD s) 100
or 1000 = lim s = Kp
s"0 s (s + 100)
Now characteristics equations is
1 + G (s) H (s) = 0
(K + KD s) 100
1000 = lims " 0 s p = Kp
s (s + 100)
Now characteristics equation is
1 + G (s) H (s) = 0

(100 + KD s) 100
or 1+ =0 Kp = 100
s (s + 10)
or s2 + (10 + 100KD) s + 10 4 = 0

Comparing with s2 + 2ξωn + ωn2 = 0 we get


2ξωn = 10 + 100KD
or KD = 0.9
MCQ 1.53 The transfer function of a plant is
T (s) = 5
(s + 5)( s2 + s + 1)
The second-order approximation of T (s) using dominant pole concept is
(A) 1 (B) 5
(s + 5)( s + 1) (s + 5)( s + 1)

(C) 5 (D) 1
s2 + s + 1 s2 + s + 1
SOL 1.53 Hence (D) is correct option.
We have T (s) = 5
(s + 5)( s2 + s + 1)
= 5 = 2 1
5`1 + j (s + s + 1)
s 2
s +s+1
5
In given transfer function denominator is (s + 5)[( s + 0.5) 2 + 43 ]. We can see easily
that pole at s =− 0.5 ! j 23 is dominant then pole at s =− 5 . Thus we have
approximated it.
MCQ 1.54 The open-loop transfer function of a plant is given as G (s) = s 1- 1 . If the plant is
2

operated in a unity feedback configuration, then the lead compensator that an


stabilize this control system is

Brought to you by: Nodia and Company Visit us at: www.nodia.co.in


PUBLISHING FOR GATE
Page 25 GATE EC 2007 www.gatehelp.com

10 (s − 1) 10 (s + 4)
(A) (B)
s+2 s+2
10 (s + 2) 2 (s + 2)
(C) (D)
s + 10 s + 10
SOL 1.54 Hence (A) is correct option.
G (s) = 2 1 = 1
s −1 (s + 1)( s − 1)
The lead compensator C (s) should first stabilize the plant i.e. remove 1 term.
(s − 1)
From only options (A), C (s) can remove this term

1 10 (s − 1)
ThusG (s) C (s) = #
(s + 1)( s − 1) (s + 2)
= 10 Only option (A) satisfies.
(s + 1)( s + 2)

MCQ 1.55 A unity feedback control system has an open-loop transfer function
G (s) = K
s (s + 7s + 12)
2

The gain K for which s = 1 + j1 will lie on the root locus of this system is
(A) 4 (B) 5.5
(C) 6.5 (D) 10
SOL 1.55 For ufb system the characteristics equation is
1 + G (s) = 0
or 1+ K =0
s (s2 + 7s + 12)
or s (s2 + 7s + 12) + K = 0
Point s =− 1 + j lie on root locus if it satisfy above equation i.e
(− 1 + j)[( − 1 + j) 2 + 7 (− 1 + j) + 12) + K] = 0
or K =+ 10
Hence (D) is correct option.
MCQ 1.56 The asymptotic Bode plot of a transfer function is as shown in the figure. The
transfer function G (s) corresponding to this Bode plot is

(A) 1 (B) 1
(s + 1)( s + 20) s (s + 1)( s + 20)
Brought to you by: Nodia and Company Visit us at: www.nodia.co.in
PUBLISHING FOR GATE
Page 26 GATE EC 2007 www.gatehelp.com

(C) 100 (D) 100


s (s + 1)( s + 20) s (s + 1)( 1 + 0.05s)
SOL 1.56 At every corner frequency there is change of -20 db/decade in slope which indicate
pole at every corner frequency. Thus
G (s) = K
s (1 + s)`1 + s j
20
Bode plot is in (1 + sT) form
20 log K = 60 dB = 1000
ω ω = 0. 1
Thus K =5
Hence G (s) = 100
s (s + 1)( 1 + .05s)
Hence (D) is correct option.
MCQ 1.57 The state space representation of a separately excited DC servo motor dynamics
is given as

−1 1 ω
> di H = =− 1 − 10G=ia G + =10Gu
dt 0
o
dt

where ω is the speed of the motor, ia is the armature current and u is the armature
ω (s)
voltage. The transfer function of the motor is
U (s)
(A) 2 10 (B) 2 1
s + 11s + 11 s + 11s + 11
(C) 2 10s + 10 (D) 2 1
s + 11s + 11 s + s + 11

SOL 1.57 Hence (A) is correct option.



−1 1 ω
We have > didt H = =
0
a G=
− 1 − 10 in G + =10G
u
dt

or dω =− ω + i ...(1)
n
dt
and dia =− ω − 10i + 10u ...(2)
a
dt
Taking laplace transform (i) we get
sω (s) =− ω (s) = Ia (s)
or (s + 1) ω (s) = Ia (s) ...(3)
Taking laplace transform (ii) we get
sIa (s) =− ω (s) − 10Ia (s) + 10U (s)
or ω (s) = (− 10 − s) Ia (s) + 10U (s)
= (− 10 − s)( s + 1) ω (s) + 10U (s) From (3)
or ω (s) =− [s2 + 11s + 10] ω (s) + 10U (s)

Brought to you by: Nodia and Company Visit us at: www.nodia.co.in


PUBLISHING FOR GATE
Page 27 GATE EC 2007 www.gatehelp.com

or(s2 + 11s + 11) ω (s) = 10U (s)


ω (s)
or = 2 10
U (s) (s + 11s + 11)
MCQ 1.58 In delta modulation, the slope overload distortion can be reduced by
(A) decreasing the step size (B) decreasing the granular noise
(C) decreasing the sampling rate (D) increasing the step size
SOL 1.58 Slope overload distortion can be reduced by increasing the step size
3 $ slope of x (t)
Ts
Hence (D) is correct option.
MCQ 1.59 The raised cosine pulse p (t) is used for zero ISI in digital communications. The
expression for p (t) with unity roll-off factor is given by
p (t) = sin 4πWt
4πWt (1 − 16W2 t2)
The value of p (t) at t = 1 is
4W
(A) − 0.5 (B) 0
(C) 0.5 (D) 3
SOL 1.59 Hence (C) is correct option.
sin (4πWt)
We have p (t) =
4πWt (1 − 16W2 t2)
at t = 1 it is 0 form. Thus applying L' Hospital rule
4W 0
4πW cos (4πWt)
p( ) =
1
4W

4πW [1 − 48W2 t2]


cos (4πWt)
= = cos π = 0.5
1 − 48W t
2 2 1−3

MCQ 1.60 In the following scheme, if the spectrum M (f) of m (t) is as shown, then the spectrum
Y (f) of y (t) will be

Brought to you by: Nodia and Company Visit us at: www.nodia.co.in


PUBLISHING FOR GATE
Page 28 GATE EC 2007 www.gatehelp.com

SOL 1.60 The block diagram is as shown below

Here M1 (f) = Mt (f)


j 2π B
Y1 (f) = M (f) c e + e -j2πB
2 m
j 2π B
Y2 (f) = M1 (f) c e − e -j2πB
2 m

Y (f) = Y1 (f) + Y2 (f)


All waveform is shown below

Brought to you by: Nodia and Company Visit us at: www.nodia.co.in


PUBLISHING FOR GATE
Page 29 GATE EC 2007 www.gatehelp.com

Hence (B) is correct option.


MCQ 1.61 During transmission over a certain binary communication channel, bit errors occur
independently with probability p. The probability of AT MOST one bit in error
in a block of n bits is given by
(A) pn (B) 1 − pn
(C) np (1 − p) n - 1 + (1 + p) n (D) 1 − (1 − p) n
SOL 1.61 By Binomial distribution the probability of error is
pe = n Cr pr (1 − p) n - r
Probability of at most one error
= Probability of no error + Probability of one error
= n C0 p0 (1 − p) n - 0 + n C1 p1 (1 − p) n - 1

= (1 − p) n + np (1 − p) n - 1
Hence (C) is correct option.
MCQ 1.62 In a GSM system, 8 channels can co-exist in 200 kHz bandwidth using TDMA.
A GSM based cellular operator is allocated 5 MHz bandwidth. Assuming a
frequency reuse factor of 1 , i.e. a five-cell repeat pattern, the maximum number of
5
simultaneous channels that can exist in one cell is
(A) 200 (B) 40
(C) 25 (D) 5
SOL 1.62 Bandwidth allocated for 1 Channel = 5 M Hz
Average bandwidth for 1 Channel 5 = 1 MHz
5
Total Number of Simultaneously Channel = 1M # 8 = 40 Channel
200k
Hence (B) is correct option.
MCQ 1.63 In a Direct Sequence CDMA system the chip rate is 1.2288 # 106 chips per second.
If the processing gain is desired to be AT LEAST 100, the data rate
(A) must be less than or equal to 12.288 # 103 bits per sec
(B) must be greater than 12.288 # 103 bits per sec
(C) must be exactly equal to 12.288 # 103 bits per sec
(D) can take any value less than 122.88 # 103 bits per sec
SOL 1.63 Hence (A) is correct option.
Chip Rate RC = 1.2288 # 106 chips/sec

Brought to you by: Nodia and Company Visit us at: www.nodia.co.in


PUBLISHING FOR GATE
Page 30 GATE EC 2007 www.gatehelp.com

Data Rate Rb = RC
G
Since the processing gain G must be at least 100, thus for Gmin we get
6
Rb max = RC = 1.2288 # 10 = 12.288 # 103 bps
Gmin 100

MCQ 1.64 An air-filled rectangular waveguide has inner dimensions of 3 cm # 2 cm. The wave
impedance of the TE20 mode of propagation in the waveguide at a frequency of 30
GHz is (free space impedance η0 = 377 Ω )
(A) 308 Ω (B) 355 Ω
(C) 400 Ω (D) 461 Ω
SOL 1.64 The cut-off frequency is
fc = c ` m j2 + ` n j2
2 a b
Since the mode is TE20, m = 2 and n = 0
8
fc = c m = 3 # 10 # 2 = 10 GHz
2 2 2 # 0.03
ηo 377
η' = fc 2
= 10 = 400Ω
1−c m
2
1 − c 10 10 m
f 3 # 10
Hence (C) is correct option.
MCQ 1.65 The H field (in A/m) of a plane wave propagating in free space is given by
H = xt 5 3 cos (ωt − βz) + yt`ωt − βz + π j .
η0 2
The time average power flow density in Watts is
η
(A) 0 (B) 100
100 η0

(C) 50η20 (D) 50


η0
SOL 1.65 Hence (D) is correct option.
2
= Hx2 + Hy2 = c 5 3 m + c 5 m = c 10 m
2 2
We have H 2
ηo ηo ηo
E 2 ηo H 2
η 2
For free space P = = = o c 10 m = 50 watts
2ηo 2 2 ηo ηo

MCQ 1.66 The E field in a rectangular waveguide of inner dimension a # b is given by


ωμ
E = 2 ` λ j H0 sin ` 2πx j sin (ωt − βz) yt
2

h 2 a
Where H0 is a constant, and a and b are the dimensions along the x − axis and the
y − axis respectively. The mode of propagation in the waveguide is
(A) TE20 (B) TM11
(C) TM20 (D) TE10

Brought to you by: Nodia and Company Visit us at: www.nodia.co.in


PUBLISHING FOR GATE
Page 31 GATE EC 2007 www.gatehelp.com

SOL 1.66 Hence (A) is correct option.


ωμ
E = 2 ` π j H0 sin ` 2πx j sin (ωt − βz) yt
2

h 2 a
This is TE mode and we know that
mπy
Ey \ sin ` mπx j cos `
a b j
Thus m = 2 and n = 0 and mode is TE20
MCQ 1.67 A load of 50 Ω is connected in shunt in a 2-wire transmission line of Z0 = 50Ω as
shown in the figure. The 2-port scattering parameter matrix (s-matrix) of the shunt
element is

−1 1
(A) > 12 1H
0 1
(B) =
1 0G
2
2 − 2

− 13 2 1
− 43
(C) > 3
H (D) > 4
1H
2
3 − 1
3 −3
4 4
SOL 1.67 The 2-port scattering parameter matrix is
S11 S12
S ==
S21 S22 G
(Z Z ) − Zo (50 50) − 50
S11 = L 0 = =− 1
(ZL Z0) + Zo (50 50) + 50 3
2 (ZL Zo) 2 (50 50)
S12 = S21 = = =2
(ZL Zo) + Zo (50 50) + 50 3
(Z Z ) − Zo (50 50) − 50
S22 = L o = =− 1
(ZL Zo) + Zo (50 50) + 50 3
Hence (C) is correct option.
MCQ 1.68 The parallel branches of a 2-wire transmission line re terminated in 100Ω and
200Ω resistors as shown in the figure. The characteristic impedance of the line is
Z0 = 50Ω and each section has a length of λ . The voltage reflection coefficient Γ
4
at the input is

Brought to you by: Nodia and Company Visit us at: www.nodia.co.in


PUBLISHING FOR GATE
Page 32 GATE EC 2007 www.gatehelp.com

(A) − j 7 (B) − 5
5 7

(C) j 5 (D) 5
7 7
SOL 1.68 The input impedance is
if l = λ
2
Zin = Zo ;
ZL 4
2 2
Zin1 = Zo1 = 50 = 25
ZL1 100
2 2
Zin2 = Zo2 = 50 = 12.5
ZL2 200
Now ZL = Zin1 Zin2
25 12.5 = 25
3
(50) 2
Zs = = 300
25/3
Γ = ZS − Zo = 300 − 50 = 5
ZS + Zo 300 + 50 7
Hence (D) is correct option.

MCQ 1.69 A λ dipole is kept horizontally at a height of λ0 above a perfectly conducting


2 2
infinite ground plane. The radiation pattern in the lane of the dipole (E plane)
looks approximately as

Brought to you by: Nodia and Company Visit us at: www.nodia.co.in


PUBLISHING FOR GATE
Page 33 GATE EC 2007 www.gatehelp.com

SOL 1.69 Using the method of images, the configuration is as shown below

Here d = λ, α = π, thus βd = 2π

Array factor is
βd cos ψ + α
= cos ; E
2
2π cos ψ + π
= cos ; E = sin (π cos ψ)
2
Hence (B) is correct option.
MCQ 1.70 A right circularly polarized (RCP) plane wave is incident at an angle 60c to the
normal, on an air-dielectric interface. If the reflected wave is linearly polarized, the
relative dielectric constant ξr2 is.

(A) 2 (B) 3
(C) 2 (D) 3
SOL 1.70 The Brewster angle is
tan θn = εr2
εr1
tan 60c = εr2
1
or εr2 = 3

Brought to you by: Nodia and Company Visit us at: www.nodia.co.in


PUBLISHING FOR GATE
Page 34 GATE EC 2007 www.gatehelp.com

Hence (D) is correct option.

Common Data Questions

Common Data for Questions 71, 72, 73 :


The figure shows the high-frequency capacitance - voltage characteristics of Metal/
Sio 2 /silicon (MOS) capacitor having an area of 1 # 10 - 4 cm 2 . Assume that the
permittivities (ε0 εr ) of silicon and Sio2 are 1 # 10 - 12 F/cm and 3.5 # 10 - 13 F/cm
respectively.

MCQ 1.71 The gate oxide thickness in the MOS capacitor is


(A) 50 nm (B) 143 nm
(C) 350 nm (D) 1 μm
SOL 1.71 At low voltage when there is no depletion region and capacitance is decide by SiO2
thickness only,
C = ε0 εr1 A
D
−13
or D = ε0 εr1 A = 3.5 # 10 −# 10−4 = 50 nm
C 7 # 10 12
Hence option (A) is correct
MCQ 1.72 The maximum depletion layer width in silicon is
(A) 0.143 μm (B) 0.857 μm
(C) 1 μm (D) 1.143 μm
SOL 1.72 The construction of given capacitor is shown in fig below

Brought to you by: Nodia and Company Visit us at: www.nodia.co.in


PUBLISHING FOR GATE
Page 35 GATE EC 2007 www.gatehelp.com

When applied voltage is 0 volts, there will be no depletion region and we get
C1 = 7 pF
When applied voltage is V , a depletion region will be formed as shown in fig an
total capacitance is 1 pF. Thus
CT = 1 pF
or CT = C1 C2 = 1 pF
C1 + C2
or 1 = 1 + 1
CT C1 C2
Substituting values of CT and C1 we get
C2 = 7 pF
6
D2 = ε0 εr2 A = 1 # 710 #- 1210 = 6 # 10 - 4 cm
- 12 -4
Now
C2 6 # 10
7
= 0.857 μm
Hence option (B) is correct.
MCQ 1.73 Consider the following statements about the C − V characteristics plot :
S1 : The MOS capacitor has as n -type substrate
S2 : If positive charges are introduced in the oxide, the C − V polt will shift to the
left.
Then which of the following is true?
(A) Both S1 and S2 are true
(B) S1 is true and S2 is false
(C) S1 is false and S2 is true
(D) Both S1 and S2 are false
SOL 1.73 Depletion region will not be formed if the MOS capacitor has n type substrate but
from C-V characteristics, C reduces if V is increased. Thus depletion region must
be formed. Hence S1 is false
If positive charges is introduced in the oxide layer, then to equalize the effect the
applied voltage V must be reduced. Thus the C − V plot moves to the left. Hence
S2 is true.
Hence option (C) is correct.

Brought to you by: Nodia and Company Visit us at: www.nodia.co.in


PUBLISHING FOR GATE
Page 36 GATE EC 2007 www.gatehelp.com

Common Data for Questions 74 & 75 :


Two 4-array signal constellations are shown. It is given that φ1 and φ2 constitute
an orthonormal basis for the two constellation. Assume that the four symbols in
both the constellations are equiprobable. Let N0 denote the power spectral density
2
of white Gaussian noise.

MCQ 1.74 The if ratio or the average energy of Constellation 1 to the average energy of
Constellation 2 is
(A) 4a2 (B) 4
(C) 2 (D) 8
SOL 1.74 Energy of constellation 1 is
Eg1 = (0) 2 + (− 2 a) 2 + (− 2 a) 2 + ( 2 a) 2 + (− 2 2 a) 2

= 2a2 + 2a2 + 2a2 + 8a2 = 16a2


Energy of constellation 2 is
Eg2 = a2 + a2 + a2 + a2 = 4a2

Eg1 16a2
Ratio = = =4
Eg2 4a2
Hence (B) is correct option.
MCQ 1.75 If these constellations are used for digital communications over an AWGN channel,
then which of the following statements is true ?
(A) Probability of symbol error for Constellation 1 is lower
(B) Probability of symbol error for Constellation 1 is higher
(C) Probability of symbol error is equal for both the constellations
(D) The value of N0 will determine which of the constellations has a lower probability
of symbol error

SOL 1.75 Noise Power is same for both which is N0 .


2
Thus probability of error will be lower for the constellation 1 as it has higher signal
energy.
Hence (A) is correct option.

Brought to you by: Nodia and Company Visit us at: www.nodia.co.in


PUBLISHING FOR GATE
Page 37 GATE EC 2007 www.gatehelp.com

Linked Answer Questions : Q.76 to Q.85 carry two marks each.

Statement for Linked Answer Questions 76 & 77:


Consider the Op-Amp circuit shown in the figure.

MCQ 1.76 The transfer function V0 (s)/ Vi (s) is


(A) 1 − sRC (B) 1 + sRC
1 + sRC 1 − sRC

(C) 1 (D) 1
1 − sRC 1 + sRC
SOL 1.76 The voltage at non-inverting terminal is
1
V+ = sC 1 Vi = 1 V
R + sC 1 + sCR i
Now V- = V+ = 1 V
1 + sCR i
Applying voltage division rule
(V + Vi)
V+ = R1 (V0 + Vi) = o
R1 + R1 2
1 (V + Vi)
or Vi = o
1 + sCR 2
or Vo =− 1 + 2
Vi 1 + sRC
V0 = 1 − sRC
Vi 1 + sRC
Hence (A) is correct option.

MCQ 1.77 If Vi = V1 sin (ωt) and V0 = V2 sin (ωt + φ), then the minimum and maximum values
of φ (in radians) are respectively
(A) − π and π (B) 0 and π
2 2 2

(C) − π and 0 (D) − π and 0


2
SOL 1.77 Hence (C) is correct option.
V0 = H (s) = 1 − sRC
Vi 1 + sRC
Brought to you by: Nodia and Company Visit us at: www.nodia.co.in
PUBLISHING FOR GATE
Page 38 GATE EC 2007 www.gatehelp.com

1 − jωRC
H (jω) =
1 + jωRC
+H (jω) = φ =− tan - 1 ωRC − tan - 1 ωRC
=− 2 tan - 2 ωRC
Minimum value, φmin = − π (at ω " 3)
Maximum value, φmax = 0( at ω = 0)

Statement for Linked Answer Questions 78 & 79 :


An 8085 assembly language program is given below.
Line 1: MVI A, B5H
2: MVI B, OEH
3: XRI 69H
4: ADD B
5: ANI 9BH
6: CPI 9FH
7: STA 3010H
8: HLT
MCQ 1.78 The contents of the accumulator just execution of the ADD instruction in line 4
will be
(A) C3H (B) EAH
(C) DCH (D) 69H
SOL 1.78 Line 1 : MVI A, B5H ; Move B5H to A
2 : MVI B, 0EH ; Move 0EH to B
3 : XRI 69H ; [A] XOR 69H and store in A
; Contents of A is CDH
4 : ADDB ; Add the contents of A to contents of B and
; store in A, contents of A is EAH
5 : ANI 9BH ; [a] AND 9BH, and store in A,
; Contents of A is 8 AH
6 : CPI 9FH ; Compare 9FH with the contents of A
; Since 8 AH < 9BH, CY = 1
7 : STA 3010 H ; Store the contents of A to location 3010 H
8 : HLT ; Stop
Thus the contents of accumulator after execution of ADD instruction is EAH.
Hence (B) is correct answer.
MCQ 1.79 After execution of line 7 of the program, the status of the CY and Z flags will be
(A) CY = 0, Z = 0 (B) CY = 0, Z = 1

Brought to you by: Nodia and Company Visit us at: www.nodia.co.in


PUBLISHING FOR GATE
Page 39 GATE EC 2007 www.gatehelp.com

(C) CY = 1, Z = 0 (D) CY = 1, Z = 1
SOL 1.79 The CY = 1 and Z = 0
Hence (C) is correct answer.

Statement for linked Answer Question 80 & 81 :


Consider a linear system whose state space representation is x (t) = Ax (t). If
1
the initial state vector of the system is x (0) = = G, then the system response is
−2
e−2x 1
−2tH . If the itial state vector of the system changes to x (0) = =
− 2G
x (t) = > , then
− 2e −t
e
the system response becomes x (t) = > −tH
−e
MCQ 1.80 The eigenvalue and eigenvector pairs (λi vi) for the system are
1 1 1 1
(A) e− 1 = Go and e− 2 = Go (B) e− 1, = Go and e2, = Go
−1 −2 −1 −2
1 1 1 1
(C) e− 1, = Go and e− 2, = Go (D) e− 2 = Go and e1, = Go
−1 −2 −1 −2

SOL 1.80 Hence (A) is correct option.


We have xo (t) = Ax (t)
p q
A ==
r sG
Let

1 e−2t
For initial state vector x (0) = = G the system response is x (t) = > H
−2 − 2e−2t
d −2t
e p q 1
==
r s G=− 2G
Thus > d dt −2t H
dt (− 2e )
t=0
−2 (0)
− 2e p q 1
or > 4e−2 (0) H = =r s G=− 2G
−2 p − 2q
= 4 G = = r − 2s G

We get p − 2q =− 2 and r − 2s = 4 ...(i)


1 e−t
For initial state vector x (0) = = G the system response is x (t) = > −tH
−1 −e
d
e−t p q 1
==
> d (− e−t)H r s G=− 1G
dt
Thus
dt
t=0
− (0)
−e p q 1
> e− (0) H = =r s G=− 1G
−1 p−q
= 1G = = r − s G

Brought to you by: Nodia and Company Visit us at: www.nodia.co.in


PUBLISHING FOR GATE
Page 40 GATE EC 2007 www.gatehelp.com

We get p − q =− 1 and r − s = 1 ...(2)


Solving (1) and (2) set of equations we get
p q 0 1
=r s G = =− 2 − 3G

The characteristic equation


λI − A = 0
λ −1
=0
2 λ+3
or λ (λ + 3) + 2 = 0
or λ =− 1, − 2
Thus Eigen values are − 1 and − 2
Eigen vectors for λ1 =− 1
(λ1 I − A) X1 = 0
λ1 − 1 x11
or = 2 λ + 3G=x G = 0
1 21

− 1 − 1 x11
= 2 2 G=x G = 0
21

or − x11 − x21 = 0
or x11 + x21 = 0
We have only one independent equation x11 =− x21.Let x11 = K , then x21 =− K , the
Eigen vector will be
x11 K 1
=x G = =− K G = K =− 1G
21

Now Eigen vector for λ2 =− 2


(λ2 I − A) X2 = 0
λ2 − 1 x12
or = 2 λ + 3G=x G = 0
2 22

− 2 − 1 x11
or = 2 1 G=x G = 0
21

or − x11 − x21 = 0
or x11 + x21 = 0
We have only one independent equation x11 =− x21.
Let x11 = K, then x21 =− K , the Eigen vector will be
x12 K 1
=x G = =− 2K G = K =− 2G
22

MCQ 1.81 The system matrix A is


0 1 1 1
(A) =
− 1 1G
(B) =
− 1 − 2G

Brought to you by: Nodia and Company Visit us at: www.nodia.co.in


PUBLISHING FOR GATE
Page 41 GATE EC 2007 www.gatehelp.com

2 1 0 1
(C) = G (D) =
−1 −1 − 2 − 3G
SOL 1.81 As shown in previous solution the system matrix is
0 1
A ==
− 2 − 3G
Hence (D) is correct option.

Statement for Linked Answer Question 82 & 83 :


An input to a 6-level quantizer has the probability density function f (x) as shown
in the figure. Decision boundaries of the quantizer are chosen so as to maximize the
entropy of the quantizer output. It is given that 3 consecutive decision boundaries
are’ − 1'.'0' and '1' .

MCQ 1.82 The values of a and b are


(A) a = 1 and b = 1 (B) a = 1 and b = 3
6 12 5 40

(C) a = 1 and b = 1 (D) a = 1 and b = 1


4 16 3 24
SOL 1.82 Area under the pdf curve must be unity
Thus 2a + 4a + 4b =1
2a + 8b = 1 ...(1)
For maximum entropy three region must by equivaprobable thus
2a = 4b = 4b ...(2)
From (1) and (2) we get
b = 1 and a = 1
12 6
Hence (A) is correct option.
MCQ 1.83 Assuming that the reconstruction levels of the quantizer are the mid-points of the
decision boundaries, the ratio of signal power to quantization noise power is
(A) 152 (B) 64
9 3

(C) 76 (D) 28
3
SOL 1.83 Hence correct option is ( )

Brought to you by: Nodia and Company Visit us at: www.nodia.co.in


PUBLISHING FOR GATE
Page 42 GATE EC 2007 www.gatehelp.com

Statement for Linked Answer Question 84 and 85 :


In the Digital-to-Analog converter circuit shown in the figure below,
VR = 10V and R = 10kΩ

MCQ 1.84 The current i is


(A) 31.25μA (B) 62.5μA
(C) 125μA (D) 250μA
SOL 1.84 Since the inverting terminal is at virtual ground the resistor network can be reduced
as follows

The current from voltage source is


I = VR = 10 = 1 mA
R 10k
This current will be divide as shown below

Brought to you by: Nodia and Company Visit us at: www.nodia.co.in


PUBLISHING FOR GATE
Page 43 GATE EC 2007 www.gatehelp.com

-3
Now i = I = 1 # 10 = 62.5 μ A
16 16
Hence (B) is correct answer.
MCQ 1.85 The voltage V0 is
(A) − 0.781 V (B) − 1.562 V
(C) − 3.125 V (D) − 6.250 V
SOL 1.85 The net current in inverting terminal of OP - amp is
I- = 1 + 1 = 5I
4 16 16
So that V0 =− R # 5I =− 3.125
16
Hence (C) is correct answer.

Answer Sheet
1. (A) 19. (A) 37. (B) 55. (D) 73. (C)
2. (B) 20. (D) 38. (D) 56. (D) 74. (B)
3. (C) 21. (C) 39. (D) 57. (A) 75. (A)
4. (A) 22. (A) 40. (C) 58. (D) 76. (A)
5. (D) 23. (C) 41. (D) 59. (C) 77. (C)
6. (A) 24. (D) 42. (B) 60. (B) 78. (B)
7. (D) 25. (B) 43. (A) 61. (C) 79. (C)
8. (C) 26. (C) 44. (C) 62. (B) 80. (A)
9. (D) 27. (A) 45. (A) 63. (A) 81. (D)
10. (C) 28. (D) 46. (C) 64. (C) 82. (A)
11. (C) 29. (D) 47. (A) 65. (D) 83. (*)
12. (A) 30. (A) 48. (A) 66. (A) 84. (B)
13. (C) 31. (D) 49. (B) 67. (C) 85. (C)

Brought to you by: Nodia and Company Visit us at: www.nodia.co.in


PUBLISHING FOR GATE
Page 44 GATE EC 2007 www.gatehelp.com

14. (B) 32. (A) 50. (B) 68. (D)


15. (D) 33. (B) 51. (B) 69. (B)
16. (D) 34. (B) 52. (B) 70 (D)
17. (C) 35. (C) 53. (D) 71 (A)
18. (B) 36. (C) 54. (A) 72 (B)
**********

Brought to you by: Nodia and Company Visit us at: www.nodia.co.in


PUBLISHING FOR GATE
GATE EC
2008

Q.1 - Q.20 carry one mark each.


p11 p12
MCQ 1.1 All the four entries of the 2 # 2 matrix = = p21 p22 G are nonzero, and one of its
eigenvalue is zero. Which of the following statements is true?
(A) p11 p12 − p12 p21 = 1 (B) p11 p22 − p12 p21 =− 1
(C) p11 p22 − p12 p21 = 0 (D) p11 p22 + p12 p21 = 0
SOL 1.1 The product of Eigen value is equal to the determinant of the matrix. Since one of
the Eigen value is zero, the product of Eigen value is zero, thus determinant of the
matrix is zero.
Thus p11 p22 − p12 p21 = 0
Hence (C) is correct answer.
MCQ 1.2 The system of linear equations
4x + 2y = 7
2x + y = 6 has
(A) a unique solution (B) no solution
(C) an infinite number of solutions (D) exactly two distinct solutions
SOL 1.2 The given system is
4 2 x 7
=2 1G=y G = = 6 G
4 2
A ==
2 1G
We have
4 2
and A = =0 Rank of matrix ρ (A) < 2
2 1
4 2 7
Now C == G Rank of matrix ρ (C) = 2
2 1 6
Since ρ (A) ! ρ (C) there is no solution.
Hence (B) is correct answer.
MCQ 1.3 The equation sin (z) = 10 has

Brought to you by: Nodia and Company Visit us at: www.nodia.co.in


PUBLISHING FOR GATE
Page 2 GATE EC 2008 www.gatehelp.com

(A) no real or complex solution


(B) exactly two distinct complex solutions
(C) a unique solution
(D) an infinite number of complex solutions
SOL 1.3 sin z can have value between − 1 to + 1. Thus no solution.
Hence (A) is correct solution.
MCQ 1.4 For real values of x , the minimum value of the function
f (x) = exp (x) + exp (− x) is
(A) 2 (B) 1
(C) 0.5 (D) 0
SOL 1.4 Hence (A) is correct answer.
We have f (x) = ex + e−x
For x > 0 , ex > 1 and 0 < e−x < 1
For x < 0 , 0 < ex < 1 and e−x > 1
Thus f (x) have minimum values at x = 0 and that is e0 + e−0 = 2 .
MCQ 1.5 Which of the following functions would have only odd powers of x in its Taylor
series expansion about the point x = 0 ?
(A) sin (x3) (B) sin (x2)
(C) cos (x3) (D) cos (x2)
SOL 1.5 Hence (A) is correct answer.
3 5
sin x = x + x + x + ...
3! 5!
2 4
cos x = 1 + x + x + ...
2! 4!
Thus only sin (x3) will have odd power of x .
dx (t)
MCQ 1.6 Which of the following is a solution to the differential equation + 3x (t) = 0 ?
dt
(A) x (t) = 3e - t (B) x (t) = 2e - 3t
(C) x (t) =− 23 t2 (D) x (t) = 3t2
SOL 1.6 Hence (B) is correct answer.
dx (t)
We have + 3x (t) = 0
dt
or (D + 3) x (t) = 0
Since m =− 3 , x (t) = Ce - 3t Thus only (B) may be solution.
MCQ 1.7 In the following graph, the number of trees (P) and the number of cut-set (Q) are

Brought to you by: Nodia and Company Visit us at: www.nodia.co.in


PUBLISHING FOR GATE
Page 3 GATE EC 2008 www.gatehelp.com

(A) P = 2, Q = 2 (B) P = 2, Q = 6
(C) P = 4, Q = 6 (D) P = 4, Q = 10
SOL 1.7 The given graph is

There can be four possible tree of this graph which are as follows:

There can be 6 different possible cut-set.

Hence (C) is correct option.


MCQ 1.8 In the following circuit, the switch S is closed at t = 0 . The rate of change of
current di (0+) is given by
dt

(A) 0 (B) Rs Is
L
(R + Rs) Is
(C) (D) 3
L
SOL 1.8 Initially i (0−) = 0 therefore due to inductor i (0+) = 0 . Thus all current Is will flow
in resistor R and voltage across resistor will be Is Rs . The voltage across inductor
will be equal to voltage across Rs as no current flow through R.

Brought to you by: Nodia and Company Visit us at: www.nodia.co.in


PUBLISHING FOR GATE
Page 4 GATE EC 2008 www.gatehelp.com

Thus vL (0+) = Is Rs
di (0+)
but vL (0+) = L
dt
di (0+) v (0+) Is Rs
Thus = L =
dt L L
Hence (B) is correct option.
MCQ 1.9 The input and output of a continuous time system are respectively denoted by
x (t) and y (t). Which of the following descriptions corresponds to a causal system ?
(A) y (t) = x (t − 2) + x (t + 4) (B) y (t) = (t − 4) x (t + 1)
(C) y (t) = (t + 4) x (t − 1) (D) y (t) = (t + 5) x (t + 5)
SOL 1.9 The output of causal system depends only on present and past states only.
In option (A) y (0) depends on x (− 2) and x (4).
In option (B) y (0) depends on x (1).
In option (C) y (0) depends on x (− 1).
In option (D) y (0) depends on x (5).
Thus only in option (C) the value of y (t) at t = 0 depends on x (− 1) past value. In
all other option present value depends on future value.
Hence (C) is correct answer
MCQ 1.10 The impulse response h (t) of a linear time invariant continuous time system is
described by h (t) = exp (αt) u (t) + exp (βt) u (− t) where u (− t) denotes the unit
step function, and α and β are real constants. This system is stable if
(A) α is positive and β is positive
(B) α is negative and β is negative
(C) α is negative and β is negative
(D) α is negative and β is positive
SOL 1.10 Hence (D) is correct answer.
We have h (t) = eαt u (t) + e βt u (− t)
This system is stable only when bounded input has bounded output For stability
αt < 0 for t > 0 that implies α < 0 and βt > 0 for t > 0 that implies β > 0 . Thus,
α is negative and β is positive.
MCQ 1.11 The pole-zero given below correspond to a

Brought to you by: Nodia and Company Visit us at: www.nodia.co.in


PUBLISHING FOR GATE
Page 5 GATE EC 2008 www.gatehelp.com

(A) Law pass filter (B) High pass filter


(C) Band filter (D) Notch filter
SOL 1.11 Percent overshoot depends only on damping ratio, ξ .
Mp = e− ξπ 1 − ξ
2

If Mp is same then ξ is also same and we get


ξ = cos θ
Thus θ = constant
The option (C) only have same angle.
Hence (C) is correct option.
MCQ 1.12 Step responses of a set of three second-order underdamped systems all have the
same percentage overshoot. Which of the following diagrams represents the poles
of the three systems ?

SOL 1.12 Transfer function for the given pole zero plot is:
(s + Z1)( s + Z2)
(s + P1)( s + P2)
From the plot Re (P1 and P2 )>(Z1 and Z2 )
So, these are two lead compensator.
Hence both high pass filters and the system is high pass filter.

Brought to you by: Nodia and Company Visit us at: www.nodia.co.in


PUBLISHING FOR GATE
Page 6 GATE EC 2008 www.gatehelp.com

Hence (C) is correct option.


MCQ 1.13 Which of the following is NOT associated with a p − n junction ?
(A) Junction Capacitance (B) Charge Storage Capacitance
(C) Depletion Capacitance (D) Channel Length Modulations
SOL 1.13 Channel length modulation is not associated with a p − n junction. It is being
associated with MOSFET in which effective channel length decreases, producing
the phenomenon called channel length modulation.
Hence option (D) is correct.
MCQ 1.14 Which of the following is true?
(A) A silicon wafer heavily doped with boron is a p+ substrate
(B) A silicon wafer lightly doped with boron is a p+ substrate
(C) A silicon wafer heavily doped with arsenic is a p+ substrate
(D) A silicon wafer lightly doped with arsenic is a p+ substrate
SOL 1.14 Trivalent impurities are used for making p − type semiconductors. So, Silicon wafer
heavily doped with boron is a p+ substrate.
Hence option (A) is correct
MCQ 1.15 For a Hertz dipole antenna, the half power beam width (HPBW) in the E -plane is
(A) 360c (B) 180c
(C) 90c (D) 45c
SOL 1.15 The beam-width of Hertizian dipole is 180c and its half power beam-width is 90c.
Hence (C) is correct option
MCQ 1.16 For static electric and magnetic fields in an inhomogeneous source-free medium,
which of the following represents the correct form of Maxwell’s equations ?
(A) 4$ E = 0 , 4# B = 0 (B) 4$ E = 0 , 4$ B = 0
(C) 4# E = 0 , 4# B = 0 (D) 4# E = 0 , 4$ B = 0
SOL 1.16 Maxwell equations
4− B = 0
4$ E = ρ/E
4# E =− B
4# Ht = D + J
For static electric magnetic fields
4$ B = 0
4$ E = ρ/E
4# E = 0
S
4# H = J
Hence (D) is correct option

Brought to you by: Nodia and Company Visit us at: www.nodia.co.in


PUBLISHING FOR GATE
Page 7 GATE EC 2008 www.gatehelp.com

MCQ 1.17 In the following limiter circuit, an input voltage Vi = 10 sin 100πt is applied. Assume
that the diode drop is 0.7 V when it is forward biased. When it is forward biased.
The zener breakdown voltage is 6.8 V
The maximum and minimum values of the output voltage respectively are

(A) 6.1 V, − 0.7 V (B) 0.7 V, − 7.5 V


(C) 7.5 V, − 0.7 V (D) 7.5 V, − 7.5 V
SOL 1.17 For the positive half of Vi , the diode D1 is forward bias, D2 is reverse bias and the
zener diode is in breakdown state because Vi > 6.8 .
Thus output voltage is
V0 = 0.7 + 6.8 = 7.5 V
For the negative half of Vi, D2 is forward bias thus
Then V0 =− 0.7 V
Hence (C) is correct option
MCQ 1.18 A silicon wafer has 100 nm of oxide on it and is furnace at a temperature above
1000c C for further oxidation in dry oxygen. The oxidation rate
(A) is independent of current oxide thickness and temperature
(B) is independent of current oxide thickness but depends on temperature
(C) slows down as the oxide grows
(D) is zero as the existing oxide prevents further oxidation
SOL 1.18 Oxidation rate is zero because the existing oxide prevent the further oxidation.
Hence option (D) is correct.
MCQ 1.19 The drain current of MOSFET in saturation is given by ID = K (VGS − VT ) 2 where
K is a constant.
The magnitude of the transconductance gm is

K (VGS − VT ) 2
(A) (B) 2K (VGS − VT )
VDS
Id K (VGS − VT ) 2
(C) (D)
VGS − VDS VGS
SOL 1.19 Hence option (B) is correct.
gm = 2ID = 2 K (VGS − VT ) 2 = 2K (VGS − VT )
2VGS 2VGS

Brought to you by: Nodia and Company Visit us at: www.nodia.co.in


PUBLISHING FOR GATE
Page 8 GATE EC 2008 www.gatehelp.com

MCQ 1.20 Consider the amplitude modulated (AM) signalAc cos ωc t + 2 cos ωm t cos ωc t . For
demodulating the signal using envelope detector, the minimum value of Ac should
be
(A) 2 (B) 1
(C) 0.5 (D) 0
SOL 1.20 Hence (A) is correct option
We have xAM (t) = Ac cos ωc + 2 cos ωm t cos ωc t

= AC c1 + 2 cos ωm t m cos ωc t
Ac
For demodulation by envelope demodulator modulation index must be less than or
equal to 1.
Thus 2 #1
Ac
Ac $ 2
Hence minimum value of Ac = 2

Q.21 to Q.75 carry two marks each

MCQ 1.21 The Thevenin equivalent impedance Zth between the nodes P and Q in the following
circuit is

(A) 1 (B) 1 + s + 1
s

(D) s2 + s + 1
2
(C) 2 + s + 1
s s + 2s + 1
SOL 1.21 Killing all current source and voltage sources we have,

Zth = (1 + s) ( s1 + 1)
(1 + s)( s1 + 1) [ s1 + 1 + 1 + s]
= =
(1 + s) + ( s1 + 1) s + s1 + 1 + 1

Brought to you by: Nodia and Company Visit us at: www.nodia.co.in


PUBLISHING FOR GATE
Page 9 GATE EC 2008 www.gatehelp.com

or Zth = 1
Alternative :
Here at DC source capacitor act as open circuit and inductor act as short circuit.
Thus we can directly calculate thevenin Impedance as 1 Ω
Hence (A) is correct option.
MCQ 1.22 The driving point impedance of the following network is given by
Z (s) = 2 0.2s
s + 0.1s + 2

The component values are


(A) L = 5 H, R = 0.5 Ω, C = 0.1 F (B) L = 0.1 H, R = 0.5 Ω, C = 5 F
(C) L = 5 H, R = 2 Ω, C = 0.1 F (D) L = 0.1 H, R = 2 Ω, C = 5 F
SOL 1.22 Hence (D) is correct option.
s
Z (s) = R 1 sL = 2 C
sC s + s
RC + 1
LC
We have been given
Z (s) = 2 0.2s
s + 0.1s + 2
Comparing with given we get
1 = 0.2 or C = 5 F
C
1 = 0.1 or R = 2 Ω
RC
1 = 2 or L = 0.1 H
LC

MCQ 1.23 The circuit shown in the figure is used to charge the capacitor C alternately from
two current sources as indicated. The switches S1 and S2 are mechanically coupled
and connected as follows:
For 2nT # t # (2n + 1) T , (n = 0, 1, 2,..) S1 to P1 and S2 to P2
For (2n + 1) T # t # (2n + 2) T, (n = 0, 1, 2,...) S1 to Q1 and S2 to Q2

Brought to you by: Nodia and Company Visit us at: www.nodia.co.in


PUBLISHING FOR GATE
Page 10 GATE EC 2008 www.gatehelp.com

Assume that the capacitor has zero initial charge. Given that u (t) is a unit step
function , the voltage vc (t) across the capacitor is given by
3
(A) / (− 1) n tu (t − nT)
n=1
3
(B) u (t) + 2 / (− 1) n u (t − nT)
n=1
3
(C) tu (t) + 2 / (− 1) n u (t − nT) (t − nT)
n=1

(D) / 60.5 − e− (t − 2nT) + 0.5e− (t − 2nT) − T @


3

n=1
SOL 1.23 Voltage across capacitor is
#
t
Vc = 1 idt
C 0
Here C = 1 F and i = 1 A. Therefore
#0 dt
t
Vc =

For 0 < t < T , capacitor will be charged from 0 V


#0 dt = t
t
Vc =

At t = T, Vc = T Volts
For T < t < 2T , capacitor will be discharged from T volts as
#T dt = 2T − t
t
Vc = T −

At t = 2T, Vc = 0 volts
For 2T < t < 3T , capacitor will be charged from 0 V
#2Tdt = t − 2T
t
Vc =

At t = 3T, Vc = T Volts
For 3T < t < 4T , capacitor will be discharged from T Volts
#3Tdt = 4T − t
t
Vc = T −

At t = 4T, Vc = 0 Volts
For 4T < t < 5T , capacitor will be charged from 0 V
#4Tdt = t − 4T
t
Vc =

Brought to you by: Nodia and Company Visit us at: www.nodia.co.in


PUBLISHING FOR GATE
Page 11 GATE EC 2008 www.gatehelp.com

At t = 5T, Vc = T Volts
Thus the output waveform is

Only option C satisfy this waveform.


Hence (C) is correct option.
MCQ 1.24 The probability density function (pdf) of random variable is as shown below

The corresponding commutative distribution function CDF has the form

SOL 1.24 CDF is the integration of PDF. Plot in option (A) is the integration of plot given
in question.
Hence (A) is correct option.
MCQ 1.25 The recursion relation to solve x = e - x using Newton - Raphson method is
(A) xn + 1 = e−x
n
(B) xn + 1 = xn − e−x n

−x x n2 − e−x (1 − xn) − 1
n

(C) xn + 1 = (1 + xn) e −x
n

(D) xn + 1 =
1+e n
xn − e−x
n

SOL 1.25 Hence (C) is correct answer.


We have x = e-x
or f (x) = x − e - x
f'( x) = 1 + e - x
Brought to you by: Nodia and Company Visit us at: www.nodia.co.in
PUBLISHING FOR GATE
Page 12 GATE EC 2008 www.gatehelp.com

The Newton-Raphson iterative formula is


f (xn)
xn + 1 = xn −
f'( xn)
Now f (xn) = xn − e - x n

f'( xn) = 1 + e - x n

(1 + xn) e - x
xn + 1 = xn − xn − e- x =
-x n n

Thus
1+e n
1 + e-x n

MCQ 1.26 The residue of the function f (z) = 1 at z = 2 is


(z + 2) (z − 2) 2
2

(A) − 1 (B) − 1
32 16

(C) 1 (D) 1
16 32
SOL 1.26 Hence (A) is correct answer.
Res f (z) z = a = 1 dn - 1 6(z − a) n f (z)@
(n − 1)! dzn - 1 z=a

Here we have n = 2 and a = 2

1 d (z − 2) 2 1
(2 − 1)! dz ; (z − 2) 2 (z + 2) 2 Ez = a
Thus Res f (z) z = 2 =

= d ; 1 −2
dz (z + 2) 2 Ez = a ; (z + 2) 3 Ez = a
=

=− 2 =− 1
64 32

0 1
Consider the matrix P = =
− 2 − 3G
MCQ 1.27 . The value of e p is

2e−2 − 3e−1 e−1 − e−2 e−1 + e−1 2e−2 − e−1


(A) > −2 H (B) > −1 H
2e − 2e−1 5e−2 − e−1 2e − 4e2 3e−1 + 2e−2
5e−2 − e−1 3e−1 − e−2 2e−1 − e−2 e−1 − e−2
(C) > −2 H (D) > H
2e − 6e−1 4e−2 + 6−1 − 2e−1 + 2e−2 − e−1 + 2e−2

SOL 1.27 Hence (D) is correct answer.


eP = L- 1 6(sI − A) - 1@
s 0 0 1 -1
= L e=
0 s G =− 2 − 3Go
-1

s − 1 -1
= L- 1 e= o
2 s + 3G
s+3 1
= L f> Hp
-1 (s + 1)( s + 2) (s + 1)( s + 2)
-2 s
(s + 1)( s + 2) (s + 1)( s + 2)
2e − e-1
e −e-2 -1 -2
== G
− 2e + 2e − e - 1 + 2e - 2
-1 -2

Brought to you by: Nodia and Company Visit us at: www.nodia.co.in


PUBLISHING FOR GATE
Page 13 GATE EC 2008 www.gatehelp.com

MCQ 1.28 In the Taylor series expansion of exp (x) + sin (x) about the point x = π , the
coefficient of (x − π) 2 is
(A) exp (π) (B) 0.5 exp (π)
(C) exp (π) + 1 (D) exp (π) − 1
SOL 1.28 Taylor series is given as
(x − a) 2
f (x) = f (a) + x − a f'( a) + f"( a) + ...
1! 2!
For x = π we have
(x − π) 2
Thus f (x) = f (π) + x − π f'( π) + f"( x)...
1! 2!
Now = ex + sin x
f (x)
= ex + cos x
f'( x)
= ex − sin x
f"( x)
= e π − sin π = e π
f"( π)
f"( π)
Thus the coefficient of (x − π) 2 is
2!
Hence (B) is correct answer.
MCQ 1.29 Px (x) = M exp _− 2 x i − N exp _− 3 x i is the probability density function for the
real random variable X , over the entire x axis, M and N are both positive real
numbers. The equation relating M and N is
(A) M − 2 N = 1 (B) 2M + 1 N = 1
3 3
(C) M + N = 1 (D) M + N = 3
SOL 1.29 Correct Option is ( )
MCQ 1.30 The value of the integral of the function g (x, y) = 4x3 + 10y 4 along the straight line
segment from the point (0, 0) to the point (1, 2) in the x − y plane is
(A) 33 (B) 35
(C) 40 (D) 56
SOL 1.30 The equation of straight line from (0, 0) to (1, 2) is y = 2x .
Now g (x, y) = 4x3 + 10y 4
or, g (x, 2x) = 4x3 + 160x 4
#0 g (x, 2x) = #0 (4x3 + 160x4) dx
1 1
Now

= [x 4 + 32x5] 10 = 33
Hence (A) is correct answer.
MCQ 1.31 A linear, time - invariant, causal continuous time system has a rational transfer
function with simple poles at s =− 2 and s =− 4 and one simple zero at s =− 1.

Brought to you by: Nodia and Company Visit us at: www.nodia.co.in


PUBLISHING FOR GATE
Page 14 GATE EC 2008 www.gatehelp.com

A unit step u (t) is applied at the input of the system. At steady state, the output
has constant value of 1. The impulse response of this system is
(A) [exp (− 2t) + exp (− 4t)] u (t)
(B) [− 4 exp (− 2t) − 12 exp (− 4t) − exp (− t)] u (t)
(C) [− 4 exp (− 2t) + 12 exp (− 4t)] u (t)
(D) [− 0.5 exp (− 2t) + 1.5 exp (− 4t)] u (t)
SOL 1.31 Hence (C) is correct answer.
K (s + 1)
G (s) = , and R (s) = 1
(s + 2)( s + 4) s
K (s + 1)
C (s) = G (s) R (s) =
s (s + 2)( s + 4)
= K + K − 3K
8s 4 (s + 2) 8 (s + 4)
Thus c (t) = K :1 + 1 e−2t − 3 e−4tD u (t)
8 4 8
At steady-state , c (3) = 1
Thus K = 1 or K = 8
8
8 (s + 1)
Then, G (s) = = 12 − 4
(s + 2)( s + 4) (s + 4) (s + 2)
h (t) = L−1 G (s) = (− 4e−2t + 12e−4t) u (t)
MCQ 1.32 The signal x (t) is described by
1 for − 1 # t # + 1
x (t) = )
0 otherwise
Two of the angular frequencies at which its Fourier transform becomes zero are
(A) π, 2π (B) 0.5π, 1.5π
(C) 0, π (D) 2π, 2.5π
SOL 1.32 Hence (A) is correct answer.
1 for − 1 # t # + 1
We have x (t) = )
0 otherwise
Fourier transform is
1
#− 33e−jωt x (t) dt = #−1 e−jωt 1dt
= 1 [e−jωt]−11
− jω
= 1 (e−jω − e jω) = 1 (− 2j sin ω)
− jω − jω
= 2 sin ω
ω
This is zero at ω = π and ω = 2π

Brought to you by: Nodia and Company Visit us at: www.nodia.co.in


PUBLISHING FOR GATE
Page 15 GATE EC 2008 www.gatehelp.com

MCQ 1.33 A discrete time linear shift - invariant system has an impulse response h [n] with
h [0] = 1, h [1] =− 1, h [2] = 2, and zero otherwise The system is given an input
sequence x [n] with x [0] = x [2] = 1, and zero otherwise. The number of nonzero
samples in the output sequence y [n], and the value of y [2] are respectively
(A) 5, 2 (B) 6, 2
(C) 6, 1 (D) 5, 3
SOL 1.33 Hence (D) is correct answer.
Given h (n) = [1, − 1, 2]
x (n) = [1, 0, 1]
y (n) = x (n)* h (n)
The length of y [n] is
= L1 + L2 − 1 = 3 + 3 − 1 = 5
3
y (n) = x (n) * h (n) = / x (k) h (n − k)
k =− 3
3
y (2) = / x (k) h (2 − k)
k =− 3

= x (0) h (2 − 0) + x (1) h (2 − 1) + x (2) h (2 − 2)


= h (2) + 0 + h (0) = 1 + 2 = 3
There are 5 non zero sample in output sequence and the value of y [2] is 3.
MCQ 1.34 Consider points P and Q in the x − y plane, with P = (1, 0) and Q = (0, 1). The
Q
line integral 2 #P (xdx + ydy) along the semicircle with the line segment PQ as its
diameter
(A) is − 1
(B) is 0
(C) is 1
(D) depends on the direction (clockwise or anit-clockwise) of the semicircle
SOL 1.34 Hence (B) is correct answer.
Q
I =2 #P (xdx + ydy)
Q Q
=2 #P xdx + 2 #P ydy
#1 #0 ydy = 0
0 1
=2 xdx + 2

MCQ 1.35 Let x (t) be the input and y (t) be the output of a continuous time system. Match
the system properties P1, P2 and P3 with system relations R1, R2, R3, R4
Properties Relations
P1 : Linear but NOT time - invariant R1 : y (t) = t2 x (t)
P2 : Time - invariant but NOT linear R2 : y (t) = t x (t)

Brought to you by: Nodia and Company Visit us at: www.nodia.co.in


PUBLISHING FOR GATE
Page 16 GATE EC 2008 www.gatehelp.com

P3 : Linear and time - invariant R3 : y (t) = x (t)


R4 : y (t) = x (t − 5)
(A) (P1, R1), (P2, R3), (P3, R4)
(B) (P1, R2), (P2, R3), (P3, R4)
(C) (P1, R3), (P2, R1), (P3, R2)
(D) (P1, R1), (P2, R2), (P3, R3)
SOL 1.35 Mode function are not linear. Thus y (t) = x (t) is not linear but this functions is
time invariant. Option (A) and (B) may be correct.
The y (t) = t x (t) is not linear, thus option (B) is wrong and (a) is correct. We can
see that
R1: y (t) = t2 x (t) Linear and time variant.
R2: y (t) = t x (t) Non linear and time variant.
R3: y (t) = x (t) Non linear and time invariant
R4: y (t) = x (t − 5) Linear and time invariant
Hence (B) is correct answer.
MCQ 1.36 A memory less source emits n symbols each with a probability p. The entropy of
the source as a function of n
(A) increases as log n (B) decreases as log ( n1 )
(C) increases as n (D) increases as n log n
SOL 1.36 The entropy is
m
H = / pi log2 p1i bits
i=1

Since p1 = p2 = ... = pn = 1
n
n
H = / n1 log n = log n
i=1
Hence (A) is correct option.
MCQ 1.37 {x (n)} is a real - valued periodic sequence with a period N . x (n) and X (k) form
N-point Discrete Fourier Transform (DFT) pairs. The DFT Y (k) of the sequence
N−1
y (n) = 1 / x (r) x (n + r) is
N r=0
N−1
(A) X (k) 2 (B) 1 / X (r) X (k + r)
N r=0
N−1
(C) 1 / X (r) X (k + r) (D) 0
N r=0

SOL 1.37 Hence (A) is correct answer.


N-1
Given : y (n) = 1
N r=0
/
x (r) x (n + r)

Brought to you by: Nodia and Company Visit us at: www.nodia.co.in


PUBLISHING FOR GATE
Page 17 GATE EC 2008 www.gatehelp.com

It is Auto correlation.
DFT
Hence y (n) = rxx (n) X (k) 2

MCQ 1.38 Group I lists a set of four transfer functions. Group II gives a list of possible step
response y (t). Match the step responses with the corresponding transfer functions.

(A) P − 3, Q − 1, R − 4, S − 2 (B) P − 3, Q − 2, R − 4, S − 1
(C) P − 2, Q − 1, R − 4, S − 2 (D) P − 3, Q − 4, R − 1, S − 2
SOL 1.38 Hence (D) is correct option.
P = 2 25 2ξωn = 0, ξ = 0 " Undamped Graph 3
s + 25

Q= 62 2ξωn = 20, ξ > 1 " Overdamped Graph 4


s + 20s + 62
2

R= 62 2ξωn = 12, ξ = 1 " Critically Graph 1


s + 12s + 62
2

S= 72 2ξωn = 7, ξ < 1 " underdamped Graph 2


s + 7s + 72
2

MCQ 1.39 A certain system has transfer function


G (s) = 2 s + 8
s + αs − 4
Brought to you by: Nodia and Company Visit us at: www.nodia.co.in
PUBLISHING FOR GATE
Page 18 GATE EC 2008 www.gatehelp.com

where α is a parameter. Consider the standard negative unity feedback configuration


as shown below

Which of the following statements is true?


(A) The closed loop systems is never stable for any value of α
(B) For some positive value of α, the closed loop system is stable, but not for all
positive values.
(C) For all positive values of α, the closed loop system is stable.
(D) The closed loop system stable for all values of α, both positive and negative.
SOL 1.39 Hence (C) is correct option.
The characteristic equation of closed lop transfer function is
1 + G (s) H (s) = 0
1+ 2 s+8 =0
s + αs − 4
or s 2 + αs − 4 + s + 8 = 0
or s2 + (α + 1) s + 4 = 0
This will be stable if (α + 1) > 0 " α > − 1. Thus system is stable for all positive
value of α.
MCQ 1.40 A signal flow graph of a system is given below

The set of equalities that corresponds to this signal flow graph is


Jx1N R β − γ 0 VJx1N R0 0 V
K O S WK O S W u1
(A) d K x2O = S γ α 0 WK x2O+ S0 1 We o
dt K O S u2
x3 S− α β 0 WWK x3O SS1 0 WW
L P L P
Jx1N RT0 α γ XVJx1N TR1 0 XV
K O S WK O S W u1
(B) d K x2O = S0 − α − γ WK x2O+ S0 1 We o
dt K O S u2
x3 S0 β − β WWK x3O SS0 0 WW
L P TR L P
Jx1N − α β 0 VXJx1N RT1 0 VX
K O S WK O S W u1
(C) d K x2O = S− β − γ 0 WK x2O+ S0 1 We o
dt K O S u2
x3 S α γ 0 WWK x3O SS0 0 WW
L P T XL P T X
Brought to you by: Nodia and Company Visit us at: www.nodia.co.in
PUBLISHING FOR GATE
Page 19 GATE EC 2008 www.gatehelp.com

Jx1N R− α 0 β VJx1N R1 0 V
K O S WK O S W u1
(D) d K x2O = S γ 0 α WK x2O+ S0 1 We o
dt K O S u2
x3 S− β 0 − α WWK x3O SS0 0 WW
L P T XL P T X
SOL 1.40 We labeled the given SFG as below :

From this SFG we have


xo1 =− γx1 + βx3 + μ1
xo2 = γx1 + αx3
xo3 =− βx1 − αx3 + u2
R V R VR V R V
Sx1 W S− γ 0 β WSx1 W S0 1 W u1
Thus Sx2 W = S γ 0 α WSx2 W+ S0 0 We o
SSx WW SS− β 0 − α WWSSx WW SS1 0 WW u2
3 3
T X T XT X T X
Hence (C) is correct option.
MCQ 1.41 The number of open right half plane of
G (s) = 5 10 is
s + 2s + 3s + 6s2 + 5s + 3
4 3

(A) 0 (B) 1
(C) 2 (D) 3
SOL 1.41 The characteristic equation is
1 + G (s) = 0
or s + 2s + 3s + 6s + 5s + 3 = 0
5 4 3 2

Substituting s = z1 we have
3z5 + 5z 4 + 6z3 + 3z2 + 2z + 1 = 0
The routh table is shown below. As there are tow sign change in first column, there
are two RHS poles.

z5 3 6 2
z4 5 3 1
z3 21
5
7
5

z2 4
3 3
z1 − 74
z0 1

Brought to you by: Nodia and Company Visit us at: www.nodia.co.in


PUBLISHING FOR GATE
Page 20 GATE EC 2008 www.gatehelp.com

Hence (C) is correct option.


MCQ 1.42 The magnitude of frequency responses of an underdamped second order system
is 5 at 0 rad/sec and peaks to 10 at 5 2 rad/sec. The transfer function of the
3
system is
(A) 2 500 (B) 2 375
s + 10s + 100 s + 5s + 75
(C) 2 720 (D) 2 1125
s + 12s + 144 s + 25s + 225

SOL 1.42 For underdamped second order system the transfer function is
T (s) = 2 Kωn2
s + 2ξωn s + ωn2
It peaks at resonant frequency. Therefore
Resonant frequency ωr = ωn 1 − 2ξ2
and peak at this frequency
μr = 5
2ξ 1 − ξ2
We have ωr = 5 2 , and μr = 10 . Only options (A) satisfy these values.
3
ωn = 10, ξ = 1
2
where ωr = 10 1 − 2` 1 j = 5 2
4
and μr = 1 5 1 = 10 Hence satisfied
22 1− 4 3
Hence (C) is correct option.
MCQ 1.43 Group I gives two possible choices for the impedance Z in the diagram. The
circuit elements in Z satisfy the conditions R2 C2 > R1 C1. The transfer functions V0
Vi
represents a kind of controller.

Match the impedances in Group I with the type of controllers in Group II

Brought to you by: Nodia and Company Visit us at: www.nodia.co.in


PUBLISHING FOR GATE
Page 21 GATE EC 2008 www.gatehelp.com

(A) Q − 1, R − 2 (B) Q − 1, R − 3
(C) Q − 2, R − 3 (D) Q − 3, R − 2
SOL 1.43 The given circuit is a inverting amplifier and transfer function is
Vo = − Z = − Z (sC1 R1 + 1)
Vi R 1
R1
sC R + 1
1 1

(sC2 R2 + 1)
For Q , Z =
sC2
Vo =− (sC2 R2 + 1) (sC1 R1 + 1) PID Controller
#
Vi sC2 R1
For R, Z = R2
(sC2 R2 + 1)
Vo =− R2 (sC1 R1 + 1)
#
Vi (sC2 R2 + 1) R1
Since R2 C2 > R1 C1, it is lag compensator.
Hence (B) is correct option.
MCQ 1.44 For the circuit shown in the following figure, transistor M1 and M2 are identical
NMOS transistors. Assume the M2 is in saturation and the output is unloaded.

The current Ix is related to Ibias as


(A) Ix = Ibias + Is (B) Ix = Ibias
(C) Ix = Ibias − cVDD − Vout m (D) Ix = Ibias − Is
RE
SOL 1.44 By Current mirror,
^ L h2
W
Ix = W Ibias
^ L h1
Brought to you by: Nodia and Company Visit us at: www.nodia.co.in
PUBLISHING FOR GATE
Page 22 GATE EC 2008 www.gatehelp.com

Since MOSFETs are identical,


Thus bW l = bW l
L 2 L 2
Hence Ix = Ibias
Hence (B) is correct option.
MCQ 1.45 The measured trans conductance gm of an NMOS transistor operating in the linear
region is plotted against the gate voltage VG at a constant drain voltage VD . Which
of the following figures represents the expected dependence of gm on VG ?

SOL 1.45 Hence option (C) is correct.


As VD = constant
Thus gm \ (VGS − VT ) Which is straight line.

MCQ 1.46 Consider the following circuit using an ideal OPAMP. The I-V characteristic of the
diode is described by the relation I = I 0 _eV − 1i where VT = 25 mV, I0 = 1μ A and
V
t

V is the voltage across the diode (taken as positive for forward bias). For an input
voltage Vi =− 1 V , the output voltage V0 is

(A) 0 V (B) 0.1 V


(C) 0.7 V (D) 1.1 V
SOL 1.46 The circuit is using ideal OPAMP. The non inverting terminal of OPAMP is at
ground, thus inverting terminal is also at virtual ground.

Brought to you by: Nodia and Company Visit us at: www.nodia.co.in


PUBLISHING FOR GATE
Page 23 GATE EC 2008 www.gatehelp.com

Thus current will flow from -ive terminal (0 Volt) to -1 Volt source. Thus the
current I is
0 − (− 1)
I = = 1
100k 100k
The current through diode is
I = I 0 _eV − 1i
V
t

Now VT = 25 mV and I0 = 1 μA
I = 10−6 8e 25 # 10 − 1B = 1 5
V
Thus −3

10
or V = 0.06 V
Now V0 = I # 4k + V = 1 # 4k + 0.06 = 0.1 V
100k
Hence (B) is correct option.
MCQ 1.47 The OPAMP circuit shown above represents a

(A) high pass filter (B) low pass filter


(C) band pass filter (D) band reject filter
SOL 1.47 The circuit is using ideal OPAMP. The non inverting terminal of OPAMP is at
ground, thus inverting terminal is also at virtual ground.

Brought to you by: Nodia and Company Visit us at: www.nodia.co.in


PUBLISHING FOR GATE
Page 24 GATE EC 2008 www.gatehelp.com

Thus we can write


vi = −Rv
R1 + sL sR C + 1
2

2 2

or v0
=− R2
vi (R1 + sL)( sR2 C2 + 1)
and from this equation it may be easily seen that this is the standard form of T.F.
of low pass filter
H (s) = K
(R1 + sL)( sR2 C2 + 1)
and form this equation it may be easily seen that this is the standard form of T.F.
of low pass filter
H (s) = 2 K
as + bs + b
Hence (B) is correct option.
MCQ 1.48 Two identical NMOS transistors M1 and M2 are connected as shown below. Vbias is
chosen so that both transistors are in saturation. The equivalent gm of the pair is
defied to be 2Iout at constant Vout
2Vi
The equivalent gm of the pair is

(A) the sum of individual gm ' s of the transistors


(B) the product of individual gm ’s of the transistors
(C) nearly equal to the gm of M1
g
(D) nearly equal to m of M2
g0
SOL 1.48 The current in both transistor are equal. Thus gm is decide by M1.
Hence (C) is correct option.
MCQ 1.49 An 8085 executes the following instructions
2710 LXI H, 30A0 H
2713 DAD H
2414 PCHL
All address and constants are in Hex. Let PC be the contents of the program
counter and HL be the contents of the HL register pair just after executing PCHL.
Which of the following statements is correct ?
PC = 2715H PC = 30A0H
(A) (B)
HL = 30A0H HL = 2715H
Brought to you by: Nodia and Company Visit us at: www.nodia.co.in
PUBLISHING FOR GATE
Page 25 GATE EC 2008 www.gatehelp.com

PC = 6140H PC = 6140H
(C) (D)
HL = 6140H HL = 2715H

SOL 1.49 2710H LXI H, 30A0H ; Load 16 bit data 30A0 in HL pair
2713H DAD H ; 6140H " HL
2714H PCHL ; Copy the contents 6140H of HL in PC
Thus after execution above instruction contests of PC and HL are same and that
is 6140H
Hence (C) is correct answer.
MCQ 1.50 An astable multivibrator circuit using IC 555 timer is shown below. Assume that
the circuit is oscillating steadily.

The voltage Vc across the capacitor varies between


(A) 3 V to 5 V (B) 3 V to 6 V
(C) 3.6 V to 6 V (D) 3.6 V to 5 V
SOL 1.50 Correct Option is ( )
MCQ 1.51 Silicon is doped with boron to a concentration of 4 # 1017 atoms cm3 . Assume the
intrinsic carrier concentration of silicon to be 1.5 # 1010 / cm 3 and the value of kT/q
to be 25 mV at 300 K. Compared to undopped silicon, the fermi level of doped
silicon
(A) goes down by 0.31 eV (B) goes up by 0.13 eV
(C) goes down by 0.427 eV (D) goes up by 0.427 eV
SOL 1.51 Hence option (C) is correct.
E2 − E1 = kT ln NA
ni
NA = 4 # 1017

Brought to you by: Nodia and Company Visit us at: www.nodia.co.in


PUBLISHING FOR GATE
Page 26 GATE EC 2008 www.gatehelp.com

ni = 1.5 # 1010
17
E2 − E1 = 25 # 10−3 e ln 4 # 10 10 = 0.427 eV
1.5 # 10
Hence fermi level goes down by 0.427 eV as silicon is doped with boron.
MCQ 1.52 The cross section of a JFET is shown in the following figure. Let Vc be − 2 V
and let VP be the initial pinch -off voltage. If the width W is doubled (with other
geometrical parameters and doping levels remaining the same), then the ratio
between the mutual trans conductances of the initial and the modified JFET is

1 − 2/Vp
(B) 1 e
2 1 − 1/2Vp o
(A) 4

1 − 2/Vp 1 − (2 − Vp )
(C) e o (D)
1 − 1/2Vp 1 − [1 (2 Vp )]
SOL 1.52 Hence option (C) is correct
2
Pinch off voltage VP = eW ND
εs
Let VP = VP1
Now VP1 = W12 = W2
VP2 W22 (2W) 2
or 4VP1 = VP2
Initial transconductance
gm = Kn ;1 − Vbi − VGS
Vp E
0 − (− 2)
gm1 = Kn =1 −
VP1 G
= Kn ;1 −
VP1 E
For first condition 2

For second condition


0 − (− 2)
gm2 = Kn =1 −
VP2 G
= K2 ;1 −
4VP1 E
2

1 − 2/VP1
=f p
gm1
Dividing
gm2 1 − 1/ (2VP1)
Hence VP = VP1
MCQ 1.53 Consider the Schmidt trigger circuit shown below
A triangular wave which goes from -12 to 12 V is applied to the inverting input of

Brought to you by: Nodia and Company Visit us at: www.nodia.co.in


PUBLISHING FOR GATE
Page 27 GATE EC 2008 www.gatehelp.com

OPMAP. Assume that the output of the OPAMP swings from +15 V to -15 V. The
voltage at the non-inverting input switches between

(A) − 12V to +12 V (B) -7.5 V to 7.5 V


(C) -5 V to +5 V (D) 0 V and 5 V
SOL 1.53 Let the voltage at non inverting terminal be V1, then after applying KCL at non
inverting terminal side we have
15 − V1 + V0 − V1 = V1 − (− 15)
10 10 10
or V1 = V0
3
If V0 swings from -15 to +15 V then V1 swings between -5 V to +5 V.
Hence (C) is correct option.
MCQ 1.54 The logic function implemented by the following circuit at the terminal OUT is

(A) P NOR Q (B) P NAND Q


(C) P OR Q (D) P AND Q
SOL 1.54 From the figure shown below it may be easily seen upper MOSFET are shorted and
connected to Vdd thus OUT is 1 only when the node S is 0,

Since the lower MOSFETs are shorted to ground, node S is 0 only when input P

Brought to you by: Nodia and Company Visit us at: www.nodia.co.in


PUBLISHING FOR GATE
Page 28 GATE EC 2008 www.gatehelp.com

and Q are 1. This is the function of AND gate.


Hence (D) is correct answer.
MCQ 1.55 Consider the following assertions.
S1 : For Zener effect to occur, a very abrupt junction is required.
S2 : For quantum tunneling to occur, a very narrow energy barrier is required.
Which of the following is correct ?
(A) Only S2 is true
(B) S1 and S2 are both true but S2 is not a reason for S1
(C) S1 and S2 and are both true but S2 is not a reason for S1
(D) Both S1 and S2 are false
SOL 1.55 Hence option (A) is correct.
MCQ 1.56 The two numbers represented in signed 2’s complement form are P + 11101101
and Q = 11100110 . If Q is subtracted from P , the value obtained in signed 2’s
complement is
(A) 1000001111 (B) 00000111
(C) 11111001 (D) 111111001
SOL 1.56 MSB of both number are 1, thus both are negative number. Now we get
11101101 = (− 19) 10
and 11100110 = (− 26) 10
P − Q = (− 19) − (− 26) = 7
Thus 7 signed two’s complements form is
(7) 10 = 00000111
Hence (B) is correct answer.
MCQ 1.57 Which of the following Boolean Expressions correctly represents the relation
between P, Q, R and M1

(A) M1 = (P OR Q) XOR R (B) M1 = (P AND Q) X OR R


(C) M1 = (P NOR Q) X OR R (D) M1 = (P XOR Q) XOR R
SOL 1.57 The circuit is as shown below

Brought to you by: Nodia and Company Visit us at: www.nodia.co.in


PUBLISHING FOR GATE
Page 29 GATE EC 2008 www.gatehelp.com

X = PQ
Y = (P + Q)
So Z = PQ (P + Q)
= (P + Q )( P + Q) = PQ + PQ = P 5 Q
and M1 = Z 5 R = (P 5 Q) 5 R
Hence (D) is correct answer
MCQ 1.58 For the circuit shown in the following, I0 − I3 are inputs to the 4:1 multiplexers, R
(MSB) and S are control bits.
The output Z can be represented by

(A) PQ + PQS + QRS


(B) PQ + PQR + PQS
(C) PQR + PQR + PARS + QRS
(D) PQR + PQRS + PQRS + QRS
SOL 1.58 Hence (A) is correct answer.
Z = I0 RS + I1 RS + I2 RS + I3 RS
= (P + Q ) RS + PRS + PQRS + PRS
= PRS + QRS + PRS + PQRS + PRS
The k − Map is as shown below

Brought to you by: Nodia and Company Visit us at: www.nodia.co.in


PUBLISHING FOR GATE
Page 30 GATE EC 2008 www.gatehelp.com

Z = PQ + PQS + QRS
MCQ 1.59 For each of the positive edge-triggered J − K flip flop used in the following figure,
the propagation delay is 3 t .

Which of the following wave forms correctly represents the output at Q1 ?

SOL 1.59 Since the input to both JK flip-flop is 11, the output will change every time with
clock pulse. The input to clock is

The output Q0 of first FF occurs after time 3 T and it is as shown below

The output Q1 of second FF occurs after time 3 T when it gets input (i.e. after 3 T
from t1) and it is as shown below

Brought to you by: Nodia and Company Visit us at: www.nodia.co.in


PUBLISHING FOR GATE
Page 31 GATE EC 2008 www.gatehelp.com

Hence (B) is correct answer.


MCQ 1.60 For the circuit shown in the figure, D has a transition from 0 to 1 after CLK
changes from 1 to 0. Assume gate delays to be negligible
Which of the following statements is true

(A) Q goes to 1 at the CLK transition and stays at 1


(B) Q goes to 0 at the CLK transition and stays 0
(C) Q goes to 1 at the CLK tradition and goes to 0 when D goes to 1
(D) Q goes to 0 at the CLK transition and goes to 1 when D goes to 1
SOL 1.60 The circuit is as shown below

The truth table is shown below. When CLK make transition Q goes to 1 and when
D goes to 1, Q goes to 0
Hence (A) is correct answer.
MCQ 1.61 A rectangular waveguide of internal dimensions (a = 4 cm and b = 3 cm) is to be
operated in TE11 mode. The minimum operating frequency is
(A) 6.25 GHz (B) 6.0 GHz
(C) 5.0 GHz (D) 3.75 GHz
SOL 1.61 Cut-off Frequency is
fc = c ` m j2 + ` n j2
2 a b

Brought to you by: Nodia and Company Visit us at: www.nodia.co.in


PUBLISHING FOR GATE
Page 32 GATE EC 2008 www.gatehelp.com

For TE11 mode,


3 # 1010
fc = ` 4 j + ` 3 j = 6.25 GHz
1 2 1 2
2
Hence (A) is correct option.
MCQ 1.62 One end of a loss-less transmission line having the characteristic impedance of 75Ω
and length of 1 cm is short-circuited. At 3 GHz, the input impedance at the other
end of transmission line is
(A) 0 (B) Resistive
(C) Capacitive (D) Inductive
SOL 1.62 Hence (D) is correct option.
Z + iZo tan (βl)
Zin = Zo L
Zo + iZL tan (βl)
For ZL = 0 ,Zin = iZo tan (βl)
The wavelength is
8
λ = c = 3 # 109 = 0.1 m or 10 cm
f 3 # 10
βl = 2π l = 2π # 1 = π
λ 10 5
Thus Zin = iZo tan π
5
Thus Zin is inductive because Zo tan π is positive
5
MCQ 1.63 A uniform plane wave in the free space is normally incident on an infinitely thick
dielectric slab (dielectric constant ε = 9 ). The magnitude of the reflection coefficient
is
(A) 0 (B) 0.3
(C) 0.5 (D) 0.8
SOL 1.63 Hence (C) is correct option.
μ
We have η =
ε
Reflection coefficient
η − η1
Γ= 2
η2 + η1
Substituting values for η1 and η2 we have
μ
− με
= 1 − εr = 1 −
o o

τ = εμε o r o 9 since εr = 9
μ
εε +
o
o r ε
o
o
1 + εr 1+ 9
=− 0.5
MCQ 1.64 In the design of a single mode step index optical fibre close to upper cut-off, the
single-mode operation is not preserved if
(A) radius as well as operating wavelength are halved

Brought to you by: Nodia and Company Visit us at: www.nodia.co.in


PUBLISHING FOR GATE
Page 33 GATE EC 2008 www.gatehelp.com

(B) radius as well as operating wavelength are doubled


(C) radius is halved and operating wavelength is doubled
(D) radius is doubled and operating wavelength is halved
SOL 1.64 In single mode optical fibre, the frequency of limiting mode increases as radius
decreases
Hence r \ 1
f
So. if radius is doubled, the frequency of propagating mode gets halved, while in
option (D) it is increased by two times.
Hence (C) is correct option.
MCQ 1.65 At 20 GHz, the gain of a parabolic dish antenna of 1 meter and 70% efficiency is
(A) 15 dB (B) 25 dB
(C) 35 dB (D) 45 dB
SOL 1.65 Hence (D) is correct option.
8
λ = c = 3 # 10 9 = 3
f 20 # 10 200
2
Gp = ηπ2 ` D j = 0.7 # π2 c 13 m = 30705.4
2
Gain
λ 100
= 44.87 dB
MCQ 1.66 Noise with double-sided power spectral density on K over all frequencies is passed
through a RC low pass filter with 3 dB cut-off frequency of fc . The noise power at
the filter output is
(A) K (B) Kfc
(C) kπfc (D) 3
SOL 1.66 Hence (C) is correct option.
PSD of noise is N0 = K ...(1)
2
The 3-dB cut off frequency is
fc = 1 ...(2)
2πRC
Output noise power is
= N0 = c N0 m 1 = Kπfc
4RC 2 2RC

MCQ 1.67 Consider a Binary Symmetric Channel (BSC) with probability of error being p. To
transmit a bit, say 1, we transmit a sequence of three 1s. The receiver will interpret
the received sequence to represent 1 if at least two bits are 1. The probability that
the transmitted bit will be received in error is
(A) p3 + 3p2 (1 − p) (B) p3

Brought to you by: Nodia and Company Visit us at: www.nodia.co.in


PUBLISHING FOR GATE
Page 34 GATE EC 2008 www.gatehelp.com

(C) (1 − p3) (D) p3 + p2 (1 − p)


SOL 1.67 At receiving end if we get two zero or three zero then its error.
Let p be the probability of 1 bit error, the probability that transmitted bit error is
= Three zero + two zero and single one
= 3 C3 p3 + 3C2 p2 (1 − p)
= p3 + p2 (1 − p)
Hence (D) is correct option.
MCQ 1.68 Four messages band limited to W, W, 2W and 3W respectively are to be multiplexed
using Time Division Multiplexing (TDM). The minimum bandwidth required for
transmission of this TDM signal is
(A) W (B) 3W
(C) 6W (D) 7W
SOL 1.68 Bandwidth of TDM is
= 1 (sum of Nyquist Rate)
2
= 1 [2W + 2W + 4W + 6W] = 7W
2
Hence (D) is correct option.
MCQ 1.69 Consider the frequency modulated signal
10 cos [2π # 105 t + 5 sin (2π # 1500t) + 7.5 sin (2π # 1000t)]
with carrier frequency of 105 Hz. The modulation index is
(A) 12.5 (B) 10
(C) 7.5 (D) 5
SOL 1.69 Hence (B) is correct option.
We have θi = 2π105 t + 5 sin (2π1500t) + 7.5 sin (2π1000t)
ωi = dθi = 2π105 + 10π1500 cos (2π1500t) + 15π1000 cos (2π1000t)
dt
Maximum frequency deviation is
3ωmax = 2π (5 # 1500 + 7.5 # 1000)
3 fmax = 15000
3f
Modulation index is = max = 15000 = 10
fm 1500

MCQ 1.70 The signal cos ωc t + 0.5 cos ωm t sin ωc t is


(A) FM only (B) AM only
(C) both AM and FM (D) neither AM nor FM
SOL 1.70 Hence (C) is correct option.

Brought to you by: Nodia and Company Visit us at: www.nodia.co.in


PUBLISHING FOR GATE
Page 35 GATE EC 2008 www.gatehelp.com

Common Data for Questions 71, 72 and 73 :


A speed signal, band limited to 4 kHz and peak voltage varying between +5 V and
− 5 V, is sampled at the Nyquist rate. Each sample is quantized and represented
by 8 bits.
MCQ 1.71 If the bits 0 and 1 are transmitted using bipolar pulses, the minimum bandwidth
required for distortion free transmission is
(A) 64 kHz (B) 32 kHz
(C) 8 kHz (D) 4 kHz
SOL 1.71 Hence (B) is correct option.
fm = 4 KHz
fs = 2fm = 8 kHz
Bit Rate Rb = nfs = 8 # 8 = 64 kbps
The minimum transmission bandwidth is
BW = Rb = 32 kHz
2

MCQ 1.72 Assuming the signal to be uniformly distributed between its peak to peak value,
the signal to noise ratio at the quantizer output is
(A) 16 dB (B) 32 dB
(C) 48 dB (D) 4 kHz
SOL 1.72 Hence (C) is correct option.
S0
c N m = 1.76 + 6n dB
0

= 1.76 + 6 # 8 = 49.76 dB We have n = 8


MCQ 1.73 The number of quantization levels required to reduce the quantization noise by a
factor of 4 wo
(A) 1024 (B) 512
(C) 256 (D) 64
SOL 1.73 Hence (B) is correct option.
As Noise \ 12
L
To reduce quantization noise by factor 4, quantization level must be two times i.e.
2L .
Now L = 2n = 28 = 256
Thus 2L = 512

Common data for questions 74 & 75 :


The following series RLC circuit with zero conditions is excited by a unit impulse

Brought to you by: Nodia and Company Visit us at: www.nodia.co.in


PUBLISHING FOR GATE
Page 36 GATE EC 2008 www.gatehelp.com

functions δ (t).

MCQ 1.74 For t > 0 , the output voltage vC ^ t h is

(A) 2 ^e − e th
−1
(B) 2 te 2 t
−1 3
2
t 2

3 3

(C) 2 e 2 t cos c 3 t m (D) 2 e 2 t sin c 3 t m


−1 −1

3 2 3 2
SOL 1.74 Writing in transform domain we have
Vc (s) 1
= 1 s = 2 1
Vs (s) ^s + s + 1h (s + s + 1)
Since Vs (t) = δ (t) " Vs (s) = 1 and
Vc (s) = 2 1
(s + s + 1)
3
or Vc (s) = 2 = 2
G
3 (s + 12 ) 2 + 43
Taking inverse laplace transform we have
Vt = 2 e− sin c 3 t m
t
2

3 2
Hence (D) is correct option.
MCQ 1.75 For t > 0 , the voltage across the resistor is
(A) 1 _e 2 t − e− 2 t i
3 1

=cos c 2 t m − c 2 mG
−1 t 3 1 sin 3 t
(B) e 2
3

(C) 2 e 2 t sin c 3 t m
−1

3 2

(D) 2 e 2 t cos c 3 t m
−1

3 2
SOL 1.75 Let voltage across resistor be vR
VR (s)
= 1 1 = 2 s
VS (s) ( s + s + 1) (s + s + 1)
Since vs = δ (t) " Vs (s) = 1 we get
VR (s) = 2 s = s
(s + s + 1) (s + 2 ) + 43
1 2

Brought to you by: Nodia and Company Visit us at: www.nodia.co.in


PUBLISHING FOR GATE
Page 37 GATE EC 2008 www.gatehelp.com

(s + 12 ) 1
= − 2
(s + 12 ) 2 + 43 (s + 12 ) 2 + 43
vR (t) = e− cos 3 t − 1 # 2 e− sin 3 t
1 1
or 2 2

2 2 3 2

= e− 2 =cos 3 t − 1 sin 3 tG
t

2 3 2
Hence (B) is correct option.

Linked Answer Questions : Q. 76 to Q.85 carry two marks each.

Statement for linked Answers Questions 76 & 77:


A two-port network shown below is excited by external DC source. The voltage and
the current are measured with voltmeters V1, V2 and ammeters. A1, A2 (all assumed
to be ideal), as indicated

Under following conditions, the readings obtained are:


(1) S1 -open, S2 - closed A1 = 0,V1 = 4.5 V,V2 = 1.5 V, A2 = 1 A
(2) S1 -open, S2 - closed A1 = 4 A,V1 = 6 V,V2 = 6 V, A2 = 0

MCQ 1.76 The z -parameter matrix for this network is


1. 5 1. 5 1.5 4.5
(A) =
4. 5 1. 5 G
(B) =
1.5 4.5G
1.5 4.5 4.5 1.5
(C) =
1.5 1.5 G
(D) =
1.5 4.5G
SOL 1.76 From the problem statement we have
z11 = v1 = 6 = 1. 5Ω
i1 i = 0 4
2

z12 = v1 = 4.5 = 4.5Ω


i2 i = 0
1
1
z21 = v2 = 6 = 1.5Ω
i1 i = 0 4
2

z22 = v2 = 1.5 = 1.5Ω


i2 i = 0
2
1
Thus z -parameter matrix is

Brought to you by: Nodia and Company Visit us at: www.nodia.co.in


PUBLISHING FOR GATE
Page 38 GATE EC 2008 www.gatehelp.com

z11 z12 1.5 4.5


=z z G = =1.5 1.5 G
21 22

Hence (C) is correct option.


MCQ 1.77 The h -parameter matrix for this network is
−3 3 −3 −1
(A) = G (B) =
− 1 0.67 3 0.67 G
3 3 3 1
(C) =
1 0.67 G
(D) =
− 3 − 0.67 G
SOL 1.77 From the problem statement we have
h12 = v1 = 4.5 = 3
v2 i = 0 1.5
1

h22 = i2 = 1 = 0.67
v2 i = 0 1.5
1

From z matrix, we have


v1 = z11 i1 + z12 i2
v2 = z21 i1 + z22 i2
If v2 = 0
Then i2 = − z21 = − 1.5 =− 1 = h
21
i1 z22 1.5
or i2 =− i1
Putting in equation for v1, we get
v1 = (z11 − z12) i1
v1 = h11 = z11 − z12 = 1.5 − 4.5 =− 3
i1 v = 0
2

Hence h −parameter will be


h11 h12 −3 3
=h h G = =− 1 0.67 G
21 22

Hence (A) is correct option.

Statement for Linked Answer Question 78 and 79 :


In the following network, the switch is closed at t = 0− and the sampling starts
from t = 0 . The sampling frequency is 10 Hz.

Brought to you by: Nodia and Company Visit us at: www.nodia.co.in


PUBLISHING FOR GATE
Page 39 GATE EC 2008 www.gatehelp.com

MCQ 1.78 The samples x (n), n = (0, 1, 2, ...) are given by


(A) 5 (1 − e−0.05n) (B) 5e−0.05n
(C) 5 (1 − e−5n) (D) 5e−5n
SOL 1.78 Current through resistor (i.e. capacitor) is
I = I (0+) e−t/RC
Here, I (0+) = V = 5 = 25μA
R 200k
RC = 200k # 10μ = 2 sec
I = 25e− μ A
t
2

= VR # R = 5e− V
t
2

Here the voltages across the resistor is input to sampler at frequency of 10 Hz. Thus
−n
x (n) = 5e = 5e−0.05n For t > 0
2 # 10

Hence (B) is correct answer.


MCQ 1.79 The expression and the region of convergence of the z −transform of the sampled
signal are
(A) 5z 5 , z < e−5 (B) 5z , z < e−0.05
z−e z − e−0.05

(C) 5z , z > e−0.05 (D) 5z −5 , z > e−5


z − e−0.05 z−e
SOL 1.79 Hence (C) is correct answer.
Since x (n) = 5e−0.05n u (n) is a causal signal
Its z transform is
X (z) = 5 : 1 5z
−0.05 −1 D =
1−e z z − e−0.05
Its ROC is e−0.05 z−1 > 1 " z > e−0.05

Statement for Linked Answer Questions 80 and 81:


In the following transistor circuit, VBE = 0.7 V, r3 = 25 mV/IE , and β and all the
capacitances are very large

MCQ 1.80 The value of DC current IE is

Brought to you by: Nodia and Company Visit us at: www.nodia.co.in


PUBLISHING FOR GATE
Page 40 GATE EC 2008 www.gatehelp.com

(A) 1 mA (B) 2 mA
(C) 5 mA (D) 10 mA
SOL 1.80 For the given DC values the Thevenin equivalent circuit is as follows

The thevenin resistance and voltage are


VTH = 10 # 9 = 3 V
10 + 20
and total RTH = 10k # 20k = 6.67 kΩ
10k + 20k
Since β is very large, therefore IB is small and can be ignored
Thus IE = VTH − VBE = 3 − 0.7 = 1 mA
RE 2.3k
Hence (A) is correct option.
MCQ 1.81 The mid-band voltage gain of the amplifier is approximately
(A) -180 (B) -120
(C) -90 (D) -60
SOL 1.81 The small signal model is shown in fig below

IC
gm = = 1m = 1 A/V IC . IE
VT 25m 25
Vo =− gm Vπ # (3k 3k )
=− 1 Vin (1.5k) Vπ = Vin
25
=− 60Vin
or Am = Vo =− 60
Vin

Brought to you by: Nodia and Company Visit us at: www.nodia.co.in


PUBLISHING FOR GATE
Page 41 GATE EC 2008 www.gatehelp.com

Hence (D) is correct option.

Statement For Linked Answer Question 82 & 83 :


In the following circuit, the comparators output is logic “1” if V1 > V2 and is logic
3
"0" otherwise. The D/A conversion is done as per the relation VDAC = 2n - 1 bn /
n=0
Volts, where b3

(MSB), b1, b2 and b0 (LSB) are the counter outputs. The counter starts from the
clear state.

MCQ 1.82 The stable reading of the LED displays is


(A) 06 (B) 07
(C) 12 (D) 13
SOL 1.82 Hence (D) is correct answer.
We have
3
VDAC = / 2n - 1bn = 2- 1b0 + 20 b1 + 21b2 + 22 b3
n=0
or VDAC = 0.5b0 + b1 + 2b2 + 4b3
The counter outputs will increase by 1 from 0000 till Vth > VDAC . The output of
counter and VDAC is as shown below

Clock b3 b3 b2 b0 VDAC
1 0001 0
2 0010 0.5
3 0011 1
4 0100 1.5
5 0101 2

Brought to you by: Nodia and Company Visit us at: www.nodia.co.in


PUBLISHING FOR GATE
Page 42 GATE EC 2008 www.gatehelp.com

6 0110 2.5
7 0111 3
8 1000 3.5
9 1001 4
10 1010 4.5
11 1011 5
12 1100 5.5
13 1101 6
14 1110 6.5
and when VADC = 6.5 V (at 1101), the output of AND is zero and the counter stops.
The stable output of LED display is 13.
MCQ 1.83 The magnitude of the error between VDAC and Vin at steady state in volts is
(A) 0.2 (B) 0.3
(C) 0.5 (D) 1.0
SOL 1.83 Hence (B) is correct answer.
The VADC − Vin at steady state is
= 6.5 − 6.2 = 0.3V

Statement for Linked Answer Question 84 & 85 :


The impulse response h (t) of linear time - invariant continuous time system is given
by h (t) = exp (− 2t) u (t), where u (t) denotes the unit step function.
MCQ 1.84 The frequency response H (ω) of this system in terms of angular frequency ω, is
given by H (ω)
(A) 1 (B) sin ω
1 + j2ω ω
1 jω
(C) (D)
2 + jω 2 + jω
SOL 1.84 Hence (C) is correct answer.
h (t) = e−2t u (t)
H (jω) = # h (t) e−jωt dt
3

−3

= #0 3e−2t e−jωt dt = #0 3e−(2 + jω)t dt = 1


(2 + jω)

MCQ 1.85 The output of this system, to the sinusoidal input x (t) = 2 cos 2t for all time t , is
(A) 0 (B) 2−0.25 cos (2t − 0.125π)
(C) 2−0.5 cos (2t − 0.125π) (D) 2−0.5 cos (2t − 0.25π)

Brought to you by: Nodia and Company Visit us at: www.nodia.co.in


PUBLISHING FOR GATE
Page 43 GATE EC 2008 www.gatehelp.com

SOL 1.85 Hence (D) is correct answer.


H (jω) = 1
(2 + jω)
The phase response at ω = 2 rad/sec is
+H (jω) =− tan−1 ω =− tan−1 2 =− π =− 0.25π
2 2 4
Magnitude respone at ω = 2 rad/sec is

H (jω) = 1 = 1
2 +w
2 2
2 2

Input is x (t) = 2 cos (2t)

Output i = 1 # 2 cos (2t − 0.25π)


2 2
= 1 cos [2t − 0.25π]
2

Answer Sheet
1. (C) 19. (B) 37. (A) 55. (A) 73. (B)
2. (B) 20. (A) 38. (D) 56. (B) 74. (D)
3. (A) 21. (A) 39. (C) 57. (D) 75. (B)
4. (A) 22. (D) 40. (C) 58. (A) 76. (C)
5. (A) 23. (C) 41. (C) 59. (B) 77. (A)
6. (B) 24. (A) 42. (C) 60. (A) 78. (B)
7. (C) 25. (C) 43. (B) 61. (A) 79. (C)
8. (B) 26. (A) 44. (B) 62. (D) 80. (A)
9. (C) 27. (D) 45. (C) 63. (C) 81. (D)
10. (D) 28. (B) 46. (B) 64. (C) 82. (D)
11. (C) 29. (*) 47. (B) 65. (D) 83. (B)
12. (C) 30. (A) 48. (C) 66. (C) 84. (C)
13. (D) 31. (C) 49. (C) 67. (D) 85. (D)
14. (A) 32. (A) 50. (*) 68. (D)
15. (C) 33. (D) 51. (C) 69. (B)
16. (D) 34. (B) 52. (C) 70 (C)
17. (C) 35. (B) 53. (C) 71 (B)
18. (D) 36. (A) 54. (D) 72 (C)

Brought to you by: Nodia and Company Visit us at: www.nodia.co.in


PUBLISHING FOR GATE
GATE EC
2009

Q.1 - Q.20 carry one mark each.

MCQ 1.1 The order of the differential equation


d2y dy 3
2 + c dt m + y = e
4 −t
dt
is
(A) 1 (B) 2
(C) 3 (D) 4
SOL 1.1 The highest derivative terms present in DE is of 2nd order.
Hence (B) is correct answer.
MCQ 1.2 The Fourier series of a real periodic function has only
(P) cosine terms if it is even
(Q) sine terms if it is even
(R) cosine terms if it is odd
(S) sine terms if it is odd
Which of the above statements are correct ?
(A) P and S (B) P and R
(C) Q and S (D) Q and R
SOL 1.2 The Fourier series of a real periodic function has only cosine terms if it is even and
sine terms if it is odd.
Hence (A) is correct answer.
MCQ 1.3 A function is given by f (t) = sin2 t + cos 2t . Which of the following is true ?
(A) f has frequency components at 0 and 1 Hz

(B) f has frequency components at 0 and 1 Hz


π

(C) f has frequency components at 1 and 1 Hz


2π π

(D) f has frequency components at 0.1 and 1 Hz


2π π
Brought to you by: Nodia and Company Visit us at: www.nodia.co.in
PUBLISHING FOR GATE
Page 2 GATE EC 2009 www.gatehelp.com

SOL 1.3 Given function is


f (t) = sin2 t + cos 2t = 1 − cos 2t + cos 2t = 1 + 1 cos 2t
2 2 2
The function has a DC term and a cosine function. The frequency of cosine terms is
ω = 2 = 2πf " f = 1 Hz
π
The given function has frequency component at 0 and 1 Hz.
π
Hence (B) is correct answer.
MCQ 1.4 A fully charged mobile phone with a 12 V battery is good for a 10 minute talk-time.
Assume that, during the talk-time the battery delivers a constant current of 2 A
and its voltage drops linearly from 12 V to 10 V as shown in the figure. How much
energy does the battery deliver during this talk-time?

(A) 220 J (B) 12 kJ


(C) 13.2 kJ (D) 14.4 J
SOL 1.4 The energy delivered in 10 minutes is
#0 VIdt = I #0Vdt
t t
E = = I # Area

= 2 # 1 (10 + 12) # 600 = 13.2 kJ


2
Hence (C) is correct option.
MCQ 1.5 In an n-type silicon crystal at room temperature, which of the following can have
a concentration of 4 # 1019 cm - 3 ?
(A) Silicon atoms (B) Holes
(C) Dopant atoms (D) Valence electrons
SOL 1.5 Only dopant atoms can have concentration of 4 # 1019 cm - 3 in n −type silicon at
room temperature.
Hence option (C) is correct.
MCQ 1.6 The full form of the abbreviations TTL and CMOS in reference to logic families are
(A) Triple Transistor Logic and Chip Metal Oxide Semiconductor
(B) Tristate Transistor Logic and Chip Metal Oxide Semiconductor
(C) Transistor Transistor Logic and Complementary Metal Oxide Semiconductor
(D) Tristate Transistor Logic and Complementary Metal Oxide Silicon

Brought to you by: Nodia and Company Visit us at: www.nodia.co.in


PUBLISHING FOR GATE
Page 3 GATE EC 2009 www.gatehelp.com

SOL 1.6 TTL " Transistor - Transistor logic


CMOS " Complementary Metal Oxide Semi-conductor
Hence (C) is correct answer.
MCQ 1.7 The ROC of z -transform of the discrete time sequence
x (n) = b 1 l u (n) − b 1 l u (− n − 1) is
n n

3 2
(A) 1 < z < 1 (B) z > 1
3 2 2

(C) z < 1 (D) 2 < z < 3


3
SOL 1.7 Hence (A) is correct answer
x [n] = b 1 l u (n) − b 1 l u (− n − 1)
n n

3 2
Taking z transform we have
n=3 n n =− 1
X (z) = / b 1 l z−n − / b 1 l z−n
n

n=0
3 n =− 3
2
n=3 n =− 1
/ b 13 z−1l b2z l
n
1 −1 n
= − /
n=0 n =− 3

First term gives 1 z−1 < 1 " 1 < z


3 3
Second term gives 1 z−1 > 1 " 1 > z
2 2
Thus its ROC is the common ROC of both terms. that is
1< z <1
3 2

MCQ 1.8 The magnitude plot of a rational transfer function G (s) with real coefficients is
shown below. Which of the following compensators has such a magnitude plot ?

(A) Lead compensator (B) Lag compensator


(C) PID compensator (D) Lead-lag compensator
SOL 1.8 This compensator is roughly equivalent to combining lead and lad compensators in
the same design and it is referred also as PID compensator.
Hence (C) is correct option
MCQ 1.9 A white noise process X (t) with two-sided power spectral density 1 # 10−10 W/Hz

Brought to you by: Nodia and Company Visit us at: www.nodia.co.in


PUBLISHING FOR GATE
Page 4 GATE EC 2009 www.gatehelp.com

is input to a filter whose magnitude squared response is shown below.

The power of the output process Y (t) is given by


(A) 5 # 10−7 W (B) 1 # 10−6 W
(C) 2 # 10−6 W (D) 1 # 10−5 W
SOL 1.9 Correct Option is ( )
MCQ 1.10 Which of the following statements is true regarding the fundamental mode of the
metallic waveguides shown ?

(A) Only P has no cutoff-frequency


(B) Only Q has no cutoff-frequency
(C) Only R has no cutoff-frequency
(D) All three have cutoff-frequencies
SOL 1.10 Rectangular and cylindrical waveguide doesn’t support TEM modes and have cut
off frequency.
Coaxial cable support TEM wave and doesn’t have cut off frequency.
Hence (A) is correct option.
MCQ 1.11 A fair coin is tossed 10 times. What is the probability that only the first two tosses
will yield heads?
(A) c 1 m (B) 10C2 b 1 l
2 2

2 2

(C) c 1 m (D) 10C2 b 1 l


10 10

2 2
10
SOL 1.11 Number of elements in sample space is 2 . Only one element
"H, H, T, T, T, T, T, T, T, T , is event. Thus probability is 10
1
2
Hence (C) is correct answer.
MCQ 1.12 If the power spectral density of stationary random process is a sine-squared function
of frequency, the shape of its autocorrelation is

Brought to you by: Nodia and Company Visit us at: www.nodia.co.in


PUBLISHING FOR GATE
Page 5 GATE EC 2009 www.gatehelp.com

SOL 1.12 Correct Option is ( )

MCQ 1.13 If f (z) = c 0 + c1 z−1 , then # 1 +zf (z) dz is given by


unit circle
(A) 2πc1 (B) 2π (1 + c0)
(C) 2πjc1 (D) 2π (1 + c0)
SOL 1.13 Hence (C) is correct answer
We have
f (z) = c0 + c1 z - 1
1 + f (z) 1 + c0 + c1 z - 1 z (1 + c0) + c1
f1 (z) = = =
z z z2
Since f1 (z) has double pole at z = 0 , the residue at z = 0 is
z (1 + c0) + c1
Res f1 (z) z = 0 = lim z2 .f1 (z) = lim z2 . c m = c1
z"0 z"0 z2
Hence
[1 + f (z)]
#
f1 (z) dz = # z
dz = 2πj [Residue at z = 0 ]
unit circle unit circle

= 2πjc1
MCQ 1.14 In the interconnection of ideal sources shown in the figure, it is known that the 60
V source is absorbing power.

Brought to you by: Nodia and Company Visit us at: www.nodia.co.in


PUBLISHING FOR GATE
Page 6 GATE EC 2009 www.gatehelp.com

Which of the following can be the value of the current source I ?


(A) 10 A (B) 13 A
(C) 15 A (D) 18 A
SOL 1.14 Circuit is as shown below

Since 60 V source is absorbing power. So, in 60 V source current flows from + to


- ve direction
So, I + I1 = 12
I = 12 − I1
I is always less then 12 A So, only option (A) satisfies this conditions.
Hence (A) is correct option.
MCQ 1.15 The ratio of the mobility to the diffusion coefficient in a semiconductor has the
units
(A) V - 1 (B) cm.V1
(C) V.cm - 1 (D) V.s
SOL 1.15 Hence option (A) is correct.
2
Unit of mobility μn is = cm
V. sec
2
Unit of diffusion current Dn is = cm
sec
μn 2 2
Thus unit of is = cm / cm = 1 = V−1
Dn V $ sec sec V

MCQ 1.16 In a microprocessor, the service routine for a certain interrupt starts from a fixed
location of memory which cannot be externally set, but the interrupt can be delayed
or rejected Such an interrupt is
(A) non-maskable and non-vectored
(B) maskable and non-vectored
(C) non-maskable and vectored
(D) maskable and vectored
SOL 1.16 Vectored interrupts : Vectored interrupts are those interrupts in which program
control transferred to a fixed memory location.
Maskable interrupts : Maskable interrupts are those interrupts which can be rejected

Brought to you by: Nodia and Company Visit us at: www.nodia.co.in


PUBLISHING FOR GATE
Page 7 GATE EC 2009 www.gatehelp.com

or delayed by microprocessor if it is performing some critical task.


Hence (D) is correct answer.
MCQ 1.17 If the transfer function of the following network is
Vo (s) 1
=
Vi (s) 2 + sCR

The value of the load resistance RL is


(A) R (B) R
4 2
(C) R (D) 2R
SOL 1.17 For given network we have
(RL XC ) Vi
V0 =
R + (RL XC )
RL
V0 (s)
= 1 + sRL C = RL
Vi (s) R+ R L R + RRL sC + RL
1 + sRL C

= RL = 1
R + RRL sC + RL 1+ R + RsC
RL
But we have been given
V (s) 1
T .F. = 0 =
Vi (s) 2 + sCR
Comparing, we get
1 + R = 2 & RL = R
RL
Hence (C) is correct option.
MCQ 1.18 Consider the system
dx = Ax + Bu with A = =1 0G and B = = p G
dt 0 1 q
where p and q are arbitrary real numbers. Which of the following statements about
the controllability of the system is true ?
(A) The system is completely state controllable for any nonzero values of p and q
(B) Only p = 0 and q = 0 result in controllability
(C) The system is uncontrollable for all values of p and q

Brought to you by: Nodia and Company Visit us at: www.nodia.co.in


PUBLISHING FOR GATE
Page 8 GATE EC 2009 www.gatehelp.com

(D) We cannot conclude about controllability from the given data


SOL 1.18 Hence (C) is correct option.
1 0 p
Here A == G and B = = G
0 1 q
1 0 p p
AB = = G= G == G
0 1 q q
p q
S = 8B AB B = =
q pG
S = pq − pq = 0
Since S is singular, system is completely uncontrollable for all values of p and q .
MCQ 1.19 For a message signal m (t) = cos (2πfm t) and carrier of frequency fc , which of the
following represents a single side-band (SSB) signal ?
(A) cos (2πfm t) cos (2πfc t) (B) cos (2πfc t)
(C) cos [2π (fc + fm) t] (D) [1 + cos (2πfm t) cos (2πfc t)
SOL 1.19 Hence (C) is correct option.
cos (2πfm t) cos (2πfc t) $ DSB suppressed carrier
cos (2πfc t) $ Carrier Only
cos [2π (fc + fm) t] $ USB Only
[1 + cos (2πfm t) cos (2πfc t)] $ USB with carrier
MCQ 1.20 Two infinitely long wires carrying current are as shown in the figure below. One
wire is in the y − z plane and parallel to the y − axis. The other wire is in the x − y
plane and parallel to the x − axis. Which components of the resulting magnetic
field are non-zero at the origin ?

(A) x, y, z components (B) x, y components


(C) y, z components (D) x, z components
SOL 1.20 Due to 1 A current wire in x − y plane, magnetic field be at origin will be in x
direction.
Due to 1 A current wire in y − z plane, magnetic field be at origin will be in z
Brought to you by: Nodia and Company Visit us at: www.nodia.co.in
PUBLISHING FOR GATE
Page 9 GATE EC 2009 www.gatehelp.com

direction.
Thus x and z component is non-zero at origin.
Hence (D) is correct option.

Q.21 to Q.60 carry two marks each.

MCQ 1.21 Consider two independent random variables X and Y with identical distributions.
The variables X and Y take values 0, 1 and 2 with probabilities 12 , 14 and 1
4
respectively. What is the conditional probability P (X + Y = 2 X − Y = 0) ?
(A) 0 (B) 1/16
(C) 1/6 (D) 1
SOL 1.21 Hence (C) is correct option.
We have
p (X = 0) = p (Y = 0) = 1
2
p (X = 1) = p (Y = 1) = 1
4
p (X = 2) = p (Y = 2) = 1
4
Let X+Y = 2 $ A
and X−Y = 0 $ B
Now
P (A + B)
P (X + Y = 2 X − Y = 0) =
P (B)
Event P (A + B) happen when X + Y = 2 and X − Y = 0 . It is only the case when
X = 1 and Y = 1.
Thus P (A + B) = 1 # 1 = 1
4 4 16
Now event P (B) happen when
X − Y = 0 It occurs when X = Y , i.e.
X = 0 and Y = 0 or
X = 1 and Y = 1 or
X = 2 and Y = 2
Thus P (B) = 1 # 1 + 1 # 1 + 1 # 1 = 6
2 2 4 4 4 4 16
P (A + B) 1/16
Now = =1
P (B) 6/16 6

MCQ 1.22 The Taylor series expansion of sin x at x = π is given by


x−π
(x − π) 2 (x − π) 2
(A) 1 + + ... (B) − 1 − + ...
3! 3!

Brought to you by: Nodia and Company Visit us at: www.nodia.co.in


PUBLISHING FOR GATE
Page 10 GATE EC 2009 www.gatehelp.com

(x − π) 2 (x − π) 2
(C) 1 − + ... (D) − 1 + + ...
3! 3!
SOL 1.22 Hence (D) is correct answer.
We have f (x) = sin x
x−π
Substituting x − π = y ,we get
sin (y + π)
= − 1 (sin y)
sin y
f (y + π) = =−
y y y
3 5
= − 1 cy − + − ...m
y y
y 3! 5!
y2 y 4
or f (y + π) =− 1 + − + ...
3! 5!
Substituting x − π = y we get
(x − π) 2 (x − π) 4
f (x) =− 1 + − + ...
3! 5!

MCQ 1.23 If a vector field V is related to another vector field A through V = 4# A , which of
the following is true? (Note : C and SC refer to any closed contour and any surface
whose boundary is C . )
(A) #CV $ dl = #S #C A $ d S (B) #C A $ dl = #S #CV $ d S
(C) #C Δ # V $ dl = #S #C Δ # A $ d S (D) #C Δ # V $ dl = #S #CV $ d S
SOL 1.23 Hence (B) is correct option.
We have V = 4# A ...(1)
By Stokes theorem
= # A $ dl ## (4 # A) $ ds ...(2)
From (1) and (2) we get
# A $ dl = ##V $ ds
MCQ 1.24 Given that F (s) is the one-side Laplace transform of f (t), the Laplace transform of
#0 f (τ) dτ is
t

(A) sF (s) − f (0) (B) 1 F (s)


s

#0
s
(C) F (τ) dτ (D) 1 [F (s) − f (0)]
s
SOL 1.24 By property of unilateral laplace transform

F (s) 1 0
# #
t L
f (τ) dτ + f (τ) dτ
−3 s s −3
Here function is defined for 0 < τ < t , Thus
F (s)
#
t L
f (τ)
0 s
Brought to you by: Nodia and Company Visit us at: www.nodia.co.in
PUBLISHING FOR GATE
Page 11 GATE EC 2009 www.gatehelp.com

Hence (B) is correct answer.


MCQ 1.25 Match each differential equation in Group I to its family of solution curves from
Group II
Group I Group II
dy y
A. = 1. Circles
dx x
dy y
B. =− 2. Straight lines
dx x
dy
C. =x 3. Hyperbolas
dx y
dy
D. =− x
dx y
(A) A − 2, B − 3, C − 3, D − 1
(B) A − 1, B − 3, C − 2, D − 1
(C) A − 2, B − 1, C − 3, D − 3
(D) A − 3, B − 2, C − 1, D − 2
SOL 1.25 Hence (A) is correct answer
dy y
(A) =
dx x

or # dyy = # dxx
or log y = log x + log c
or y = cx Straight Line
Thus option (A) and (C) may be correct.
dy y
(B) =−
dx x

or # dyy =− # dxx
or log y =− log x + log c
or log y = log 1 + log c
x

or y = c Hyperbola
x

MCQ 1.26 The Eigen values of following matrix are


R V
S− 1 3 5 W
S− 3 − 1 6 W
SS 0 0 3 WW
(A) 3, 3 + 5j, T6 − j X (B) − 6 + 5j, 3 + j, 3 − j

Brought to you by: Nodia and Company Visit us at: www.nodia.co.in


PUBLISHING FOR GATE
Page 12 GATE EC 2009 www.gatehelp.com

(C) 3 + j, 3 − j, 5 + j (D) 3, − 1 + 3j, − 1 − 3j


SOL 1.26 Sum of the principal diagonal element of matrix is equal to the sum of Eigen values.
Sum of the diagonal element is − 1 − 1 + 3 = 1.In only option (D), the sum of Eigen
values is 1.
Hence (D) is correct answer.
MCQ 1.27 An AC source of RMS voltage 20 V with internal impedance Zs = (1 + 2j) Ω feeds a
load of impedance ZL = (7 + 4j) Ω in the figure below. The reactive power consumed
by the load is

(A) 8 VAR (B) 16 VAR


(C) 28 VAR (D) 32 VAR
SOL 1.27 From given circuit the load current is
IL = V = 20+0c = 20+0c
Zs + ZL (1 + 2j) + (7 + 4j) 8 + 6j
= 1 (8 − 6j) = 20+0c = 2+ − φ where φ = tan - 1 3
5 10+φ 4
The voltage across load is
VL = IL ZL
The reactive power consumed by load is
Pr = VL IL* = IL ZL # IL* = ZL IL 2
2
= (7 # 4j) 20+0c = (7 + 4j) = 28 + 16j
8 + 6j
Thus average power is 28 and reactive power is 16.
Hence (B) is correct option.
MCQ 1.28 The switch in the circuit shown was on position a for a long time, and is move to
position b at time t = 0 . The current i (t) for t > 0 is given by

Brought to you by: Nodia and Company Visit us at: www.nodia.co.in


PUBLISHING FOR GATE
Page 13 GATE EC 2009 www.gatehelp.com

(A) 0.2e−125t u (t) mA (B) 20e−1250t u (t) mA


(C) 0.2e−1250t u (t) mA (D) 20e−1000t u (t) mA
SOL 1.28 At t = 0− , the circuit is as shown in fig below :

V (0−) = 100 V
Thus V (0+) = 100 V
At t = 0+ , the circuit is as shown below

I (0+) = 100 = 20 mA
5k
At steady state i.e. at t = 3 is I (3)= 0
i (t) = I (0+) e− u (t)
t
Now RCeq

(0.5μ + 0.3μ) 0.2μ


Ceq = = 0.16 μ F
0.5μ + 0.3μ + 0.2μ
1 = 1 = 1250
RCeq 5 # 103 # 0.16 # 10−6
i (t) = 20e−1250t u (t) mA
Hence (B) is correct option.
MCQ 1.29 In the circuit shown, what value of RL maximizes the power delivered to RL ?

(A) 2.4 Ω (B) 8 Ω


3
Brought to you by: Nodia and Company Visit us at: www.nodia.co.in
PUBLISHING FOR GATE
Page 14 GATE EC 2009 www.gatehelp.com

(C) 4 Ω (D) 6 Ω
SOL 1.29 For Pmax the load resistance RL must be equal to thevenin resistance Req i.e. RL
= Req . The open circuit and short circuit is as shown below

The open circuit voltage is


Voc = 100 V

From fig I1 = 100 = 12.5 A


8
Vx =− 4 # 12.5 =− 50 V

I2 = 100 + Vx = 100 − 50 = 12.5 A


4 4
Isc = I1 + I2 = 25 A

Rth = Voc = 100 = 4 Ω


Isc 25
Thus for maximum power transfer RL = Req = 4 Ω
Hence (C) is correct option.
MCQ 1.30 The time domain behavior of an RL circuit is represented by
L di + Ri = V0 (1 + Be−Rt/L sin t) u (t).
dt
For an initial current of i (0) = V0 , the steady state value of the current is given by
R
(A) i (t) " V0 (B) i (t) " 2V0
R R

(C) i (t) " V0 (1 + B) (D) i (t) " 2V0 (1 + B)


R R
SOL 1.30 Steady state all transient effect die out and inductor act as short circuits and forced
response acts only. It doesn’t depend on initial current state. From the given time
domain behavior we get that circuit has only R and L in series with V0 . Thus at
steady state
i (t) " i (3) = V0
R
Hence (A) is correct option.

Brought to you by: Nodia and Company Visit us at: www.nodia.co.in


PUBLISHING FOR GATE
Page 15 GATE EC 2009 www.gatehelp.com

MCQ 1.31 In the circuit below, the diode is ideal. The voltage V is given by

(A) min (Vi, 1) (B) max (Vi, 1)


(C) min (− Vi, 1) (D) max (− Vi, 1)
SOL 1.31 Let diode be OFF. In this case 1 A current will flow in resistor and voltage across
resistor will be V = 1.V
Diode is off, it must be in reverse biased, therefore
Vi − 1 > 0 " Vi > 1
Thus for Vi > 1 diode is off and V = 1V
Option (B) and (C) doesn’t satisfy this condition.
Let Vi < 1. In this case diode will be on and voltage across diode will be zero and
V = Vi
Thus V = min (Vi, 1)
Hence (A) is correct option.
MCQ 1.32 Consider the following two statements about the internal conditions in a n −channel
MOSFET operating in the active region.
S1 : The inversion charge decreases from source to drain
S2 : The channel potential increases from source to drain.
Which of the following is correct?
(A) Only S2 is true
(B) Both S1 and S2 are false
(C) Both S1 and S2 are true, but S2 is not a reason for S1
(D) Both S1 and S2 are true, and S2 is a reason for S1
SOL 1.32 Both S1 and S2 are true and S2 is a reason for S1.
Hence option (D) is correct.
MCQ 1.33 In the following a stable multivibrator circuit, which properties of v0 (t) depend on
R2 ?

Brought to you by: Nodia and Company Visit us at: www.nodia.co.in


PUBLISHING FOR GATE
Page 16 GATE EC 2009 www.gatehelp.com

(A) Only the frequency


(B) Only the amplitude
(C) Both the amplitude and the frequency
(D) Neither the amplitude nor the frequency
SOL 1.33 The R2 decide only the frequency.
Hence (A) is correct option
MCQ 1.34 In the circuit shown below, the op-amp is ideal, the transistor has VBE = 0.6 V and
β = 150 . Decide whether the feedback in the circuit is positive or negative and
determine the voltage V at the output of the op-amp.

(A) Positive feedback, V = 10 V. (B) Positive feedback, V = 0 V


(C) Negative feedback, V = 5 V (D) Negative feedback, V = 2 V
SOL 1.34 The circuit is shown in fig below

Brought to you by: Nodia and Company Visit us at: www.nodia.co.in


PUBLISHING FOR GATE
Page 17 GATE EC 2009 www.gatehelp.com

The voltage at non inverting terminal is 5 V because OP AMP is ideal and inverting
terminal is at 5 V.
Thus IC = 10 − 5 = 1 mA
5k
VE = IE RE = 1m # 1.4k = 1.4V IE = IC
= 0.6 + 1.4 = 2V
Thus the feedback is negative and output voltage is V = 2V .
Hence (D) is correct option.
MCQ 1.35 A small signal source Vi (t) = A cos 20t + B sin 106 t is applied to a transistor amplifier
as shown below. The transistor has β = 150 and hie = 3Ω . Which expression best
approximate V0 (t)

(A) V0 (t) =− 1500 (A cos 20t + B sin 106 t)


(B) V0 (t) = − 1500( A cos 20t + B sin 106 t)
(C) V0 (t) =− 1500B sin 106 t
(D) V0 (t) =− 150B sin 106 t
SOL 1.35 The output voltage is
hfe RC
V0 = Ar Vi .− Vi
hie
Here RC = 3 Ω and hie = 3 kΩ
Thus V0 . − 150 # 3k Vi
3k
.− 150 (A cos 20t + B sin 106 t)
Since coupling capacitor is large so low frequency signal will be filtered out, and
best approximation is
V0 .− 150B sin 106 t
Hence (D) is correct option.
MCQ 1.36 If X = 1 in logic equation 6X + Z {Y + (Z + XY )}@ {X + X (X + Y)} = 1, then
(A) Y = Z (B) Y = Z
(C) Z = 1 (D) Z = 0

Brought to you by: Nodia and Company Visit us at: www.nodia.co.in


PUBLISHING FOR GATE
Page 18 GATE EC 2009 www.gatehelp.com

SOL 1.36 We have 6X + Z {Y + (Z + XY )}@ [X + Z (X + Y)] = 1


Substituting X = 1 and X = 0 we get
[1 + Z {Y + (Z + 1Y )}][ 0 + Z (1 + Y)] = 1
or [1][ Z (1)] = 1 1 + A = 1 and 0 + A = A
or Z =1)Z=0
Hence (D) is correct answer
MCQ 1.37 What are the minimum number of 2- to -1 multiplexers required to generate a 2-
input AND gate and a 2- input Ex-OR gate
(A) 1 and 2 (B) 1 and 3
(C) 1 and 1 (D) 2 and 2
SOL 1.37 The AND gate implementation by 2:1 mux is as follows

Y = AI 0 + AI1 = AB

The EX − OR gate implementation by 2:1 mux is as follows

Y = BI0 + BI1 = AB + BA
Hence (A) is correct answer.
MCQ 1.38 Refer to the NAND and NOR latches shown in the figure. The inputs (P1, P2) for
both latches are first made (0, 1) and then, after a few seconds, made (1, 1). The
corresponding stable outputs (Q1, Q2) are

(A) NAND: first (0, 1) then (0, 1) NOR: first (1, 0) then (0, 0)
(B) NAND : first (1, 0) then (1, 0) NOR : first (1, 0) then (1, 0)
(C) NAND : first (1, 0) then (1, 0) NOR : first (1, 0) then (0, 0)
(D) NAND : first (1, 0) then (1, 1) NOR : first (0, 1) then (0, 1)

Brought to you by: Nodia and Company Visit us at: www.nodia.co.in


PUBLISHING FOR GATE
Page 19 GATE EC 2009 www.gatehelp.com

SOL 1.38 For the NAND latche the stable states are as follows

For the NOR latche the stable states are as follows

Hence (C) is correct answer.

MCQ 1.39 What are the counting states (Q1, Q2) for the counter shown in the figure below

(A) 11, 10, 00, 11, 10,... (B) 01, 10, 11, 00, 01...
(C) 00, 11, 01, 10, 00... (D) 01, 10, 00, 01, 10...
SOL 1.39 The given circuit is as follows.

The truth table is as shown below. Sequence is 00, 11, 10, 00 ...

CLK J1 K1 Q1 J2 K2 Q2
1 1 1 0 1 1 0
2 1 1 1 1 1 1
3 0 0 1 0 1 0
4 1 1 0 1 1 0

Brought to you by: Nodia and Company Visit us at: www.nodia.co.in


PUBLISHING FOR GATE
Page 20 GATE EC 2009 www.gatehelp.com

Hence (A) is correct answer.


MCQ 1.40 A system with transfer function H (z) has impulse response h (.) defined as
h (2) = 1, h (3) =− 1 and h (k) = 0 otherwise. Consider the following statements.
S1 : H (z) is a low-pass filter.
S2 : H (z) is an FIR filter.
Which of the following is correct?
(A) Only S2 is true
(B) Both S1 and S2 are false
(C) Both S1 and S2 are true, and S2 is a reason for S1
(D) Both S1 and S2 are true, but S2 is not a reason for S1
SOL 1.40 We have h (2) = 1, h (3) =− 1 otherwise h (k) = 0 . The diagram of response is as
follows :

It has the finite magnitude values. So it is a finite impulse response filter. Thus S2
is true but it is not a low pass filter. So S1 is false.
Hence (A) is correct answer.
MCQ 1.41 Consider a system whose input x and output y are related by the equation
# x (t − τ) g (2τ) dτ where h (t) is shown in the graph.
3
y (t) =
−3

Which of the following four properties are possessed by the system ?


BIBO : Bounded input gives a bounded output.
Causal : The system is causal,
LP : The system is low pass.
LTI : The system is linear and time-invariant.
(A) Causal, LP (B) BIBO, LTI
(C) BIBO, Causal, LTI (D) LP, LTI
SOL 1.41 Here h (t) ! 0 for t < 0 . Thus system is non causal. Again any bounded input x (t)
gives bounded output y (t). Thus it is BIBO stable.

Brought to you by: Nodia and Company Visit us at: www.nodia.co.in


PUBLISHING FOR GATE
Page 21 GATE EC 2009 www.gatehelp.com

Here we can conclude that option (B) is correct.


Hence (B) is correct answer.
MCQ 1.42 The 4-point Discrete Fourier Transform (DFT) of a discrete time sequence {1,0,2,3}
is
(A) [0, − 2 + 2j , 2, − 2 − 2j ] (B) [2, 2 + 2j , 6, 2 − 2j ]
(C) [6, 1 − 3j , 2, 1 + 3j ] (D) [6, − 1 + 3j , 0, − 1 − 3j ]
SOL 1.42 Hence (D) is correct answer
We have x [n] = {1, 0, 2, 3) and N = 4
N−1
X [k ] = / x [ n] e −j2πnk/N
k = 0, 1...N − 1
n=0
3
For N = 4 , X [k ] = / x [ n] e −j2πnk/4
k = 0, 1,... 3
n=0
3
Now X [ 0] = / x [n]
n=0
= x [0] + x [1] + x [2] + x [3] = 1 + 0 + 2 + 3 = 6
3
x [1] = / x [ n] e −jπn/2

n=0

= x [0] + x [1] e−jπ/2 + x [2] e−jπ + x [3] e−jπ3/2


= 1 + 0 − 2 + j3 =− 1 + j3
3
X [ 2] = / x [ n] e −jπn

n=0

= x [0] + x [1] e−jπ + x [2] e−j2π + x [3] e−jπ3


= 1+0+2−3 = 0
3
X [ 3] = / x [ n] e −j3πn/2

n=0

= x [0] + x [1] e−j3π/2 + x [2] e−j3π + x [3] e−j9π/2


= 1 + 0 − 2 − j3 =− 1 − j3
Thus [6, − 1 + j3, 0, − 1 − j3]
MCQ 1.43 The feedback configuration and the pole-zero locations of
G (s) = s2 − 2s + 2
2

s + 2s + 2
are shown below. The root locus for negative values of k , i.e. for − 3 < k < 0 , has
breakaway/break-in points and angle of departure at pole P (with respect to the
positive real axis) equal to

Brought to you by: Nodia and Company Visit us at: www.nodia.co.in


PUBLISHING FOR GATE
Page 22 GATE EC 2009 www.gatehelp.com

(A) ! 2 and 0c (B) ! 2 and 45c


(C) ! 3 and 0c (D) ! 3 and 45c
SOL 1.43 The characteristic equation is
1 + G (s) H (s) = 0
K (s2 − 2s + 2)
or 1+ =0
s2 + 2s + 2
or s2 + 2s + 2 + K (s2 − 2s + 2) = 0
K =− s2 + 2s + 2
2
or
s − 2s + 2
For break away & break in point differentiating above w.r.t. s we have
dK =− (s − 2s + 2) (2s + 2) − (s + 2s + 2) (2s − 2) = 0
2 2

ds (s2 − 2s + 2) 2
Thus (s2 − 2s + 2)( 2s + 2) − (s2 + 2s + 2)( 2s − 2) = 0
or s =! 2
Let θd be the angle of departure at pole P , then

− θd − θp1 + θz1 + θz2 = 180c


− θd = 180c − (− θp1 + θz1 + θ2)
= 180c − (90c + 180 − 45c) =− 45c
Hence (B) is correct option.
An LTI system having transfer function s +s 2+s 1+ 1 and input x (t) = sin (t + 1) is in
2
MCQ 1.44 2

steady state. The output is sampled at a rate ωs rad/s to obtain the final output
{x (k)}. Which of the following is true ?
(A) y (.) is zero for all sampling frequencies ωs
(B) y (.) is nonzero for all sampling frequencies ωs
(C) y (.) is nonzero for ωs > 2 , but zero for ωs < 2
(D) y (.) is zero for ωs > 2 , but nonzero for ω2 < 2

Brought to you by: Nodia and Company Visit us at: www.nodia.co.in


PUBLISHING FOR GATE
Page 23 GATE EC 2009 www.gatehelp.com

SOL 1.44 Hence (A) is correct answer.


MCQ 1.45 The unit step response of an under-damped second order system has steady state
value of -2. Which one of the following transfer functions has theses properties ?
(A) 2 − 2.24 (B) 2 − 3.82
s + 2.59s + 1.12 s + 1.91s + 1.91
(C) 2 − 2.24 (D) 2 − 382
s − 2.59s + 1.12 s − 1.91s + 1.91
SOL 1.45 For under-damped second order response
T (s) = 2 kωn2 where ξ < 1
s + 2ξωn s + ωn2
Thus (A) or (B) may be correct
For option (A) ωn = 1.12 and 2ξωn = 2.59 " ξ = 1.12
For option (B) ωn = 1.91 and 2ξωn = 1.51 " ξ = 0.69
Hence (B) is correct option.
MCQ 1.46 A discrete random variable X takes values from 1 to 5 with probabilities as shown
in the table. A student calculates the mean X as 3.5 and her teacher calculates the
variance of X as 1.5. Which of the following statements is true ?

k 1 2 3 4 5
P (X = k) 0.1 0.2 0.3 0.4 0.5
(A) Both the student and the teacher are right
(B) Both the student and the teacher are wrong
(C) The student is wrong but the teacher is right
(D) The student is right but the teacher is wrong
SOL 1.46 Hence (B) is correct option.
The mean is
X = Σxi pi (x)
= 1 # 0.1 + 2 # 0.2 + 3 # 0.4 + 4 # 0.2 + 5 # 0.1
= 0.1 + 0.4 + 1.2 + 0.8 + 0.5 = 3.0
X = Σxi2 pi (x)
2

= 1 # 0.1 + 4 # 0.2 + 9 # 0.4 + 16 # 0.2 + 25 # 0.1


= 0.1 + 0.8 + 3.6 + 3.2 + 2.5 = 10.2
Variance σx = X2 − ^X h2
2

= 10.2 − (3) 2 = 1.2


MCQ 1.47 A message signal given by m (t) = ( 12 ) cos ω1 t − ( 12 ) sin ω2 t amplitude - modulated
with a carrier of frequency ωC to generator s (t)[ 1 + m (t)] cos ωc t . What is the
power efficiency achieved by this modulation scheme ?
(A) 8.33% (B) 11.11%
(C) 20% (D) 25%

Brought to you by: Nodia and Company Visit us at: www.nodia.co.in


PUBLISHING FOR GATE
Page 24 GATE EC 2009 www.gatehelp.com

SOL 1.47 Hence (C) is correct option.


m (t) = 1 cos ω1 t − 1 sin ω2 t
2 2
sAM (t) = [1 + m (t)] cos ωc t
Modulation index
m (t) max
=
Vc
1 2 1 2
`2j +`2j =
m = 1
2
` 2j
1 2
2
η = m # 100 % = # 100% = 20%
m2 + 2 ` 2j +2
1 2

MCQ 1.48 A communication channel with AWGN operating at a signal to noise ration
SNR >> 1 and bandwidth B has capacity C1. If the SNR is doubled keeping
constant, the resulting capacity C2 is given by
(A) C2 . 2C1 (B) C2 . C1 + B
(C) C2 . C1 + 2B (D) C2 . C1 + 0.3B
SOL 1.48 Hence (B) is correct option.
We have C1 = B log2 `1 + S j
N
. B log2 ` S j As S >> 1
N N
If we double the S ratio then
N
C2 . B log2 ` 2S j
N
. B log2 2 + B log2 S
N
. B + C1
MCQ 1.49 A magnetic field in air is measured to be
y
B = B0 c 2 x 2 yt − 2 xt m
x +y x + y2
What current distribution leads to this field ?
[Hint : The algebra is trivial in cylindrical coordinates.]
t t
(A) J = B0 z c 2 1 2 m, r ! 0 (B) J =− B0 z c 2 2 2 m, r ! 0
μ0 x + y μ0 x + y
t
(C) J = 0, r ! 0 (D) J = B0 z c 2 1 2 m, r ! 0
μ0 x + y
SOL 1.49 Hence (C) is correct option.
y
We have Bv = B0 c 2 x 2 ay − 2 ax m ...(1)
x +y x + y2
To convert in cylindrical substituting
Brought to you by: Nodia and Company Visit us at: www.nodia.co.in
PUBLISHING FOR GATE
GATE IES PSU’S-2021
MADE EASY/ACE
Leading inStitUte for eSe/gate/PSU’S

IES GS NOTES-2021
NOTED-: These Are Only Sample Notes,
If You Want Complete Full MADE
EASY+ACE IES GS ALL Branch ALL
SUBJECT FULL NOTES BUY Click below
Link Buy

Noted-: Download File Zip format so use zip rar app extract zip
file easily open folder available all subject pdf.
Page 25 GATE EC 2009 www.gatehelp.com

x = r cos φ and y = r sin φ


ax = cos φar − sin φaφ
and ay = sin φar + cos φaφ
In (1) we have
Bv = Bv0 aφ
v Bv a
Now Hv = B = 0 φ constant
μ0 μ0
Jv = 4# Hv = 0 since H is constant

MCQ 1.50 A transmission line terminates in two branches, each of length λ , as shown.
4
The branches are terminated by 50Ω loads. The lines are lossless and have the
characteristic impedances shown. Determine the impedance Zi as seen by the source.

(A) 200Ω (B) 100Ω


(C) 50Ω (D) 25Ω

SOL 1.50 The transmission line are as shown below. Length of all line is λ
4

2 2
Zi1 = Z01 = 100 = 200Ω
ZL1 50
2 2
Zi2 = Z02 = 100 = 200Ω
ZL2 50
ZL3 = Zi1 Zi2 = 200Ω 200Ω = 100Ω
2 2
Zi = Z0 = 50 = 25Ω
ZL3 100
Hence (D) is correct option.

Common Date for Question 51 and 52 :

Brought to you by: Nodia and Company Visit us at: www.nodia.co.in


PUBLISHING FOR GATE
Page 26 GATE EC 2009 www.gatehelp.com

Consider a silicon p − n junction at room temperature having the following


parameters:
Doping on the n -side = 1 # 1017 cm - 3
Depletion width on the n -side = 0.1μm
Depletion width on the p −side = 1.0μm
Intrinsic carrier concentration = 1.4 # 1010 cm - 3
Thermal voltage = 26 mV
Permittivity of free space = 8.85 # 10 - 14 F.cm - 1
Dielectric constant of silicon = 12

MCQ 1.51 The built-in potential of the junction


(A) is 0.70 V
(B) is 0.76 V
(C) is 0.82 V
(D) Cannot be estimated from the data given
SOL 1.51 Hence option (B) is correct.
We know that
NA WP = ND WN
17 −6
or NA = ND WN = 1 # 10 # 0.1−6# 10 = 1 # 1016
WP 1 # 10
The built-in potential is
Vbi = VT 1nc NA N
n i2 m
D

17 16
= 26 # 10−3 ln e 1 # 10 # 1 10 o
# 10 = 0.760
2
(1.4 # 10 )

MCQ 1.52 The peak electric field in the device is


(A) 0.15 MV . cm - 1, directed from p −region to n −region
(B) 0.15 MV . cm - 1, directed from n −region to p −region
(C) 1.80 MV . cm - 1, directed from p-retion to n −region
(D) 1.80 MV . cm - 1, directed from n −region to p −region
SOL 1.52 The peak electric field in device is directed from p to n and is
E =− eND xn from p to n
εs
= eND xn from n to p
εs
−19 17 −5
= 1.6 # 10 # 1 #−10 14
# 1 # 10 = 0.15 MV/cm
8.85 # 10 # 12
Hence option (B) is correct.

Brought to you by: Nodia and Company Visit us at: www.nodia.co.in


PUBLISHING FOR GATE
Page 27 GATE EC 2009 www.gatehelp.com

Common Data for Questions 53 and 54 :


The Nyquist plot of a stable transfer function G (s) is shown in the figure are
interested in the stability of the closed loop system in the feedback configuration
shown.

MCQ 1.53 Which of the following statements is true ?


(A) G (s) is an all-pass filter
(B) G (s) has a zero in the right-half plane
(C) G (s) is the impedance of a passive network
(D) G (s) is marginally stable
SOL 1.53 The plot has one encirclement of origin in clockwise direction. Thus G (s) has a zero
is in RHP.
Hence (B) is correct option.
MCQ 1.54 The gain and phase margins of G (s) for closed loop stability are
(A) 6 dB and 180c (B) 3 dB and 180c
(C) 6 dB and 90c (D) 3 dB and 90c
SOL 1.54 The Nyzuist plot intersect the real axis ate - 0.5. Thus
G. M. =− 20 log x =− 20 log 0.5 = 6.020 dB
And its phase margin is 90c.
Hence (C) is correct option.

Common data for Questions 55 & 56 :


The amplitude of a random signal is uniformly distributed between -5 V and 5 V.
MCQ 1.55 If the signal to quantization noise ratio required in uniformly quantizing the signal
is 43.5 dB, the step of the quantization is approximately
(A) 0.033 V (B) 0.05 V
(C) 0.0667 V (D) 0.10 V
SOL 1.55 Hence (C) is correct option.
We have SNR = 1.76 + 6n

Brought to you by: Nodia and Company Visit us at: www.nodia.co.in


PUBLISHING FOR GATE
Page 28 GATE EC 2009 www.gatehelp.com

or 43.5 = 1.76 + 6n
6n = 43.5 + 1.76
6n = 41.74 $ n . 7
No. of quantization level is
27 = 128
Step size required is
5 − (− 5)
= VH − VL = = 10
128 128 128
= .078125
. .0667
MCQ 1.56 If the positive values of the signal are uniformly quantized with a step size of 0.05
V, and the negative values are uniformly quantized with a step size of 0.1 V, the
resulting signal to quantization noise ration is approximately
(A) 46 dB (B) 43.8 dB
(C) 42 dB (D) 40 dB
SOL 1.56 For positive values step size
s+ = 0.05 V
For negative value step size
s- = 0.1 V
No. of quantization in + ive is
= 5 = 5 = 100
s+ 0.05
Thus 2n + = 100 $ n+ = 7
No. of quantization in − ve
Q1 = 5 = 5 = 50
s- 0.1
2n = 50 $ n - = 6
-
Thus
` N j+ = 1.76 + 6n = 1.76 + 42 = 43.76 dB
S +

` N j- = 1.76 + 6n = 1.76 + 36 = 37.76 dB


S -

` N j0 = 43.76 dB
Best S

Hence (B) is correct option.

Statement for Linked Answer Question 57 and 58 :


Consider for CMOS circuit shown, where the gate voltage v0 of the n-MOSFET is
increased from zero, while the gate voltage of the p −MOSFET is kept constant at
3 V. Assume, that, for both transistors, the magnitude of the threshold voltage is
1 V and the product of the trans-conductance parameter is 1mA. V - 2

Brought to you by: Nodia and Company Visit us at: www.nodia.co.in


PUBLISHING FOR GATE
Page 29 GATE EC 2009 www.gatehelp.com

MCQ 1.57 For small increase in VG beyond 1V, which of the following gives the correct
description of the region of operation of each MOSFET
(A) Both the MOSFETs are in saturation region
(B) Both the MOSFETs are in triode region
(C) n-MOSFETs is in triode and p −MOSFET is in saturation region
(D) n- MOSFET is in saturation and p −MOSFET is in triode region
SOL 1.57 For small increase in VG beyond 1 V the n − channel MOSFET goes into saturation
as VGS "+ ive and p − MOSFET is always in active region or triode region.
Hence (D) is correct option.
MCQ 1.58 Estimate the output voltage V0 for VG = 1.5 V. [Hints : Use the appropriate current-
voltage equation for each MOSFET, based on the answer to Q.57]
(A) 4 − 1 (B) 4 + 1
2 2

(C) 4 − 3 (D) 4 + 3
2 2
SOL 1.58 Hence (C) is correct option.

Statement for Linked Answer Question 59 & 60 :


Two products are sold from a vending machine, which has two push buttons P1
and P2 .
When a buttons is pressed, the price of the corresponding product is displayed in
a 7 - segment display. If no buttons are pressed, '0' is displayed signifying ‘Rs 0’.
If only P1 is pressed, ‘2’ is displayed, signifying ‘Rs. 2’
If only P2 is pressed ‘5’ is displayed, signifying ‘Rs. 5’
If both P1 and P2 are pressed, 'E' is displayed, signifying ‘Error’
The names of the segments in the 7 - segment display, and the glow of the display
for ‘0’, ‘2’, ‘5’ and ‘E’ are shown below.

Brought to you by: Nodia and Company Visit us at: www.nodia.co.in


PUBLISHING FOR GATE
Page 30 GATE EC 2009 www.gatehelp.com

Consider
(1) push buttons pressed/not pressed in equivalent to logic 1/0 respectively.
(2) a segment glowing/not glowing in the display is equivalent to logic 1/0
respectively.

MCQ 1.59 If segments a to g are considered as functions of P1 and P2 , then which of the
following is correct
(A) g = P 1 + P2, d = c + e (B) g = P1 + P2, d = c + e
(C) g = P1 + P2, e = b + c (D) g = P1 + P2, e = b + c
SOL 1.59 The given situation is as follows

The truth table is as shown below

P1 P2 a b c d e f g
0 0 1 1 1 1 1 1 0
0 1 1 0 1 1 0 1 1
1 0 1 1 0 1 1 0 1
1 1 1 0 0 1 1 1 1
From truth table we can write
a =1
b = P 1 P 2 + P1 P 2 = P 2 1 NOT Gate
c = P1 P2 + P1 P2 = P1 1 NOT Gate
d = 1 = c+e
and c = P1 P2 = P1 + P2 1 OR GATE
f = P1 P2 = P1 + P2 1 OR GATE
g = P1 P2 = P1 + P2 1 OR GATE
Thus we have g = P1 + P2 and d = 1 = c + e . It may be observed easily from figure
that
Led g does not glow only when both P1 and P2 are 0. Thus

Brought to you by: Nodia and Company Visit us at: www.nodia.co.in


PUBLISHING FOR GATE
Page 31 GATE EC 2009 www.gatehelp.com

g = P1 + P2
LED d is 1 all condition and also it depends on
d = c+e
Hence (B) is correct answer.
MCQ 1.60 What are the minimum numbers of NOT gates and 2 - input OR gates required to
design the logic of the driver for this 7 - Segment display
(A) 3 NOT and 4 OR (B) 2 NOT and 4 OR
(C) 1 NOT and 3 OR (D) 2 NOT and 3 OR
SOL 1.60 As shown in previous solution 2 NOT gates and 3-OR gates are required.
Hence (D) is correct answer.

Answer Sheet
1. (B) 13. (C) 25. (A) 37. (A) 49. (C)
2. (A) 14. (A) 26. (D) 38. (C) 50. (D)
3. (B) 15. (A) 27. (B) 39. (A) 51. (B)
4. (C) 16. (D) 28. (B) 40. (A) 52. (B)
5. (C) 17. (C) 29. (C) 41. (B) 53. (B)
6. (C) 18. (C) 30. (A) 42. (D) 54. (C)
7. (A) 19. (C) 31. (A) 43. (B) 55. (C)
8. (C) 20. (D) 32. (D) 44. (A) 56. (B)
9. (*) 21. (C) 33. (A) 45. (B) 57. (D)
10. (A) 22. (D) 34. (D) 46. (B) 58. (C)
11. (C) 23. (B) 35. (D) 47. (C) 59. (B)
12. (*) 24. (B) 36. (D) 48. (B) 60. (D)

Brought to you by: Nodia and Company Visit us at: www.nodia.co.in


PUBLISHING FOR GATE
Page 34 GATE EC 2009 www.gatehelp.com

Brought to you by: Nodia and Company Visit us at: www.nodia.co.in


PUBLISHING FOR GATE
GATE EC
2010

Q. No. 1 - 25 Carry One Mark Each

MCQ 1.1 The eigen values of a skew-symmetric matrix are


(A) always zero (B) always pure imaginary
(C) either zero or pure imaginary (D) always real
SOL 1.1 Eigen value of a Skew-symmetric matrix are either zero or pure imaginary in
conjugate pairs.
Hence (C) is correct option.

MCQ 1.2 The trigonometric Fourier series for the waveform f (t) shown below contains

(A) only cosine terms and zero values for the dc components
(B) only cosine terms and a positive value for the dc components
(C) only cosine terms and a negative value for the dc components
(D) only sine terms and a negative value for the dc components
SOL 1.2 For a function x (t) trigonometric fourier series is
3
x (t) = Ao + / [An cos nωt + Bn sin nωt]
n=1

Where, Ao = 1 # x (t) dt T0 "fundamental period


T0 T
0

An = 2 # x (t) cos nωt dt


T0 T
0

Brought to you by: Nodia and Company Visit us at: www.nodia.co.in


PUBLISHING FOR GATE
Page 2 GATE EC 2010 www.gatehelp.com

Bn = 2 # x (t) sin nωt dt


T0 T
0

For an even function x (t), Bn = 0


Since given function is even function so coefficient Bn = 0 , only cosine and constant
terms are present in its fourier series representation.
Constant term :
A0 = 1 #
3T/4
x (t) dt
T −T/4
= 1 : # Adt + # − 2AdtD
T/4 3T/4

T −T/4 T/4

= 1 :TA − 2AT D =− A
T 2 2 2
Constant term is negative.
Hence (C) is correct option.
d 2 n (x) n (x)
MCQ 1.3 A function n (x) satisfied the differential equation − 2 =0
dx 2 L
where L is a constant. The boundary conditions are : n (0) = K and n (3) = 0 . The
solution to this equation is
(A) n (x) = K exp (x/L) (B) n (x) = K exp (− x/ L )
(C) n (x) = K 2 exp (− x/L) (D) n (x) = K exp (− x/L)
SOL 1.3 Given differential equation
d 2 n (x) n (x)
− 2 =0
dx 2 L
Let n (x) = Aeλx
λx
So, Aλ2 eλx − Ae2 = 0
L
λ2 − 12 = 0 & λ = ! 1
L L
Boundary condition, n (3) = 0 so take λ =− 1
L
x
n (x) = Ae− L
n (0) = Ae0 = K & A = K
So, n (x) = Ke− (x/L)
Hence (D) is correct option.
MCQ 1.4 For the two-port network shown below, the short-circuit admittance parameter
matrix is

Brought to you by: Nodia and Company Visit us at: www.nodia.co.in


PUBLISHING FOR GATE
Page 3 GATE EC 2010 www.gatehelp.com

4 −2 1 − 0.5
(A) >
−2 4 H
(B) >
− 0.5 1 H
S S

1 0.5 4 2
(C) >
0.5 1 H
S (D) > H S
2 4
SOL 1.4 Given circuit is as shown below

By writing node equation at input port


I1 = V1 + V1 − V2 = 4V1 − 2V2 ...(1)
0.5 0.5
By writing node equation at output port
I2 = V2 + V2 − V1 =− 2V1 + 4V2 ...(2)
0.5 0.5
From (1) and (2), we have admittance matrix
4 −2
Y =>
− 2 4H
Hence (A) is correct option.
MCQ 1.5 For parallel RLC circuit, which one of the following statements is NOT correct ?
(A) The bandwidth of the circuit decreases if R is increased
(B) The bandwidth of the circuit remains same if L is increased
(C) At resonance, input impedance is a real quantity
(D) At resonance, the magnitude of input impedance attains its minimum values.
SOL 1.5 A parallel RLC circuit is shown below :

Brought to you by: Nodia and Company Visit us at: www.nodia.co.in


PUBLISHING FOR GATE
Page 4 GATE EC 2010 www.gatehelp.com

Input impedance Z in = 1
1 + 1 + jω C
R jω L
At resonance 1 = ωC
ωL
So, Z in = 1 = R (maximum at resonance)
1/R
Thus (D) is not true.
Furthermore bandwidth is ωB i.e ωB \ 1 and is independent of L,
R
Hence statements A, B, C, are true.
Hence (D) is correct option.
MCQ 1.6 At room temperature, a possible value for the mobility of electrons in the inversion
layer of a silicon n -channel MOSFET is
(A) 450 cm2 / V-s (B) 1350 cm2 / V-s
(C) 1800 cm2 / V-s (D) 3600 cm2 / V-s
SOL 1.6 At room temperature mobility of electrons for Si sample is given μn = 1350 cm2 /Vs.
For an n -channel MOSFET to create an inversion layer of electrons, a large positive
gate voltage is to be applied. Therefore, induced electric field increases and mobility
decreases.
So, Mobility μn < 1350 cm2 /Vs for n -channel MOSFET
Hence (A) is correct option.
MCQ 1.7 Thin gate oxide in a CMOS process in preferably grown using
(A) wet oxidation (B) dry oxidation
(C) epitaxial oxidation (D) ion implantation
SOL 1.7 Dry oxidation is used to achieve high quality oxide growth.
Hence (B) is correct option.
MCQ 1.8 In the silicon BJT circuit shown below, assume that the emitter area of transistor
Q1 is half that of transistor Q2

The value of current Io is approximately


(A) 0.5 mA (B) 2 mA

Brought to you by: Nodia and Company Visit us at: www.nodia.co.in


PUBLISHING FOR GATE
Page 5 GATE EC 2010 www.gatehelp.com

(C) 9.3 mA (D) 15 mA


SOL 1.8 Since, emitter area of transistor Q1 is half of transistor Q2 , so current
IE = 1 IE and IB = 1 IB
1
2 2 1
2 2

The circuit is as shown below :

VB =− 10 − (− 0.7) =− 9.3 V
Collector current
0 − (− 9.3)
I1 = = 1 mA
(9.3 kΩ)
β 1 = 700 (high), So IC . IE 1

Applying KCL at base we have


1 − IE = IB + IB
1 2

1 − (β 1 + 1) IB = IB + IB
1 1 2

I
1 = (700 + 1 + 1) B + IB
2

2 2

IB . 2
2
702
I 0 = IC = β 2 : IB = 715 # 2 . 2 mA
2 2
702
Hence (B) is correct option.
MCQ 1.9 The amplifier circuit shown below uses a silicon transistor. The capacitors CC and
CE can be assumed to be short at signal frequency and effect of output resistance
r0 can be ignored. If CE is disconnected from the circuit, which one of the following
statements is true

Brought to you by: Nodia and Company Visit us at: www.nodia.co.in


PUBLISHING FOR GATE
Page 6 GATE EC 2010 www.gatehelp.com

(A) The input resistance Ri increases and magnitude of voltage gainAV decreases
(B) The input resistance Ri decreases and magnitude of voltage gain AV increases
(C) Both input resistance Ri and magnitude of voltage gain AV decreases
(D) Both input resistance Ri and the magnitude of voltage gain AV increases
SOL 1.9 The equivalent circuit of given amplifier circuit (when CE is connected, RE is short-
circuited)

Input impedance Ri = RB || r π
Voltage gain AV = gm RC
Now, if CE is disconnected, resistance RE appears in the circuit

Input impedance R in = RB || [rπ + (β + 1)] RE


Input impedance increases
gm RC
Voltage gain AV = Voltage gain decreases.
1 + gm R E
Hence (A) is correct option.
MCQ 1.10 Assuming the OP-AMP to be ideal, the voltage gain of the amplifier shown below

Brought to you by: Nodia and Company Visit us at: www.nodia.co.in


PUBLISHING FOR GATE
Page 7 GATE EC 2010 www.gatehelp.com

is

(A) − R2 (B) − R 3
R1 R1

(D) −b R2 + R 3 l
R2 || R 3
(C) −
R1 R1
SOL 1.10 The circuit is as shown below :

So, 0 − Vi + 0 − Vo = 0
R1 R2
or Vo =− R2
Vi R1
Hence (A) is correct option.
MCQ 1.11 Match the logic gates in Column A with their equivalents in Column B

Brought to you by: Nodia and Company Visit us at: www.nodia.co.in


PUBLISHING FOR GATE
Page 8 GATE EC 2010 www.gatehelp.com

(A) P-2, Q-4, R-1, S-3 (B) P-4, Q-2, R-1, S-3
(C) P-2, Q-4, R-3, S-1 (D) P-4, Q-2, R-3, S-1
SOL 1.11 Hence Correct Option is (D)

MCQ 1.12 For the output F to be 1 in the logic circuit shown, the input combination should
be

(A) A = 1, B = 1, C = 0 (B) A = 1, B = 0, C = 0
(C) A = 0, B = 1, C = 0 (D) A = 0, B = 0, C = 1
SOL 1.12 In the circuit F = (A 5 B) 9 (A 9 B) 9 C
For two variables A 5 B = A9B
So,(A 5 B) 9 (A 9 B) = 0 (always)
F = 09C = 0$C+1$C = C
So, F = 1 when C = 1 or C = 0
Hence (A) (B) (C) are correct options.
MCQ 1.13 In the circuit shown, the device connected Y5 can have address in the range

Brought to you by: Nodia and Company Visit us at: www.nodia.co.in


PUBLISHING FOR GATE
Page 9 GATE EC 2010 www.gatehelp.com

(A) 2000-20FF (B) 2D00-2DFF


(C) 2E00-2EFF (D) FD00-FDFF
SOL 1.13 Since G2 is active low input, output of NAND gate must be 0
G2 = A15 : A14 A13 A12 A11 = 0
So, A15 A14 A13 A12 A11 = 00101
To select Y5 Decoder input
ABC = A 8 A 9 A10 = 101
Address range
A15 A14 A13 A12 A11 A10 A 9 A 8 ...............A 0

S S
0011101........A 0
2 D

^2D00 − 2DFF h
Hence (B) is correct option.
MCQ 1.14 Consider the z -transform x (z) = 5z2 + 4z−1 + 3; 0 < z < 3. The inverse z -
transform x [n] is
(A) 5δ [n + 2] + 3δ [n] + 4δ [n − 1] (B) 5δ [n − 2] + 3δ [n] + 4δ [n + 1]

(C) 5u [n + 2] + 3u [n] + 4u [n − 1] (D) 5u [n − 2] + 3u [n] + 4u [n + 1]


SOL 1.14 Hence (A) is correct option. Hence (A) is correct option.
Inverse Z − transform
We know that αZ ! a αδ [n ! a]
−1
Given that X (z) = 5z + 4z + 3
2

Inverse z-transform x [n] = 5δ [n + 2] + 4δ [n − 1] + 3δ [n]


MCQ 1.15 Two discrete time system with impulse response h1 [n] = δ [n − 1] and h2 [n] = δ [n − 2]
are connected in cascade. The overall impulse response of the cascaded system is
(A) δ [n − 1] + δ [n − 2] (B) δ [n − 4]

(C) δ [n − 3] (D) δ [n − 1] δ [n − 2]

Brought to you by: Nodia and Company Visit us at: www.nodia.co.in


PUBLISHING FOR GATE
Page 10 GATE EC 2010 www.gatehelp.com

SOL 1.15 Hence (C) is correct option


We have h1 [n] = δ [n − 1] or H1 [Z ] = Z − 1
and h 2 [n] = δ [n − 2] or H2 (Z ) = Z − 2
Response of cascaded system
H (z ) = H1 (z ) : H2 (z ) = z−1 : z−2 = z−3
or, h [n] = δ [n − 3]
MCQ 1.16 For a N -point FET algorithm N = 2m which one of the following statements is
TRUE ?
(A) It is not possible to construct a signal flow graph with both input and output
in normal order
(B) The number of butterflies in the m th stage in N/m
(C) In-place computation requires storage of only 2N data
(D) Computation of a butterfly requires only one complex multiplication.
SOL 1.16 For an N-point FET algorithm butterfly operates on one pair of samples and
involves two complex addition and one complex multiplication.
Hence (D) is correct option.
MCQ 1.17 The transfer function Y (s) /R (s) of the system shown is

(A) 0 (B) 1
s+1

(C) 2 (D) 2
s+1 s+3
SOL 1.17 From the given block diagram

H (s) = Y (s) − E (s) $ 1


s+1
E (s) = R (s) − H (s)

Brought to you by: Nodia and Company Visit us at: www.nodia.co.in


PUBLISHING FOR GATE
Page 11 GATE EC 2010 www.gatehelp.com

E (s)
= R (s) − Y (s) +
(s + 1)
E (s) :1 − 1
s + 1D
= R (s) − Y (s)

sE (s)
= R (s) − Y (s) ...(1)
(s + 1)
E (s)
Y (s) = ...(2)
s+1
From (1) and (2) sY (s) = R (s) − Y (s)
(s + 1) Y (s) = R (s)
Transfer function
Y (s)
= 1
R (s) s + 1
Hence (B) is correct option.
A system with transfer function X^^s hh = s +s p has an output y (t) = cos ^2t − π3 h for the
Ys
MCQ 1.18
input signal x (t) = p cos ^2t − π2 h. Then, the system parameter p is
(A) 3 (B) 2
3

(C) 1 (D) 3
2
SOL 1.18 Transfer function is given as
Y (s)
H (s) = = s
X (s) s + p

H (jω) =
jω + p
Amplitude Response
H (jω) = ω
ω +p2
2

Phase Response θh (ω) = 90c − tan−1 a ω k


p

Input x (t) = p cos a2t − π k


2
Output y (t) = H (jω) x (t − θh) = cos a2t − π k
3
H (jω) = p = ω
ω +p2
2

1 = 2 , (ω = 2 rad/ sec)
p 4+p2
or 4p 2 = 4 + p 2 & 3p 2 = 4

or p = 2/ 3
Alternative :
Brought to you by: Nodia and Company Visit us at: www.nodia.co.in
PUBLISHING FOR GATE
Page 12 GATE EC 2010 www.gatehelp.com

θh = 9− π − a− π kC = π
3 2 6
π = π − tan−1 ω
So,
6 2 apk

tan−1 a ω k = π − π = π
p 2 6 3
ω = tan π = 3
p a3k
2 = 3 , (ω = 2 rad/ sec)
p
or p = 2/ 3
Hence (B) is correct option
MCQ 1.19 For the asymptotic Bode magnitude plot shown below, the system transfer function
can be

(A) 10s + 1 (B) 100s + 1


0.1s + 1 0.1s + 1

(C) 100s (D) 0.1s + 1


10s + 1 10s + 1
SOL 1.19 Initial slope is zero, so K = 1
At corner frequency ω 1 = 0.5 rad/ sec , slope increases by + 20 dB/decade, so there
is a zero in the transfer function at ω 1
At corner frequency ω 2 = 10 rad/ sec , slope decreases by − 20 dB/decade and
becomes zero, so there is a pole in transfer function at ω 2
K a1 + s k
ω1
Transfer function H (s) =
a1 + ω 2 k
s

1 a1 + s k (1 + 10s)
0. 1
= =
(1 + 0.1s)
a1 + 0.1 k
s

Hence (A) is correct option


MCQ 1.20 Suppose that the modulating signal is m (t) = 2 cos (2πfm t) and the carrier signal
is xC (t) = AC cos (2πfC t), which one of the following is a conventional AM signal

Brought to you by: Nodia and Company Visit us at: www.nodia.co.in


PUBLISHING FOR GATE
Page 13 GATE EC 2010 www.gatehelp.com

without over-modulation
(A) x (t) = AC m (t) cos (2πfC t)
(B) x (t) = AC [1 + m (t)] cos (2πfC t)
(C) x (t) = AC cos (2πfC t) + AC m (t) cos (2πfC t)
4
(D) x (t) = AC cos (2πfm t) cos (2πfC t) + AC sin (2πfm t) sin (2πfC t)
SOL 1.20 Conventional AM signal is given by
x (t) = AC [1 + μm (t)] cos (2πfC t)
Where μ < 1, for no over modulation.
In option (C)
x (t) = AC :1 + 1 m (t)D cos (2πfC t)
4
Thus μ = 1 < 1 and this is a conventional AM-signal without over-modulation
4
Hence (C) is correct option.
MCQ 1.21 Consider an angle modulated signal
x (t) = 6 cos [2π # 106 t + 2 sin (800πt)] + 4 cos (800πt)
The average power of x (t) is
(A) 10 W (B) 18 W
(C) 20 W (D) 28 W
SOL 1.21 Hence (B) is correct option.
(6) 2
Power P = = 18 W
2

MCQ 1.22 If the scattering matrix [S ] of a two port network is

0.2 0c 0.9 90c


[S ] = > H, then the network is
0.9 90c 0.1 90c

(A) lossless and reciprocal (B) lossless but not reciprocal


(C) not lossless but reciprocal (D) neither lossless nor reciprocal
SOL 1.22 For a lossless network
S11 2 + S21 2 = 1
For the given scattering matrix
S11 = 0.2 0c , S12 = 0.9 90c
S21 = 0.9 90c , S22 = 0.1 90c
Here, (0.2) 2 + (0.9) 2 ! 1 (not lossless)
Reciprocity :
S12 = S21 = 0.9 90c (Reciprocal)
Hence (C) is correct option.

Brought to you by: Nodia and Company Visit us at: www.nodia.co.in


PUBLISHING FOR GATE
Page 14 GATE EC 2010 www.gatehelp.com

MCQ 1.23 A transmission line has a characteristic impedance of 50 Ω and a resistance of


0.1 Ω/m . If the line is distortion less, the attenuation constant(in Np/m) is
(A) 500 (B) 5
(C) 0.014 (D) 0.002
SOL 1.23 For distortion less transmission line characteristics impedance
Z0 = R
G
Attenuation constant
α = RG

So, α = R = 0.1 = 0.002


Z0 50

Hence (D) is correct option.


MCQ 1.24 Consider the pulse shape s (t) as shown. The impulse response h (t) of the filter
matched to this pulse is

SOL 1.24 Impulse response of the matched filter is given by


h (t) = S (T − t)

Brought to you by: Nodia and Company Visit us at: www.nodia.co.in


PUBLISHING FOR GATE
Page 15 GATE EC 2010 www.gatehelp.com

Hence (C) is correct option.


MCQ 1.25 The electric field component of a time harmonic plane EM wave traveling in a
nonmagnetic lossless dielectric medium has an amplitude of 1 V/m. If the relative
permittivity of the medium is 4, the magnitude of the time-average power density
vector (in W/m2 ) is
(A) 1 (B) 1
30π 60π

(C) 1 (D) 1
120π 240π
SOL 1.25 Intrinsic impedance of EM wave
μ μ0
η = = = 120π = 60π
ε 4ε0 2
Time average power density
2
Pav = 1 EH = 1 E = 1 = 1
2 2 η 2 # 60π 120π
Hence (C) is correct option.

Q. No. 26-51 carry two marks each :

MCQ 1.26 If ey = x1/x , then y has a


(A) maximum at x = e (B) minimum at x = e
−1
(C) maximum at x = e (D) minimum at x = e−1
SOL 1.26 Hence (A) is correct option.
1
Given that ey = x x
1
or ln ey = ln x x
or y = 1 ln x
x

Brought to you by: Nodia and Company Visit us at: www.nodia.co.in


PUBLISHING FOR GATE
Page 16 GATE EC 2010 www.gatehelp.com

= 1 1 + ln x ^− x− x h = 12 − ln2
dy 1
Now 2

dx xx x x
For maxima and minima :
dy
= 12 (1 − ln x) = 0
dx x
ln x = 1 " x = e 1
d 2y
Now =− 23 − ln x b− 23 l − 12 b 1 l
dx 2 x x x x
=− 22 + 2 ln3 x − 13
x x x
= −22 + 23 − 13 < 0
2
d x
dy 2 at x = e
1 e e e
So, y has a maximum at x = e1
MCQ 1.27 A fair coin is tossed independently four times. The probability of the event “the
number of time heads shown up is more than the number of times tail shown up”
(A) 1 (B) 1
16 8

(C) 1 (D) 5
4 16
SOL 1.27 According to given condition head should comes 3 times or 4 times
P (Heads comes 3 times or 4 times) = 4C 4 b 1 l + 4C 3 b 1 l b 1 l
4 3

2 2 2
= 1: 1 +4:1 :1 = 5
16 8 2 16
Hence (D) is correct option.
MCQ 1.28 v = xyatx + x 2 aty , then
If A # Av .dlv over the path shown in the figure is
C

(A) 0 (B) 2
3

(C) 1 (D) 2 3
SOL 1.28 Hence (C) is correct option
Av = xyatx + x 2 aty

Brought to you by: Nodia and Company Visit us at: www.nodia.co.in


PUBLISHING FOR GATE
Page 17 GATE EC 2010 www.gatehelp.com

v = dxatx + dyaty
dl
# Av : dl
v = # (xyatx + x 2 aty) : (dxatx + dyaty)
C C

= # (xydx + x 2 dy)
C

= #1/
2/ 3
xdx +
1/ 3
#2/ 3xdx + #1
3 4 dy + #3
1 1 dy
3 3 3 3
= 1 : 4 − 1 D + 3 :1 − 4 D + 4 [3 − 1] + 1 [1 − 3]
2 3 3 2 3 3 3 3
=1

MCQ 1.29 The residues of a complex function x (z) = 1 − 2z at its poles are
z (z − 1) (z − 2)
(A) 1 , − 1 and 1 (B) 1 , − 1 and − 1
2 2 2 2

(C) 1 , − 1 and − 3 (D) 1 , − 1 and 3


2 2 2 2
SOL 1.29 Hence (C) is correct option.
Given function
X (z ) = 1 − 2z
z (z − 1) (z − 2)
Poles are located at z = 0, z = 1, and z = 2
At Z = 0 residues is
R 0 = z : X (z) Z = 0 = 1 − 2 # 0 = 1
(0 − 1) (0 − 2) 2
at z = 1, R1 = (Z − 1) : X (Z ) Z = 1

= 1−2#1 = 1
1 (1 − 2)
At z = 2 , R2 = (z − 2) : X (z) z = 2

= 1 − 2 # 2 =− 3
2 (2 − 1) 2

Consider differential equation dx^ h − y (x) = x , with the initial condition y (0) = 0 .
dy x
MCQ 1.30
Using Euler’s first order method with a step size of 0.1, the value of y (0.3) is
(A) 0.01 (B) 0.031
(C) 0.0631 (D) 0.1
SOL 1.30 Hence (B) is correct option.
Taking step size h = 0.1, y (0) = 0
x y dy dy
= x+y yi + 1 = yi + h
dx dx
0 0 0 y1 = 0 + 0.1 (0) = 0

Brought to you by: Nodia and Company Visit us at: www.nodia.co.in


PUBLISHING FOR GATE
Page 18 GATE EC 2010 www.gatehelp.com

x y dy dy
= x+y yi + 1 = yi + h
dx dx
0.1 0 0.1 y2 = 0 + 0.1 (0.1) = 0.01
0.2 0.01 0.21 y 3 = 0.01 + 0.21 # 0.1 = 0.031
0.3 0.031
From table, at x = 0.3, y (x = 0.3) = 0.031

Given f (t) = L−1 ; 3 3s + 1


s + 4s2 + (k − 3) s E
MCQ 1.31 . If lim f (t) = 1, then the value of k is
t"3

(A) 1 (B) 2
(C) 3 (D) 4
SOL 1.31 Hence (D) is correct option.
f (t) = L − 1 ; 3 3s + 1
s + 4s 2 + (k − 3) s E
We have

and lim f (t) = 1


t"3

By final value theorem


lim f (t) = lim sF (s) = 1
t"3 s"0

s. (3s + 1)
or lim =1
s"0 s + 4s2 + (k − 3) s
3

s (3s + 1)
or lim 2 =1
s " 0 s [s + 4s + (k − 3)]

1 =1
k−3
or k =4
MCQ 1.32 In the circuit shown, the switch S is open for a long time and is closed at t = 0 .
The current i (t) for t $ 0+ is

(A) i (t) = 0.5 − 0.125e−1000t A (B) i (t) = 1.5 − 0.125e−1000t A

(C) i (t) = 0.5 − 0.5e−1000t A (D) i (t) = 0.375e−1000t A


SOL 1.32 Hence (A) is correct option.
Let the current i (t) = A + Be−t/τ τ " Time constant
Brought to you by: Nodia and Company Visit us at: www.nodia.co.in
PUBLISHING FOR GATE
Page 19 GATE EC 2010 www.gatehelp.com

When the switch S is open for a long time before t < 0 , the circuit is

At t = 0 , inductor current does not change simultaneously, So the circuit is

Current is resistor (AB)


i (0) = 0.75 = 0.375 A
2
Similarly for steady state the circuit is as shown below

i (3) = 15 = 0.5 A
3
−3
τ = L = 15 # 10 = 10−3 sec
Req 10 + (10 || 10)
t
i (t) = A + Be− 1 # 10 = A + Be−100t
−3

Now i (0) = A + B = 0.375


and i (3) = A = 0.5
So, B = 0.375 − 0.5 =− 0.125
Hence i (t) = 0.5 − 0.125e−1000 t A
MCQ 1.33 The current I in the circuit shown is

(A) − j1 A (B) j1 A
(C) 0 A (D) 20 A

Brought to you by: Nodia and Company Visit us at: www.nodia.co.in


PUBLISHING FOR GATE
Page 20 GATE EC 2010 www.gatehelp.com

SOL 1.33 Circuit is redrawn as shown below

Where, Z1 = jωL = j # 103 # 20 # 10−3 = 20j


Z2 = R || XC
XC = 1 = 1 =− 20j
jωC j # 103 # 50 # 10−6
1 (− 20j)
Z2 = R = 1Ω
1 − 20j
Voltage across Z2
− 20j
c 1 − 20j m
VZ = Z2 : 20 0 = : 20
Z1 + Z 2
c 20j − 1 − 20j m
2
20j

(− 20j)
=c
20j + 400 − 20j m
: 20 =− j

Current in resistor R is
V j
I = Z =− =− j A
2

R 1
Hence (A) is correct option.
MCQ 1.34 In the circuit shown, the power supplied by the voltage source is

(A) 0 W (B) 5 W
(C) 10 W (D) 100 W
SOL 1.34 The circuit can be redrawn as

Brought to you by: Nodia and Company Visit us at: www.nodia.co.in


PUBLISHING FOR GATE
Page 21 GATE EC 2010 www.gatehelp.com

Applying nodal analysis


VA − 10 + 1 + VA − 0 = 0
2 2
2VA − 10 + 2 = 0 = V4 = 4 V
Current, I1 = 10 − 4 = 3 A
2
Current from voltage source is
I 2 = I1 − 3 = 0
Since current through voltage source is zero, therefore power delivered is zero.
Hence (A) is correct option.
MCQ 1.35 In a uniformly doped BJT, assume that NE , NB and NC are the emitter, base and
collector doping in atoms/cm3 , respectively. If the emitter injection efficiency of the
BJT is close unity, which one of the following condition is TRUE
(A) NE = NB = NC (B) NE >> NB and NB > NC

(C) NE = NB and NB < NC (D) NE < NB < NC


SOL 1.35 Emitter injection efficiency is given as
γ = 1
1 + NB
NE
To achieve γ = 1, NE >> NB
Hence (B) is correct option.
MCQ 1.36 Compared to a p-n junction with NA = ND = 1014 /cm3 , which one of the following
statements is TRUE for a p-n junction with NA = ND = 1020 /cm3 ?
(A) Reverse breakdown voltage is lower and depletion capacitance is lower
(B) Reverse breakdown voltage is higher and depletion capacitance is lower
(C) Reverse breakdown voltage is lower and depletion capacitance is higher
(D) Reverse breakdown voltage is higher and depletion capacitance is higher
SOL 1.36 Reverse bias breakdown or Zener effect occurs in highly doped PN junction through
tunneling mechanism. In a highly doped PN junction, the conduction and valence

Brought to you by: Nodia and Company Visit us at: www.nodia.co.in


PUBLISHING FOR GATE
Page 22 GATE EC 2010 www.gatehelp.com

bands on opposite sides of the junction are sufficiently close during reverse bias
that electron may tunnel directly from the valence band on the p-side into the
conduction band on n -side.
Breakdown voltage VB \ 1
NA ND
So, breakdown voltage decreases as concentration increases
Depletion capacitance
C =' eεs NA ND
1
1/2

2 (Vbi + VR) (NA + ND)


Thus C \ NA ND
Depletion capacitance increases as concentration increases
Hence (C) is correct option.
MCQ 1.37 Assuming that the flip-flop are in reset condition initially, the count sequence
observed at QA , in the circuit shown is

(A) 0010111... (B) 0001011...


(C) 0101111... (D) 0110100....
SOL 1.37 Let QA (n), QB (n), QC (n) are present states and QA (n + 1), QB (n + 1), QC (n + 1) are
next states of flop-flops.
In the circuit
QA (n + 1) = QB (n) 9 QC (n)
QB (n + 1) = QA (n)
QC (n + 1) = QB (n)
Initially all flip-flops are reset
1st clock pulse
QA = 0 9 0 = 1
QB = 0
QC = 0
2 nd clock pulse
QA = 0 9 0 = 1
QB = 1
QC = 0
rd
3 clock pulse

Brought to you by: Nodia and Company Visit us at: www.nodia.co.in


PUBLISHING FOR GATE
Page 23 GATE EC 2010 www.gatehelp.com

QA = 1 9 0 = 0
QB = 1
QC = 1
4 th clock pulse
QA = 1 9 1 = 1
QB = 0
QC = 1
So, sequence QA = 01101.......
Hence (D) is correct option.
MCQ 1.38 The transfer characteristic for the precision rectifier circuit shown below is (assume
ideal OP-AMP and practical diodes)

SOL 1.38 The circuit is as shown below

Brought to you by: Nodia and Company Visit us at: www.nodia.co.in


PUBLISHING FOR GATE
Page 24 GATE EC 2010 www.gatehelp.com

Current I = 20 − 0 + Vi − 0 = 5 + Vi
4R R R
If I > 0, diode D2 conducts
So, for 5 + VI > 0 & VI > − 5, D2 conducts
2
Equivalent circuit is shown below

Output is Vo = 0 . If I < 0 , diode D2 will be off


5 + VI < 0 & V < − 5, D is off
I 2
R
The circuit is shown below

0 − Vi + 0 − 20 + 0 − Vo = 0
R 4R R

or Vo =− Vi − 5

At Vi =− 5 V, Vo = 0
At Vi =− 10 V, Vo = 5 V
Hence (B) is correct option.
MCQ 1.39 The Boolean function realized by the logic circuit shown is

Brought to you by: Nodia and Company Visit us at: www.nodia.co.in


PUBLISHING FOR GATE
Page 25 GATE EC 2010 www.gatehelp.com

(A) F = Σm (0, 1, 3, 5, 9, 10, 14) (B) F = Σm (2, 3, 5, 7, 8, 12, 13)

(C) F = Σm (1, 2, 4, 5, 11, 14, 15) (D) F = Σm (2, 3, 5, 7, 8, 9, 12)


SOL 1.39 Output of the MUX can be written as
F = I 0 S 0 S1 + I1 S 0 S1 + I 2 S 0 S1 + I 3 S 0 S1
Here, I 0 = C, I1 = D, I2 = C , I 3 = CD
and S 0 = A, S1 = B
So, F = C A B + D A B + C A B + C DA B
Writing all SOP terms
F = A B C D + A B C D + A BCD + A B C D
1 44 2
m
44 3 1 44 2 m
44 3 S m7
1 44 2
m
4
43
3 2 5

+A B C D + A B C D + ABC D
1 44 2
m
4
4 3 1 44 2
m
44 3 S
m 12
9 8

F = / m (2, 3, 5, 7, 8, 9, 12)

Hence (D) is correct option.


MCQ 1.40 For the 8085 assembly language program given below, the content of the accumulator
after the execution of the program is

(A) 00H (B) 45H


(C) 67H (D) E7H
SOL 1.40 By executing instruction one by one
MVI A, 45 H & MOV 45 H into accumulator, A = 45 H
STC & Set carry, C = 1
CMC & Complement carry flag, C = 0
RAR & Rotate accumulator right through carry

Brought to you by: Nodia and Company Visit us at: www.nodia.co.in


PUBLISHING FOR GATE
Page 26 GATE EC 2010 www.gatehelp.com

A = 00100010
XRA B& XOR A and B
A = A 5 B = 00100010 5 01000101 = 01100111 = 674
Hence (C) is correct option.
MCQ 1.41 A continuous time LTI system is described by

d 2 y (t) dy (t) dx (t)


2 +4 + 3y (t) = 2 + 4x (t)
dt dt dt
Assuming zero initial conditions, the response y (t) of the above system for the
input x (t) = e−2t u (t) is given by

(A) (et − e3t) u (t) (B) (e−t − e−3t) u (t)

(C) (e−t + e−3t) u (t) (D) (et + e3t) u (t)


SOL 1.41 System is described as
d 2 y (t) dt (t) dx (t)
+4 + 3y (t) = 2 + 4x (t)
dt 2 dt dt
Taking laplace transform on both side of given equation
s 2 Y (s) + 4sY (s) + 3Y (s) = 2sX (s) + 4X (s)
(s 2 + 4s + 3) Y (s) = 2 (s + 2) X (s) s
Transfer function of the system
Y (s) 2 (s + 2) 2 (s + 2)
H (s) = = 2 =
X (s) s + 4s + 3 (s + 3) (s + 1)
Input x (t) = e−2t u (t)
or, X (s) = 1
(s + 2)
Output Y (s) = H (s) : X (s)
2 (s + 2)
Y (s) = : 1
(s + 3) (s + 1) (s + 2)
By Partial fraction
Y (s) = 1 − 1
s+1 s+3
Taking inverse laplace transform
y (t) = (e−t − e−3t) u (t)
Hence (B) is correct option.
MCQ 1.42 The transfer function of a discrete time LTI system is given by

Brought to you by: Nodia and Company Visit us at: www.nodia.co.in


PUBLISHING FOR GATE
Page 27 GATE EC 2010 www.gatehelp.com

2 − 3 z−1
H (z) = 4
1 − z + 1 z−2
3 −1
4 8
Consider the following statements:
S1: The system is stable and causal for ROC: z > 1/2
S2: The system is stable but not causal for ROC: z < 1/4
S3: The system is neither stable nor causal for ROC: 1/4 < z < 1/2
Which one of the following statements is valid ?
(A) Both S1 and S2 are true (B) Both S2 and S3 are true
(C) Both S1 and S3 are true (D) S1, S2 and S3 are all true
SOL 1.42 Hence (C) is correct option.
We have
2 − 34 z − 1
H (z) =
1 − 34 z − 1 + 18 z − 2
By partial fraction H (z ) can be written as
H (z ) = 1 1
1 −1 +
^1 − 2 z h ^ 1 − 1 −1
4z h
For ROC : z > 1/2

h [n] = b 1 l u [n] + b 1 l u [n], n > 0


n n
1 = an u [n], z > a
2 4 1 − z −1
Thus system is causal. Since ROC of H (z ) includes unit circle, so it is stable also.
Hence S1 is True
For ROC : z < 1
4
h [n] =−b 1 l u [− n − 1] + b 1 l u (n), z > 1 , z < 1
n n

2 4 4 2
System is not causal. ROC of H (z ) does not include unity circle, so it is not stable
and S 3 is True
MCQ 1.43 The Nyquist sampling rate for the signal
sin (500πt) sin (700) πt
s (t) = # is given by
πt πt
(A) 400 Hz (B) 600 Hz
(C) 1200 Hz (D) 1400 Hz
SOL 1.43 Hence(C) is correct option.
S (t) = sin c (500t) sin c (700t)
S (f ) is convolution of two signals whose spectrum covers f 1 = 250 Hz and f 2 = 350 Hz
. So convolution extends
f = 25 + 350 = 600 Hz
Nyquist sampling rate

Brought to you by: Nodia and Company Visit us at: www.nodia.co.in


PUBLISHING FOR GATE
Page 28 GATE EC 2010 www.gatehelp.com

N = 2f = 2 # 600 = 1200 Hz

MCQ 1.44 A unity negative feedback closed loop system has a plant with the transfer function
G (s) = s + 21s + 2 and a controller Gc (s) in the
2

feed forward path. For a unit set input, the transfer function of the controller that
gives minimum steady state error is
(A) Gc (s) = s + 1 (B) Gc (s) = s + 2
s+2 s+1
(s + 1) (s + 4)
(C) Gc (s) = (D) Gc (s) = 1 + 2 + 3s
(s + 2) (s + 3) s
SOL 1.44 Steady state error is given as
sR (s)
eSS = lim
s " 0 1 + G (s) GC (s)

R (s) = 1 (unit step unit)


s
eSS = lim 1
s " 0 1 + G (s) GC (s)

= lim 1
s"0 GC (s)
1+ 2
s + 2s + 2
eSS will be minimum if lim GC (s) is maximum
s"0
In option (D)
lim GC (s) = lim 1 + 2 + 3s = 3
s"0 s"0 s
So, eSS = lim 1 = 0 (minimum)
s"0 3

Hence (D) is correct option.


MCQ 1.45 X (t) is a stationary process with the power spectral density Sx (f ) > 0 , for all f .
The process is passed through a system shown below

Let Sy (f ) be the power spectral density of Y (t). Which one of the following
statements is correct
(A) Sy (f ) > 0 for all f
(B) Sy (f ) = 0 for f > 1 kHz
(C) Sy (f ) = 0 for f = nf0, f0 = 2 kHz kHz, n any integer

Brought to you by: Nodia and Company Visit us at: www.nodia.co.in


PUBLISHING FOR GATE
Page 29 GATE EC 2010 www.gatehelp.com

(D) Sy (f ) = 0 for f = (2n + 1) f0 = 1 kHz , n any integer


SOL 1.45 For the given system, output is written as
y (t) = d [x (t) + x (t − 0.5)]
dt
dx (t) dx (t − 0.5)
y (t) = +
dt dt
Taking laplace on both sides of above equation
Y (s) = sX (s) + se−0.5s X (s)
Y (s)
H (s) = = s (1 + e−0.5s)
X (s)
H (f ) = jf (1 + e−0.5 # 2πf ) = jf (1 + e− πf )
Power spectral density of output
SY (f ) = H (f ) 2 SX (f ) = f 2 (1 + e− πf ) 2 SX (f )

For SY (f ) = 0 , 1 + e− πf = 0
f = (2n + 1) f0
or f0 = 1 KHz
Hence (D) is correct option.
MCQ 1.46 A plane wave having the electric field components Evi = 24 cos ^3 # 108 − βy h atx
V/m and traveling in free space is incident normally on a lossless medium with
μ = μ0 and ε = 9ε0 which occupies the region y $ 0 . The reflected magnetic field
component is given by
(A) 1 cos (3 # 108 t + y) atx A/m
10π

(B) 1 cos (3 108 t + y) at A/m


20π # x

(C) − 1 cos (3 # 108 t + y) atx A/m


20π

(D) − 1 cos (3 # 108 t + y) atx A/m


10π
SOL 1.46 In the given problem

Reflection coefficient
η2 − η 1
τ = = 400π − 120π =− 1
η2 + η 1 40π + 120π 2

Brought to you by: Nodia and Company Visit us at: www.nodia.co.in


PUBLISHING FOR GATE
Page 30 GATE EC 2010 www.gatehelp.com

τ is negative So magnetic field component does not change its direction Direction
of incident magnetic field
atE # atH = atK
atZ # atH = aty
atH = atx ( + x direction)
So, reflection magnetic field component
Hr = τ # 24 cos (3 # 108 + βy) atx , y $ 0
η
= 1 # 24 cos (3 # 108 + βy) atx , y $ 0
2 # 120π

β = ω = 3 # 108 = 1
8

vC 3 # 10
So, Hr = 1 cos (3 # 108 + y) atx , y $ 0
10π
Hence (A) is correct option.
MCQ 1.47 In the circuit shown, all the transmission line sections are lossless. The Voltage
Standing Wave Ration(VSWR) on the 60 Ω line is

(A) 1.00 (B) 1.64


(C) 2.50 (D) 3.00
SOL 1.47 For length of λ/4 transmission line
Z + jZo tan βl
Z in = Zo ; L
Zo + jZL tan βl E
ZL = 30 Ω , Zo = 30 Ω, β = 2π , l = λ
λ 4
So, tan βl = tan b 2π : λ l = 3
λ 4
R ZL V
S tan βl + jZo W 2
Z in = Zo S W = Z 0 = 60 Ω
S Zo + jZL W ZL
S tan βl W
T X
For length of λ/8 transmission line
Z + jZo tan βl
Z in = Zo ; L
Zo + jZL tan βl E

Brought to you by: Nodia and Company Visit us at: www.nodia.co.in


PUBLISHING FOR GATE
Page 31 GATE EC 2010 www.gatehelp.com

Zo = 30 Ω, ZL = 0 (short)
tan βl = tan b 2π : λ l = 1
λ 8
Z in = jZo tan βl = 30j
Circuit is shown below.

Reflection coefficient
60 + 3j − 60
τ = ZL − Zo = = 1
ZL + Zo 60 + 3j + 60 17
1+ τ
VSWR = = 1 + 17 = 1.64
1− τ 1 − 17
Hence (B) is correct option.

Common Data Questions: 48 & 49 :


Consider the common emitter amplifier shown below with the following circuit
parameters:
β = 100, gm = 0.3861 A/V, r0 = 259 Ω, RS = 1 kΩ, RB = 93 kΩ,
RC = 250 kΩ, RL = 1 kΩ, C1 = 3 and C2 = 4.7 μF

MCQ 1.48 The resistance seen by the source vS is


(A) 258 Ω (B) 1258 Ω
(C) 93 kΩ (D) 3
SOL 1.48 By small signal equivalent circuit analysis

Brought to you by: Nodia and Company Visit us at: www.nodia.co.in


PUBLISHING FOR GATE
Page 32 GATE EC 2010 www.gatehelp.com

Input resistance seen by source vs


R in = vs = Rs + Rs || rs
is
= (1000 Ω) + (93 kΩ || 259 Ω) = 1258 Ω
Hence (B) is correct option.
MCQ 1.49 The lower cut-off frequency due to C2 is
(A) 33.9 Hz (B) 27.1 Hz
(C) 13.6 Hz (D) 16.9 Hz
SOL 1.49 Cut-off frequency due to C2
fo = 1
2π (RC + RL) C2
fo = 1 = 271 Hz
2 # 3.14 # 1250 # 4.7 # 10−6
Lower cut-off frequency
f
fL . o = 271 = 27.1 Hz
10 10
Hence (B) is correct option.

Common Data Question : 50 & 51 :


The signal flow graph of a system is shown below:

MCQ 1.50 The state variable representation of the system can be


1 1 0 −1 1 0
xo = > H x +> Hu
(A) x = >− 1 0H x + >2H u
o
(B) −1 0 2
yo = [0 0.5] x yo = 80 0.5B x

1 1 0 −1 1 0
xo = > H x +> Hu xo = > H x +> Hu
(C) −1 0 2 (D) −1 0 2
yo = 80.5 0.5B x yo = 80.5 0.5B x

Brought to you by: Nodia and Company Visit us at: www.nodia.co.in


PUBLISHING FOR GATE
Page 33 GATE EC 2010 www.gatehelp.com

SOL 1.50 Assign output of each integrator by a state variable

xo1 =− x1 + x2
xo2 =− x1 + 2u
y = 0.5x1 + 0.5x2
State variable representation
−1 1 0
xo = > H x + > Hu
−1 0 2
yo = [0.5 0.5] x
Hence (D) is correct option.
MCQ 1.51 The transfer function of the system is
(A) s2+ 1 (B) s2− 1
s +1 s +1

(C) s+1 (D) s−1


s +s+1
2
s +s+1
2

SOL 1.51 By masson’s gain formula

Transfer function
H (s) =
Y (s)
=
/ PK ΔK
U (s) Δ
Forward path given
P1 (abcdef ) = 2 # 1 # 1 # 0.5 = 12
s s s
P2 (abcdef ) = 2 # 1 # 1 # 0.5
3
Loop gain L1 (cdc) =− 1
s
L2 (bcdb) = 1 # 1 # − 1 = −21
s s s
Δ = 1 − [L1 + L2] = 1 − :− 1 − 12 D = 1 + 1 + 12
s s s s
Δ1 = 1, Δ2 = 2

Brought to you by: Nodia and Company Visit us at: www.nodia.co.in


PUBLISHING FOR GATE
Page 34 GATE EC 2010 www.gatehelp.com

= P1 Δ 1 + P2 Δ 2
Y (s)
So, H (s) =
U (s) Δ
1 :1+1:1
2 s (1 + s)
=s = 2
1
1+ + 2 1 (s + s + 1)
s s
Hence (C) is correct option.

Linked Answer Questions: Q. 52 to Q. 55

Statements for Linked Answer Question : 52 & 53 :


The silicon sample with unit cross-sectional area shown below is in thermal
equilibrium. The following information is given: T = 300 K electronic charge
= 1.6 # 10−19 C , thermal voltage = 26 mV and electron mobility = 1350 cm2 / V-s

MCQ 1.52 The magnitude of the electric field at x = 0.5 μm is


(A) 1 kV/cm (B) 5 kV/cm
(C) 10 kV/cm (D) 26 kV/cm
SOL 1.52 Sample is in thermal equilibrium so, electric field

E = 1 = 10 kV/cm
1 μm

Hence (C) is correct option.


MCQ 1.53 The magnitude of the electron of the electron drift current density at x = 0.5 μm is
(A) 2.16 # 10 4 A/cm2 (B) 1.08 # 10 4 A/m2
(C) 4.32 # 103 A/cm2 (D) 6.48 # 102 A/cm2
SOL 1.53 Electron drift current density
Jd = ND μn eE
= 1016 # 1350 # 1.6 # 10−19 # 10 # 1013
= 2.16 # 10 4 A/cm2
Hence (A) is correct option.

Brought to you by: Nodia and Company Visit us at: www.nodia.co.in


PUBLISHING FOR GATE
Page 35 GATE EC 2010 www.gatehelp.com

Statement for linked Answer Question : 54 & 55 :


Consider a baseband binary PAM receiver shown below. The additive channel noise
n (t) is with power spectral density Sn (f ) = N 0 /2 = 10−20 W/Hz . The low-pass filter
is ideal with unity gain and cut-off frequency 1 MHz. Let Yk represent the random
variable y (tk ).
Yk = Nk , if transmitted bit bk = 0
Yk = a + Nk if transmitted bit bk = 1
Where Nk represents the noise sample value. The noise sample has a probability
density function, PNk (n) = 0.5αe− α n (This has mean zero and variance 2/α 2 ).
Assume transmitted bits to be equiprobable and threshold z is set to a/2 = 10−6 V .

MCQ 1.54 The value of the parameter α (in V − 1 ) is


(A) 1010 (B) 107
(C) 1.414 # 10−10 (D) 2 # 10−20
SOL 1.54 Let response of LPF filters
1, f < 1 MHz
H (f ) = *
0, elsewhere
Noise variance (power) is given as
2
H (f ) No df = 22 (given)
fo
P = σ2 = #0
α
2 # 10−20 df = 22
1 # 106
#0
α
2 # 10−20 # 106 = 22
α
α = 1014
2

or α = 107
Hence (B) is correct option.
MCQ 1.55 The probability of bit error is
(A) 0.5 # e−3.5 (B) 0.5 # e−5
(C) 0.5 # e−7 (D) 0.5 # e−10
SOL 1.55 Probability of error is given by

Brought to you by: Nodia and Company Visit us at: www.nodia.co.in


PUBLISHING FOR GATE
Page 36 GATE EC 2010 www.gatehelp.com

Pe = 1 [P (0/1) + P (1/0)]
2
α/2
P (0/1) = #− 3 0.5e− α n − a dn = 0.5e−10
where a = 2 # 10−6 V and α = 107 V − 1

P (1/0) = #a/32 0.5e− α n dn = 0.5e−10

Pe = 0.5e−10
Hence (D) is correct option.

Q. No. 56 - 60 Carry One Mark Each :

MCQ 1.56 Which of the following options is closest in meaning to the world below:
(A) Cyclic (B) Indirect
(C) Confusing (D) Crooked
SOL 1.56 Circuitous means round about or not direct. Indirect is closest in meaning to this
circuitous
(A) Cyclic : Recurring in nature
(B) Indirect : Not direct
(C) Confusing : lacking clarity of meaning
(D) Crooked : set at an angle; not straight
Hence (B) is correct option.
MCQ 1.57 The question below consists of a pair of related words followed by four pairs of
words. Select the pair that best expresses the relation in the original pair.
Unemployed: Worker
(A) fallow : land (B) unaware: sleeper
(C) wit : jester (D) renovated : house
SOL 1.57 A worker may by unemployed. Like in same relation a sleeper may be unaware.
Hence (B) is correct option.
MCQ 1.58 Choose the most appropriate word from the options given below to complete the
following sentence;
If we manage to ____ our natural resources, we would leave a better planet for
our children.
(A) uphold (B) restrain
(C) Cherish (D) conserve
SOL 1.58 Here conserve is most appropriate word.
Hence (D) is correct option.

Brought to you by: Nodia and Company Visit us at: www.nodia.co.in


PUBLISHING FOR GATE
Page 37 GATE EC 2010 www.gatehelp.com

MCQ 1.59 Choose the most appropriate word from the options given below to complete the
following sentence:
His rather casual remarks on politics ___ his lack of seriousness about the subject
(A) masked (B) belled
(C) betrayed (D) suppressed
SOL 1.59 Betrayed means reveal unintentionally that is most appropriate.
Hence (C) is correct option.
MCQ 1.60 25 persons are in a room, 15 of them play hockey, 17 of them football and 10 of
them play both hockey and football. Then the number of persons playing neither
hockey nor football is ;
(A) 2 (B) 17
(C) 13 (D) 3
SOL 1.60 Hence (D) is correct option.
Number of people who play hockey n (A) = 15
Number of people who play football n (B) = 17
Persons who play both hockey and football n (A + B) = 10
Persons who play either hockey or football or both :
n (A , B) = n (A) + n (B) − n (A + B)
= 15 + 17 − 10 = 22
Thus people who play neither hockey nor football = 25 − 22 = 3

Q. No. 61-65 Carry Two Marks Each

MCQ 1.61 Modern warfare has changed from large scale clashes of armies to suppression of
civilian populations. Chemical agents that do their work silently appear to be
suited to such warfare; and regretfully, there exist people in military establishments
who think that chemical agents are useful tools for their cause.
Which of the following statements best sums up the meaning of the above passage :
(A) Modern warfare has resulted in civil strife.
(B) Chemical agents are useful in modern warfare.
(C) Use of chemical agents in warfare would be undesirable
(D) People in military establishment like to use agents in war
SOL 1.61 Hence (D) is correct option.
MCQ 1.62 If 137 + 276 = 435 how much is 731 + 672 ?
(A) 534 (B) 1403
(C) 1623 (D) 1513
SOL 1.62 Since 7 + 6 = 13 but unit digit is 5 so base may be 8 as 5 is the remainder when 13

Brought to you by: Nodia and Company Visit us at: www.nodia.co.in


PUBLISHING FOR GATE
Page 38 GATE EC 2010 www.gatehelp.com

is divided by 8. Let us check.


137 8 731 8
276 8 672 8
435 Thus here base is 8. Now 1623
Hence (C) is correct option.
MCQ 1.63 5 skilled workers can build a wall in 20 days; 8 semi-killed worker can build a wall
in 25 days; 10 unskilled workers can build a wall in 30 days. If a team has 2 killed,
6 semi-killed and 5 unskilled workers, how long will it take to build the wall
(A) 20 days (B) 18 days
(C) 16 days (D) 15 days
SOL 1.63 Hence (D) is correct option.
Let W be the total work.
Per day work of 5 skilled workers =W
20
Per day work of one skill worker = W =W
5 # 20 100
Similarly per day work of 1 semi-skilled workers = W = W
8 # 25 200
Similarly per day work of one semi-skill worker = W = W
10 # 30 300
Thus total per day work of 2 skilled, 6 semi-skilled and 5 unskilled workers is
= 2W + 6W + 5W = 12W + 18W + 10W = W
100 200 300 600 15
Therefore time to complete the work is 15 days.
MCQ 1.64 Given digits 2, 2, 3, 3, 4, 4, 4 how many distinct 4 digit numbers greater than 3000
can be formed
(A) 50 (B) 51
(C) 52 (D) 54
SOL 1.64 As the number must be greater than 3000, it must be start with 3 or 4. Thus we
have two case:
Case (1) If left most digit is 3 an other three digits are any of 2, 2, 3, 3, 4, 4, 4, 4.
(1) Using 2, 2, 3 we have 3223, 3232, 3322 i.e. 3! = 3 no.
2!
(2) Using 2, 2, 4 we have 3224, 3242, 3422 i.e. 3! = 3 no.
2!
(3) Using 2, 3, 3 we have 3233, 3323, 3332 i.e. 3! = 3 no.
2!
(4) Using 2, 3, 4 we have 3! = 6 no.
(5) Using 2, 4, 4 we have 3244, 3424, 3442 i.e. 3! = 3 no.
2!

Brought to you by: Nodia and Company Visit us at: www.nodia.co.in


PUBLISHING FOR GATE
Page 39 GATE EC 2010 www.gatehelp.com

(6) Using 3, 3, 4 we have 3334, 3343, 3433 i.e. 3! = 3 no.


2!

(7) Using 3, 4, 4 we have 3344, 3434, 3443 i.e. 3! = 3 no.


2!

(8) Using 4, 4, 4 we have 3444 i.e. 3! = 1 no.


3!
Total 4 digit numbers in this case is
1 + 3 + 3 + 3 + 6 + 3 + 3 + 3 + 1 = 25
Case 2 : If left most is 4 and other three digits are any of 2, 2, 3, 3, 3, 4, 4, 4.
(1) Using 2, 2, 3 we have 4223, 4232, 4322 i.e. . 3! = 3 no
2!

(2) Using 2, 2, 4 we have 4224, 4242, 4422 i.e. . 3! = 3 no


2!

(3) Using 2, 3, 3 we have 4233, 4323, 4332 i.e. . 3! = 3 no


2!
(4) Using 2, 3, 4 we have i.e. . 3! = 6 no
(5) Using 2, 4, 4 we have 4244, 4424, 4442 i.e. . 3! = 3 no
2!

(6) Using 3, 3, 3 we have 4333 i.e 3! = 1. no.


3!

(7) Using 3, 3, 4 we have 4334, 4343, 4433 i.e. . 3! = 3 no


2!

(8) Using 3, 4, 4 we have 4344, 4434, 4443 i.e. . 3! = 3 no


2!

(9) Using 4, 4, 4 we have 4444 i.e. 3! = 1. no


3!
Total 4 digit numbers in 2nd case = 3 + 3 + 3 + 6 + 3 + 3 + 1 + 3 + 1 = 26
Thus total 4 digit numbers using case (1) and case (2) is = 25 + 26 = 51
Hence (B) is correct option.
MCQ 1.65 Hari(H), Gita(G), Irfan(I) and Saira(S) are sibilings (i.e. brothers and sisters). All
were born on Ist January. The age difference between any two successive siblings
(that is born one after another) is less than 3 years. Given the following facts:
(i) Hari’s age + Gita’s age > Irfan’s age + Saira’s age
(ii) The age difference between Gita and Saira is 1 year. However, Gita is not the
oldest and Saira is not the youngest
(iii) There are not twins.
In what order were they born (oldest first)
(A) HSIG (B) SGHI

Brought to you by: Nodia and Company Visit us at: www.nodia.co.in


PUBLISHING FOR GATE
Page 40 GATE EC 2010 www.gatehelp.com

(C) IGSH (D) IHSG


SOL 1.65 Let H , G , S and I be ages of Hari, Gita, Saira and Irfan respectively.
Now from statement (1) we have H + G > I + S
Form statement (2) we get that G − S = 1 or S − G = 1
As G can’t be oldest and S can’t be youngest thus either GS or SG possible.
From statement (3) we get that there are no twins
(A) HSIG : There is I between S and G which is not possible
(B) SGHI : SG order is also here and S > G > H > I and G + H > S + I which is
possible.
(C) IGSH : This gives I > G and S > H and adding these both inequalities we
have I + S > H + G which is not possible.
(D) IHSG : This gives I > H and S > G and adding these both inequalities we
have I + S > H + G which is not possible.
Hence (B) is correct option.

Answer Sheet
1. (C) 13. (B) 25. (C) 37. (D) 49. (B) 61. (D)
2. (C) 14. (A) 26. (A) 38. (B) 50. (D) 62. (C)
3. (D) 15. (C) 27. (D) 39. (D) 51. (C) 63. (D)
4. (A) 16. (D) 28. (C) 40. (C) 52. (C) 64. (B)
5. (D) 17. (B) 29. (C) 41. (B) 53. (A) 65. (B)
6. (A) 18. (B) 30. (B) 42. (C) 54. (B)
7. (B) 19. (A) 31. (D) 43. (C) 55. (D)
8. (B) 20. (C) 32. (A) 44. (D) 56. (B)
9. (A) 21. (B) 33. (A) 45. (D) 57. (B)
10. (A) 22. (C) 34. (A) 46. (A) 58. (D)
11. (D) 23. (D) 35. (B) 47. (B) 59. (C)
12. (*) 24. (C) 36. (C) 48. (B) 60. (D)

Brought to you by: Nodia and Company Visit us at: www.nodia.co.in


PUBLISHING FOR GATE
GATE EC
2011

Q. No. 1 – 25 Carry One Mark Each

MCQ 1.1 Consider the following statements regarding the complex Poynting vector Pv for the
power radiated by a point source in an infinite homogeneous and lossless medium.
Re(Pv ) denotes the real part of Pv.S denotes a spherical surface whose centre is
at the point source, and nt denotes the unit surface normal on S. Which of the
following statements is TRUE?
(A) Re(Pv ) remains constant at any radial distance from the source
(B) Re(Pv ) increases with increasing radial distance from the source
(C) ## Re (Pv) .nt dS remains constant at any radial distance from the source
S

(D) ## Re (Pv) .nt dS decreases with increasing radial distance form the source
S
SOL 1.1 Power radiated from any source is constant.
Hence (C) is correct option..
MCQ 1.2 A transmission line of characteristic impedance 50 Ω is terminated by a 50 Ω
load. When excited by a sinusoidal voltage source at 10 GHz, the phase difference
between two points spaced 2 mm apart on the line is found to be π radians. The
4
phase velocity of the wave along the line is
(A) 0.8 # 108 m/s (B) 1.2 # 108 m/s

(C) 1.6 # 108 m/s (D) 3 # 108 m/s


SOL 1.2 We have d = 2 mm and f = 10 GHz
Phase difference = 2π d = π ;
λ 4
or = λ = 8d = 8 # 2 mm = 16 mm
v = fλ = 10 # 109 # 16 # 10−3
= 1.6 # 108 m/ sec
Hence (C) is correct option.
MCQ 1.3 An analog signal is band-limited to 4 kHz, sampled at the Nyquist rate and

Brought to you by: Nodia and Company Visit us at: www.nodia.co.in


PUBLISHING FOR GATE
Page 2 GATE EC 2011 www.gatehelp.com

the samples are quantized into 4 levels. The quantized levels are assumed to be
independent and equally probable. If we transmit two quantized samples per second,
the information rate is ________ bits / second.
(A) 1 (B) 2
(C) 3 (D) 4
SOL 1.3 Quantized 4 level require 2 bit representation i.e. for one sample 2 bit are required.
Since 2 sample per second are transmitted we require 4 bit to be transmitted per
second.
Hence (D) is correct option.
MCQ 1.4 The root locus plot for a system is given below. The open loop transfer function
corresponding to this plot is given by

s (s + 1)
(A) G(s) H(s) = k
(s + 2) (s + 3)
(s + 1)
(B) G(s) H(s) = k
s (s + 2) (s + 3) 2

(C) G(s) H(s) = k 1


s (s − 1) (s + 2) (s + 3)
(s + 1)
(D) G(s) H(s) = k
s (s + 2) (s + 3)
SOL 1.4 For given plot root locus exists from − 3 to 3, So there must be odd number of
poles and zeros. There is a double pole at s =− 3
Now poles = 0, − 2, − 3, − 3
zeros =− 1
k (s + 1)
Thus transfer function G (s) H (s) =
s (s + 2) (s + 3) 2
Hence (B) is correct option.
MCQ 1.5 A system is defined by its impulse response h (n) = 2n u (n − 2). The system is
(A) stable and causal (B) causal but not stable
(C) stable but not causal (D) unstable and non-causal

Brought to you by: Nodia and Company Visit us at: www.nodia.co.in


PUBLISHING FOR GATE
Page 3 GATE EC 2011 www.gatehelp.com

SOL 1.5 Function h (n) = an u (n) stable if a < 1 and Unstable if a H 1


We have h (n) = 2n u (n − 2);
Here a = 2 therefore h (n) is unstable and since h (n) = 0 for n < 0
Therefore h (n) will be causal. So h (n) is causal and not stable.
Hence (B) is correct option.
MCQ 1.6 If the unit step response of a network is (1 − e− αt), then its unit impulse response is
(A) αe− αt (B) α−1 e− αt
(C) (1 − α−1) eαt (D) (1 − α) e− αt
SOL 1.6 Hence (A) is correct option.
Impulse response = d (step response)
dt
= d (1 − e− αt)
dt
= 0 + αe− αt = αe− αt

MCQ 1.7 The output Y in the circuit below is always ‘1’ when

(A) two or more of the inputs P, Q, R are ‘0’


(B) two or more of the inputs P, Q, R are ‘1’
(C) any odd number of the inputs P, Q, R is ‘0’
(D) any odd number of the inputs P, Q, R is ‘1’
SOL 1.7 The given circuit is shown below:

(PQ QR ) PR = (PQ + QR PR )
= PQ + QR + PR
= PQ + QR + PR

Brought to you by: Nodia and Company Visit us at: www.nodia.co.in


PUBLISHING FOR GATE
Page 4 GATE EC 2011 www.gatehelp.com

If any two or more inputs are ‘1’ then output y will be 1.


Hence (B) is correct option.
MCQ 1.8 In the circuit shown below, capacitors C1 and C2 are very large and are shorts at
the input frequency. vi is a small signal input. The gain magnitude vo at 10 M
vi
rad/s is

(A) maximum (B) minimum


(C) unity (D) zero
SOL 1.8 For the parallel RLC circuit resonance frequency is,
ωr = 1 = 1 = 10 M rad/s
LC 10 # 10 # 1 # 10−9
−6

Thus given frequency is resonance frequency and parallel RLC circuit has maximum
impedance at resonance frequency
Gain of the amplifier is gm # (ZC RL) where ZC is impedance of parallel RLC
circuit.
At ω = ωr , ZC = R = 2 kΩ = ZC max .
Hence at this frequency (ωr ), gain is
Gain ω = ω = gm (ZC RL) = gm (2k 2k) = gm # 103 which is maximum. Therefore
r

gain is maximum at ωr = 10 M rad/ sec .


Hence (A) is correct option.
MCQ 1.9 Drift current in the semiconductors depends upon
(A) only the electric field
(B) only the carrier concentration gradient
(C) both the electric field and the carrier concentration
(D) both the electric field and the carrier concentration gradient
SOL 1.9 Hence (C) is correct option.
Drift current Id = qnμn E
It depends upon Electric field E and carrier concentration n

Brought to you by: Nodia and Company Visit us at: www.nodia.co.in


PUBLISHING FOR GATE
Page 5 GATE EC 2011 www.gatehelp.com

MCQ 1.10 A Zener diode, when used in voltage stabilization circuits, is biased in
(A) reverse bias region below the breakdown voltage
(B) reverse breakdown region
(C) forward bias region
(D) forward bias constant current mode
SOL 1.10 Zener diode operates in reverse breakdown region.

Hence (B) is correct option.


MCQ 1.11 The circuit shown below is driven by a sinusoidal input vi = v p cos (t/RC ) . The
steady state output vo

(A) (v p /3) cos (t/RC ) (B) (v p /3) sin (t/RC )


(C) (v p /2) cos (t/RC ) (D) (v p /2) sin (t/RC )
SOL 1.11 The given circuit is shown below

For parallel combination of R and C equivalent impedance is


R$ 1
jω C R
Zp = =
R+ 1 1 + j ωRC
jω C
Transfer function can be written as

Brought to you by: Nodia and Company Visit us at: www.nodia.co.in


PUBLISHING FOR GATE
Page 6 GATE EC 2011 www.gatehelp.com

R
Vout = Z p 1 + jωRC
=
Vin Zs + Zp R+ 1 + R
jωC 1 + jωRC
jωRC
=
jωRC + (1 + jωRC) 2
j
= Here ω = 1
j + (1 + j) 2 RC
Vout = j
=1
Vin (1 + j) + j
2 3

v out = b p l cos (t/RC)


V
Thus
3
Hence (A) is correct option.
MCQ 1.12 Consider a closed surface S surrounding volume V. If rv is the position vector of a
point inside S, with nt the unit normal on S, the value of the integral ## 5rt.nv dS is
(A) 3V (B) 5V
(C) 10V (D) 15V
SOL 1.12 From Divergence theorem, we have
v = #A
v Adv
### 4$ v $ nt ds
s
The position vector
rv = ^utx x + uty y + utz z h
v
Here, A = 5rv, thus
c 2x 2z m ^ h
4$ A v = utx 2 + uty 2 + utz 2 : utx x + uty y + utz z
2y
= c dx +
dx dy dz m
dy dz
+ 5 = 3 # 5 = 15

So, ##s 5rv $ nt ds = ### 15 dv = 15V


Hence (D) is correct option
TE
MCQ 1.13 The modes in a rectangular waveguide are denoted by TM where mn
mn

m and n are the eigen numbers along the larger and smaller dimensions of the
waveguide respectively. Which one of the following statements is TRUE?
(A) The TM 10 mode of the wave does not exist
(B) The TE 10 mode of the wave does not exist
(C) The TM 10 and TE 10 the modes both exist and have the same cut-off frequencies
(D) The TM 10 and TM 01 modes both exist and have the same cut-off frequencies
SOL 1.13 TM11 is the lowest order mode of all the TMmn modes.
Hence (A) is correct option.
dy
MCQ 1.14 The solution of the differential equation = ky, y (0) = c is
dx
Brought to you by: Nodia and Company Visit us at: www.nodia.co.in
PUBLISHING FOR GATE
Page 7 GATE EC 2011 www.gatehelp.com

(A) x = ce−ky (B) x = kecy

(C) y = cekx (D) y = ce−kx


SOL 1.14 Hence (C) is correct answer
dy
We have = ky
dx

Integrating # dy
y
= # k dx + A

or ln y = kx + A
Since y (0) = c thus ln c = A
So, we get, ln y = kx + ln c
or ln y = ln ekx + ln c
or y = cekx
MCQ 1.15 The Column-I lists the attributes and the Column-II lists the modulation systems.
Match the attribute to the modulation system that best meets it
Column-I
P. Power efficient transmission of signals
Q. Most bandwidth efficient transmission of voice signals
R. Simplest receiver structure
S. Bandwidth efficient transmission of signals with Significant dc component
Column-II
1. Conventional AM
2. FM
3. VSB
4. SSB-SC
(A) P-4;Q-2;R-1;S-3 (B) P-2;Q-4;R-1;S-3
(C) P-3;Q-2;R-1;S-4 (D) P-2;Q-4;R-3;S-1
SOL 1.15 In FM the amplitude is constant and power is efficient transmitted. No variation
in power.
There is most bandwidth efficient transmission in SSB- SC. because we transmit
only one side band.
Simple Diode in Non linear region ( Square law ) is used in conventional AM that
is simplest receiver structure.
In VSB dc. component exists.
Hence (B) is correct option.
d 2y dy
MCQ 1.16 The differential equation 100 − 20 + y = x (t) describes a system with an in
dt 2 dt

Brought to you by: Nodia and Company Visit us at: www.nodia.co.in


PUBLISHING FOR GATE
Page 8 GATE EC 2011 www.gatehelp.com

put x (t) and an output y (t). The system, which is initially relaxed, is excited by a
unit step input. The output y(t) can be represented by the waveform

SOL 1.16 Hence (A) is correct option.


d2 y dy
We have 100 2 − 20 + y = x (t)
dt dt

Applying Laplace transform we get


100s2 Y (s) − 20sY (s) + Y (s) = X (s)

Y (s) 1
or H (s) = =
X (s) 100s2 − 20s + 1
1/100 A
= 2 =
s − (1/5) s + 1/100 s2 + 2ξωn s + ω2
Here ωn = 1/10 and 2ξωn =− 1/5 giving ξ =− 1
Roots are s = 1/10, 1/10 which lie on Right side of s plane thus unstable.
MCQ 1.17 For the transfer function G (jω) = 5 + jω , the corresponding Nyquist plot for
positive frequency has the form

Brought to you by: Nodia and Company Visit us at: www.nodia.co.in


PUBLISHING FOR GATE
Page 9 GATE EC 2011 www.gatehelp.com

SOL 1.17 We have G (jω) = 5 + jω


Here σ = 5 . Thus G (jω) is a straight line parallel to jω axis.
Hence (A) is correct option.
MCQ 1.18 The trigonometric Fourier series of an even function does not have the
(A) dc term (B) cosine terms
(C) sine terms (D) odd harmonic terms
SOL 1.18 For an even function Fourier series contains dc term and cosine term (even and odd
harmonics).
Hence (C) is correct option.
MCQ 1.19 When the output Y in the circuit below is ‘1’, it implies that data has

(A) changed from 0 to 1 (B) changed from 1 to 0


(C) changed in either direction (D) not changed
SOL 1.19 For the output to be high, both inputs to AND gate should be high.
The D-Flip Flop output is the same, after a delay.
Let initial input be 0; (Consider Option A)
then Q = 1 (For 1 D-Flip Flop). This is given as input to 2 FF.
st nd

Let the second input be 1. Now, considering after 1 time interval; The output of 1st
Flip Flop is 1 and 2nd FF is also 1. Thus Output = 1.
Hence (A) is correct option.
MCQ 1.20 The logic function implemented by the circuit below is (ground implies logic 0)

Brought to you by: Nodia and Company Visit us at: www.nodia.co.in


PUBLISHING FOR GATE
Page 10 GATE EC 2011 www.gatehelp.com

(A) F = AND ^P, Q h (B) F = OR ^P, Q h


(C) F = X NOR ^P, Q h (D) F = X OR ^P, Q h
SOL 1.20 Hence (D) is correct option.
F = S1 S 0 I 0 + S1 S 0 I1 + S1 S 0 I 2 + S1 S 0 I 3
I0 = I3 = 0
F = PQ + PQ = XOR (P, Q) ( S1 = P, S 0 = Q )
MCQ 1.21 The circuit below implements a filter between the input current ii and the output
voltage vo . Assume that the opamp is ideal. The filter implemented is a

(A) low pass filter (B) band pass filter


(C) band stop filter (D) high pass filter
SOL 1.21 The given circuit is shown below :

From diagram we can write


Ii = Vo + Vo
R1 sL1

Brought to you by: Nodia and Company Visit us at: www.nodia.co.in


PUBLISHING FOR GATE
Page 11 GATE EC 2011 www.gatehelp.com

Transfer function
H (s) = Vo = sR1 L1
I1 R1 + sL1
jω R 1 L 1
or H (jω) =
R 1 + jω L 1
At ω = 0
H (jω) = 0
At ω = 3
H (jω) = R1 = constant . Hence HPF.
Hence (D) is correct option.
MCQ 1.22 A silicon PN junction is forward biased with a constant current at room temperature.
When the temperature is increased by 10ºC, the forward bias voltage across the
PN junction
(A) increases by 60 mV (B) decreases by 60 mV
(C) increases by 25 mV (D) decreases by 25 mV
SOL 1.22 For every 1c C increase in temperature, forward bias voltage across diode decreases
by 2.5 mV. Thus for 10c C increase, there us 25 mV decreases.
Hence (D) is correct option.
MCQ 1.23 In the circuit shown below, the Norton equivalent current in amperes with respect
to the terminals P and Q is

(A) 6.4 − j 4.8 (B) 6.56 − j 7.87


(C) 10 + j 0 (D) 16 + j 0
SOL 1.23 Replacing P − Q by short circuit as shown below we have

Using current divider rule the current Isc is

Brought to you by: Nodia and Company Visit us at: www.nodia.co.in


PUBLISHING FOR GATE
Page 12 GATE EC 2011 www.gatehelp.com

ISC = 25 (16 0 ) = (6.4 − j4.8) A


25 + 15 + j30
Hence (A) is correct option.
MCQ 1.24 In the circuit shown below, the value of RL such that the power transferred to RL
is maximum is

(A) 5 Ω (B) 10 Ω
(C) 15 Ω (D) 20 Ω
SOL 1.24 Power transferred to RL will be maximum when RL is equal to the thevenin
resistance. We determine thevenin resistance by killing all source as follows :

RTH = 10 # 10 + 10 = 15 Ω
10 + 10
Hence (C) is correct option.

MCQ 1.25 The value of the integral # − 3z + 4 dz where c is the circle z = 1 is given by
c
(z 2 + 4z + 5)
(A) 0 (B) 1/10
(C) 4/5 (D) 1

SOL 1.25 C R Integrals is # − 3z + 4 dz where C is circle z = 1


C
z 2 + 4z + 5

C
# f (z) dz = 0 if poles are outside C.

Now z 2 + 4z + 5 = 0

(z + 2) 2 + 1 = 0
Thus z1, 2 =− 2 ! j & z1, 2 > 1
So poles are outside the unit circle.
Hence (A) is correct option.

Brought to you by: Nodia and Company Visit us at: www.nodia.co.in


PUBLISHING FOR GATE
Page 13 GATE EC 2011 www.gatehelp.com

Q. No. 26 – 51 Carry Two Marks Each

MCQ 1.26 A current sheet J = 10uty A/m lies on the dielectric interface x = 0 between two
dielectric media with εr 1 = 5, μr 1 = 1 in Region − 1 (x < 0) and εr 2 = 5, μr 2 = 2 in
Region − 2 (x > 0). If the magnetic field in Region− 1 at x = 0− is Hv1 = 3utx + 30uty A/m
the magnetic field in Region-2 at x = 0+ is

(A) H2 = 1.5utx + 30uty − 10utz A/m


(B) Hv2 = 1.5utx + 30uty − 10utz A/m
(C) Hv2 = 1.5utx + 40uty A/m
(D) Hv2 = 3utx + 30uty + 10utz A/m
SOL 1.26 From boundary condition
Bn1 = Bn2
μ1 Hx1 = μ2 Hx2
or Hx2 = Hx1 = 1.5
2
or Hx2 = 1.5utx
Further if H z = 1.5utx + Auty + Buz
Then from Boundary condition
10ut
(3utx + 30uty) utx = (1.5utx + Auty + Butz ) xt + v y
J
=− 30utz =− Autz + Buty + 10uty
Comparing we get A = 30 and B =− 10
So H z = 1.5utx + 30uty − 10utz A/m
Hence (A) is correct option.
MCQ 1.27 A transmission line of characteristic impedance 50 W is terminated in a load
impedance ZL . The VSWR of the line is measured as 5 and the first of the voltage
maxima in the line is observed at a distance of λ from the load. The value of ZL is
4
(A) 10 Ω (B) 250 Ω
(C) (19.23 + j 46.15) Ω (D) (19.23 − j 46.15) Ω
SOL 1.27 Since voltage maxima is observed at a distance of λ/4 from the load and we know
that the separation between one maxima and minima equals to λ/4 so voltage
minima will be observed at the load, Therefore load can not be complex it must be
pure resistive.
Now Γ = s−1
s+1
Brought to you by: Nodia and Company Visit us at: www.nodia.co.in
PUBLISHING FOR GATE
Page 14 GATE EC 2011 www.gatehelp.com

also RL = R 0 (since voltage maxima is formed at the load)


s
RL = 50 = 10 Ω
5
Hence (A) is correct option.
MCQ 1.28 x (t) is a stationary random process with auto-correlation function. Rx (τ) = exp (πr 2) .
This process is passed through the system shown below. The power spectral density
of the output process y (t) is

(A) (4π 2 f 2 + 1) exp (− πf 2) (B) (4π 2 f 2 − 1) exp (− πf 2)


(C) (4π 2 f 2 + 1) exp (− πf ) (D) (4π 2 f 2 − 1) exp (− πf )
SOL 1.28 Hence (A) is correct option.
We have Sx (f) = F {Rx (τ)} = F {exp (− πτ2)}
2
= e− πf
The given circuit can be simplified as

Power spectral density of output is


Sy (f) = G (f) 2 Sx (f)
2
= j2πf − 1 2 e− πf
= ( (2πf) 2 + 1) 2 e− πf
2

Sy (f) = (4π2 f 2 + 1) e− πf
2
or
MCQ 1.29 The output of a 3-stage Johnson (twisted ring) counter is fed to a digital-to analog
(D/A) converter as shown in the figure below. Assume all the states of the counter
to be unset initially. The waveform which represents the D/A converter output vo is

Brought to you by: Nodia and Company Visit us at: www.nodia.co.in


PUBLISHING FOR GATE
Page 15 GATE EC 2011 www.gatehelp.com

SOL 1.29 All the states of the counter are initially unset.

State Initially are shown below in table :


Q2 Q1 Q0
0 0 0 0
1 0 0 4
1 1 0 6
1 1 1 7
0 1 1 3
0 0 1 1
0 0 0 0
Hence (A) is correct option.

Brought to you by: Nodia and Company Visit us at: www.nodia.co.in


PUBLISHING FOR GATE
Page 16 GATE EC 2011 www.gatehelp.com

MCQ 1.30 Two D flip-flops are connected as a synchronous counter that goes through the
following QB QA sequence 00 " 11 " 01 " 10 " 00 " ....
The combination to the inputs DA and DB are
(A) DA = QB; DB = QA
(B) DA = QA; DB = QB
(C) DA = (QA QB + QA QB); DB = QA
(D) DA = (QA QB + QA QB); DB = QB
SOL 1.30 The sequence is QB QA
00 " 11 " 01 " 10 " 00 " ...
QB QA QB (t + 1) QA (t + 1)
0 0 1 1
1 1 0 1
0 1 1 0
1 0 0 0
QB ^t + 1h

QB ^t + 1h = Q A

DA = Q A Q B + QA QB
Hence (D) is correct option.
MCQ 1.31 In the circuit shown below, for the MOS transistors, μn Cox = 100 μ/A/V 2 and the
threshold voltage VT = 1 V . The voltage Vx at the source of the upper transistor is

(A) 1 V (B) 2 V

Brought to you by: Nodia and Company Visit us at: www.nodia.co.in


PUBLISHING FOR GATE
Page 17 GATE EC 2011 www.gatehelp.com

(C) 3 V (D) 3.67 V


SOL 1.31 Given circuit is shown below.

For transistor M2 ,
VGS = VG − VS = Vx − 0 = Vx

VDS = VD − VS = Vx − 0 = Vx

Since VGS − VT = Vx − 1 < VDS , thus M2 is in saturation.


By assuming M1 to be in saturation we have
IDS (M ) = IDS (M )
1 2

μn C 0x μ C
(4) (5 − Vx − 1) 2 = n 0x 1 (Vx − 1) 2
2 2

4 (4 − Vx ) 2 = (Vx − 1) 2
or 2 (4 − Vx ) = ! (Vx − 1)
Taking positive root,
8 − 2Vx = Vx − 1
Vx = 3 V
At Vx = 3 V for M1,VGS = 5 − 3 = 2 V < VDS . Thus our assumption is true and
Vx = 3 V .
Hence (C) is correct option.
MCQ 1.32 An input x (t) = exp (− 2t) u (t) + δ (t − 6) is applied to an LTI system with impulse
response h (t) = u (t) . The output is
(A) [1 − exp (− 2t)] u (t) + u (t + 6) (B) [1 − exp (− 2t)] u (t) + u (t − 6)
(C) 0.5 [1 − exp (− 2t)] u (t) + u (t + 6) (D) 0.5 [1 − exp (− 2t)] u (t) + u (t − 6)
SOL 1.32 Hence (D) is correct option.
We have x (t) = exp (− 2t) μ (t) + s (t − 6) and h (t) = u (t)
Taking Laplace Transform we get
X (s) = b 1 + e−6s l and H (s) = 1
s+2 s
Now Y (s) = H (s) X (s)
−6s
= 1 : 1 + e−6sD = 1 +e
s s+2 s (s + 2) s

Brought to you by: Nodia and Company Visit us at: www.nodia.co.in


PUBLISHING FOR GATE
Page 18 GATE EC 2011 www.gatehelp.com

−6s
or Y (s) = 1 − 1 +e
2s 2 (s + 2) s
Thus y (t) = 0.5 [1 − exp (− 2t)] u (t) + u (t − 6)
MCQ 1.33 For a BJT the common base current gain α = 0.98 and the collector base junction
reverse bias saturation current Ico = 0.6 μA . This BJT is connected in the common
emitter mode and operated in the active region with a base drive current IB = 204 A
. The collector current Ic for this mode of operation is
(A) 0.98 mA (B) 0.99 mA
(C) 1.0 mA (D) 1.01 mA
SOL 1.33 Hence (D) is correct option.
We have α = 0.98
Now β = α = 4.9
1−α
In active region, for common emitter amplifier,
IC = βIB + (1 + β) ICO ...(1)
Substituting ICO = 0.6 μA and IB = 20 μA in above eq we have,
IC = 1.01 mA
2 (s + 1)
MCQ 1.34 If F (s) = L [f (t)] = then the initial and final values of f (t) are respectively
s2 + 4s + 7
(A) 0, 2 (B) 2, 0
(C) 0, 2/7 (D) 2/7, 0
SOL 1.34 Correct Option is ( )
MCQ 1.35 In the circuit shown below, the current I is equal to

(A) 14 0c A (B) 2.0 0c A


(C) 2.8 0c A (D) 3.2 0c A
SOL 1.35 From star delta conversion we have

Brought to you by: Nodia and Company Visit us at: www.nodia.co.in


PUBLISHING FOR GATE
Page 19 GATE EC 2011 www.gatehelp.com

Thus R1 = Ra Rb = 6.6 = 2Ω
Ra + Rb + Rc 6 + 6 + 6
Here R1 = R 2 = R 3 = 2 Ω
Replacing in circuit we have the circuit shown below :

Now the total impedance of circuit is


(2 + j4) (2 − j4)
Z = +2 = 7Ω
(2 + j4) (2 − j4)
14 0c
Current I = = 2 0c
7
Hence (B) is correct option.
MCQ 1.36 A numerical solution of the equation f (x) + x − 3 = 0 can be obtained using
Newton- Raphson method. If the starting value is x = 2 for the iteration, the value
of x that is to be used in the next step is
(A) 0.306 (B) 0.739
(C) 1.694 (D) 2.306
SOL 1.36 Hence (C) is correct option.
We have f (x) = x + x − 3 = 0
f l (x) = 1 + 1
2 x
Substituting x 0 = 2 we get
f l (x 0) = 1.35355 and f (x 0) = 2 + 2 − 3 = 0.414
Newton Raphson Method
f (x 0)
x1 = x 0 −
f l (x 0)
Substituting all values we have
x 1 = 2 − 0.414 = 1.694
1.3535

MCQ 1.37 The electric and magnetic fields for a TEM wave of frequency 14 GHz in a
homogeneous medium of relative permittivity εr and relative permeability μr = 1
are given by

Brought to you by: Nodia and Company Visit us at: www.nodia.co.in


PUBLISHING FOR GATE
Page 20 GATE EC 2011 www.gatehelp.com

Ev = E p e j (ωt − 280πy) utz V/m Hv = 3e j (ωt − 280πy) utx A/m


Assuming the speed of light in free space to be 3 # 108 m/s , the intrinsic impedance
of free space to be 120π , the relative permittivity εr of the medium and the electric
field amplitude E p are
(A) εr = 3, E p = 120π (B) εr = 3, Eb = 360π
(C) εr = 9, E p = 360π (D) εr = 9, E p = 120π
SOL 1.37 From the expressions of Ev & Hv , we can write,
β = 280 π
or 2 π = 280 π & λ = 1
λ 140
v
Wave impedance, Zw = E = p = 120 π
E
Hv 3 εr
again, f = 14 GHz
8
Now λ = C = 3 # 10 9 = 3
εr f εr 14 # 10 140 εr
or 3 = 1
140 εr 140
or εr = 9
= 120π = E p = 120π
Ep
Now
3 9
Hence (D) is correct option.
MCQ 1.38 A message signal m (t) = cos 200πt + 4 cos πt modulates the carrier c (t) = cos 2πfc t
where fc = 1 MHz to produce an AM signal. For demodulating the generated AM
signal using an envelope detector, the time constant RC of the detector circuit
should satisfy
(A) 0.5 ms < RC < 1 ms (B) 1 μs << RC < 0.5 ms
(C) RC << μs (D) RC >> 0.5 ms
SOL 1.38 Highest frequency component in m (t) is fm = 4000π/2π = 2000 Hz
Carrier frequency fC = 1 MHz
For Envelope detector condition
1/fC << RC << 1/fm
1 μs << RC << 0.5 ms
Hence (B) is correct option.
MCQ 1.39 The block diagram of a system with one input it and two outputs y1 and y2 is given
below.

Brought to you by: Nodia and Company Visit us at: www.nodia.co.in


PUBLISHING FOR GATE
Page 21 GATE EC 2011 www.gatehelp.com

A state space model of the above system in terms of the state vector x and the
output vector y = [y1 y2]T is
(A) xo = [2] x + [1] u ; y = [1 2] x
1
(B) xo = [− 2] x + [1] u; y = > H x
2

−2 0 1
(C) xo = > H x + > H u ; y = 81 2B x
0 −2 1

2 0 1 1
(D) xo = > H x + > Hu ; y = > Hx
0 2 1 2
SOL 1.39 Hence (B) is correct option.
dy
Here x = y1 and xo = 1
dx
y1 x 1
y = > H = > H = > Hx
y2 2x 2
Now y1 = 1 u
s+2
y1 (s + 2) =u
yo1 + 2y1 =u
xo + 2x =u
xo =− 2x + u
xo = [− 2] x + [1] u
Drawing SFG as shown below

Thus xo1 = [− 2] x1 + [1] u


y1 = x1 ; y2 = 2x1

y1 1
y = > H = > H x1
y2 2

Brought to you by: Nodia and Company Visit us at: www.nodia.co.in


PUBLISHING FOR GATE
Page 22 GATE EC 2011 www.gatehelp.com

Here x1 = x
MCQ 1.40 Two systems H1 (Z ) and H2 (Z ) are connected in cascade as shown below. The overall
output y (n) is the same as the input x (n) with a one unit delay. The transfer
function of the second system H2 (Z ) is

1 − 0.6z−1 z−1 (1 − 0.6z−1)


(A) (B)
z (1 − 0.4z−1)
−1
(1 − 0.4z−1)
z−1 (1 − 0.4z−1) 1 − 0.4 z−1
(C) (D)
(1 − 0.6z−1) z (1 − 0.6z−1)
−1

SOL 1.40 Hence (B) is correct option.


y (n) = x (n − 1)
or Y (z) = z−1 X (z)
Y (z)
or = H (z) = z−1
X (z)
Now H1 (z) H2 (z) = z−1
1 − 0.4z−1
c 1 − 0.6z−1 m H2 (z) = z
−1

z−1 (1 − 0.6z−1)
H2 (z) =
(1 − 0.4z−1)

MCQ 1.41 An 8085 assembly language program is given below. Assume that the carry flag is
initially unset. The content of the accumulator after the execution of the program
is

(A) 8 CH (B) 64 H
(C) 23 H (D) 15 H
SOL 1.41 Initially Carry Flag, C = 0
MVI A, 07 H ; A = 0000 0111
RLC ; Rotate left without carry. A = 0000 1110
MVO B, A ; B = A = 0000 1110

Brought to you by: Nodia and Company Visit us at: www.nodia.co.in


PUBLISHING FOR GATE
Page 23 GATE EC 2011 www.gatehelp.com

RLC ; A = 0001 1100


RLC ; A = 0011 1000
ADD B ; A = 0011 1000
; + 0000 1110
; 0100 0110
RRC Rotate Right with out carry, A = 0010 0011
;
Thus A = 23 H
Hence (C) is correct option.
MCQ 1.42 The first six points of the 8-point DFT of a real valued sequence are
5, 1 − j 3, 0, 3 − j 4, 0 and 3 + j 4..... The last two points of the DFT are respectively
(A) 0, 1 − j 3 (B) 0, 1 + j 3
(C) 1 + j 3, 5 (D) 1 − j3, 5
SOL 1.42 For 8 point DFT, x* [1] = x [7]; x* [2] = x [6]; x* [3] = x [5] and it is conjugate symmetric
about x [4], x [6] = 0 ; x [7] = 1 + j3
Hence (B) is correct option.
MCQ 1.43 For the BJT QL in the circuit shown below, β = 3, VBE on = 0.7 V, V = 0.7 V . The
CE sat

switch is initially closed. At time t = 0 , the switch is opened. The time t at which
Q1 leaves the active region is

(A) 10 ms (B) 25 ms
(C) 50 ms (D) 100 ms
SOL 1.43 Hence (C) is correct option
In active region VBEon = 0.7 V
Emitter voltage VE = VB − VBEon =− 5.7 V

VE − (− 10) − 5.7 − (− 10)


Emitter Current IE = = = 1 mA
4.3k 4.3k
Now IC . IE = 1 mA
Applying KCL at collector

Brought to you by: Nodia and Company Visit us at: www.nodia.co.in


PUBLISHING FOR GATE
Page 24 GATE EC 2011 www.gatehelp.com

i1 = 0.5 mA
Since i1 = C dVC
dt
or VC = 1 # i1 dt = i1 t ...(1)
C C

with time, the capacitor charges and voltage across collector changes from 0 towards
negative.
When saturation starts,VCE = 0.7 & VC =+ 5 V (across capacitor)

Thus from (1) we get, + 5 = 0.5 mA T


5 μA
−6
or T = 5 # 5 # 10 = 50 m sec
0.5 # 10−3
.
MCQ 1.44 In the circuit shown below, the network N is described by the following Y matrix:
0.1 S − 0.01 S
Y=>
0.1 S 0.1 S H
. the voltage gain V2 is
V1

(A) 1/90 (B) –1/90


(C) –1/99 (D) –1/11
SOL 1.44 From given admittance matrix we get
I1 = 0.1V1 − 0.01V2 and ...(1)
I2 = 0.01V1 + 0.1V2 ...(2)
Now, applying KVL in outer loop;
V2 =− 100I2

Brought to you by: Nodia and Company Visit us at: www.nodia.co.in


PUBLISHING FOR GATE
Page 25 GATE EC 2011 www.gatehelp.com

or I2 =− 0.01V2 ...(3)
From eq (2) and eq (3) we have
− 0.01V2 = 0.01V1 + 0.1V2
− 0.11V2 = 0.01V1
V2 = − 1
V1 11
Hence (D) is correct option.
MCQ 1.45 In the circuit shown below, the initial charge on the capacitor is 2.5 mC, with the
voltage polarity as indicated. The switch is closed at timet = 0 . The current i ^ t h at
a time t after the switch is closed is

(A) i (t) = 15 exp (− 2 # 103 t) A (B) i (t) = 5 exp ^− 2 # 103 t h A


(C) i (t) = 10 exp (− 2 # 103 t) A (D) i (t) =− 5 exp (− 2 # 103 t) A
SOL 1.45 Here we take the current flow direction as positive.
At t = 0− voltage across capacitor is
−3
Q
VC (0−) =− =− 2.5 # 10−6 =− 50 V
C 50 # 10
+
Thus VC (0 ) =− 50 V
In steady state capacitor behave as open circuit thus
V (3) = 100 V
Now, VC (t) = VC (3) + (VC (0+) − VC (3)) e−t/RC
−t
= 100 + (− 50 − 100) e 10 # 50 # 10
−6

= 100 − 150e− (2 # 10 t)
3

Now ic (t) = C dV
dt
= 50 # 10−6 # 150 # 2 # 103 e−2 # 10 t A
3

= 15e−2 # 10 t
3

ic (t) = 15 exp (− 2 # 103 t) A


Hence (A) is correct option.
MCQ 1.46 The system of equations
x+y+z = 6
x + 4y + 6y = 20

Brought to you by: Nodia and Company Visit us at: www.nodia.co.in


PUBLISHING FOR GATE
Page 26 GATE EC 2011 www.gatehelp.com

x + 4y + λz = μ
has NO solution for values of λ and μ given by
(A) λ = 6, μ = 20 (B) λ = 6, μ =
Y 20
(C) λ =
Y 6, μ = 20 (D) λ =
Y 6, μ = 20
SOL 1.46 Writing A: B we have
R V
S1 1 1 : 6 W
S1 4 6 : 20W
S W
S1 4 λ : μ W
T X
Apply R 3 " R 3 − R2
R V
S1 1 1 : 6 W
S1 4 6 : 20 W
S W
S0 0 λ − 6 : μ − 20W
T X
For equation to have solution, rank of A and A: B must be same. Thus for no
solution; λ = 6, μ ! 20
Hence (B) is correct option
MCQ 1.47 A fair dice is tossed two times. The probability that the second toss results in a
value that is higher than the first toss is
(A) 2/36 (B) 2/6
(C) 5/12 (D) ½
SOL 1.47 Total outcome are 36 out of which favorable outcomes are :
(1, 2), (1, 3), (1, 4), (1, 5), (1, 6), (2, 3), (2, 4), (2, 5), (2, 6);
(3, 4), (3, 5), (3, 6), (4, 5), (4, 6), (5, 6) which are 15.

Thus P (E) = No. of favourable outcomes = 15 = 5


No. of total outcomes 36 12
Hence (C) is correct option.

Common Data Questions: 48 & 49


The channel resistance of an N-channel JFET shown in the figure below is 600 Ω
when the full channel thickness (tch ) of 10μm is available for conduction. The built-
in voltage of the gate P+ N junction (Vbi ) is − 1 V . When the gate to source voltage
(VGS ) is 0 V, the channel is depleted by 1 μm on each side due to the built in voltage
and hence the thickness available for conduction is only 8 μm

Brought to you by: Nodia and Company Visit us at: www.nodia.co.in


PUBLISHING FOR GATE
Page 27 GATE EC 2011 www.gatehelp.com

MCQ 1.48 The channel resistance when VGS =− 3 V is


(A) 360 Ω (B) 917 Ω
(C) 1000 Ω (D) 3000 Ω
SOL 1.48 Full channel resistance is
ρ L
r # = 600 Ω ...(1)
W#a
If VGS is applied, Channel resistance is
ρ L
where b = a c1 −
Vp m
rl = # VGS
W#b
Pinch off voltage,
qN
Vp = D a2 ...(2)

If depletion on each side is d = 1 μm at VGS = 0 .
qN
Vj = D d2

qN qND
or 1 = D (1 # 10−6) 2 & = 1012
2ε 2ε
Now from equation (2), we have
Vp = 1012 # (5 # 10−6) 2

or Vp =− 25 V
At VGS =− 3 V ;
b = 5 b1 − − 3 μm = 3.26 μm
− 25 l
ρL ρL a = 600 5
rl = = # 3.26 = 917 Ω
W # b Wa # b
Hence (B) is correct option.
MCQ 1.49 The channel resistance when VGS = 0 V is
(A) 480 Ω (B) 600 Ω
(C) 750 Ω (D) 1000 Ω

Brought to you by: Nodia and Company Visit us at: www.nodia.co.in


PUBLISHING FOR GATE
Page 28 GATE EC 2011 www.gatehelp.com

SOL 1.49
At VGS = 0 V , b = 4 μm
since 2b = 8 μm
ρL a = 600 5 = 750 Ω
Thus rl =
Wa # b #4

Hence (C) is correct option.

Common Data Questions: 50 & 51


The input-output transfer function of a plant H (S ) = 100 . The plant is
s (s + 10) 2
placed in a unity negative feedback configuration as shown in the figure below.

MCQ 1.50 The gain margin of the system under closed loop unity negative feedback is
(A) 0 dB (B) 20 dB
(C) 26 dB (D) 46 dB
SOL 1.50 Hence (C) is correct option.
We have G (s) H (s) = 100
s (s + 10) 2
Now G (jω) H (jω) = 100
jω (jω + 10) 2
If ωp is phase cross over frequency G (jω) H (jω) = 180c
ωp
− 180c = 100 tan−1 0 − tan−1 3 − 2 tan−1 a
10 k
Thus

or − 180c =− 90 − 2 tan−1 (0.1ωp)


or 45c = tan−1 (0.1ωp)
or tan 45c = 0.1ωp = 1
or ωp = 10 rad/se
Now G (jω) H (jω) = 100
ω (ω + 100)
2

At ω = ωp
G (jω) H (jω) = 100 = 1
10 (100 + 100) 20
Gain Margin =− 20 log 10 G (jω) H (jω)
=− 20 log 10 b 1 l
20
= 26 dB

Brought to you by: Nodia and Company Visit us at: www.nodia.co.in


PUBLISHING FOR GATE
Page 29 GATE EC 2011 www.gatehelp.com

MCQ 1.51 The signal flow graph that DOES NOT model the plant transfer function H (S ) is

SOL 1.51 Hence (D) is correct option


From option (D) TF = H (s)
= 100 ! 100
s (s2 + 100) s (s + 10) 2

Linked Answer Questions: Q.52 to Q.55 Carry Two Marks Each

Statement for Linked Answer Questions: 52 & 53

MCQ 1.52 The bias current IDC through the diodes is


(A) 1 mA (B) 1.28 mA
(C) 1.5 mA (D) 2 mA
SOL 1.52 Hence (A) is correct option.
The current flows in the circuit if all the diodes are forward biased. In forward
biased there will be 0.7 V drop across each diode.
12.7 − 4 (0.7)
Thus IDC = = 1 mA
9900

Brought to you by: Nodia and Company Visit us at: www.nodia.co.in


PUBLISHING FOR GATE
Page 30 GATE EC 2011 www.gatehelp.com

MCQ 1.53 The ac output voltage Vac is


(A) 0.25 cos ^ωt h mV (B) 1 cos (ωt) mV
(C) 2 cos (ωt) mV (D) 22 cos (ωt) mV
SOL 1.53 Hence (B) is correct option.
The forward resistance of each diode is
r = VT = 25 mV = 25 Ω
IC 1 mA
4 (r)
Vac = Vi # e
4 (r) + 9900 o
Thus

= 100 mV cos (ωt) 0.01 = 1 cos (ωt) mV

Statement for Linked Answer Questions: 54 & 55


A four-phase and an eight-phase signal constellation are shown in the figure below.

MCQ 1.54 For the constraint that the minimum distance between pairs of signal points be d
for both constellations, the radii r 1 , and r 2 of the circles are
(A) r 1 = 0.707d, r2 = 2.782d (B) r 1 = 0.707d, r 2 = 1.932d
(C) r 1 = 0.707d, r 2 = 1.545d (D) r 1 = 0.707d, r 2 = 1.307d
SOL 1.54 Four phase signal constellation is shown below

Now d2 = r 12 + r 12
d2 = 2r 12
r1 = d/ 2 = 0.707d

Brought to you by: Nodia and Company Visit us at: www.nodia.co.in


PUBLISHING FOR GATE
Page 31 GATE EC 2011 www.gatehelp.com

θ = 2π = 2π = π
M 8 4
Applying Cooine law we have
d2 = r 22 + r 22 − 2r 22 cos π
4
= 2r 22 − 2r 22 1/ 2 = (2 − 2 ) r 22
or r2 = d = 1.3065d
2− 2
Hence (D) is correct option.
MCQ 1.55 Assuming high SNR and that all signals are equally probable, the additional average
transmitted signal energy required by the 8-PSK signal to achieve the same error
probability as the 4-PSK signal is
(A) 11.90 dB (B) 8.73 dB
(C) 6.79 dB (D) 5.33 dB
SOL 1.55 Here Pe for 4 PSK and 8 PSK is same because Pe depends on d . Since Pe is same,
d is same for 4 PSK and 8 PSK.

Additional Power SNR


= (SNR) 2 − (SNR) 1

= 10 log b ES2 l − 10 log b ES1 l


No No
= 10 log b ES2 l
ES1
= 10 log a r2 k & 20 log a r2 k = 20 log 1.3065d
2
r1 r1 0.707d

Brought to you by: Nodia and Company Visit us at: www.nodia.co.in


PUBLISHING FOR GATE
Page 32 GATE EC 2011 www.gatehelp.com

Additional SNR = 5.33 dB


Hence (D) is correct option.

Q. No. 56 – 60 Carry One Mark Each

MCQ 1.56 There are two candidates P and Q in an election. During the campaign, 40% of
the voters promised to vote for P, and rest for Q. However, on the day of election
15% of the voters went back on their promise to vote for P and instead voted forQ.
25% of the voters went back on their promise to vote for Q and instead voted for
P. Suppose, P lost by 2 votes, then what was the total number of voters?
(A) 100 (B) 110
(C) 90 (D) 95
SOL 1.56 Let us assume total voters are 100. Thus 40 voter (i.e. 40 %) promised to vote for
P and 60 (rest 60 % ) promised to vote fore Q.
Now, 15% changed from P to Q (15 % out of 40)
Changed voter from P to Q 15 40 = 6
100 #
Now Voter for P 40 − 6 = 34
Also, 25% changed form Q to P (out of 60%)
Changed voter from Q to P 25 60 = 15
100 #
Now Voter for P 34 + 15 = 49
Thus P P got 49 votes and Q got 51 votes, and P lost by 2 votes, which is given.
Therefore 100 voter is true value.
Hence (A) is correct option.
MCQ 1.57 Choose the most appropriate word from the options given below to complete the
following sentence:
It was her view that the country’s problems had been_________ by foreign
technocrats, so that to invite them to come back would be counter-productive.
(A) Identified (B) ascertained
(C) Texacerbated (D) Analysed
SOL 1.57 The sentence implies that technocrats are counterproductive (negative). Only (C)
can bring the same meaning.
Hence (C) is correct option
MCQ 1.58 Choose the word from the options given below that is most nearly opposite in
meaning to the given word:
Frequency
(A) periodicity (B) rarity
(C) gradualness (D) persistency

Brought to you by: Nodia and Company Visit us at: www.nodia.co.in


PUBLISHING FOR GATE
Page 33 GATE EC 2011 www.gatehelp.com

SOL 1.58 Periodicity is almost similar to frequency. Gradualness means something happening
with time. Persistency is endurance. Rarity is opposite to frequency.
Hence (B) is correct option.
MCQ 1.59 Choose the most appropriate word from the options given below to complete the
following sentence:
Under ethical guidelines recently adopted by the Indian Medical Association, human
genes are to be manipulated only to correct diseases for which______________
treatments are unsatisfactory.
(A) Similar (B) Most
(C) Uncommon (D) Available
SOL 1.59 Available is appropriate because manipulation of genes will be done when other
treatments are not useful.
Hence (D) is correct option.
MCQ 1.60 The question below consists of a pair of related words followed by four pairs of
words. Select the pair that best expresses the relation in the original pair:
Gladiator : Arena
(A) dancer : stage (B) commuter: train
(C) teacher : classroom (D) lawyer : courtroom
SOL 1.60 A gladiator performs in an arena. Commutators use trains. Lawyers performs,
but do not entertain like a gladiator. Similarly, teachers educate. Only dancers
performs on a stage.
Hence (A) is correct option.

Q. No. 61 – 65 Carry Two Marks Each

MCQ 1.61 The fuel consumed by a motorcycle during a journey while traveling at various
speeds is indicated in the graph below.

The distances covered during four laps of the journey are listed in the table below

Brought to you by: Nodia and Company Visit us at: www.nodia.co.in


PUBLISHING FOR GATE
Page 34 GATE EC 2011 www.gatehelp.com

Lap Distance (kilom- Average speed (kilometers per


eters) hour)
P 15 15
Q 75 45
R 40 75
S 10 10
From the given data, we can conclude that the fuel consumed per kilometre was
least during the lap
(A) P (B) Q
(C) R (D) S
SOL 1.61 Since fuel consumption/litre is asked and not total fuel consumed, only average
speed is relevant. Maximum efficiency comes at 45 km/hr, So least fuel consumer
per litre in lap Q
Hence (B) is correct option.
MCQ 1.62 Three friends, R, S and T shared toffee from a bowl. R took 1/3rd of the toffees,
but returned four to the bowl. S took 1/4th of what was left but returned three
toffees to the bowl. T took half of the remainder but returned two back into the
bowl. If the bowl had 17 toffees left, how many toffees-were originally there in the
bowl?
(A) 38 (B) 31
(C) 48 (D) 41
SOL 1.62 Let total no of toffees be x . The following table shows the all calculations.
Friend Bowl Status
= x −4 = 2x + 4
3 3

= 1 :2x + 4D − 3 = 2x + 4 − x + 2
4 3 3 6
= x +1−3 = x −2 = x +6
6 6 2

= 1 a x + 6k − 2 = x +6−x −1
2 2 2 4
= x +1 = x +5
4 4

Now, x + 5 = 17
4
or x = 17 − 5 = 12
4
x = 12 # 4 = 48

Brought to you by: Nodia and Company Visit us at: www.nodia.co.in


PUBLISHING FOR GATE
Page 35 GATE EC 2011 www.gatehelp.com

Hence (C) is correct option.


y
MCQ 1.63 Given that f ^y h = , and q is any non-zero real number, the value
y
of f ^q h − f ^− q h is
(A) 0 (B) − 1
(C) 1 (D) 2
SOL 1.63 Hence (D) is correct option.
y
f (y) =
y
−y
Now f (− y) = =− f (y)
y
or f (q) − f (− q) = 2f (q) = 2

MCQ 1.64 The sum of n terms of the series 4 + 44 + 444 + f. is


(A) (4/81) [10n + 1 − 9n − 1] (B) (4/81) [10n − 1 − 9n − 1]
(C) (4/81) [10n + 1 − 9n − 10] (D) (4/81) [10n − 9n − 10]
SOL 1.64 Hence (C) is correct option.
4 + 44 + 444 + .............. 4 (1 + 11 + 111 + .......)
= 4 (9 + 99 + 999 + ............)
9
= 4 [(10 − 1) + (100 − 1) + ........]
9
= 4 [10 (1 + 10 + 102 + 103) − n]
9

= 4 :10 # 10 − 1 − nD
n

9 10 − 1
= 4 610n + 1 − 10 − 9n@
81

MCQ 1.65 The horse has played a little known but very important role in the field of medicine.
Horses were injected with toxins of diseases until their blood built up immunities.
Then a serum was made from their blood. Serums to fight with diphtheria and
tetanus were developed this way.
It can be inferred from the passage that horses were
(A) given immunity to diseases
(B) generally quite immune to diseases
(C) given medicines to fight toxins
(D) given diphtheria and tetanus serums
SOL 1.65 Option B fits the sentence, as they built up immunities which helped humans

Brought to you by: Nodia and Company Visit us at: www.nodia.co.in


PUBLISHING FOR GATE
Page 36 GATE EC 2011 www.gatehelp.com

create serums from their blood.


Hence (B) is correct option.

Answer Sheet
1. (C) 13. (A) 25. (A) 37. (D) 49. (C) 61. (B)
2. (C) 14. (C) 26. (A) 38. (B) 50. (C) 62. (C)
3. (D) 15. (B) 27. (A) 39. (B) 51. (D) 63. (D)
4. (B) 16. (A) 28. (A) 40. (B) 52. (A) 64. (C)
5. (B) 17. (A) 29. (A) 41. (C) 53. (B) 65. (B)
6. (A) 18. (C) 30. (D) 42. (B) 54. (D)
7. (B) 19. (A) 31. (C) 43. (C) 55. (D)
8. (A) 20. (D) 32. (D) 44. (D) 56. (A)
9. (C) 21. (D) 33. (D) 45. (A) 57. (C)
10. (B) 22. (D) 34. (*) 46. (B) 58. (B)
11. (A) 23. (A) 35. (B) 47. (C) 59. (D)
12. (D) 24. (C) 36. (C) 48. (B) 60. (A)

Brought to you by: Nodia and Company Visit us at: www.nodia.co.in


PUBLISHING FOR GATE
GATE EC - 1991 www.gateforum.com
Join discussion of this test paper at http://forum.gatementor.com

1.1 An excitation is applied to a system at t = T and its response is zero for


−∞ < t < T . Such a system is
x(t)
(a) non-causal system
(b) stable system
(c) causal system t
t=T
(d) unstable system

1.2 In a series RLC high Q circuit, the current peaks at a frequency


(a) equal to the resonant frequency
(b) greater than the resonant frequency
(c) less than the resonant frequency
(d) none of the above

1.3 The voltage across an impedance in a network is V ( s ) = z ( s ) I ( s ) , where V(s) ,


Z(s) are the Laplace transforms of the corresponding time function
v ( t ) , z ( t ) and i ( t ) . The voltage v ( t ) is:
1
(a) ν ( t ) = z ( t ) .ν ( t ) (b) ν ( t ) = ∫ i (t ) .z (t − τ ) dτ
0

1
(c) ν ( t ) = ∫ i (t ) .z (t + τ ) dτ
0
(d) ν ( t ) = z ( t ) + i ( t )

1.4 Two two-port networks are connected in cascade. The combination is to be


represented as a single two-port network. The parameters of the network are
obtained by multiplying the individual
(a) z-parameter matrix (b) h-parameter matrix
(c) y-parameter matrix (d) ABCD parameter matrix

1.5 The pole-zero pattern of a certain filter is shown in the figure below. The filter
must be of the following type. jw
+j2
(a) low pass (b) high pass
(c) all pass (d) band pass × +j1

×
-1 +1

× -j1

-j2

Join All India Mock GATE Classroom Test Series - 2007 conducted by GATE Forum in over 25 cities all over India. Question
Papers including section tests and full tests are designed by IISc alumni according to the latest syllabus. Percentile, All India Rank,
interaction with IISc alumni in our online discussion forums, and more.
visit
www.gateforum.com
Think GATE Think GATE Forum
GATE EC - 1991 www.gateforum.com
Join discussion of this test paper at http://forum.gatementor.com

1.6 The necessary and sufficient condition for a rational function of s. T(s) to be
driving point impedance of an RC network is that all poles and zeros should be
(a) simple and lie on the negative axis in the s-plane
(b) complex and lie in the left half of the s-plane
(c) complex and lie in the right half of the s-plane
(d) simple and lie on the positive real axis of the s-plane

1.7 In the signal flow graph of Figure, the gain c/r will be
5

T 1 2 3 4 1 C

-1 -1 -1

11 22 24 44
(a) (b) (c) (d)
9 15 23 23

1.8 A second order system has a transfer function given by


25
G (s) = 2
s + 8s + 25
If the system, initially at rest is subjected to a unit step input at t = 0, the
second peak in response will occur at
π 2π π
(a) π sec (b) sec (c) sec (d) sec
3 3 2

1.9 The open loop transfer function of a feedback control system is:
1
G (s) H (s) = 3
( s + 1)
The gain margin of the system is:
(a) 2 (b) 4 (c) 8 (d) 16

1.10 A unity feedback control system has the open loop transfer function
4 (1 + 2s )
G (s) =
s2 ( s + 2 )
If the input to the system is a unit ramp, the steady state error will be
(a) 0 (b) 0.5 (c) 2 (d) infinity

Join All India Mock GATE Classroom Test Series - 2007 conducted by GATE Forum in over 25 cities all over India. Question
Papers including section tests and full tests are designed by IISc alumni according to the latest syllabus. Percentile, All India Rank,
interaction with IISc alumni in our online discussion forums, and more.
visit
www.gateforum.com
Think GATE Think GATE Forum
GATE EC - 1991 www.gateforum.com
Join discussion of this test paper at http://forum.gatementor.com

1.11 The characteristic equation of a feedback control system is given by


s3 + 5s2 + ( K + 6 ) s + K = 0

Where K > 0 is a scalar variable parameter. In the root loci diagram of the
system the asymptotes of the root locus for large values of K meet at a point in
the s-plane whose coordinates are
(a) (-3,0) (b) (-2,0) (c) (-1,0) (d) (2,0)

1.12 A linear second order single input continuous time system is described by the
following set of differential equations
x1 ( t ) = −2 x1 ( t ) + 4 x2 ( t )
x2 ( t ) = 2 x1 ( t ) − x2 ( t ) + u ( t )

Where x1 ( t ) and x2 ( t ) are the state variables and u ( t ) is the control variable.
The system is:
(a) controllable and stable (b) controllable but unstable
(c) uncontrollable and unstable (d) uncontrollable and stable

1.13 A linear time-invariant discrete-time system is described by the vector matrix


difference equation
x ( k + 1) = F X ( k ) + Gu ( k )

Where X ( k ) is the state vector, F is an n × n constant matrix, G is a


( n × r ) constant matrix and u ( k ) is the control vector. The state transition matrix
of the system is given by inverse Z-transform of
−1 −1
(a) ZI - F (b) (ZI – F) Z (c) ( ZI − F ) G (d) ( ZI − F ) Z

1.14 A silicon sample is uniformly doped with 1016 phosphorous atoms/cm3 and
2 × 1016 boron atoms/cm3. If all the dopants are fully ionized, the material is
(a) n-type with carrier concentration of 1016 / cm3

(b) p-type with carrier concentration of 1016 / cm3

(c) p-type with carrier concentration of 2 × 1016 / cm3


(d) T2 will get damaged and T1 will be safe

1.15 An n-type silicon sample, having electron mobility µ n = twice the hole mobility
µ p , is subjected to a steady illumination such that the electron concentration
doubles from its thermal equilibrium value. As a result, the conductivity of the
sample increases by a factor of …

Join All India Mock GATE Classroom Test Series - 2007 conducted by GATE Forum in over 25 cities all over India. Question
Papers including section tests and full tests are designed by IISc alumni according to the latest syllabus. Percentile, All India Rank,
interaction with IISc alumni in our online discussion forums, and more.
visit
www.gateforum.com
Think GATE Think GATE Forum
GATE EC - 1991 www.gateforum.com
Join discussion of this test paper at http://forum.gatementor.com

1.16 The small signal capacitances of an abrupt P1 − n junction is 1nF / Cm2 at zero
bias. If the built in voltage is 1 volt, the capacitance at a reverse bias voltage of
99 volts is equal to …

1.17 Referring to the figure. The switch S is in position 1 initially and steady state
conditions exist from time t = 0 to t = t0. The switch is suddenly thrown into
position 2. The current 1 through the 10K resistor as a function of time t from t =
0, is …. (Give the sketch showing the magnitudes of the current at t = 0,
t = t0 and t = ∞ )

S
2

20V 10K 1
20V

1.18 Discrete transistors T1 and T2 having maximum collector current rating of 0.75
amps are connected in parallel as shown in the figure. This combination is treated
as a single transistor to carry a total current of 1 ampere, when biased with self
bias circuit. When the circuit is switched on, T1 draws 0.55 amps and T2 draws
0.45 amps. If the supply is kept on continuously, ultimately it is very likely that

T1 T2

(a) both T1 and T2 get damaged (b) both T1 and T2 will be safe
(c) T1 will get damaged and T2 will be safe
(d) T2 will get damaged and T1 will be safe

Join All India Mock GATE Classroom Test Series - 2007 conducted by GATE Forum in over 25 cities all over India. Question
Papers including section tests and full tests are designed by IISc alumni according to the latest syllabus. Percentile, All India Rank,
interaction with IISc alumni in our online discussion forums, and more.
visit
www.gateforum.com
Think GATE Think GATE Forum
GATE EC - 1991 www.gateforum.com
Join discussion of this test paper at http://forum.gatementor.com

1.19. The built-in potential of the gate junction of a n-channel JFET is 0.5 volts. The
drain current saturates at VDS = 4.0 volts when VGS = 0. The pinch off voltage is
________.

1.20. In figure, all transistors are identical and have a high value of beta. The voltage
VDC is equal to ______.
10 volts

5mA 1kΩ

VDC=?
Q1 Q4

Q2 Q3

1.21. In figure, both transistors are identical and have a high value of beta. Take the
dc base-emitter voltage drop as 0.7 volt and KT/q = 25 mV. The small signal low
frequency voltage gain (Vo Vi ) is equal to _______
10 volts

1.2kΩ

VO/V=?

~ Vi

1kΩ

-5.7 volts

Join All India Mock GATE Classroom Test Series - 2007 conducted by GATE Forum in over 25 cities all over India. Question
Papers including section tests and full tests are designed by IISc alumni according to the latest syllabus. Percentile, All India Rank,
interaction with IISc alumni in our online discussion forums, and more.
visit
www.gateforum.com
Think GATE Think GATE Forum
GATE EC - 1991 www.gateforum.com
Join discussion of this test paper at http://forum.gatementor.com

1.22. In figure the input V1 is a 100 Hz triangular wave having a peak to peak
amplitude of 2 volts and an average value of zero volts. Given that the diode is
ideal, the average value of the output V0 is ________.
Vi VO
1K

1K
0.6 volts

1.23. In figure, the n-channel MOSFETs are identical and their current voltage
characteristics are given by the following expressions: 5 volts

 V 2 DS  IDC=?
For VDS , ID = (VGS − 1) VDS −  mA 2mA
 2 
2
For VDS ≥ (VGS − 1) , (VGS − 1) = (VGS − 1) mA

-2 volts
Where VGS and VDS are the gate source and drain source voltages respectively
and I0 is the drain current.
The current IDC flowing through the transistor ‘M’ is equal to _______

1.24. In order that the circuit of Figure works properly as differentiator, it should be
modified to _____ (draw the modified circuit)

R
+


1.25 Two non-inverting amplifiers, one having a unity gain and the other having a gain
of twenty are made using identical operational amplifiers. As a compared to the
unity gain amplifier, the amplifier with gain twenty has
(a) less negative feedback (b) greater input impedance
(c) less bandwidth (d) none of the above

Join All India Mock GATE Classroom Test Series - 2007 conducted by GATE Forum in over 25 cities all over India. Question
Papers including section tests and full tests are designed by IISc alumni according to the latest syllabus. Percentile, All India Rank,
interaction with IISc alumni in our online discussion forums, and more.
visit
www.gateforum.com
Think GATE Think GATE Forum
GATE EC - 1991 www.gateforum.com
Join discussion of this test paper at http://forum.gatementor.com

1.26 Two dimensional addressing of 256 × 8 bit ROM using to 1 selectors requires
____ (how many?) NAND gates.

1.27 The CMOS equivalent of the following nMOS gate (in figure) is ______ (draw the
circuit).
+

O/P

A
C

1.28 In figure, the Boolean expression for the output in terms of inputs A, B and C
when the clock ‘CK’ is high, is given by _______

p-channel

O/P=?

CK A B

n-channel

1.29 An S-R FLIP-FLOP can be converted into a T FLIP FLOP by connecting ______ to
Q and _______ to Q.

Join All India Mock GATE Classroom Test Series - 2007 conducted by GATE Forum in over 25 cities all over India. Question
Papers including section tests and full tests are designed by IISc alumni according to the latest syllabus. Percentile, All India Rank,
interaction with IISc alumni in our online discussion forums, and more.
visit
www.gateforum.com
Think GATE Think GATE Forum
GATE EC - 1991 www.gateforum.com
Join discussion of this test paper at http://forum.gatementor.com

1.30 A bit stored in a FAMOS device can be erased by _______.

1.31 A signal has frequency components from 300 Hz to 1.8 KHz. The minimum
possible rate at which the signal has to be sampled is _______.

1.32 A sequential multiplexer is connected as shown in figure. Each time the


multiplexer receives the clock, it switches to the next channel (From 6 it goes to
1). If the input signals are
1
(
A = 5 cos 2π 4 × 103 t) A
B 2
B = 2 cos 2π (3.8 × 10 t ) 3
C 3 MUX
4
C = 6 cos 2π ( 2.2 × 10 t ) 3
5
Sampling and
D = 4 cos 2π (1.7 × 10 t ) 3
D 6
quantizing

Clock

he minimum clock frequency should be _____ KHz.

1.33 For the signal constellation shown in figure below, the type of modulation is
______.
n
cos2π t
T

S2 S1 n
sin2π t
T
S3 S4

T=symbol duration

1.34 Two resistors R1 and R2 (in ohms) at temperatures T1 and T2 K respectively, are
connected in series. Their equivalent noise temperatures is _____ K.

1.35 A binary source has symbol probabilities 0.8 and 0.2. If extension coding (blocks
of 4 symbols) is used. The lower and upper bounds on the average code word
length are
(a) lower _______
(b) higher _______

1.36 In a reflex Klystron oscillator


(a) the maximum possible efficiency is 58%
(b) the frequency of oscillation varies linearly with the reflector voltage
(c) the power output varies continuously with reflector voltage
(d) the power output is maximum at fixed frequency.

Join All India Mock GATE Classroom Test Series - 2007 conducted by GATE Forum in over 25 cities all over India. Question
Papers including section tests and full tests are designed by IISc alumni according to the latest syllabus. Percentile, All India Rank,
interaction with IISc alumni in our online discussion forums, and more.
visit
www.gateforum.com
Think GATE Think GATE Forum
GATE EC - 1991 www.gateforum.com
Join discussion of this test paper at http://forum.gatementor.com

1.37 The input impedance of a short-circuited lossless transmission line quarter wave
length is
(a) purely reactive (b) purely resistive
(c) infinite
(d) dependent on the characteristic impedance of the line.

1.38 The electric field component of a uniform plane electromagnetic wave


propagating in the Y-direction in a lossless medium will satisfy the equation.
∂ 2 Ey ∂ 2 Ey ∂ 2 Ey ∂ 2 Ey
(a) =µ∈ (b) =µ∈
∂y 2 ∂t 2 ∂x 2 ∂t 2

∂2 E x ∂2 E x E x2 + E z2 µ
(c) = µ ∈ (d) =
∂y 2 ∂t 2 2
H +H 2 ∈
x z

1.39 A radio wave is incident on a layer of ionosphere at an angle of 30 degree with


the vertical. If the critical frequency is 1.2 MHz, the maximum usable frequency
is
(a) 1.2 MHz (b) 2.4 MHz (c) 0.6 MHz (d) 1.386 MHz

1.40 In a broad side array of 20 isotropic radiators, equally spaced at a distance of


λ
, the beam width between first nulls is
2
(a) 51.3 degrees (b) 11.46 degrees
(c) 22.9 degrees (d) 102.6 degrees

2. (a) Find the Laplace transform of the waveform x ( t ) shown in figure.

z(t) 1

0
T 2T 3T 4T 5T 6T
-1 t

-2

1
(b) The network shown in figure is R1=5Ω
initially under steady state
i(t)
condition with the switch in
position 1. The switch is moved 2
10V L=2H
from position 1 to position 2 at t ≠
0. Calculate the current R2=5Ω
i ( t ) through R1 after switching.

3. The open loop transfer function of a


Join All India Mock GATE Classroom Test Series - 2007 conducted by GATE Forum in over 25 cities all over India. Question
Papers including section tests and full tests are designed by IISc alumni according to the latest syllabus. Percentile, All India Rank,
interaction with IISc alumni in our online discussion forums, and more.
visit
www.gateforum.com
Think GATE Think GATE Forum
GATE EC - 1991 www.gateforum.com
Join discussion of this test paper at http://forum.gatementor.com

feedback control system incorporating a dead time element is given by


Ke −Ts
G (s) =
s ( s + 1)

Where K > 0, and T > 0 are variable scalar parameters.


(a) For a given value of T show that the closed loop system is stable for all value
of K < K0 where K0 = ω0cosecω0T and ω0 is the smallest value of ω satisfying
the equation ω = cot ωT

4. The current I in a forward biased P + N junction shown in figure (a) is entirely due
to diffusion of holes from x = 0 to x = L. The injected hole concentration
distribution in the m-region is linear as shown in figure (b), with
1012
p (0) = and L = 10−3 cm . Determine:
cm3
(a) The current density in the diode assuming that the diffusion coefficient holes
is 12 cm2/sec.
(b) The velocity of holes in the n-region at x = 0.

p(o)

p(x)

Hole
I Density
x
p+ O L
n
L
O x

5. It is required to use a JFET of figure as linear resistor. The parameters of the


JFET are as follows:
W = 100 µm, L = µm, a = 2.5µm.
The doping in the n-layer is
16 3
N0 = 10 / cm and the electron mobility is
1500 cm2 / V − sec . The depletion layer
width of each junction due to the built in
potential is 0.25µm. The two p+ − gate
regions are connected together externally.
The resistances of the regions outside the
p+ Gate
gate are negligible. Determine the
q n Layer
minimum value of the linear resistor which
can be realized using this JFET without Source p+ Gate Drain
forward biasing the gate junctions. L

Join All India Mock GATE Classroom Test Series - 2007 conducted by GATE Forum in over 25 cities all over India. Question
Papers including section tests and full tests are designed by IISc alumni according to the latest syllabus. Percentile, All India Rank,
interaction with IISc alumni in our online discussion forums, and more.
visit
www.gateforum.com
Think GATE Think GATE Forum
GATE EC - 1991 www.gateforum.com
Join discussion of this test paper at http://forum.gatementor.com

6. In figure, the operational amplifier is ideal and its output can swing between – 15
and +15 volts. The input ν p which is zero for t < 0, is switched to 5 volts at the
instant t = 0. Given that the output ν 0 is +15 volts for t <0, sketch on the same
diagram the waveforms of ν 0 and ν i . You must give the values of important
parameters of this sketch.
1µF

5 volts

1K
0 −
τ=0
Vi
V1=? VO=?
+

7. In figure, the operational amplifiers are ideal and their output can swing between
-15 and +15 volts. Sketch on same diagram, the waveform of voltages
V1 and V2 as a function of time. You must give the values of important parameters
of this sketch.
3µF

V1=?
− 1K

+
+ V2=?
3K
1K

1K

8. The program given below is run on an 8085 based microcomputer system.


Determine the contents of the registers: PC, SP, B, C, H, L after a half instruction
is executed.
LOC
2000 START LXI SP 1000H
LXI H2F37 H
XRA A
MOV A, H
INX H
PUSH H
CZ 20 FF H
JMP 3000 H
HLT
20FF ADD H
Join All India Mock GATE Classroom Test Series - 2007 conducted by GATE Forum in over 25 cities all over India. Question
Papers including section tests and full tests are designed by IISc alumni according to the latest syllabus. Percentile, All India Rank,
interaction with IISc alumni in our online discussion forums, and more.
visit
www.gateforum.com
Think GATE Think GATE Forum
GATE EC - 1991 www.gateforum.com
Join discussion of this test paper at http://forum.gatementor.com

RZ
POP B
PUSH B
RMZ
HLT
3000
HLT

9. The four variable function f is given in terms of min-terms as:


f ( A, B, C , D ) = ∑ m (2, 3, 8,10,11,12,14,15) .
Using the K-map minimize the function in the sum of products form. Also, given
the realization using only two-input NAND gates.

10. (a) A signal A sin ω m t is input to a square – law device ( e0 − em2 ) . The output of
which is given to an FM modulator as the modulating signal.
The frequency deviation characteristics of the FM modulator is f = fc + Ke ( t ) .
Where e ( t ) is the modulating signal and K is a constant.

Determine the FM signal and the frequency components in its spectrum.


(b) In the figure, x ( t ) is the modulating signal and tc is the carrier frequency.
Determine the value of gain K so that the output is a suppressed carrier DSB
signal.

x(t) ein +
Σ K Q.e2in Σ eout (t )

+ ein
Σ b.e2 in

11. (a) A Gaussian random variable with zero mean and variance σ is input to a
limiter with input output characteristic given by
eout = ein for ein < σ
eout = σ for ein ≥ σ
eout = −σ for ein ≤ σ
Determine the probability density function of the output random variable.
(b) A random process X(t) is wide sense stationary. If

Join All India Mock GATE Classroom Test Series - 2007 conducted by GATE Forum in over 25 cities all over India. Question
Papers including section tests and full tests are designed by IISc alumni according to the latest syllabus. Percentile, All India Rank,
interaction with IISc alumni in our online discussion forums, and more.
visit
www.gateforum.com
Think GATE Think GATE Forum
GATE EC - 1991 www.gateforum.com
Join discussion of this test paper at http://forum.gatementor.com

Y ( t ) = x (t ) − x (t − a)

Determine the auto correlation function Ry ( i ) and power spectral density


Sy (ω ) of Y ( t ) in terms of those of X ( t ) .

12. A uniform plane electromagnetic wave traveling in free space enters into a
lossless medium at normal incidence. In the medium its velocity reduces by 50%
and in free space sets up a standing wave having a reflection coefficient of -
0.125. Calculate the permeability and the permittivity of the medium.

13. In the radiation pattern of a 3-element array of isotropic radiators equally spaced
λ
at distances of it is required to place a null at an angle of 33.56 degrees off
4
the end-fire direction. Calculate the progressive phase shifts to be applied to the
elements. Also calculate the angle at which the main beam is placed for this
phase distribution.

Join All India Mock GATE Classroom Test Series - 2007 conducted by GATE Forum in over 25 cities all over India. Question
Papers including section tests and full tests are designed by IISc alumni according to the latest syllabus. Percentile, All India Rank,
interaction with IISc alumni in our online discussion forums, and more.
visit
www.gateforum.com
Think GATE Think GATE Forum
GATE EC - 2000 www.gateforum.com
Join discussion of this test paper at http://forum.gatementor.com

SECTION – A (75 marks)


1. This question consists of TWENTY-FIVE sub-questions (1.1 – 1.25) of ONE mark
each. For each of these sub-questions, four possible alternatives (A,B, C and D)
are given, out of which ONLY ONE is correct. Indicate the correct answers in the
boxes corresponding to the questions only on the FIRST sheet of the answer
book.
1.1 In the circuit of Fig.P.11, the voltage v(t) is 1Ω 1Ω

(a) e at − e bt (b) e at + e bt +

(c) ae at
− be bt
(d) ae at
+ be bt eat V(t) 1H ebt
-

1.2 In the circuit of Fig.P1.2, the value of the voltage source E is V2 + -


0V
- +
(a) -16 V 2V - 1V
(b) 4 V
(c) -6V +
E=?
(d) 16 V -

+
+
4V 5V
V1 10V

t
s+2 s2 + 1
1.3 Given that L =  f ( t )  = 2
s +1
, L f ( t )  =
( s + 3) ( s + 2)
, h (t ) = ∫ f (τ ) g (t − τ ) dτ .
0

L h ( t )  is

s2 + 1 1
(a) (b)
s+3 s+3
s2 + 1 s+2
(c) + 2 (d) None of the above
( s + 3 )( s + 2 ) s +1
VCC
1.4 In the differential amplifier of Fig.P1.4, if the
source resistance of the current source IEE is
R R
infinite, then the common-mode gain is
(a) zero
Vin1
(b) infinite Vin2

(c) indeterminate
Vin1 + Vin2
(d)
2VT IEE

-VEE

Join All India Mock GATE Classroom Test Series - 2007 conducted by GATE Forum in over 25 cities all over India. Question
Papers including section tests and full tests are designed by IISc alumni according to the latest syllabus. Percentile, All India Rank,
interaction with IISc alumni in our online discussion forums, and more. For more details,
visit
www.gateforum.com
Think GATE Think GATE Forum
GATE EC - 2000 www.gateforum.com
Join discussion of this test paper at http://forum.gatementor.com
+15V
1.5 In the circuit of Fig.P.15, Vo is
(a) -1 V
-
(b) 2 V +1V Vo
+
(c) +1 V R
(d) +15 V -15V

1.6 Introducing a resistor in the emitter of a common amplifier stabilizes the dc


operating point against variations in
(a) only the temperature (b) only the β of the transistor
(c) both temperature and β (d) none of the above

1.7 The current gain of a bipolar transistor drops at high frequencies because of
(a) transistor capacitances (b) high current effects in the base
(c) parasitic inductive elements (d) the Early effect

1.8 An amplifier with resistive negative feedback has two left half-plane poles in its
open-loop transfer function. The amplifier
(a) will always be unstable at high frequencies
(b) will be stable for all frequencies
(c) may be unstable, depending on the feedback factor
(d) will oscillate at low frequencies

C
1.9 If the op-amp in Fig.P.1.9, is ideal, then vo , is
(a) zero V1 sinωt C
-
(b) (V1 − V2 ) sin ω t Vo
V2 sinωt +
(c) − (V1 + V2 ) sin ω t C

(d) (V1 + V2 ) sin ωt


R1 R2
1.10 The configuration of Fig.P.1.10 is a
(a) precision integrator
(b) Hartley oscillator -
Vo
(c) Butterworth highpass filter + R
(d) Wien-bridge oscillator
C

R C

Join All India Mock GATE Classroom Test Series - 2007 conducted by GATE Forum in over 25 cities all over India. Question
Papers including section tests and full tests are designed by IISc alumni according to the latest syllabus. Percentile, All India Rank,
interaction with IISc alumni in our online discussion forums, and more. For more details,
visit
www.gateforum.com
Think GATE Think GATE Forum
GATE EC - 2000 www.gateforum.com
Join discussion of this test paper at http://forum.gatementor.com

1.11 Assume that the op-amp of Fig.P1.11 is ideal. If vi is a triangular wave, then
vo will be R
(a) square wave
C
(b) triangular wave Vo -
(c) parabolic wave Vo
+
(d) sine wave

2
1.12 The Fourier Transform of the signal x ( t ) = e −3t is of the following form, where A
and B are constants:
−B f 2
(a) Ae (b) Ae − Bf (c) A + B f (d) Ae − Bf

1.13 A system with an input x ( t ) and output y ( t ) is described by the relation:


y ( t ) = tx ( t ) . This system is

(a) linear and time-invariant (b) linear and time varying


(c) non-linear and time-invariant (d) non-linear and time-varying

1.14 The amplitude modulated wave from s ( t ) = Ac 1 + K a m ( t )  cos ωC t is fed to an


ideal envelope detector. The maximum magnitude of K0 m ( t ) is greater than 1.
Which of the following could be the detector output?
2
(a) Ac m ( t ) (b) Ac 2 1 + K am ( t ) 
2
(c) Ac 1 + K a m ( t )  (d) Ac 1 + K a m ( t ) 

1.15 An 8 bit successive approximation analog to digital converter has full scale
reading of 2.55 V and its conversion time for an analog input of 1V is 20µs. The
conversion for a 2V input will be
(a) 10 µs (b) 20 µs (c) 40 µs (d) 50 µs

1.16 The number of hardware interrupts (which require an external signal to interrupt)
present in an 8085 microprocessor are
(a) 1 (b) 4 (c) 5 (d) 13

1.17 The most commonly used amplifier in sample and hold circuits is
(a) a unity gain inverting amplifier
(b) a unity gain non-inverting amplifier
(c) an inverting amplifier with a gain of 10
(d) an inverting amplifier with a gain of 100
Join All India Mock GATE Classroom Test Series - 2007 conducted by GATE Forum in over 25 cities all over India. Question
Papers including section tests and full tests are designed by IISc alumni according to the latest syllabus. Percentile, All India Rank,
interaction with IISc alumni in our online discussion forums, and more. For more details,
visit
www.gateforum.com
Think GATE Think GATE Forum
GATE EC - 2000 www.gateforum.com
Join discussion of this test paper at http://forum.gatementor.com

1.18 The number of comparators in a 4 bit flash ADC is


(a) 4 (b) 5 (c) 15 (d) 16

1.19. For the logic circuit shown in Fig.P1.19, the required input condition (A,B,C) to
make the output (X)=1 is
A
(a) 1, 0, 1
B
(b) 0, 0, 1
(c) 1, 1, 1
(d) 0, 1, 1
x
C

1.20. In the 8085 microprocessor, the RST6 instruction transfers the program
execution to the following location
(a) 30 H (b) 24 H (c) 48 H (d) 60 H

1.21. The magnitudes of the open-circuit and short-circuit input impedances of a


transmission line are 100Ω and 25Ω respectively. The characteristic impedance of
the line is,
(a) 25Ω (b) 50Ω (c) 75Ω (d) 100Ω

1.22. A TEM wave is incident normally upon a perfect conductor. The E and H fields at
the boundary will be, respectively.
(a) minimum and minimum (b) maximum and maximum
(c) minimum and maximum (d) maximum and minimum

1.23. The frequency range for satellite communication is


(a) 1 KHz to 100 KHz (b) 100 KHz to 10 KHz
(c) 10 MHz to 30 MHz (d) 1 GHz to 30 GHz

λ λ λ
1.24. If the diameter of a dipole antenna is increased from to , then its
2 100 50
(a) bandwidth increases (b) bandwidth decreases
(c) gain increases (d) gain decreases
L1
1.25 The circuit of Fig.P1.25 represents a RS
Vo
(a) low pass filter
(b) high pass filter C1 L2
RL
~ VS
(c) band pass filter
C2
(d) band reject filter

Join All India Mock GATE Classroom Test Series - 2007 conducted by GATE Forum in over 25 cities all over India. Question
Papers including section tests and full tests are designed by IISc alumni according to the latest syllabus. Percentile, All India Rank,
interaction with IISc alumni in our online discussion forums, and more. For more details,
visit
www.gateforum.com
Think GATE Think GATE Forum
GATE EC - 2000 www.gateforum.com
Join discussion of this test paper at http://forum.gatementor.com

2. This question consists of TWENTY-FIVE sub-questions (2.1 – 2.25) of ONE mark


each. For each of these sub-questions, four possible alternatives (A,B, C and D)
are given, out of which ONLY ONE is correct. Indicate the correct answers in the
boxes corresponding to the questions only on the SECOND sheet of the answer
book.
2 −1 0 0
 
0 3 0 0
2.1 The eigen values of the matrix  are
0 0 −2 0
 
0 0 −1 4
(a) 2, -2, 1, -1 (b) 2, 3, -2, 4
(c) 2, 3, 1, 4 (d) None of the above

2.2. Use the data of Fig.2(a). The current i in the circuit of Fig.(b) is
(a) -2A R2 R2

(b) 2A
R1 R3 R1 R3
(c) -4A
(d) +4A
+ R4 R4
I=? 20V
10V 2A
+

2.3. For the circuit in Fig.P2.3, the voltage vo is 2Ω 2Ω


(a) 2V (b) 1V -
+ -
4V 2V
(c) -1V 2Ω vo
- +
(d) None of the above +

2.4. A linear time invariant system has an impulse response e2t , t > 0 . If the initial
conditions are zero and the input is e3t , the output for t > 0 is

(a) e3t − e2t (b) e5t

(c) e3t + e2t (d) None of the above


15V

2.5. In the circuit of Fig.P2.5, assume that the transistor is in


active region. It has a large β and its base-emitter voltage is RC
0.7V. The value of Ic is 10kΩ
IC
(a) Indeterminate since Rc is not given
(b) 1 mA
5kΩ
(c) 5 mA
430Ω
(d) 10 mA

Join All India Mock GATE Classroom Test Series - 2007 conducted by GATE Forum in over 25 cities all over India. Question
Papers including section tests and full tests are designed by IISc alumni according to the latest syllabus. Percentile, All India Rank,
interaction with IISc alumni in our online discussion forums, and more. For more details,
visit
www.gateforum.com
Think GATE Think GATE Forum
GATE EC - 2000 www.gateforum.com
Join discussion of this test paper at http://forum.gatementor.com

2.6. If the op-amp in Fig.P2.6, has an input offset voltage of 5 mV and an open-loop
voltage gain of 10,000, then vo will be
+15V
(a) 0V
(b) 5 mV -
Vo
(c) +15 V or –15 V +
(d) +50 V or –50 V -15V

2.7. For the logic circuit shown in Fig.P2.7, the simplified Boolean expression for the
output Y is A
(a) A+B+C B

(b) A
(c) B Y
(d) C
C

2.8. For the 4 bit DAC shown in Fig.P2.8, the output voltage vo is
1K 7K

+15V

-
R R R Vo
+

2R 2R 2R 2R 2R -15V

1V 1V
(a) 10 V (b) 5 V (c) 4 V (d) 8 V

2.9. A sequential circuit using D flip-flop and logic gates is shown in Fig.P2.9, where X
and Y are the inputs and Z is the output. The circuit is

X
D Q Z
CLK
Y Q Z

(a) S – R Flip-Flop with inputs X = R and Y = S


(b) S – R Flip-Flop with inputs X = S and Y = R
(c) J – K Flip-Flop with inputs X = J and Y = K
(d) J – K Flip-Flop with inputs X = K and Y = J
Join All India Mock GATE Classroom Test Series - 2007 conducted by GATE Forum in over 25 cities all over India. Question
Papers including section tests and full tests are designed by IISc alumni according to the latest syllabus. Percentile, All India Rank,
interaction with IISc alumni in our online discussion forums, and more. For more details,
visit
www.gateforum.com
Think GATE Think GATE Forum
GATE EC - 2000 www.gateforum.com
Join discussion of this test paper at http://forum.gatementor.com

2.10. The contents of Register (B) and Accumulator (A) of 8085 microprocessor are
49H and 3AH respectively. The contents of A and the status of carry flag (CY) and
sign flag (S) after executing SUB B instructions are
(a) A = F1, CY = 1, S =1 (b) A = 0F, CY = 1, S =1
(c) A = F0, CY = 0, S =0 (d) A = 1F, CY = 1, S =1

2.11. In Fig.P2.11, the J and K inputs of all the four Flip-Flops are made high. The
frequency of the signal at output Y is

J Q J Q J Q J Q

CLK CLK CLK CLK


F=10kHz

K K K K
CLR CLR CLR CLR

(a) 0.833 KHz (b) 1.0 KHz (c) 0.91 KHz (d) 0.77 KHz

2.12. One period (0,T) each of two periodic waveforms, W1 and W2 are shown in
Fig.P2.12. The magnitudes of the nth Fourier series coefficients of W1 and W2, for
n ≥ 1, n odd, are respectively proportional to

(a) n−3 and n−2 1


1
W1 W2
(b) n−2 and n−3
T/2 T
T
−1 −2 0 0 T/2
(c) n and n

(d) n−4 and n−2 -1 -1

2.13. Let u(t) be the step function. Which of the waveforms in Fig.P2.13(a) – (d)
corresponds to the convolution of u(t) – u(t – 1) with u(t) – u(t – 2)?
(a) 1 (b)
1

0 1 2 3
0 1 2

Join All India Mock GATE Classroom Test Series - 2007 conducted by GATE Forum in over 25 cities all over India. Question
Papers including section tests and full tests are designed by IISc alumni according to the latest syllabus. Percentile, All India Rank,
interaction with IISc alumni in our online discussion forums, and more. For more details,
visit
www.gateforum.com
Think GATE Think GATE Forum
GATE EC - 2000 www.gateforum.com
Join discussion of this test paper at http://forum.gatementor.com

1.5 1
(c) (d)

0 t
t 1 3
0 1.5 2 3

2.14. In Fig.P2.14, the steady state output voltage corresponding to the input voltage 3
+ 4 sin 100t V is
4  π 1KΩ
(a) 3 + sin  100t −  V
2  4
input output
 π 10µF
(b) 3 + 4 2 sin 100t −  V
 4

3 4  π
(c) + sin  100t +  V
2 2  4

 π
(d) 3 + 4 sin 100t +  V
 4

2.15. In a digital communication system employing Frequency Shift Keying (FSK), the
0 and 1 bit are represented by sine waves of 10 KHz and 25 KHz respectively.
These waveforms will be orthogonal for a bit interval of
(a) 45 µsec (b) 200 µsec (c) 50 µsec (d) 250 µsec

2.16. A message m(t) band-limited to the frequency fm has a power of Pm. The power
of the output signal in the Fig.P2.16 is
Pm cos θ
(a)
2
(multiply) Ideal low pass
P m(t)cosωot filter cut off
(b) m Output signal
4 F=fm pass
band gain =1 (ωo>2πfm)
P sin2 θ
(c) m
4
cos(ωot+θ)
Pm cos2 θ
(d)
4

2.17. The Hilbert transform of cos ω1t + sin ω2t is


(a) sin ω1t − cos ω2t (b) sin ω1t + cos ω2t
(c) cosω1t − sin ω2t (d) sin ω1t + sin ω2t

Join All India Mock GATE Classroom Test Series - 2007 conducted by GATE Forum in over 25 cities all over India. Question
Papers including section tests and full tests are designed by IISc alumni according to the latest syllabus. Percentile, All India Rank,
interaction with IISc alumni in our online discussion forums, and more. For more details,
visit
www.gateforum.com
Think GATE Think GATE Forum
GATE EC - 2000 www.gateforum.com
Join discussion of this test paper at http://forum.gatementor.com

2.18. A system has a phase response given by φ(ω), where ω is the angular frequency.
The phase delay and group delay at ω = ωo are respectively given by
φ (ω o ) dφ (ω ) d 2φ (ωo )
(a) − ,− ω = ωo (b) φ (ωo ) , − ω = ωo
ωo dω dω 2

ωo dφ (ω ) ωo
(c)
φ (ω o )
,−

ω = ωo (d) ωoφ (ωo ) , ∫−∞
φ ( λ ) dλ

2.19. A system described by the transfer function


1
H (s) = 3 2
is stable. The constraints on α and k are.
s + α s + ks + 3
(a) α > 0, αk < 3 (b) α > 0, αk > 3 (c) α > 0, αk > 0 (d) α > 0, αk < 0

2.20. In an FM system, a carrier of 100 MHz is modulated by a sinusoidal signal of 5


KHz. The bandwidth by Carson’s approximation is 1 MHz. If y(t) = (modulated
waveform)3, than by using Carson’s approximation, the bandwidth of y(t) around
300 MHz and the and the spacing of spectral components are, respectively.
(a) 3 MHz, 5 KHz (b) 1 MHz, 15 KHz
(c) 3 MHz, 15 KHz (d) 1 MHz, 5 KHz

2.21. A uniform plane wave in air impinges at 45° angle on a lossless dielectric
material with dielectric constant ε r . The transmitted wave propagates in a 30°
direction with respect to the normal. The value of ε r is

(a) 1.5 (b) 1.5 (c) 2 (d) 2

2.22. For an 8 feet (2.4 m) parabolic disk antenna operating at 4 GHz, the minimum
distance required for far field measurement is closest to
(a) 7.5 cm (b) 15 cm (c) 15 m (d) 150 m

2.23. A rectangular wave guide has dimensions 1 cm × 0.5 cm. Its cut-off frequency is
(a) 2 dB (b) 5 dB (c) 8 dB (d) 12 dB

2.24. A rectangular wave guide has dimensions 1 cm × 0.5 cm. Its cut-off frequency is
(a) 5 GHz (b) 10 GHz (c) 15 GHz (d) 20 GHz

2.25. Two coaxial cables 1 and 2 are filled with different dielectric constants
λ 
ε r1 and ε r 2 respectively. The ratio of the wavelengths in the two cables,  1  is
 λ2 
ε r1 εr 2 ε r1 εr2
(a) (b) (c) (d)
εr 2 ε r1 εr2 ε r1
Join All India Mock GATE Classroom Test Series - 2007 conducted by GATE Forum in over 25 cities all over India. Question
Papers including section tests and full tests are designed by IISc alumni according to the latest syllabus. Percentile, All India Rank,
interaction with IISc alumni in our online discussion forums, and more. For more details,
visit
www.gateforum.com
Think GATE Think GATE Forum
GATE EC - 2000 www.gateforum.com
Join discussion of this test paper at http://forum.gatementor.com

SECTION – B (75 marks)


This section consists of TWENTY questions of FIVE marks each. Attempt ANY FIFTEEN
questions out of them. If more number of questions are attempted, score off the
answers not be evaluated, else only the first fifteen unscored answers will be
considered.
3. For the circuit in Fig.P3
(a) Find the Thevenin equivalent of the sub circuit faced by the capacitor across
the terminals a, b.
2Ω 2Ω
(b) Find vc ( t ) , t > 0, given vc ( 0 ) = 0
a
(c) Find i ( t ) , t > 0 + 2Ω
VC 2F ⇒ 4V
-
b i(t)

Fig.P3
4. For the circuit in Fig.P4, which is in steady state
(a) Find the frequency ωo at which the magnitude of the impedance across
terminals a, b reaches a maximum.
(b) Find the impedance across a, b at the frequency ωo

(c) If v s ( t ) = V sin (ωot ) , find iL ( t ) , iR ( t ) . 2Ω


+
a
+ 2Ω VC
4F
Vi(t) ~ iL iR
-
-
b

5. For the circuit in Fig.P5, write the state equations using vc and iL as state
variables.
VC 1Ω

+ 2Ω 2H
VC 1F
-
εv
iL

6. The network N in Fig.P6 consists only of two elements: a resistor of 1Ω and an


inductor of L Henry. A 5 V source is connected at the input at t = 0 seconds. The
inductor current is zero at t =0. The output voltage is found to be 5e −3t V , for
(a) Find the voltage transfer function of the network.
(b) Find L, and draw the configuration of the network.
(c) Find the impulse response of the network.

+
Input + N Output
voltage - voltage
-

Fig.P6
Join All India Mock GATE Classroom Test Series - 2007 conducted by GATE Forum in over 25 cities all over India. Question
Papers including section tests and full tests are designed by IISc alumni according to the latest syllabus. Percentile, All India Rank,
interaction with IISc alumni in our online discussion forums, and more. For more details,
visit
www.gateforum.com
Think GATE Think GATE Forum
GATE EC - 2000 www.gateforum.com
Join discussion of this test paper at http://forum.gatementor.com

7. For the linear, time invariant system whose block diagram is shown in Fig.P7(a),
with input x(t) and output y(t),
(a) Find the transfer function.
(b) For the step response of the system [i.e. find y(t) when x(t) is a unit step
function and the initial conditions are zero]
(c) Find y(t), if x(t) is as shown in Fig.P7(b), and the initial conditions are zero.

+ x(t)
Σ
x(t) y(t)
- -
4 1

3
t
1 2
Fig.P7(a)
Fig.P7(b)

8. A certain linear, time-invariant system has the state and output representation
shown below:
 x&1   −2 1   x1   1 
 =   +  u
 x& 2   0 −3   x2   0 
x 
y = (1 1)  1 
 x2 
(a) Find the eigen values (natural frequencies) of the system.
(b) If u(t) = δ(t) and x1 ( 0+ ) = x2 ( 0+ ) = 0, find x1 ( t ) , x2 ( t ) and y ( t ) , for t > 0.

(c) When the input is zero, choose initial conditions


x1 ( 0+ ) and x2 ( 0+ ) such that y ( t ) = Ae −2t for t > 0.

9. The block diagram of a feedback system is shown in Fig.P9(a).


(a) Find the closed loop transfer function.
(b) Find the minimum value of G for which the step response of the system
would exhibit an overshoot, as shown in Fig.P9(b).
(c) For G equal to twice this minimum value, find the time period T indicated in
Fig.P9(b).

Vo(t)

+ G
Step response

Σ
Input s ( s + 5) Output
-
(VS)

(G>0)

Join All India Mock GATE Classroom Test Series - 2007 conducted by GATE Forum in over 25 cities all over India. Question
Papers including section tests and full tests are designed by IISc alumni according to the latest syllabus. Percentile, All India Rank,
interaction with IISc alumni in our online discussion forums, and more. For more details,
visit
www.gateforum.com
Think GATE Think GATE Forum
GATE EC - 2000 www.gateforum.com
Join discussion of this test paper at http://forum.gatementor.com

10. (a) For Fig.P10(a), Plot vo under steady state conditions, with and without C.
Assume that the diode is ideal.
(b) Design a circuit using two ideal diodes, one resistor and two voltage sources
that would convert the input voltage of Fig.P10(b) to the output voltage of
Fig.P10(c). The resistor value need not be specified.
Vi

vi vo
t
R C

Fig.P10(a)
Vi Vi

10V

0 t 5V t

-10V -5V

Fig.P10(b)
Fig.P10(c)

11. For the amplifier of Fig.P11,


T
IC = 1.3mA, RC = 2k Ω, RE = 500Ω, VT = = 26mV , β = 100, VCC = 15V
q VCC
ν s = 0.01 sin (ω t ) V and Cb = Ce = 10µ F .
νo RC
(a) What is the small-signal voltage gain, Aν = ?
νs R1

(b) What is the approximate Aν if Ce is removed? IC


Vo
(c) What will ν o be if Cb is short-circuited?
Cb

VS ~ R2
RE CE

12. For a feedback amplifier, the open loop transfer function has three poles at 100k
rad/s, 1 M rad/s and 10 M rad/s. The low frequency open loop gain is 1000 and
the feedback factor (β) is 1. Use Bode plots to determine the phase margin of the
amplifier. Is the amplifier stable?

Join All India Mock GATE Classroom Test Series - 2007 conducted by GATE Forum in over 25 cities all over India. Question
Papers including section tests and full tests are designed by IISc alumni according to the latest syllabus. Percentile, All India Rank,
interaction with IISc alumni in our online discussion forums, and more. For more details,
visit
www.gateforum.com
Think GATE Think GATE Forum
GATE EC - 2000 www.gateforum.com
Join discussion of this test paper at http://forum.gatementor.com

13. Fig.P13 shows a common base amplifier.


(a) Write expressions for the time-constants associated with the capacitors,
CB and CS .

(b) What is the approximate lower cutoff frequency of the amplifier?


VCC

RC
R1

Vo
Rs

CB R2 Cs
RE ~ Vs

14. For the CMOS monostable multi-vibrator of Fig.P14, R = 50 kW, C = 0.01µF,


VDD = 5V , and the CMOS NOR gates have a threshold voltage (VT ) of 1.5 V. ν in is a
trigger pulse τ p ( )
RC as shown in the figure.

(a) Plot ν 01 and ν R as functions of time.

(b) Write the equation for ν R ( t ) , for t > 0.

(c) Find the time period of the output pulse.


Vin Vin V01
V0

5V
VR
t C
0 τP
R

15. The operating conditions (ON = 1, OFF = 0) of three pumps (x,y,z) are to be
monitored. x = 1 implies that pump X is on. It is required that the indicator (LED)
on the panel should glow when a majority of the pumps fail.
(a) Enter the logical values in the K-map in the format shown in Fig.15(a). derive
the minimal Boolean sum of products expression whose output is zero when
a majority of the pumps fail.
(b) The above expression is implemented using logic gates, and point P is the
output of this circuit, as shown in Fig.P15(b). P is at 0 V when a majority of
the pumps fails and is at 5 V otherwise. Design a circuit to drive the LED
using this output. The current through the LED should be 10 mA and the

Join All India Mock GATE Classroom Test Series - 2007 conducted by GATE Forum in over 25 cities all over India. Question
Papers including section tests and full tests are designed by IISc alumni according to the latest syllabus. Percentile, All India Rank,
interaction with IISc alumni in our online discussion forums, and more. For more details,
visit
www.gateforum.com
Think GATE Think GATE Forum
GATE EC - 2000 www.gateforum.com
Join discussion of this test paper at http://forum.gatementor.com

voltage drop across it is 1V. Assume that P can source or sink 10 mA and a 5
V supply is available.
z z
x y
P
x y Logic
x y Circuit
x y

16. A one-bit full adder is to be implemented using 8-to-1 multiplexers (MUX).


(a) Write the truth table for sum (S) and carry to the next stage ( CN ) in terms of
the two bits (A, B) and carry from the previous stage C p . The truth table ( )
should be in the ascending order of (A,B, C p ), i.e. (000,001,010,…etc.).

(b) Implement S and CN using 8-to-1 multiplexers.

17. (a) The program and machine code for an 8085 microprocessor are given by

3E MVI A C3
C3
00 NOP
80 ADD B
3D DEC A
C2 JNZ 800A
0A
80
C3 JMP 800C
0C
80
D3 OUT 10
10
76 HLT
The starting address of the above program is 7FFF H. What would happen if
it is executed from 8000 H?
(b) For the instructions given below, how many memory operations (read/write)
are performed during the execution in an 8085 µP?
(c) Write an instruction which takes the minimum possible time to clear the
accumulator of the 8085.

Join All India Mock GATE Classroom Test Series - 2007 conducted by GATE Forum in over 25 cities all over India. Question
Papers including section tests and full tests are designed by IISc alumni according to the latest syllabus. Percentile, All India Rank,
interaction with IISc alumni in our online discussion forums, and more. For more details,
visit
www.gateforum.com
Think GATE Think GATE Forum
GATE EC - 2000 www.gateforum.com
Join discussion of this test paper at http://forum.gatementor.com

18. A band limited signal x(t) with a′ spectrum X(f) as shown in Fig.P18(a) is
processed as shown in Fig.P18(b). p(t) is a periodic train of impulses as in
Fig.P18(c). The ideal bandpass filter has a passband from 26 KHz to 34 KHz.
(a) Calculate the Fourier series coefficients Cn in the Fourier expansion of p(t) in
+∞
 jn2π t 
the form p ( t ) = ∑ C ex
n = −∞
n p 
 T 

(b) Find the Fourier Transform of p(t).


(c) Obtain and sketch the spectrum of xs ( t ) .

(d) Obtain and sketch the spectrum of y ( t ) .

x(f) 1

Ideal band
x(t)
xS(t) pass filter

0 4KHz
f in KHz p(t) y(t)

Fig.P18(a) Fig.P18(b)

Train of impulses of unit strength

-T 0 T 2T 3T Time 
1
= 10KHz T
T
Fig.P18(c)

19. Zero mean white Gaussian noise with a two-sided power spectral density of 4
W/Khz is passed through an ideal lowpass filter with a cut-off frequency of 2 KHz
and a pass band gain of 1, to produce the noise output n(t).
(a) Obtain the total power in n(t).
(b) Find the autocorrelation function E[n(t)n(t+τ) of the noise n(t) as a function
of τ.
(c) Two noise samples are taken at times t1 and t2. Find the spacing t1 − t2 so
that the product n ( t1 ) n ( t2 ) has the most negative expected value and obtain
this most negative expected value.

Join All India Mock GATE Classroom Test Series - 2007 conducted by GATE Forum in over 25 cities all over India. Question
Papers including section tests and full tests are designed by IISc alumni according to the latest syllabus. Percentile, All India Rank,
interaction with IISc alumni in our online discussion forums, and more. For more details,
visit
www.gateforum.com
Think GATE Think GATE Forum
GATE EC - 2000 www.gateforum.com
Join discussion of this test paper at http://forum.gatementor.com

Given E = 10e (
− j 4 x − kt )
20. ˆ V m in free space.
y
(a) Write all the four Maxwell’s equations in free space.
(b) Find ∇ × E.
(c) Find H.

21. The three regions shown in Fig.P21 are all lossless and non-magnetic. Find
(a) Wave impedance in mediums 2 and 3.
λ
(b) d such that medium 2 acts as a quarter wave   transformer.
4
(c) Reflection coefficient ( Γ ) and voltage standing wave ratio (VSWR) at the
λ
interface of the medium 1 and 2, when d =  
4

Medium 1 Medium 2 Medium 3


εr1=1 εr2=5 εr3=9

Incident Team
Wave (f=1GHz)
d

22. Design a lossless impedance matching network shown in Fig.P22 to transform


ZL = 10 + j10Ω to Zin = 50Ω. Find the values of L, C and quality factor (Q) of the
circuit at f = 1GHz.

Zin C
ZL

Join All India Mock GATE Classroom Test Series - 2007 conducted by GATE Forum in over 25 cities all over India. Question
Papers including section tests and full tests are designed by IISc alumni according to the latest syllabus. Percentile, All India Rank,
interaction with IISc alumni in our online discussion forums, and more. For more details,
visit
www.gateforum.com
Think GATE Think GATE Forum
GATE EC - 2001 www.gateforum.com
Join discussion of this test paper at http://forum.gatementor.com

SECTION - A

1. This question consists of TWENTY-FIVE sub-questions (1.1 – 1.25) of ONE mark


each. For each of these sub-questions, four possible alternatives (A,B, C and D)
are given, out of which ONLY ONE is correct. Indicate the correct answer by
darkening the appropriate bubble against the question number on the left hand
side of the Objective Response Sheet (ORS). You may use the answer book
provided for any rough work, if needed.

4Ω 2Ω
1.1 The voltage e0 in figure 1.1 is
(a) 2V
+
4
(b) V 4Ω eO 2Ω
3 12V -
(c) 4V
(d) 8V

1.2. If each branch of a Delta circuit has impedance 3Z , then each branch of the
equivalent Wye circuit has impedance.
Z Z
(a) (b) 3Z (c) 3 3Z (d)
3 3

1
1.3. The transfer function of a system is given by H ( s ) = 2
. The impulse
s ( s − 2)
response of the system is: (* denotes convolution, and U(t) is unit step function)
(a) (t2 * e-2t)U(t) (b) (t * e2t)U(t) (c) (te-2t)U(t) (d) (te-2t)U(t)

1.4. The admittance parameter Y12 in the 2-port network in Figure 1.4 is
(a) -0.2 mho 20Ω
I1 I2
(b) 0.1 mho
(c) -0.05 mho
E1 5Ω 10Ω
(d) 0.05 mho E2

1.5. The region of convergence of the z-transform of a unit step function is


(a) z >1 (b) z <1

(c) (Real part of z) > 0 (d) (Real part of z) < 0

Join All India Mock GATE Classroom Test Series - 2007 conducted by GATE Forum in over 25 cities all over India. Question
Papers including section tests and full tests are designed by IISc alumni according to the latest syllabus. Percentile, All India Rank,
interaction with IISc alumni in our online discussion forums, and more. For more details,
visit
www.gateforum.com
Think GATE Think GATE Forum
GATE EC - 2001 www.gateforum.com
Join discussion of this test paper at http://forum.gatementor.com

1.6. The current gain of a BJT is


gm gm
(a) gmr0 (b) (c) gmrπ (d)
ro rπ
1.7. MOSFET can be used as a
(a) current controlled capacitor (b) voltage controlled capacitor
(c) current controlled inductor (d) voltage controlled inductor

1.8. The effective channel length of a MOSFET in saturation decreases with increase in
(a) gate voltage (b) drain voltage
(c) source voltage (d) body voltage

1.9. The ideal OP-AMP has the following characteristics.


(a) Ri = ∞, A = ∞, R0 = 0 (b) Ri = 0, A = ∞, R0 = 0
(c) Ri = ∞, A = ∞, R0 = ∞ (d) Ri = 0, A = ∞, R0 = ∞

1.10. The 2’s complement representation of –17 is


(a) 01110 (b) 01111 (c) 11110 (d) 10001

1.11. Consider the following two statements:


Statement 1: A stable multi-vibrator can be used for generating square wave.
Statement 2: B stable multi-vibrator can be used for storing binary information.
(a) Only statement 1 is correct
(b) Only statement 2 is correct
(c) Both the statements 1 and 2 are correct
(d) Both the statements 1 and 2 are incorrect

1.12. For the ring oscillator shown in Figure 1.12, the propagation delay of each
inverter is 100 pico second. What is the fundamental frequency of the oscillator
output?
(a) 10 MHz
V0
(b) 100 MHz
(c) 1 GHz
(d) 2 GHz

1.13. An 8085 microprocessor based system uses a 4K × 8-bit RAM whose starting
address is AA00. The address of the last byte in this RAM is
(a) 0FFFH (b) 1000 H (c) B9FF H (d) BA00 H

Join All India Mock GATE Classroom Test Series - 2007 conducted by GATE Forum in over 25 cities all over India. Question
Papers including section tests and full tests are designed by IISc alumni according to the latest syllabus. Percentile, All India Rank,
interaction with IISc alumni in our online discussion forums, and more. For more details,
visit
www.gateforum.com
Think GATE Think GATE Forum
GATE EC - 2001 www.gateforum.com
Join discussion of this test paper at http://forum.gatementor.com

1.14. The equivalent of the block diagram in Figure 1.14 is given in

E G1 G2 C

F H

(a) E G1 C (b)
E G1G2 C

F H/G2
F HG2

(c) E G1 C (d)
E G1G2 C

F HG2
F H/G2

15. If the characteristic equation of a closed-loop system is s2 + 2s + 2 = 0 , then the


system is
(a) over damped (b) critically damped
(c) underdamped (d) undamped

1.16. The root-locus diagram for a closed loop feedback system is shown in Figure
1.16. The system is overdamped. jω

K=5
K=∞ K=0 K=0
x x σ
∞ K -3 -2 -1
K=1

(a) only if 0 ≤K ≤ 1
(b) only if 1 < K < 5
(c) only if K > 5
(d) if 0 ≤K < 1 or K > 5

Join All India Mock GATE Classroom Test Series - 2007 conducted by GATE Forum in over 25 cities all over India. Question
Papers including section tests and full tests are designed by IISc alumni according to the latest syllabus. Percentile, All India Rank,
interaction with IISc alumni in our online discussion forums, and more. For more details,
visit
www.gateforum.com
Think GATE Think GATE Forum
GATE EC - 2001 www.gateforum.com
Join discussion of this test paper at http://forum.gatementor.com

1.17. The Nyquist plot for the open-loop transfer function G(s) of a unity negative
feedback system is shown in figure 1.17 if G(s) has no pole in the right half of s-
plane, the number of roots of the system characteristic equation in the right half
of s-plane is Im(s)
(a) 0
(b) 1 G(s)-Plane
(c) 2
(d) 3
-1 Re(s)


 3t 
1.18. Let δ(t) denote the delta function. The value of the integral ∫ δ (t ) cos  2  dt is
−∞

π
(a) 1 (b) -1 (c) 0 (d)
2

1.19. A band limited signal is sampled at the Nyquist rate. The signal can be recovered
by passing the samples through
(a) an RC filter (b) an envelope detector
(c) a PLL
(d) an ideal low-pass filter with appropriate bandwidth

2
− ( x − 4)
1
1.20. The PDF of a Gaussian random variable X is given by Px ( x ) = e 18 . The
3 2π
probability of the event {X = 4} is
1 1 1
(a) (b) (c) 0 (d)
2 3 2π 4

1.21. If a signal f(t) has energy E, the energy of the signal f(2t) is equal to
E
(a) E (b) (c) 2E (d) 4E
2

1.22. A transmission line is distortion-less if


1
(a) RL = (b) RL = GC (c) LG = RC (d) RG = LC
GC

Join All India Mock GATE Classroom Test Series - 2007 conducted by GATE Forum in over 25 cities all over India. Question
Papers including section tests and full tests are designed by IISc alumni according to the latest syllabus. Percentile, All India Rank,
interaction with IISc alumni in our online discussion forums, and more. For more details,
visit
www.gateforum.com
Think GATE Think GATE Forum
GATE EC - 2001 www.gateforum.com
Join discussion of this test paper at http://forum.gatementor.com

∂2 E x ∂2 E x
1.23. If a plane electromagnetic wave satisfies the equation = c2 , the wave
∂x 2 ∂t 2
propagates in the
(a) x-direction (b) z-direction
(c) y-direction
(d) xy plane at an angle of 45° between the x and z directions

1.24. The phase velocity of waves propagating in a hollow metal waveguide is


(a) greater than the velocity of light in free space.
(b) less than the velocity of light in free space.
(c) equal to the velocity of light in free space.
(d) equal to the group velocity.

1.25. The dominant mode in a rectangular waveguide is TE10, because this mode has
(a) no attenuation (b) no cut-off
(c) no magnetic field component (d) the highest cut-off wavelength

2. This question consists of TWENTY-FIVE sub-questions (2.1 – 2.25) of TWO marks


each. For each of these sub-questions, four possible alternatives (A, B, C and D)
are given, out of which ONLY ONE is correct. Indicate the correct answer by
darkening the appropriate bubble against the question number on the left hand
side of the Objective Response Sheet (ORS). You may use the answer book
provided for any rough work, if needed. 2Ω

2.1 The voltage e0 in figure 2.1 is 16V +


(a) 48 V (b) 24 V 8A 10Ω 12Ω e0
6Ω
(c) 36 V (d) 28 V -

2.2. In figure 2.2, the value of the load resistor R which maximizes the power
delivered to it is 10Ω 1H
(a) 14.14Ω
(b) 10Ω
(c) 200Ω
Emcos10t ~
(d) 28.28Ω R

Join All India Mock GATE Classroom Test Series - 2007 conducted by GATE Forum in over 25 cities all over India. Question
Papers including section tests and full tests are designed by IISc alumni according to the latest syllabus. Percentile, All India Rank,
interaction with IISc alumni in our online discussion forums, and more. For more details,
visit
www.gateforum.com
Think GATE Think GATE Forum
GATE EC - 2001 www.gateforum.com
Join discussion of this test paper at http://forum.gatementor.com

2.3. When the angular frequency ω in Figure 2.3 is varied from 0 to ∞, the locus of the
current phasor I2 is given by
i(t)

i1(t) i2(t)
16V
Emcosωt ~ R1 R2

(a) ω=0 (b)


Em ω=0 E = Em ∠0°
I2 2R2
ω=∞ Em
I2
2R2
E = Em ∠0°
Em Em ω=∞
2R2 2R2
(c) (d)
Em ω=0 E = Em ∠0°
I2
ω=0 R1
ω=∞ Em
2R2
E = Em ∠0°
Em
Em Em Em I2
2R2
R1 2R2 2R2
ω=∞

2.4 The Z parameters Z11 and Z21 for the 2-port network in figure 2.4 are
6 16 2Ω I2
(a) Z11 = − Ω; Z21 = ∞ Ω; I1
11 11
4Ω
6 4
(b) Z11 = Ω; Z21 = Ω; E1
11 11 E2
10E1
6 16
(c) Z11 = Ω; Z21 = − Ω;
11 11
4 4
(d) Z11 = Ω; Z21 = Ω;
11 11

2.5 An npn BJT has gm = 38m A/V, Cµ=10-14 F, Cπ=4×10-13F, and DC current gain β 0 =
90. for this transistor fT and fβ are
(a) fT = 1.64 × 108 Hz and fβ = 1.47 × 1010 Hz

(b) fT = 1.47 × 1010 Hz and f β = 1.64 × 108 Hz

Join All India Mock GATE Classroom Test Series - 2007 conducted by GATE Forum in over 25 cities all over India. Question
Papers including section tests and full tests are designed by IISc alumni according to the latest syllabus. Percentile, All India Rank,
interaction with IISc alumni in our online discussion forums, and more. For more details,
visit
www.gateforum.com
Think GATE Think GATE Forum
GATE EC - 2001 www.gateforum.com
Join discussion of this test paper at http://forum.gatementor.com

(c) fT = 1.33 × 1012 Hz and fβ = 1.47 × 1010 Hz

(d) fT = 1.47 × 1010 Hz and f β = 1.33 × 1012 Hz

2.6 The transistor shunt regulator


shown in Figure 2.6 has a 20Ω
regulated output voltage of 10
V, when the input varies from IZ IC
20 V to 30 V. The relevant
parameters for the Zener diode VZ
and the transistor are: Vz = Vin=20-30V VO=10V
9.5, VSE=0.3V, β=99. Neglect VBE+
-
the current through RB. Then RB
the maximum power dissipated
in the Zener diode (Pz) and the
transistor (PT) are
(a) PZ=75mW, PT=7.9W (b) PZ=85mW, PT=8.9W
(c) PZ=95mW, PT=9.9W (d) PZ=115mW, PT=11.9W

2.7 The oscillator circuit shown in Figure 2.7 is


-VCC

LC

L=10µH
CC R1 Vo

C1 = 2pF C2= 2pF

R2 Re Ce

(a) Hartley oscillator with foscillation = 79.6MHz


(b) Colpitts oscillator with foscillation = 79.6MHz
(c) Hartley oscillator with foscillation = 159.2MHz
(d) Colpitts oscillator with foscillation = 159.2MHz

R2=10K
2.8 The inverting OP-AMP shown in Figure
R1=1K
2.8 has an open-loop gain of 100. The - Vo
v Vi + +
closed loop gain 0 is Vs
-
vs

(a) -8 (b) -9 (c) -10 (d) -11


Join All India Mock GATE Classroom Test Series - 2007 conducted by GATE Forum in over 25 cities all over India. Question
Papers including section tests and full tests are designed by IISc alumni according to the latest syllabus. Percentile, All India Rank,
interaction with IISc alumni in our online discussion forums, and more. For more details,
visit
www.gateforum.com
Think GATE Think GATE Forum
GATE EC - 2001 www.gateforum.com
Join discussion of this test paper at http://forum.gatementor.com

2.9 In Figure 2.9, assume the OP-AMPs to be ideal. The output v0 of the circuit is:
10mH
10µF

10Ω 100Ω
Vs -
-
Vs=10cos(100t) VO
+
+

t
(a) 10 cos(100t) ∫
(b) 10 cos (100τ ) dτ
0

t
d

(c) 10−4 cos (100τ ) dτ
0
(d) 10−4
dt
cos (100t )

2.10 In Figure 2.10, the LED


VCC=5V

1K 1K 1K
LED

1K

S1 S2

(a) emits light when both S1 and S2 are closed.


(b) emits light when both S1 and S2 are open.
(c) emits light when only S1 or S2 is closed.
(d) does not emit light, irrespective of the switch positions.

2.11 In the TTL circuit in Figure 2.11, S2 to S0 are select lines and X7 and X0are input
lines. S0 and X0 are LSBs. The output Y is
(a) indeterminate (b) A ⊕ B

(c) A ⊕ B ( )
(d) C. A ⊕ B + C. ( A ⊕ B )

Join All India Mock GATE Classroom Test Series - 2007 conducted by GATE Forum in over 25 cities all over India. Question
Papers including section tests and full tests are designed by IISc alumni according to the latest syllabus. Percentile, All India Rank,
interaction with IISc alumni in our online discussion forums, and more. For more details,
visit
www.gateforum.com
Think GATE Think GATE Forum
GATE EC - 2001 www.gateforum.com
Join discussion of this test paper at http://forum.gatementor.com

x0 x1 x2 x3 x4 x5 x6 x7

C S2
B S1

A S0

Fig.2.11
y

2.12 The digital block in figure 2.12 is realized using two positive edge triggered D-
flip-flops. Assume that for t < t0, Q1 = Q2 =0. The circuit in the digital block is
given by:

X Y
DIGITAL
BLOCK

t2 t3 t4
t0 t1 t2 t3 t0 t1

(a) Figure 2.12 (a) (b) Figure 2.12 (b)


(c) Figure 2.12 (c) (d) Figure 2.12 (d)

1 D1 Q1 1 D2 Q2 Y 1 D1 Q1 1 D2 Q2 Y
O
X Q1 Q2 X O Q1 Q2

Figure (a) Figure (b)

1 D1 Q1 D2 Q2 Y 1 D1 Q1 D2 Q2 Y

X X O O
Q1 Q2 Q1 Q2

Figure (c) Figure (d)

Join All India Mock GATE Classroom Test Series - 2007 conducted by GATE Forum in over 25 cities all over India. Question
Papers including section tests and full tests are designed by IISc alumni according to the latest syllabus. Percentile, All India Rank,
interaction with IISc alumni in our online discussion forums, and more. For more details,
visit
www.gateforum.com
Think GATE Think GATE Forum
GATE EC - 2001 www.gateforum.com
Join discussion of this test paper at http://forum.gatementor.com

2.13 In the DRAM cell in Figure 2.13, the Vt of the NMOSFET is 1 V. For the following
three combinations of WL and BL voltages.
Word Line (WL)

Bit Line (BL)


C

(a) 5 V; 3V; 7V (b) 4 V; 3V; 4V (c) 5 V; 5V; 5V (d) 4 V; 4V; 4V

2.14 The impulse response functions of four linear systems S1, S2, S3, S4 are given
respectively by
h1 ( t ) = 1
h2 ( t ) = U ( t )
U (t )
h3 ( t ) =
t +1
h4 ( t ) = e−3t U ( t )
where U(t) is the unit step function. Which of these systems is time invariant,
causal, and stable?
(a) S1 (b) S2 (c) S3 (d) S4

2.15 An electrical system and its signal-flow graph representations are shown in Figure
2.15(a) and 2.15(b) respectively. The values of G2 and H, respectively are
Z3 ( s ) − Z3 ( s )
(a) ,
Z2 ( s ) + Z3 ( s ) + Z4 ( s ) Z1 ( s ) + Z3 ( s )

− Z3 ( s ) − Z3 ( s )
(b) ,
Z2 ( s ) − Z3 ( s ) + Z4 ( s ) Z1 ( s ) + Z3 ( s )

Z3 ( s ) Z3 ( s )
(c) ,
Z2 ( s ) + Z3 ( s ) + Z4 ( s ) Z1 ( s ) + Z3 ( s )

− Z3 ( s ) Z3 ( s )
(d) ,
Z2 ( s ) − Z3 ( s ) + Z4 ( s ) Z1 ( s ) + Z3 ( s )

Join All India Mock GATE Classroom Test Series - 2007 conducted by GATE Forum in over 25 cities all over India. Question
Papers including section tests and full tests are designed by IISc alumni according to the latest syllabus. Percentile, All India Rank,
interaction with IISc alumni in our online discussion forums, and more. For more details,
visit
www.gateforum.com
Think GATE Think GATE Forum
GATE EC - 2001 www.gateforum.com
Join discussion of this test paper at http://forum.gatementor.com

Z1(s) Z2(s) Vo(s)

I1(s) I2(s)
Figure (b)
Z3(s) Z4(s)
Vi(s)
G1 I1(s) G2 I2(s) G3

Vi(s) Vo(s)
Figure (a)

2.16 The open-loop DC gain of a unity negative feedback system with closed-loop
s+4
transfer function 2 is
s + 7s + 13
4 4
(a) (b) (c) 4 (d) 13
13 9

2.17 The feedback control system in Figure 2.17 is stable


(a) for all K ≥ 0 S −2
R(s) + C(s)
(b) only if K ≥1 K≥0 2

-
( S + 2)
(c) only if 0 ≤ K < 1
(d) only if 0 ≤ K ≤ 1 SK - 2

2.18 A video transmission system transmits 625 picture frames per second. Each
frame consists of a 400 × 400 pixel grid with 64 intensity levels per pixel. The
data rate of the system is
(a) 16 Mbps (b) 100 Mbps (c) 600 Mbps (d) 6.4 Gbps

2.19 The Nyquist sampling interval, for the signal Sinc (700t) + Sinc (500t) is
1 π 1 π
(a) sec (b) sec (c) sec (d) sec
350 350 700 175

2.20 During transmission over a communication channel, bit errors occur


independently with probability p. If a block of n bits is transmitted, the probability
of at most one bit error is equal to
(a) 1 – (1 – p)n (b) p + (n – 1) ( 1 – p)
n-1
(c) np(1 – p) (d) (1 – p)n + np(1 – p)n-1

Join All India Mock GATE Classroom Test Series - 2007 conducted by GATE Forum in over 25 cities all over India. Question
Papers including section tests and full tests are designed by IISc alumni according to the latest syllabus. Percentile, All India Rank,
interaction with IISc alumni in our online discussion forums, and more. For more details,
visit
www.gateforum.com
Think GATE Think GATE Forum
GATE EC - 2001 www.gateforum.com
Join discussion of this test paper at http://forum.gatementor.com

2.21 The PSD and the power of a signal g(t) are, respectively, Sg(ω) and Pg. The PSD
and the power of the signal ag(t) are, respectively
(a) a2 Sg (ω ) and a2 Pg (b) a2 Sg (ω ) and aPg

(c) aSg (ω ) and a2 Pg (d) aSg (ω ) and aPg

2.22 A material has conductivity of 10-2 mho/m and a relative permittivity of 4. The
frequency at which the conduction current in the medium is equal to the
displacement current is
(a) 45 MHz (b) 90 MHz (c) 450 MHz (d) 900 MHz

2.23 A uniform plane electromagnetic wave incident normally on a plane surface of a


dielectric material is reflected with a VSWR of 3. What is the percentage of
incident power that is reflected?
(a) 10% (b) 25% (c) 50% (d) 75%

2.24 A medium wave radio transmitter operating at a wavelength of 492 m has a


tower antenna of height 124m. What is the radiation resistance of the antenna?
(a) 25Ω (b) 36.5Ω (c) 50Ω (d) 73Ω

λ
2.25 In a uniform linear array, four isotropic radiating elements are spaced apart.
4
The progressive phase shift between the elements required for forming the main
beam at 60° off the end-fire is:
π π π
(a) - π radians (b) − radians (c) − radians (d) − radians
2 4 8

SECTION – B
This section consists of TWENTY questions of FIVE marks each. Attempt ANY FIFTEEN
questions. Answers must be given in the answer book provided.

3. For the circuit shown in figure 3, determine the phasors E2, E0, I and I1.
1Ω
0.5Ω j2Ω
I E1

I1

E1=10∠20°
-jΩ 0.5Ω E0
~

1Ω

Join All India Mock GATE Classroom Test Series - 2007 conducted by GATE Forum in over 25 cities all over India. Question
Papers including section tests and full tests are designed by IISc alumni according to the latest syllabus. Percentile, All India Rank,
interaction with IISc alumni in our online discussion forums, and more. For more details,
visit
www.gateforum.com
Think GATE Think GATE Forum
GATE EC - 2001 www.gateforum.com
Join discussion of this test paper at http://forum.gatementor.com

4. The circuit shown in Figure 4 is operating in steady-state with switch S1 closed.


The switch S1 is opened at t = 0.
(a) Find iL(0+).
(b) Find e1(0+).
(c) Using nodal equations and Laplace transform approach, find an expression
for the voltage across the capacitor for all t > 0.
2Ω 2Ω 5Ω
e1(t) e2(t)

S1
iL(t)

15V 1 L=0.5H 10V


C= F
8

5. The admittance parameters of a 2-port network shown in figure 5 are given by


Y11 = 2 mho, Y12 = -0.5 mho, Y21 = 4.8 mho, Y22 = 1 mho. The output port is
terminated with a load admittance YL = 0.2 mho. Find E2 for each of the following
conditions?
(a) E1=10∠0° V
(b) I1=10∠0° A
(c) A source 10∠0° V in series with a 0.25Ω resistor is connected to the input
port.

6. For the circuit shown in figure 6, D1 and D2 are indentical diodes with idealilty
factor of unity. The thermal voltage VT = 25 mV.
(a) Calculate Vf and Vr.
(b) If the reverse saturation current, Is, for the diode is 1 pA, then compute the
current I through the circuit.

D1 D2

+ - + -
Vf Vr

50mV

Join All India Mock GATE Classroom Test Series - 2007 conducted by GATE Forum in over 25 cities all over India. Question
Papers including section tests and full tests are designed by IISc alumni according to the latest syllabus. Percentile, All India Rank,
interaction with IISc alumni in our online discussion forums, and more. For more details,
visit
www.gateforum.com
Think GATE Think GATE Forum
GATE EC - 2001 www.gateforum.com
Join discussion of this test paper at http://forum.gatementor.com

7. An emitter-follower amplifier is shown in Figure 7. Zi is the impedance looking


into the base of the transistor and Z0 is the impedance looking into the emitter of
the transistor.
(a) Draw the small signal equivalent circuit of the amplifier.
(b) Obtain an expression for Zi.
(c) Obtain an expression for Z0.
(d) Determined Zi and Z0 if a capacitor CL is connected across RL.

VCC

IC

RS

Zo
Vo
+ Zi
VS RL
-

-VEE

8. Assume that the OP-AMP in Figure 8 is ideal.


(a) obtain an expression for vo in terms of vs, R, and the reverse saturation
current Is of the transistor.
(b) If R = 1Ω, Is = 1 pA and the thermal voltage VT = 25 mV, then what is the
value of the output voltage vo for an input voltage vs = 1V?
(c) Suppose that the transistor in the feedback path is replaced by a p-n junction
diode with a reverse saturation current of Is. The p-side of the diode is
connected to node A and the n-side to node B. Then what is the expression
for vo in terms of vs, R and Is?

R
A - B +
VS + Vo
+ -
-

9. A monochrome video signal that ranges from 0 to 8V, is digitized using an 8-bit
ADC.
(a) Determine the resolution of the ADC in V/bit.
(b) Calculate the mean squared quantization error.
(c) Suppose the ADC is counter controlled. The counter is up count and positive
edge triggered with clock frequency 1 MHz. What is the time taken in
seconds to get a digital equivalent of 1.59 V?
Join All India Mock GATE Classroom Test Series - 2007 conducted by GATE Forum in over 25 cities all over India. Question
Papers including section tests and full tests are designed by IISc alumni according to the latest syllabus. Percentile, All India Rank,
interaction with IISc alumni in our online discussion forums, and more. For more details,
visit
www.gateforum.com
Think GATE Think GATE Forum
GATE EC - 2001 www.gateforum.com
Join discussion of this test paper at http://forum.gatementor.com

10. In figure 10, the output of the oscillator, V1 has 10V peak amplitude with zero DC
value. The transfer characteristic of the Schmitt inverter is also shown in figure
10. Assume that the JK flip-flop is resent at time t = 0.
(a) What is the period and duty cycle of the waveform V2?
(b) What is the period and duty cycle of the waveform V3?
(c) Sketch V1, V2 and V3 for the duration 0 ≤ t ≤ 6µs. Clearly indicate the exact
timings when the waveforms V2 and V3 make high-to-low and low-to-high
transitions.
VCC=5V

J Q

0.5 MHz V1 V2
sinewave V2
oscillator
K
5V

oscillator Schmitt J-K flip-flop


inverter

0V
V1
2.5V 3.5V

11. For the digital block shown in Figure 11(a), the output Y=f(S3,S2,S1,S0) where S3
is MSB and S0 is LSB. Y is given in terms of minterms as
Y = ∑ m (1,5, 6,7,11,12,13,15) and its complement is

Y = ∑ m ( 0, 2, 3, 4, 8, 9,10,14 )

(a) Enter the logical values in the given Karnaugh map [Fig.11(b)] for the output
Y.
(b) Write down the expression for Y in sum-of products from using minimum
number of terms
(c) Draw the circuit for the digital logic boxes using four 2-input NAND gates
only for each of the boxes.
S1S0
S1 S3 S2 S3S2 00 01 11 10
S0
00

DIGITAL DIGITAL Y 01
LOGIC LOGIC
BOX 1 BOX 2 11

10

Join All India Mock GATE Classroom Test Series - 2007 conducted by GATE Forum in over 25 cities all over India. Question
Papers including section tests and full tests are designed by IISc alumni according to the latest syllabus. Percentile, All India Rank,
interaction with IISc alumni in our online discussion forums, and more. For more details,
visit
www.gateforum.com
Think GATE Think GATE Forum
GATE EC - 2001 www.gateforum.com
Join discussion of this test paper at http://forum.gatementor.com

12. Consider the following sequence of instructions for an 8085 microprocessor based
system.

Memory address Instructions


FF00 MVI, A FF H
FF02 INR A
FF03 JC FF0C H
FF06 ORI A8H
FF08 JM FF15 H
FF0B XRA A
FF0C OUT PORT 1
FF0E HLT
FF10 XRI FF H
FF12 OUT PORT 2
FF14 HLT
FF15 MVI, A FF H
FF17 ADI 02 H
FF19 RAL
FF1A JZ FF23 H
FF1D JC FF10 H
FF20 JNC FF12 H
FF23 CMA
FF24 OUT PORT 3
FF26 HLT

(a) If the program execution begins at the location FF00 H, write down the
sequence of instructions which are actually executed till a HLT instruction.
(Assume all flags are initially RESET)
(b) Which of the three ports (PORT1, PORT2 and PORT3) will be loaded with data
and what is the bit pattern of the data?

13. A feedback control system is shown in figure 13.


(a) Draw the signal-flow graph that represents the system.
(b) Find the total number of loops in the graph and determine the loop-gains of
all the loops.
(c) Find the number of all possible combination of non-touching loops taken two
at a time.
(d) Determine the transfer function of the system using the signal-flow graph.

Join All India Mock GATE Classroom Test Series - 2007 conducted by GATE Forum in over 25 cities all over India. Question
Papers including section tests and full tests are designed by IISc alumni according to the latest syllabus. Percentile, All India Rank,
interaction with IISc alumni in our online discussion forums, and more. For more details,
visit
www.gateforum.com
Think GATE Think GATE Forum
GATE EC - 2001 www.gateforum.com
Join discussion of this test paper at http://forum.gatementor.com

+
+ + +
G1 G2 C(s)
R(s) -
- -

Fig.13

14. Consider the feedback control system shown in figure 14.


(a) Find the transfer function of the system and its characteristic equation.
(b) Use the Routh-Hurwitz criterion to determine the range of K for which the
system is stable.

+ +
R(s) K ( s + 1) 1 C(s)
- s - 2

1
s + 0.1

15. For the feedback control system shown in figure 15, the process transfer function
1
is Gp ( s ) = , and the complification factor of the power amplifier is K ≥ 0.
s ( s + 1)
The design specifications required for the system are a time constant of 1 sec
and a damping ratio of 0.707.
(a) Find the desired locations of the closed loop poles.
(b) Write down the required characteristic equation for the system. Hence
determine the PD controller transfer function G0 ( s ) when K = 1.

(c) Sketch the root-locus for the system.

R(s) + PD Power C(s)


Process
controller amplifier
-

Join All India Mock GATE Classroom Test Series - 2007 conducted by GATE Forum in over 25 cities all over India. Question
Papers including section tests and full tests are designed by IISc alumni according to the latest syllabus. Percentile, All India Rank,
interaction with IISc alumni in our online discussion forums, and more. For more details,
visit
www.gateforum.com
Think GATE Think GATE Forum
GATE EC - 2001 www.gateforum.com
Join discussion of this test paper at http://forum.gatementor.com

16. The Fourier transform G(ω) of the signal g(t) in Figure 16(a) is given as
1
( )
G (ω ) = 2 e jω − jω e jω − 1 . Using this information and the time-shifting and
ω
time-scaling properties, determine the Fourier transform of signals in Figures
16(b), 16(c) and 16(d).

g(t) g1(t)

1 1

t t
-1 0 0 1

g2(t) g2(t)

1 1

t t
-1 0 1 01
-
2 2

17. The periodic modulating signal m(t) is shown in Fig.17. Using Carson’s rule
estimate BFM (bandwidth of the FM signal) and BPM (bandwidth of the PM signal)
π
for Kf = π × 104 and kp = . Assume the essential bandwidth of m(t) to consist
4
only up to and including the third harmonic.

m(t)

2×10-4
1

t
(sec)

-1

Join All India Mock GATE Classroom Test Series - 2007 conducted by GATE Forum in over 25 cities all over India. Question
Papers including section tests and full tests are designed by IISc alumni according to the latest syllabus. Percentile, All India Rank,
interaction with IISc alumni in our online discussion forums, and more. For more details,
visit
www.gateforum.com
Think GATE Think GATE Forum
GATE EC - 2001 www.gateforum.com
Join discussion of this test paper at http://forum.gatementor.com

18. A baseband signal g(t) bandlimited to 100 Hz modulates a carrier of frequency f0


Hz. The modulated singal g(t)cos 2πf t is transmitted over a channel whose input
x and output y are related by y = 2x + x2. The spectrum of g(t) is shown in
Figure 18. Sketch the spectrum of the transmitted signal and the spectrum of the
received signal.
G(f)

f(Hz)
-100 0 100

19. A periodic signal g(t) is shown in Figure 19. Determine the PSD of g(t).

g(t)

t
π π 2π sec
-2π - 4π
2 2

20. A system of three electric charges lying in a straight line is in equilibrium. Two of
the charges are positive with magnitudes Q and 2Q, and are 50 cm apart.
Determine the sign, magnitude and position of the third charge.

21. A medium has breakdown strength of 16 KV/m r.m.s. Its relative permeability is
1.0 and relative permittivity is 4.0 A plane electromagnetic wave is transmitted
through the medium. Calculate the maximum possible power flow density and the
associated magnetic filed.

22. A rectangular hollow metal waveguide has dimensions a= 2.29 cm and b = 1.02
cm. Microwave power at 10 GHz is transmitted through the waveguide in the TE10
mode.
(a) Calculate the cut-off wavelength and the guide wavelength for this mode.
(b) What are the other (TE or TM) modes that can propagate through the
waveguide?
(c) If a = b = 2.29cm, What are the modes which can propagate through the
waveguide?
Join All India Mock GATE Classroom Test Series - 2007 conducted by GATE Forum in over 25 cities all over India. Question
Papers including section tests and full tests are designed by IISc alumni according to the latest syllabus. Percentile, All India Rank,
interaction with IISc alumni in our online discussion forums, and more. For more details,
visit
www.gateforum.com
Think GATE Think GATE Forum
GATE EC - 2002 www.gateforum.com
Join discussion of this test paper at http://forum.gatementor.com

SECTION – A (75 marks)


1. This question consists of TWENTY-FIVE sub-questions (1.1 – 1.25) of ONE mark
each. For each of these sub-questions, four possible alternatives (A,B, C and D)
are given, out of which ONLY ONE is correct. Indicate the correct answer by
darkening the appropriate bubble against the question number on the left hand
side of the Objective Response Sheet (ORS). You may use the answer book
provided for any rough work, if needed.

5Ω
1.1 The dependent current source shown in Figure 1.1
(a) delivers 80 W
(b) absorbs 80 W +
I V1
V1=20V 5Ω A
(c) delivers 40 W - 5
(d) absorbs 40 W

1.2 In figure 1.2, the switch was closed for a long time before opening at t = 0. the
voltage Vx at t = 0+ is t=0
(a) 25 V
(b) 50 V
2.5A
(c) -50 V
20Ω
(d) 0 V
5H

20Ω
- +
Vx

1.3 Convolution of x(t + 5) with impulse function δ(t – 7) is equal to


(a) x(t - 12) (b) x(t + 12) (c) x(t - 2) (d) x(t + 2)

1.4 Which of the following cannot be the Fourier series expansion of a periodic
signal?
(a) x(t) = 2cos t + 3 cos 3t (b) x(t) = 2cos πt + 7 cos t
(c) x(t) = cos t + 0.5 (d) x(t) = 2cos 1.5πt + sin 3.5 πt

1.5 In Figure 1.5, a silicon is carrying a constant current of 1 mA. When the
temperature of the diode is 20°C, VD is found to be 700 mV. If the temperature
rises to 40°C, VD becomes approximately equal to
(a) 740 mV +

(b) 660 mV 1mA VD


(c) 680 mV
(d) 700 mV -

Join All India Mock GATE Classroom Test Series - 2007 conducted by GATE Forum in over 25 cities all over India. Question
Papers including section tests and full tests are designed by IISc alumni according to the latest syllabus. Percentile, All India Rank,
interaction with IISc alumni in our online discussion forums, and more. For more details,
visit
www.gateforum.com
Think GATE Think GATE Forum
GATE EC - 2002 www.gateforum.com
Join discussion of this test paper at http://forum.gatementor.com

1.6 In a negative feedback amplifier using voltage-series (i.e. voltage-sampling


series mixing) feedback. (Ri and Ro denote the input and output resistance
respectively)
(a) Ri decreases and Ro decreases (b) Ri decreases and Ro increases
(c) Ri increases and Ro decreases (d) Ri increases and Ro increases

1.7 A 741-type op-amp has a gain-bandwidth product of 1 MHz. A non-inverting


amplifier using this op-amp and having a voltage gain of 20 dB will exhibit a 3-dB
bandwidth of
1000 1000
(a) 50 KHz (b) 100 KHz (c) KHz (d)
17 7.07

1.8 Three identical RC-coupled transistor amplifiers are cascaded. If each of the
amplifiers has a frequency response as shown in Figure 1.8, the overall frequency
response is as given in
Av dB

0
-3

f
20Hz 1KHz

Av dB Av dB

(a) 0 (b) 0
-3 -3
2
f f
40Hz 0.5KHz 40Hz 1KHz

Av dB
(c) (d) Av dB

0
-3 0
-3
f
40Hz 2KHz f
10Hz 0.5KHz

1.9 4-bit 2’s complement representation of a decimal number is 1000. The number is
(a) +8 (b) 0 (c) -7 (d) -8

Join All India Mock GATE Classroom Test Series - 2007 conducted by GATE Forum in over 25 cities all over India. Question
Papers including section tests and full tests are designed by IISc alumni according to the latest syllabus. Percentile, All India Rank,
interaction with IISc alumni in our online discussion forums, and more. For more details,
visit
www.gateforum.com
Think GATE Think GATE Forum
GATE EC - 2002 www.gateforum.com
Join discussion of this test paper at http://forum.gatementor.com

1.10 If the input to the digital circuit (Figure 1.10) consisting of a cascade of 20 XOR-
gates is X, then the output Y is equal to
1

(a) 0 (b) 1 (c) X (d) X

1.11 The number of comparators required in a 3-bit comparator type ADC is


(a) 2 (b) 3 (c) 7 (d) 8

1.12 If the transistor in Figure 1.12 is in saturation, then

C
IC

IB

βdc denotes the dc


current gain

(a) IC is always equal to β dcIB (b) IC is always equal to -β dcIB


(c) IC is greater than or equal to βdcIB (d) IC is less than or equal to βdcIB

s+6
1.13 Consider a system with the transfer function G ( s ) = 2
. Its damping ratio
ks + s + 6
will be 0.5 when the value of k is
2 1
(a) (b) 3 (c) (d) 6
6 6

1.14 Which of the following points is NOT on the root locus of a system with the open
k
loop transfer function G ( s ) H ( s ) =
s ( s + 1) ( s + 3 )

(a) s = − j 3 (b) s= - 1.5 (c) s= -3 (d) s = -∞

Join All India Mock GATE Classroom Test Series - 2007 conducted by GATE Forum in over 25 cities all over India. Question
Papers including section tests and full tests are designed by IISc alumni according to the latest syllabus. Percentile, All India Rank,
interaction with IISc alumni in our online discussion forums, and more. For more details,
visit
www.gateforum.com
Think GATE Think GATE Forum
GATE EC - 2002 www.gateforum.com
Join discussion of this test paper at http://forum.gatementor.com

1.15 The phase margin of a system with the open-loop transfer function

G (s) H (s) =
(1 − s ) is
(1 + s ) (2 + s )
(a) 0° (b) 63.4° (c) 90° (d) ∞

1.16 The transfer function Y ( s ) U ( s ) of a system described by the state equations


x(t) = -2x(t) + 2u(t) and y(t) = 0.5x(t) is
0.5 1 0.5 1
(a) (b) (c) (d)
( − 2)
s ( − 2)
s ( + 2)
s ( + 2)
s

1  1 
1.17 {
The Fourier transform F e −1u ( t ) is equal to } 1 + j2π f
. Therefore, F 
 1 + j 2π t
 is

(a) ef u ( f ) (b) e − f u ( f ) (c) ef u ( −f ) (d) e − f u ( −f )

1.18 A linear phase channel with phase delay TP and group delay Tg must have
(a) TP= Tg=constant (b) TP ∝f and Tg∝f
(c) TP = constant and Tg∝f (d) TP ∝f and Tg =constant

1.19. A 1 MHz sinusoidal carrier is amplitude modulated by a symmetrical square wave


of period 100µsec. Which of the following frequencies will NOT be present in the
modulated signal?
(a) 990 KHz (b) 1010 KHz (c) 1020 KHz (d) 1030 KHz


+∞
1.20. Consider a sampled signal y ( t ) = 5 × 10−6 x ( t ) δ ( t − nTs ) where x(t) =10
n = −∞
cos(8π×103)t and Ts =100µsec. When y(t) is passed through an ideal low-pass
filter with a cutoff frequency of 5KHz, the output of the filter is

(
(a) 5 × 10−6 cos 8π × 103 t ) (
(b) 5 × 10−5 cos 8π × 103 t )
(c) 5 × 10−1 cos ( 8π × 10 ) t3
(
(d) 10 cos 8π × 103 t )
1.21. For a bit-rate of 8 Kbps, the best possible values of the transmitted frequencies
in a coherent binary FSK system are
(a) 16 KHz and 20 KHz (b) 20 KHz and 32 KHz
(c) 20 KHz and 40 KHz (d) 32 KHz and 40 KHz

1.22. The line-of-sight communication requires the transmit and receive antenna to
face each other. If the transmit antenna is vertically polarized, for best reception
the receive antenna should be
(a) horizontally polarized (b) vertically polarized

Join All India Mock GATE Classroom Test Series - 2007 conducted by GATE Forum in over 25 cities all over India. Question
Papers including section tests and full tests are designed by IISc alumni according to the latest syllabus. Percentile, All India Rank,
interaction with IISc alumni in our online discussion forums, and more. For more details,
visit
www.gateforum.com
Think GATE Think GATE Forum
GATE EC - 2002 www.gateforum.com
Join discussion of this test paper at http://forum.gatementor.com

(c) at 45° with respect to horizontal polarization


(d) at 45° with respect to vertical polarization

1.23. The VSWR can have any value between


(a) 0 and 1 (b) -1 and +1 (c) 0 and ∞ (d) 1 and ∞

1.24. In an impedance Smith chart, a clockwise movement along a constant resistance


circle gives rise to
(a) a decrease in the value of reactance
(b) an increase in the value of reactance
(c) no change in the reactance value
(d) no change in the impedance value

1.25 The phase velocity for the TE10-mode in an air-filled rectangular waveguide is (c
is the velocity of plane waves in free space)
(a) less than c (b) equal to c
(c) greater than c (d) none of the above

2. This question consists of TWENTY-FIVE sub-questions (2.1 – 2.25) of TWO marks


each. For each of these sub-questions, four possible alternatives (A,B, C and D)
are given, out of which ONLY ONE is correct. Indicate the correct answer by
darkening the appropriate bubble against the question number on the left hand
side of the Objective Response Sheet (ORS). You may use the answer book
provided for any rough work, if needed.
2.1 In the network of Figure 2.1, the maximum power is delivered to RL if its value is

I1

40Ω

0.5I1 20Ω RL
50V

40
(a) 16Ω (b) Ω (c) 60Ω (d) 20Ω
3

Join All India Mock GATE Classroom Test Series - 2007 conducted by GATE Forum in over 25 cities all over India. Question
Papers including section tests and full tests are designed by IISc alumni according to the latest syllabus. Percentile, All India Rank,
interaction with IISc alumni in our online discussion forums, and more. For more details,
visit
www.gateforum.com
Think GATE Think GATE Forum
GATE EC - 2002 www.gateforum.com
Join discussion of this test paper at http://forum.gatementor.com

2.2. If the 3-phase balanced source in Figure 2.2 delivers 1500 W at a leading power
factor of 0.844, then the value of ZL (in ohm) is approximately
(a) 90∠32.44°
ZL ZL
(b) 80∠32.44°
400V
(c) 80∠ − 32.44° 3 PHASE
BALANCED ZL
(d) 90∠ − 32.44° SOURCE

5−s
2.3. The Laplace transform of a continuous-time signal x(t) is X ( s ) = 2
. If the
s −s−2
Fourier transform of this signal exists, then x(t) is
(a) e2t u ( t ) − 2e −t u ( t ) (b) −e2t u ( −t ) + 2e −t u ( t )

(c) −e2t u ( −t ) − 2e −t u ( t ) (d) e2t u ( −t ) − 2e −t u ( t )

2.4. If the impulse response of a discrete-time system is h  n = −5n u  −n − 1 , then
the system function H(z) is equal to
−z z
(a) and the system is stable (b) and the system is stable
z −5 z −5
−z z
(c) and the system is unstable (d) and the system is unstable
z −5 z −5

2.5. An amplifier using an opamp with a slew-rate SR = 1V/µsec has a gain of 40 dB.
If this amplifier has to faithfully amplify sinusoidal signals from dc to 20 KHz
without introducing any slew-rate induced distortion, then the input signal level
must not exceed.
(a) 795 mV (b) 395 mV (c) 79.5 mV (d) 39.5 mV

Join All India Mock GATE Classroom Test Series - 2007 conducted by GATE Forum in over 25 cities all over India. Question
Papers including section tests and full tests are designed by IISc alumni according to the latest syllabus. Percentile, All India Rank,
interaction with IISc alumni in our online discussion forums, and more. For more details,
visit
www.gateforum.com
Think GATE Think GATE Forum
GATE EC - 2002 www.gateforum.com
Join discussion of this test paper at http://forum.gatementor.com

2.6. The circuit in Figure 2.6 employs positive feedback and is intended to generate
Vf ( f ) 1
sinusoidal oscillation. If at a frequency f0 B ( f ) = ∆ = ∠0°, then to sustain
V0 ( f ) 6
oscillation at this frequency
R2

R1
-

+ +

V0(f)
+ Network B(f)
Vf(f)
-
-

R1 R1
(a) R2 = 5R1 (b) R2 = 6R1 (c) R2 = (d) R2 =
6 5
2.7. A zener diode regulator in Figure 2.7 is to be designed to meet the specifications:
IL = 10 mA, V0 = 10 V and Vin varies from 30 V to 50 V. The zener diode has
Vz = 10V and I zk (knee current) = 1 mA. For satisfactory opertion
R
(a) R ≤ 1800Ω IL=10mA
+ +
(b) 2000Ω ≤ R ≤ 2200Ω I2
Dz
(c) 3700Ω≤R≤4000Ω Vin Vo RL
(d) R>4000Ω
- -

VDD=+10V
Vo
2.8. The voltage gain A∆ = of the
Vi
JFET amplifier shown in Figure ID=1mA
RD(3KΩ)
2.8 is
C2
(a) +18
+
vo
(b) -18
C1
(c) +6
(d) -6 +

vi
RG(1MΩ) RS(2.5KΩ)
CS
-

Join All India Mock GATE Classroom Test Series - 2007 conducted by GATE Forum in over 25 cities all over India. Question
Papers including section tests and full tests are designed by IISc alumni according to the latest syllabus. Percentile, All India Rank,
interaction with IISc alumni in our online discussion forums, and more. For more details,
visit
www.gateforum.com
Think GATE Think GATE Forum
GATE EC - 2002 www.gateforum.com
Join discussion of this test paper at http://forum.gatementor.com

2.9. The gates G1 and G2 in Figure 2.9 have propagation delays of 10 nsec and 20
nsec respectively. If the input Vi makes an abrupt change from logic 0 to 1 at
time t = t0 then the output waveform V0 is
G1
G2
0

V0
1 Vi Vi
1

0 Vi
to

(a) (b)

1
1

0
0
t0 t1 t2 t3
t0 t1 t2 t3

(c) (d)
1 1

0 0
t0 t1 t2 t3 t0 t1 t2 t3

2.10. The circuit in Figure 2.10 has two CMOS NOR gates. This circuit functions as a:

R
VSS V0 (output)

VSS

0 Vi C

(a) flip flop (b) Schmitt trigger


(c) Monostable multivibrator (d) Astable multivibrator

Join All India Mock GATE Classroom Test Series - 2007 conducted by GATE Forum in over 25 cities all over India. Question
Papers including section tests and full tests are designed by IISc alumni according to the latest syllabus. Percentile, All India Rank,
interaction with IISc alumni in our online discussion forums, and more. For more details,
visit
www.gateforum.com
Think GATE Think GATE Forum
GATE EC - 2002 www.gateforum.com
Join discussion of this test paper at http://forum.gatementor.com

2.11. Consider the following statements in connection with the CMOS inverter in Figure
2.11 where both the MOSFETs are of enhancement type and both have a thresh
old voltage of 2V.
Statement 1: T1 conducts when Vi ≥ 2V.
Statement 2: T1is always in saturation when V0 = 0V.
+5V

T2

VI V0

T1

Which of the following is correct?


(a) Only Statement 1 is TRUE (b) Only Statement 2 is TRUE
(c) Both the statements are TRUE (d) Both the statements are FALSE
2.12. If the input X3 , X2 , X1 , X 0 to the ROM in figure 2.12 are 8-4-2-1 BCD numbers,
then the outputs Y3Y2 Y1Y0 are
1 0 1 0
X3 X2 X1 X0

ROM

BCD-to-Decimal DECODER

D0 D1 ………………………….. D8 D9
0 1

X X X X Y3
X X X X X X Y2
X X X X Y1
X X X X X Y0

(a) gray code numbers (b) 2-4-2-1 BCD numbers


(c) excess 3 code numbers (d) none of the above
Join All India Mock GATE Classroom Test Series - 2007 conducted by GATE Forum in over 25 cities all over India. Question
Papers including section tests and full tests are designed by IISc alumni according to the latest syllabus. Percentile, All India Rank,
interaction with IISc alumni in our online discussion forums, and more. For more details,
visit
www.gateforum.com
Think GATE Think GATE Forum
GATE EC - 2002 www.gateforum.com
Join discussion of this test paper at http://forum.gatementor.com

2.13. Consider the following assembly language program.


MVI B,87H
MOV A,B
START: JMP NEXT
MVI B,00H
XRA B
OUT PORT1
HLT
NEXT: XRA B
JP START
OUT PORT2
HLT
The execution of the above program in an 8085 microprocessor will result in
(a) an output of 87H at PORT1
(b) an output of 87H at PORT2
(c) infinite looping of the program execution with accumulator data remaining at
00H
(d) infinite looping of the program execution with accumulator data alternating
between 00H and 87H

2.14. The system shown in Figure 2.14 remains stable when

R(s)
1 k S-1 1
Y(s)

(a) k < - 1 (b) - 1 < k < 1 (c) 1 < k < 3 (d) k > 3

100
2.15. The transfer function of a system is G ( s ) = . For a unit step input
( s + 1) ( s + 100 )
to the system the approximate settling time for 2% criterion is
(a) 100 sec (b) 4 sec (c) 1 sec (d) 0.01 sec

Join All India Mock GATE Classroom Test Series - 2007 conducted by GATE Forum in over 25 cities all over India. Question
Papers including section tests and full tests are designed by IISc alumni according to the latest syllabus. Percentile, All India Rank,
interaction with IISc alumni in our online discussion forums, and more. For more details,
visit
www.gateforum.com
Think GATE Think GATE Forum
GATE EC - 2002 www.gateforum.com
Join discussion of this test paper at http://forum.gatementor.com

2.16. The characteristic polynomial of a system is q ( s ) = 2s5 + s 4 + 4s3 + 2s2 + 2s + 1.


The system is
(a) stable (b) marginally stable (c) unstable (d) oscillatory

1
2.17. The system with the open loop transfer function G ( s ) H ( s ) = has a
( 2
s s + s +1 )
gain margin of
(a) - 6 dB (b) 0 dB (c) 3.5 dB (d) 6 dB

2.18. An angle modulated signal is given by

(
s ( t ) = cos 2π 2 × 106 t + 30 sin150t + 40 cos150t . )
The maximum frequency and phase deviations of s(t) are
(a) 10.5 KHz, 140π rad (b) 6 KHz, 80π rad
(c) 10.5 KHz, 100π rad (d) 7.5 KHz, 100π rad

2 sin 2π t sin199π t
2.19. In figure 2.19 m ( t ) = , s ( t ) = cos 200π t and n ( t ) = . The output
t t
y(t) will be
Lowpass
m(t) y(t)
filter

Cutoff frequency = 1Hz


Passband Gain =1
sin 2π t s(t) n(t) s(t) sin 2π t sin π t
(a) (b) + cos 3π t
t t t
sin 2π t sin 0.5π t sin 2π t sin π t
(c) + cos1.5π t (d) + cos 0.75π t
t t t t

2.20. A signal x(t) = 100 cos(24π×103)t is ideally sampled with a sampling period of 50
µsec and then passed through an ideal low-pass filter with cutoff frequency of 15
KHz. Which of the following frequencies is/are present at the filter output?
(a) 12 KHz only (b) 8 KHz only
(c) 12 KHz and 9 KHz (d) 12 KHz and 8 KHz

2.21. If the variance σ x2 of d ( n ) = x ( n ) − x ( n − 1) is one-tenth the variance σ x2 of a


stationary zero mean
discrete time signal x(n), then the normalized
Rxx ( k )
autocorrelation function 2
at k = 1 is
σx
(a) 0.95 (b) 0.90 (c) 0.10 (d) 0.05

Join All India Mock GATE Classroom Test Series - 2007 conducted by GATE Forum in over 25 cities all over India. Question
Papers including section tests and full tests are designed by IISc alumni according to the latest syllabus. Percentile, All India Rank,
interaction with IISc alumni in our online discussion forums, and more. For more details,
visit
www.gateforum.com
Think GATE Think GATE Forum
GATE EC - 2002 www.gateforum.com
Join discussion of this test paper at http://forum.gatementor.com

ur  jπ 
2.22. A plane wave is characterized by E =  0.5x
ˆ + ye
ˆ 2  e jω t − jkz . This wave is
 
 
(a) linearly polarized (b) circularly polarized
(c) elliptically polarized (d) unpolarized

2.23. Distilled water at 25° C is characterized by σ = 1.7 × 10-4 mho/m and ∈=78∈0 at
a frequency of 3 GHz. Its loss tangent tan δ is (∈ = 10-9 /(36π)F/m)
(a) 1.3 × 10-5 (b) 1.3 × 10-3
(c) 1.7 × 10-4/78 (d) 1.7 × 10-4/(78∈0)

2.24. The electric field on the surface of a perfect conductor is 2 V/m. The conductor is
immersed in water with ∈=80∈0. The surface charge density on the conductor is
(∈ = 10-9 /(36π)F/m)
(a) 0 C/m2 (b) 2 C/m2
(c) 1.8 × 10-11 C/m2 (d) 1.14 × 10-9 C/m2

2.25. A person with a receiver is 5 Km away from the transmitter. What is the distance
that this person must move further to detect a 3-dB decrease in signal strength?
(a) 942 m (b) 2070 m (c) 4978 m (d) 5320 m

SECTION – B (75 Marks)


This section consists of TWENTY questions (EC3-EC22) of FIVE marks each. Attempt ANY
FIFTEEN questions. Answers must be given in the answer book provided. Answer for
each question must start on a fresh page and must appear at one place only. (Answers
to all parts of a question must appear together).
3. The switch in Figure 3 has been in position 1 for a long time and is then moved to
position 2 at t = 0.
10u(-t)V
25kΩ t=0
+ -
1
IL
20kΩ 5H
+
4H 1kΩ
6V VC
20µF -

50kΩ

dVc ( t )
(a) Determine VC 0+ ( ) and IL 0+ ( ) (b) Determine
dt
at t = 0+

(c) Determine Vc(t) for t > 0


Join All India Mock GATE Classroom Test Series - 2007 conducted by GATE Forum in over 25 cities all over India. Question
Papers including section tests and full tests are designed by IISc alumni according to the latest syllabus. Percentile, All India Rank,
interaction with IISc alumni in our online discussion forums, and more. For more details,
visit
www.gateforum.com
Think GATE Think GATE Forum
GATE EC - 2002 www.gateforum.com
Join discussion of this test paper at http://forum.gatementor.com

4. For the network shown in Figure 4, R = 1 KΩ, L1 = 2H, L2= 5H, L3 = 1H, L4 = 4H
and C = 0.2 µF. The mutual inductances are M12 = 3H and M34 = 2H. Determine
(a) the equivalent inductance for the combination of L3 and L4.
(b) the equivalent inductance across the points A and B in the network.
(c) the resonant frequency of the network.
I1
L1 L3
R A

I2

Vs(t) ~ M12 L2 M34


L4

B
C

5. Consider the network in Figure 5.

10Ω

5Ω
1 2
Ix
I2+I1
PORT-1 PORT-2
10Ω 2Ix

1’ 2’
I2

(a) Find its short-circuit admittance parameters.


(b) Find the open-circuit impedance Z22.

6. A triangular voltage waveform Vi(t) figure 6(a) is applied at the input to the
circuit of Figure 6(b). Assume the diodes to be ideal.
(a) Determine the output V0(t).
(b) Neatly sketch the output waveform superimposed on the input Vi(t) and label
the key points. D1 0.5KΩ
+ +
Vi(t)

D2 1KΩ
12V
Vi(t) V0(t)
+ + 1.5KΩ
7V 5V
- -
0 t - -

Join All India Mock GATE Classroom Test Series - 2007 conducted by GATE Forum in over 25 cities all over India. Question
Papers including section tests and full tests are designed by IISc alumni according to the latest syllabus. Percentile, All India Rank,
interaction with IISc alumni in our online discussion forums, and more. For more details,
visit
www.gateforum.com
Think GATE Think GATE Forum
GATE EC - 2002 www.gateforum.com
Join discussion of this test paper at http://forum.gatementor.com

7. Figure 7 shows a 2-stage VCC=+10V


amplifier. The transistors Q1
and Q2 are identical with
current gain β = 100; further
βdc = β ac = β. The Zener diode
RC2 4KΩ
Dz has a break down voltage R 930Ω
Vz= 10.7 volt. Assume that Dz
C2
is in breakdown region and its
+
dynamic resistance rz is zero. Vo
RL 4KΩ
The capacitors C1 and C2 are Q2 -
+
large and provide negligible DZ VZ
impedance at signal - VC1
frequencies. VEE=-10V IC1

+ Q1
VS - C1
RB 930kΩ

(a) Identify the configuration


in each of the amplifier VEE=-10V
stages (i.e., whether CE,
CC, CB etc.)
(b) Determine the quiescent quantities IC1 and VC1.
V0
(c) Derive an expression for the voltage gain AV and determine its value.
Vs
(Assume VBE = 0.7V, r0=∞ and Thermal voltage VT = 25 mV)

8. Consider the circuit of Figure 8. The op-amp used is ideal.


VCC1=+15V

+5V

VCC2=+12V

R Vi RB
+ Vop
100KΩ - Q
150KΩ
VEE2=-12V

RF

10KΩ
IO
RL
10Ω

VEE1=-15V

Join All India Mock GATE Classroom Test Series - 2007 conducted by GATE Forum in over 25 cities all over India. Question
Papers including section tests and full tests are designed by IISc alumni according to the latest syllabus. Percentile, All India Rank,
interaction with IISc alumni in our online discussion forums, and more. For more details,
visit
www.gateforum.com
Think GATE Think GATE Forum
GATE EC - 2002 www.gateforum.com
Join discussion of this test paper at http://forum.gatementor.com

(a) In which mode is the BJT operating (i.e. active, saturation or cutoff)? Justify
your answer.
(b) Obtain an expression relating the output current I0 and the input voltage Vi.
(c) Determine I0 and V0P if Vi = 2 Volt (V0P:output of opamp)
(Assume β = 99 and VBE = 0.7 V)

9. The inputs to a digital circuit shown in Figure 9(a) are the external signals A, B
and C.

+5V

Table 9
A D0 S
S Y
DIGITAL
B X Y 0 D0
CIRCUIT
C D1 D1
1

( A, B and C are not available). The +5V power supply (logic 1) and the ground
(logic 0) are also available. The output of the circuit is X = AB + A B C .
(a) Write down the output function in its canonical SOP and POS forms.
(b) Implement the circuit using only two 2:1 multiplexers shown in Figure 9(b),
where S is the data-select line, D0 and D1 are the input data lines and Y is
the output lines. The function table for the multiplexer is given in table 9.

10. Each transistor in Figure 10 has dc current gain β dc = 50, cut-in voltage
Vγ = 0.65V and VBE, sat = 0.75 V. The output voltage V0 for T2 in saturation can be
as high as 0.2 V. Assume 0.7 V drop across a conducting p-n junction.
Determine?

RC2=1kΩ

RB1 Vo(output)

IB2
Vi(input)
T2
T1

Rb=15kΩ

-
2V
+

Join All India Mock GATE Classroom Test Series - 2007 conducted by GATE Forum in over 25 cities all over India. Question
Papers including section tests and full tests are designed by IISc alumni according to the latest syllabus. Percentile, All India Rank,
interaction with IISc alumni in our online discussion forums, and more. For more details,
visit
www.gateforum.com
Think GATE Think GATE Forum
GATE EC - 2002 www.gateforum.com
Join discussion of this test paper at http://forum.gatementor.com

(a) the minimum value IB2 necessary to keep T2 saturation.


(b) the maximum permissible value for the resistance RB1.
(c) the worst-case high input (logic 1) and the worst-case low input (logic 0) for
which T2 will be either in saturation or in cut off.

11. It is required to design a binary mod-5 synchronous counter using AB flip-flops


such that the output Q2Q1Q0 changes as 000  001  010 ….. and so on. The
excitation table for the AB flip-flop is given in table 11.
(a) Write down the state table for the mod-5 counter.
(b) Obtain simplified SOP expressions for the inputs A2, B2, A1, B1, A0 and B0 in
terms of Q2, Q1, Q0 and their complements.
(c) Hence, complete the circuit diagram for the mod-5 counter given in Figure 11
using minimum number of 2-input NAND-gate only.

A B Qn+1
0 0 1

0 1 Qn

1 0 Qn
1 1 0

A2 Q2 A1 Q1 A0 Q0

B2 Q2 B1 Q1 B0 Q0

Clock

12. An 8085 microprocessor operating at 5 MHz clock frequency execute the


following routine.
START MOVE A,B
OUT 55H
DCR B
STA FFF8H
JMP START
(a) Determine the total number of machine cycles required to execute this
routine till the JMP instruction is executed for the first time.
(b) Determine the time interval between two consecutive MEMW signals.

Join All India Mock GATE Classroom Test Series - 2007 conducted by GATE Forum in over 25 cities all over India. Question
Papers including section tests and full tests are designed by IISc alumni according to the latest syllabus. Percentile, All India Rank,
interaction with IISc alumni in our online discussion forums, and more. For more details,
visit
www.gateforum.com
Think GATE Think GATE Forum
GATE EC - 2002 www.gateforum.com
Join discussion of this test paper at http://forum.gatementor.com

(c) If the external logic controls the READY line so that three WAIT states are
introduced in the I/O WRITE machine cycle, determine the time interval
between two consecutive MEMW signals.

1
13. A unity feedback system has the plant transfer function Gp ( s ) =
( s + 1) (2s + 1)
(a) Determine the frequency at which the plant has a phase lag of 90°
k
(b) An integral controller with transfer function Gc ( s ) =
is placed in the feed-
s
forward path of the feedback system. Find the value of k such that the
compensated system has an open-loop gain margin of 2.5.
(c) Determine the steady state errors of the compensated system to unit-step
and unit-ramp inputs.

14. The Nyquist plot of an all-pole second order open-loop system is shown in Figure
14. Obtain the transfer function of the system. Imag

-0.5 0.5 1 1.5 2 2.5


Real
ω=∞
-0.5 ω=0

-1

-1.5

-2

ω=2 -2.5

15. The block diagram of a linear time invariant system is given in Figure 15.

+
- X2(s) + X1(s)
U(s) Σ 1/s Σ 1/s Σ Y(s)
- -

Join All India Mock GATE Classroom Test Series - 2007 conducted by GATE Forum in over 25 cities all over India. Question
Papers including section tests and full tests are designed by IISc alumni according to the latest syllabus. Percentile, All India Rank,
interaction with IISc alumni in our online discussion forums, and more. For more details,
visit
www.gateforum.com
Think GATE Think GATE Forum
GATE EC - 2002 www.gateforum.com
Join discussion of this test paper at http://forum.gatementor.com

(a) Write down the state variable equations for the system in matrix form
T
assuming the state vector to be  x1 ( t ) x2 ( t )  .

(b) Find out the state transition matrix.


(c) Determine y(t), t ≥ 0, when the initial values of the state at time t = 0 are x1
(0) =1, and x2 (0) = 1.

16. A deterministic signal x(t) = cos2πt is passed through a differentiator as shown in


Figure 16.

x(t) 1 d y(t)

2π dt

(a) Determine the autocorrelation Rxx (T) and the power spectral density Sxx(f).
(b) Find the output power spectral density Syy(f).
1
(c) Evaluate Rxy(0) and Rxy   .
4

17. A DSBSC modulated signal s(t) = 10 cos (2π × 106t+φ)m(t-) is corrupted by an


additive white Gaussian noise of power spectral density 10-4 W/Hz. The message
power spectral density Smm(f) is as shown in Figure 17 and φ is uniformly
distributed over the range 0 to 2π.
Smm(f)

10-3 W/Hz

f
-3KHz +3KHz
(a) Express the signal autocorrelation function RSS(T) in terms of the message
autocorrelation function Rmm(T). Clearly state the necessary assumptions.
(b) Determine the signal power spectral density SSS(f).
(c) Find the power of the modulated signal and the noise power in the
transmissions bandwidth.

18. A continuous time signal with finite energy, band limited from 3 MHz to 5 MHz is
ideally sampled, encoded by a fixed length PCM coder and then transmitted over
a digital channel of capacity 7 Mbps. The probability density function (pdf) at the
output of the sampler is uniform over the range –2V to +2V.
(a) Determine the minimum sampling rate necessary for perfect reconstruction.
(b) Determine the maximum SNR (in dB) that may be achieved.

Join All India Mock GATE Classroom Test Series - 2007 conducted by GATE Forum in over 25 cities all over India. Question
Papers including section tests and full tests are designed by IISc alumni according to the latest syllabus. Percentile, All India Rank,
interaction with IISc alumni in our online discussion forums, and more. For more details,
visit
www.gateforum.com
Think GATE Think GATE Forum
GATE EC - 2002 www.gateforum.com
Join discussion of this test paper at http://forum.gatementor.com

19. A discrete memory-less source generates either 0 or 1 at a rate of 160 Kbps; 0 is


generated three times more frequently than 1. A coherent binary PSK modulator
is employed to transmit these bits over a noisy channel. The received bits are
detected in a correlator fed with the basis function of unit energy (for this binary
PSK scheme) as the reference signal. The receiver makes a decision in favour of
1 if the correlator output is positive, else decides in favour of 0. If 0 and 1 are
( ) (
represented as − 6 2 cos 640π × 103 t V and 6 2 cos 640π × 103 t V respectively, )
then
(a) determine the transmitted signal energy per bit.
(b) determine the basis function of unit energy for this binary PSK scheme.
(c) determine the probability that the receiver makes a decision in favour of 1
when the channel noise is characterized as zero-mean AWGN with power
N
spectral density 0 = 3.125 × 10−4W / Hz
2
20. Transmission line transformation of a load ZL and Z is given by
ZL + jZ0 tan ( β l )
Z = Z0
Z0 + jZL tan ( β l )

(a) Show that the above transformation implies that the impedance Z gets
transformed to ZL* for real Z.
(b) What is the importance of the result derived in (a)?

21. Consider a parallel plate wave-guide with plate separation d as shown in Figure
21 electric and magnetic fields for the TEM-mode are given by

E x = E0e − jkz + jω t
E0 − jkz + jω t
Hy = e
n
where k = ηωε
(a) Determine the surface charge densities pr s on the plates at x = 0 and x = d.
(b) Determine the surface current densities J s on the same plates.
r
(c) Prove that ps and J s satisfy the current continuity condition.

d ∈

Join All India Mock GATE Classroom Test Series - 2007 conducted by GATE Forum in over 25 cities all over India. Question
Papers including section tests and full tests are designed by IISc alumni according to the latest syllabus. Percentile, All India Rank,
interaction with IISc alumni in our online discussion forums, and more. For more details,
visit
www.gateforum.com
Think GATE Think GATE Forum
GATE EC - 2002 www.gateforum.com
Join discussion of this test paper at http://forum.gatementor.com

22. Consider a linear array of two half-wave dipoles A and B as shown in Figure 22.
λ
The dipoles are apart and are excited in such a way that the current on
4
element B legs that on element A by 90° in phase.
(a) Obtain the expression for the radiation pattern for E0in the XY plane (i.e,
θ=90°)
(b) Sketch the radiation pattern obtained in (a).

A B
Y

λ/4

φ=0°

Join All India Mock GATE Classroom Test Series - 2007 conducted by GATE Forum in over 25 cities all over India. Question
Papers including section tests and full tests are designed by IISc alumni according to the latest syllabus. Percentile, All India Rank,
interaction with IISc alumni in our online discussion forums, and more. For more details,
visit
www.gateforum.com
Think GATE Think GATE Forum
GATE EC - 1992 www.gateforum.com
Join discussion of this test paper at http://forum.gatementor.com

Time: 3 hours Maximum Marks: 200 Part A: 80 Marks Part B: 120 Marks
1. This paper consists of two parts: “Part A” and “Part B”.
2. Part A contains 40 questions, each carrying 2 marks.
3. Part B contains 15 questions, each carrying 8 marks.
4. Read the instructions given at the beginning of each part carefully.
5. Attempt ALL questions.
6. There is no negative marking.
PART – A

1.1 Relative to a given fixed tree of a network,


(a) Link currents form an independent set
(b) Branch voltage from an independent set
(c) Link currents from an independent set
(d) Branch voltage from an independent set

1.2 For a 2-port network to be reciprocal


(a) z11 = z22 (b) y21 = y12 (c) h21 = −h12 (d) AD − BC = 0

1.3 For the series R-L circuit of figure(a), the partial fissure diagram at a certain
frequency is shown in figure (b).The operating frequency of the circuit is:

+ − + −
+
VR
VR VL +
V VC V


VC
(a) (b)

(a) equal to the resonance frequency


(b) less than the resonance frequency
(c) greater than resonance frequency (d) not zero

1.4 For the compensated attenuator of figure, the impulse response under the
condition R1C1 = R2C2 is: R1

R2  
1
+
1 − e 1 1  u ( t )
RC +
(a)
R1 + R2   C1 V2(t)
V1(t)
R2 C2 R2(t)
(b) δ (t )
R1 + R2
− C −
1
R2 R2
(c) u (t ) (d) 1−e R1C1
.u ( t )
R1 + R2 R1 + R2

Join All India Mock GATE Classroom Test Series - 2007 conducted by GATE Forum in over 25 cities all over India. Question
Papers including section tests and full tests are designed by IISc alumni according to the latest syllabus. Percentile, All India Rank,
interaction with IISc alumni in our online discussion forums, and more.
visit
www.gateforum.com
Think GATE Think GATE Forum
GATE EC - 1992 www.gateforum.com
Join discussion of this test paper at http://forum.gatementor.com

1.5 Of the four networks, N1 , N2 , N3 and N4 of figure, the networks having identical
driving point functions are
2 1
2 1

2 1
2
1
1
N2
N1

2
1 1

1 2
1
1
1

N3 N4

(a) N1 and N1 (b) N2 and N4 (c) N1 and N3 (d) N1 and N4

1.6 A linear time invariant system is described by the state variable model
 x1   −1 0   x1  0  x1 
 =
x   x  +   u = 1 2  x 
 2  0 −2   2   1  2
(a) The system is completely controllable
(b) The system is not completely controllable
(c) The system is completely observable
(d) The system is not completely observable

ke − sT4
1.7 A process with open-loop model G ( s ) = is controlled by a PID controller.
τs + 1
For this process
(a) the integral mode improves transient performance
(b) the integral mode improves steady state performance
(c) the derivative mode improves transient performance
(d) the derivative mode improves steady state performance

1.8 A linear discrete time system has the characteristic equation, z 3 − 0.81z = 0. The
system
(a) is stable (b) is marginally stable
(c) in unstable
(d) stability cannot be assessed from the given information

Join All India Mock GATE Classroom Test Series - 2007 conducted by GATE Forum in over 25 cities all over India. Question
Papers including section tests and full tests are designed by IISc alumni according to the latest syllabus. Percentile, All India Rank,
interaction with IISc alumni in our online discussion forums, and more.
visit
www.gateforum.com
Think GATE Think GATE Forum
GATE EC - 1992 www.gateforum.com
Join discussion of this test paper at http://forum.gatementor.com

1.9 Bode plot of a stable system is shown in figure. The transfer function of the
system is _______.
dB

20
20 dB/Decade

log ω

1.10 Given a unity feedback system with open loop transfer function,
K
G (s) = . The root locus plot of the system is of the form.
s ( s + 1) ( s + 2 )
jω jω

× × ×
× × × σ
σ

jω jω

× × × × × ×
σ σ

1.11 A semiconductor is irradiated with light such that carriers are uniformly
generated throughout its volume. The semiconductor is n-type with ND = 1019 per
cm3. If the excess electron concentration in the steady state id ∆n = 1015 per
cm3 and if τ p = 10µ sec [minority carrier life time] the generation rate due to
irradiation
(a) is 1020 e − h pair/cm3 / s (b) is 1024 e − h pair/cm3 / s

(c) is 1010 e − h pair/cm3 / s


(d) cannot be determined as the given data is insufficient

1.12 A P – N junction in series with a 100 ohms resistor, is forward biased so that a
current of 100 mA flows. If the voltage across this combination is instantaneously
reversed to 10 V at t = 0, the reverse current that flows through the diode at t =
0 is approximately given by
(a) 0 mA (b) 100 mA (c) 200 mA (d) 50 mA

Join All India Mock GATE Classroom Test Series - 2007 conducted by GATE Forum in over 25 cities all over India. Question
Papers including section tests and full tests are designed by IISc alumni according to the latest syllabus. Percentile, All India Rank,
interaction with IISc alumni in our online discussion forums, and more.
visit
www.gateforum.com
Think GATE Think GATE Forum
GATE EC - 1992 www.gateforum.com
Join discussion of this test paper at http://forum.gatementor.com

1.13 An infra red LED is usually fabricated from


(a) Ge (b) Si (c) Ga As (d) Ga As P

1.14 In a transistor having finite B, the forward bias across the base emitter junction
is kept constant and the reverse bias across the collector base junction is
increased. Neglecting the leakage across the collector base junction and the
depletion region generating current, the base current will ____.
(increase/decrease/remain constant).

1.15 An n-channel JFET has a pinch-off voltage of Vp = −5V , VDS (max ) = 20V , and
δ m = 2mA / V . The minimum ‘ON’ resistance is achieved in the JEFT for
(a) VGS = −7V and VDS = 0V (b) VGS = 7V and VDS = 0V
(c) VGS = 0V and VDS = 20V (d) VGS = −7V and VDS = 20V

1.16 The JFET in the circuit shown in figure has an IDSS = 10mA and VP = 5V . The value
of the resistance Rs for a drain current IDS = 6.4mA is (select the nearest value)

IDS
+
10V

RS

(a) 150 ohms (b) 470 ohms (c) 560 ohms (d) 1 kilo ohm

1.17 An op.amp has an offset voltage of 1 mV and is ideal in all other respects. If this
op.amp is used in the circuit shown in figure, the output voltage will be (select
the nearest value)

1MΩ

1KΩ

VO
+

(a) 1 mV (b) 1 V (c) ± 1 V (d) 0 V

Join All India Mock GATE Classroom Test Series - 2007 conducted by GATE Forum in over 25 cities all over India. Question
Papers including section tests and full tests are designed by IISc alumni according to the latest syllabus. Percentile, All India Rank,
interaction with IISc alumni in our online discussion forums, and more.
visit
www.gateforum.com
Think GATE Think GATE Forum
GATE EC - 1992 www.gateforum.com
Join discussion of this test paper at http://forum.gatementor.com

1.18 If the transistors in figure, have high values of β and a VBE of 0.65 volt, the
current I, flowing through the 2 kilo ohms resistance will be ______.

6.5kΩ I 2kΩ

1.85kΩ +
10V

1.65kΩ
1kΩ

1.19. The circuit of figure uses an ideal op amp. For small positive values of Vin , the
circuit works as
(a) a halfwave rectifier
R
(b) a differentiator Vin −
Vout
(c) a logarithmic amplifier +
(d) an exponential amplifier

1.20. Assume that the operational amplifier in figure is ideal. The current I, through the
1 K ohm resistor is ____.
2kΩ

I

+ 1kΩ
2mA 2K

1.21. The 6 V Zener diode shown in figure has zero zener resistance and a knee current
of 5 mA. The minimum value of R so that the voltage across it does not fall below
6 V is: 50Ω
(a) 1.2 K ohms
(b) 80 ohms
(c) 50 ohms +
10V V R

(d) 0 ohms

Join All India Mock GATE Classroom Test Series - 2007 conducted by GATE Forum in over 25 cities all over India. Question
Papers including section tests and full tests are designed by IISc alumni according to the latest syllabus. Percentile, All India Rank,
interaction with IISc alumni in our online discussion forums, and more.
visit
www.gateforum.com
Think GATE Think GATE Forum
GATE EC - 1992 www.gateforum.com
Join discussion of this test paper at http://forum.gatementor.com

1.22. The logic realized by the circuit shown in figure is:


(a) F = A.C I0
C
(b) F = A + C 4×1
I1
(c) F = B.C MUX F
I2
(d) F = B + C
C I3 S1 S0

1.23. Choose the correct statement(s) from the following:


(a) PROM contains a programmable AND array and a fixed OR array
(b) PLA contains a fixed AND array and a programmable OR array
(c) PROM contains a fixed AND array and a programmable OR array
(d) PLA contains a programmable AND array and a programmable OR array

1.24. The figure shows the circuit of a gate in the Resistor Transistor Logic (RTL)
family. The circuit represents a VCC

(a) NAND
(b) AND
(c) NOR V0
(d) OR

Vi1

Vi2

1.25. The initial contents of the 4-bit serial-in-parallel-out, right-shift, Shift Register
shown in figure, is 0110. After three clock pulses are applied, the contents of the
Shift Register will be
(a) 0 0 0 0 Clock
Serial in 0 1 1 0
(b) 0 1 0 1
(c) 1 0 1 0

(d) 1 1 1 1

1.26. In an 8085 microprocessor system with memory mapped I/O,


(a) I/O devices have 16 bit addresses
(b) I/O devices are accessed using IN and OUT instructions
(c) there can be a maximum of 256 input devices and 256 output devices
(d) arithmetic and logic operations can be directly performed with the I/O data.

Join All India Mock GATE Classroom Test Series - 2007 conducted by GATE Forum in over 25 cities all over India. Question
Papers including section tests and full tests are designed by IISc alumni according to the latest syllabus. Percentile, All India Rank,
interaction with IISc alumni in our online discussion forums, and more.
visit
www.gateforum.com
Think GATE Think GATE Forum
GATE EC - 1992 www.gateforum.com
Join discussion of this test paper at http://forum.gatementor.com

1.27. The following program is run on 8085 microprocessor:


Memory Address in hex Instruction
2000 LXI SP, 1000
2003 PUSH H
2004 PUSH D
2005 CALL 2050
2008 POP H
2009 HLT
At the completion of execution of the program, the program Counter of the 8085
contains ________ and the Stack Pointer contains ________.

1.28. Dual slope integration type Analog-to-digital converters provide


(a) higher speeds compared to all other types of A/D converters
(b) very good accuracy without putting extreme requirements on component
stability
(c) good rejection of power supply hum
(d) better resolution compared to all other types of A/D converters for the same
number of bits.

1.29. Which of the following signals is/are periodic?


(a) S ( t ) = cos 2t + cos 3t + cos 5t (b) S ( t ) = exp ( j8π t )

(c) S ( t ) = exp ( −7t ) sin10π t (d) S ( t ) = cos 2t cos 4t

1.30. If G ( f ) represents the Fourier transform of a signal g ( t ) which is real and odd
symmetric in time, then
(a) G ( f ) is complex (b) G ( f ) is imaginary

(c) G ( f ) is real (d) G ( f ) is real and non-negative

1.31. The maximum power efficiency of an AM modulator is


(a) 25% (b) 50% (c) 75% (d) 100%

1.32. For a random variable x following the probability density function, p ( x ) , shown in
figure the mean and the variance are, respectively,
1 2 4 p(x)
(a) and (b) 1 and 1
2 3 3
4
2 4
(c) 1 and (d) 2 and x
3 3 -1 0 3

Join All India Mock GATE Classroom Test Series - 2007 conducted by GATE Forum in over 25 cities all over India. Question
Papers including section tests and full tests are designed by IISc alumni according to the latest syllabus. Percentile, All India Rank,
interaction with IISc alumni in our online discussion forums, and more.
visit
www.gateforum.com
Think GATE Think GATE Forum
GATE EC - 1992 www.gateforum.com
Join discussion of this test paper at http://forum.gatementor.com

1.33. The bit stream 01001 is differentially encoded using ‘Delay and Ex OR’ scheme
for DPSK transmission. Assuming the reference bit as a ‘1’ and assigning phases
of ‘0’ and π for 1’s and 0’s respectively, in the encoded sequence, the transmitted
phase sequence becomes
(a) π0ππ0 (b) 0ππ00 (c) 0πππ0 (d) ππ0ππ

1.34. Coherent demodulation of FSK signal can be effected using


(a) correlation receiver
(b) band pass filters and envelope detectors
(c) matched filter
(d) discriminator detection

1.35. Source encoding in a data communication system is done in order to


(a) enhance the information transmission rate
(b) reduce the transmission errors
(c) conserve the transmitted power
(d) facilitate clock recovery in the receiver

1.36. A transmission line whose characteristic impedance is a pure resistance


(a) must be a lossless line (b) must be a distortionless line
(c) may not be a lossless line (d) may not be distrotionless line

1.37. Which of the following statements is/are correct?


(a) All the resonant frequencies of a microwave cavity are harmonics of a single
frequency
(b) No two of the resonant frequencies of a microwave cavity are harmonics of a
single frequency
(c) Resonant frequencies of a microwave cavity form distinct sets of
harmonically related frequency
(d) None of the above, because a microwave cavity does not resonate at a
number of frequencies.

1.38. Two dissimilar antennas having their maximum directivities equal,


(a) must have their beam-widths also equal
(b) cannot have their beam widths equal because they are dissimilar antennas
(c) may not necessarily have their maximum power gains equal
(d) must have their effective aperture areas (capture areas) also equal.

Join All India Mock GATE Classroom Test Series - 2007 conducted by GATE Forum in over 25 cities all over India. Question
Papers including section tests and full tests are designed by IISc alumni according to the latest syllabus. Percentile, All India Rank,
interaction with IISc alumni in our online discussion forums, and more.
visit
www.gateforum.com
Think GATE Think GATE Forum
GATE EC - 1992 www.gateforum.com
Join discussion of this test paper at http://forum.gatementor.com

1.39. The beam width between first null of uniform linear array of N equally spaced
(element spacing = d), equally excited antennas is determined by
(a) N alone and not by d (b) d alone and not by N
N
(c) the ratio,   (d) the product, ( Nd )
d

1.40. In a multi-cavity magnetron, strapping is employed primarily


(a) to prevent mode jumping
(b) to increase the separation between the resonant frequencies in the π mode
and in the adjacent modes
(c) to reduce the back heating of the cathode
(d) to increase the output of the magnetron.

PART– B
(15 × 8 = 120 MARKS)

2. The two-port Darlington impedance booster of figure uses identical transistors


( hie = 1K , hfe = 100, hre = hoe = 0) . Calculate the z-parameters of the network (use
relevant approximations)

I1
+
+

I2

V2
V1
1K

− −

3. Block diagram model of a position control system is shown in figure.

Amplifier Motor

R(s) 1 Y(s)
KA
+ S ( 0.5s + 1)
+
− −

sKt

Join All India Mock GATE Classroom Test Series - 2007 conducted by GATE Forum in over 25 cities all over India. Question
Papers including section tests and full tests are designed by IISc alumni according to the latest syllabus. Percentile, All India Rank,
interaction with IISc alumni in our online discussion forums, and more.
visit
www.gateforum.com
Think GATE Think GATE Forum
GATE EC - 1992 www.gateforum.com
Join discussion of this test paper at http://forum.gatementor.com

(a) In absence of derivative feedback ( Kt = 0 ) , determine damping ratio of the


system for amplifier gain K A = 5. Also find the steady state error to unit
ramp input.
(b) Find suitable values of the parameters K A and Kt so that damping ratio of the
system is increased to 0.7 without affecting the steady state error as
obtained in part (a).
1
4. A unity feedback system has open loop transfer function G ( s ) = .
s (2s + 1) ( s + 1)
Sketch Nyquist plot for the system and there from obtain the gain margin and the
phase margin.

5. A P-N-P transistor shown in figure has uniform doping in the emitter, based and
collector regions, where in the doping concentrations are 1019 per cm3 , 1017 per
cm3 , and 1015 per cm3 , respectively. The minority carrier diffusion lengths in the
emitter and the base regions are 5 microns and 100 microns, respectively.
Assuming low level injection conditions and using the law of the junction,
calculate the collector current density and the base current density and the base
current density due to base recombination. [Suitable approximations may be
made if required]. In all the regions of the transistor Dp = 8cm2 per sec.

Dn = 16cm2 per sec, ni = 1.5 × 1010 per cm2 , KT / q = 26mV , q = 1.6 × 10−19 c

Emitter Base Collector


P N P
N A = 1019 per cm3 ND = 1017 per cm3 N A = 1015 per cm3
Ln = 5 Microns Lp = 100 Microns
10 Microns 5 Microns 200 Microns

10 volts
0.78 volts
+ − + −

6. An n-channel MOSFET(T) having a VT of 2 V [Threshold voltage] is used in the


circuit shown in figure. Initially, t is
off an in steady state. At time t =
0, a step voltage of magnitude 4V 1K
is applied to the input so that the T
VO
MOSFET turns ‘ON’
instantaneously. Draw the
equivalent circuit and calculate the +
+ 10V
time taken for the output Vo to fall −
to 5V. The device constant of the 100pF
VIN
MOSFET, K = 5 mA/V2,
RDS = ∞, C DS = 0; C DG = 0.

Join All India Mock GATE Classroom Test Series - 2007 conducted by GATE Forum in over 25 cities all over India. Question
Papers including section tests and full tests are designed by IISc alumni according to the latest syllabus. Percentile, All India Rank,
interaction with IISc alumni in our online discussion forums, and more.
visit
www.gateforum.com
Think GATE Think GATE Forum
GATE EC - 1992 www.gateforum.com
Join discussion of this test paper at http://forum.gatementor.com

7. The transistors in the differential amplifier shown in figure are identical with
hfe = 100 and re = 25 ohms at 1 mA collector current. The circuit has a Common
Mode Rejection Ratio (CMRR) of 100.
ν0
(a) What is the differential gain of the circuit?
(ν1 − ν 2 )
(b) What is the common mode gain of the circuit?
(c) If d.c. voltage of 1010 mV and 990 mV are allied to inputs 1 and 2
respectively with reference to ground, what will be the output voltage Vo ?
+5V

output
V
Input 1

V1

V2 Input 2

2mA

-5V

8. Consider the circuit shown in figure. This circuit uses an ideal operational
amplifier. Assuming that the impedances at nodes A and B do not load the
preceding bridge circuit, calculate the output voltage V0.
(a) when Ra = Rb = Rd = 100 ohms (b) when Ra = Rb = Rc = 100 ohms

+10V
and Rd = 120 ohms

12kΩ

Ra Rd
A 10kΩ

VO
10kΩ
+
B
Rb Rc
12kΩ

Join All India Mock GATE Classroom Test Series - 2007 conducted by GATE Forum in over 25 cities all over India. Question
Papers including section tests and full tests are designed by IISc alumni according to the latest syllabus. Percentile, All India Rank,
interaction with IISc alumni in our online discussion forums, and more.
visit
www.gateforum.com
Think GATE Think GATE Forum
GATE EC - 1992 www.gateforum.com
Join discussion of this test paper at http://forum.gatementor.com

9. A combinational circuit has three inputs A, B and C and an output F.F. is true only
for the following input combinations?
A is false and B is true
A is false and C is true
A, B and C are all false
A, B and C are all true
(a) Write the truth table for F. use the convention, true = 1 and false = 0.
(b) Write the simplified expression for F as a Sum of Products.
(c) Write the simplified expression for F as a product of Sums.
(d) Draw a logic circuit implementation of F using the minimum number of 2
input NAND gates only.

10. A new clocked X-Y flip flop is defined with two inputs, X and Y is addition to the
clock input. The flip flop functions as follows:
If XY=00, the flip flop changes stage with each clock pulse
If XY=01, the flip flop state Q becomes 1 with the next clock pulse
If XY=10, the flip flop state Q becomes 0 with the next clock pulse
If XY=11, the change of state occurs with the clock pulse
(a) Write the Truth table for the X-Y flip flop
(b) Write the Excitation table for the X-Y flip flop
(c) It is desired to convert a J-K flip flop into the X-Y flip flop by adding some
external gates, if necessary. Draw a circuit to show how you will implement
in X-Y flip flop using a J-K flip flop.

11. A sinusoidal signal of 4 kHz frequency is used as a modulating signal for an FM


and an AM transmitter. Both of the transmitters use the same amplitude for the
unmodulated carrier. The peak frequency deviation in the FM transmitter is set to
four times the bandwidth of the AM transmitter and, the magnitudes of the
spectral components at fc ± 4kHz ( fc = carrier frequecy ) are same for beoth of the
transmitters.
Determine the modulation indices for the AM and FM transmitters.
[If required use the values of the Besset function given below:
J1 (2 ) = 0.577, J1 ( 4 ) 0.066, J1 ( 8 ) = 0.235, J1 (16 ) = 0.094 ]

12. Figure shows the memory circuit of an 8085 microprocessor.


(a) What is the total size of the memory in the circuit?
(b) What are the beginning and ending addresses of the memory in Chip 1?
(Give the answer in hexadecimal numbers)
(c) What are the beginning and ending addresses of the memory in Chip 2?
(d) Are the memory chips in the circuit ROM or RAM?

Join All India Mock GATE Classroom Test Series - 2007 conducted by GATE Forum in over 25 cities all over India. Question
Papers including section tests and full tests are designed by IISc alumni according to the latest syllabus. Percentile, All India Rank,
interaction with IISc alumni in our online discussion forums, and more.
visit
www.gateforum.com
Think GATE Think GATE Forum
GATE EC - 1992 www.gateforum.com
Join discussion of this test paper at http://forum.gatementor.com

(e) How will you replace the two NAND gates in the circuit with one 3 to 8
decoder without changing the memory size or the memory addresses?
Assume that the decoder has one active high enable E1 and one active low
enable E2.

A15
A14
A13
A12
A11 A0-10
CS
11 Memory
8085
16 chip
A0-15 D0-7
OE
D0-7 8 10/ M 8

10/ M
RD A0-10
RD OE
11 Memory
WR D0-7 chip
CS
A15 8
A14
A13
A12
A11

13. (a) A low pass signal x(t) has a spectrum given by


f
x (f ) = 1 − 1 , for f ≤ 2000Hz.
2000
0, elsewhere.
Assuming that x(t) is ideally sampled at a sampling frequency of 3 kHz sketch.
(i) X ( f ) , and

(ii) the spectrum of the sampled signal for f ≤ 3kHz.

(b) An analog low pass signal of 4 kHz bandwidth is sampled at the Nyquist rate,
subsequently quantized and encoded into a binary PCM wave with 128
quantization levels. The word synchronization is effected by adding a
synchronizing pulse at the end of each code word. Determine the bit ratio to the
resulting PCM signal.

14. A received binary NRZ signal assumes the voltage levels of 500 millvolts and -
500 millivolts respectively for ‘1’ and ‘0’ transmission with a bit rate or r
bits/second. The signal is corrupted by additive white Gaussian noise with a two
sided spectral density of volts2/Hz. The received signal is processed by an
integrate and Dump circuit in every bit interval and compared with a zero
threshold to take a bit decision.
Assuming ‘1’ and ‘0’ transmission to be equally likely, determine the maximum
value of r such that the bit error probability ≤ 10−5
Join All India Mock GATE Classroom Test Series - 2007 conducted by GATE Forum in over 25 cities all over India. Question
Papers including section tests and full tests are designed by IISc alumni according to the latest syllabus. Percentile, All India Rank,
interaction with IISc alumni in our online discussion forums, and more.
visit
www.gateforum.com
Think GATE Think GATE Forum
GATE EC - 1992 www.gateforum.com
Join discussion of this test paper at http://forum.gatementor.com

− z2
1 ∞
Given x =


x
e 2
dz = 10−5 at x = 4.27

− z2
1 ∞
erfc x =
π
∫ x
e 2
2dz = 2 × 10−5 at x = 3.02

15. A uniform plane wave traveling in free space along the +z direction and having
its electric field along the x-direction, is normally incident on a thick brass sheet,
infinite in extent along the x-and y-directions). The electric field intensity of the
wave is given by
(
E x = 1225 cos 5.89 × 1010 t − β z volts/metre. )
Calculate the power per square metre that causes heating of the brass sheet
taking ε r = 1, µ r = 1 and σ = 1.649 × 107 mhos/metre for brass.

16. Elements of a linear array of three equally spaced (element spacing = 0.5 λ )
vertical mast radiators, are excited as given in figure. For the horizontal plane
radiation pattern of the array, determine the direction of the major lobe (main
lobe or principal lobe), and calculate its half-power beam width in degrees.

0.5 0.5

I∠ + 90° 2I∠0° I∠ − 90°

Join All India Mock GATE Classroom Test Series - 2007 conducted by GATE Forum in over 25 cities all over India. Question
Papers including section tests and full tests are designed by IISc alumni according to the latest syllabus. Percentile, All India Rank,
interaction with IISc alumni in our online discussion forums, and more.
visit
www.gateforum.com
Think GATE Think GATE Forum
GATE EC - 1993 www.gateforum.com
Join discussion of this test paper at http://forum.gatementor.com

Time : 3 hours PART I Maximum Marks : 200


SECTION - A
1. This questions 1.1 to 1.7 below one or more the alternatives are correct.
Write the code letter(s), (A,B, C and D) corresponding to the correct
alternatives in the answer book. Marks will be given only if all the correct
alternatives have been selected and no incorrect alternative is picked up.

0 0 0
 
1.1 The eigen vector(s) of the matrix  0 0 0  , a ≠ 0, is/are:
0 0 0
 
(a) (0, 0, α ) (b) (α , 0, 0 ) (c) (0, 0,1) (d) ( 0, α , 0 )

d 2 y dy
1.2 The differential equation, + + sin y = 0, is:
dx 2 dx
(a) linear (b) non-linear (c) homogeneous (d) of degree two

1.3 Simpson’s rule for integration gives exact result when f ( x ) is a polynomial of
degree
(a) 1 (b) 2 (c) 3 (d) 4

1.4 Which of the following is (are) valid FORTRAN 77 statement(s)?


(a) DO 131 = 1 (b) A = DIM***7 (c) READ = 15.0 (d) GOTO 3 = 10

1.5 Fourier series of the periodic function (period 2π) defined by


 0 −π < x < 0
f (x) =  is
x 0 < x < π
π ∞
1  1 
+ ∑  n2  cos nπ − 1 cos nx − cos nπ sin nx  
4 1  π  n 
By putting x = π in the above, one can deduce that the sum of the series
1 1 1
1+ 2
+ 2 + 2 + K , is
3 5 7
π2 π2 π2 π2
(a) (b) (c) (d)
4 6 8 12

1.6 Which of the following improper integrals is (are) convergent?


1 ∞ ∞ 1
sin x cos x x 1 − cos x
(a) ∫0 1 − cos x dx (b) ∫0 1 + x dx ` (c) ∫0 1 + x 2 dx (d) ∫
0
5
dx
2
x

Join All India Mock GATE Classroom Test Series - 2007 conducted by GATE Forum in over 25 cities all over India. Question
Papers including section tests and full tests are designed by IISc alumni according to the latest syllabus. Percentile, All India Rank,
interaction with IISc alumni in our online discussion forums, and more.
visit
www.gateforum.com
Think GATE Think GATE Forum
GATE EC - 1993 www.gateforum.com
Join discussion of this test paper at http://forum.gatementor.com

1.7 The function f ( x, y ) = x 2 y − 3xy + 2y + x, has

(a) no local extremum


(b) one local minimum but no local maximum
(c) one local maximum but no local minimum
(d) one local minimum but one local maximum

2. In questions 2.1 to 2.10 below, each blank (_________) is to be suitably


filled in. in the answer book write the question number and the answer
only. Do not copy the question. Also no explanations for the answers are
to be given.

2.1 lim
( )
x e x − 1 + 2 ( cos x − 1)
is _________.
x →0 x (1 − cos x )

(3m) ! x 3m is _________
2.2 The radius of convergence of the power series ∑ 3
0 ( m !)
ur ur r
2.3 If the linear velocity V is given by V = x 2 yiˆ + xyzjˆ − yz 2 k the angular velocity ω at
the point (1, 1, -1) is _________.

2.4 Given the differential equation, y = x − y with the initial condtion y ( 0 ) = 0. the
value of y ( 0.1) calculated numerically up to the third place of decimal by the
second order Runge-Kutta method with step size h = 0.1 is _________

2.5 For X = 4.0, the value of I in the FORTRAN 77 statement


3
I = −2 * *2 + 5.0 * X X * 3 + is _________
4

1
1 x
x
2.6 The value of the double integral ∫∫1+y
0 x
2
dxdy is _________

1 0 0 1
 
0 −1 0 −1 
2.7 If A = 
0 0 i i 
 
0 0 0 −i 

The matrix A4 , calculated by the use of Coyley – Mamilton theorem or otherwise,


is _________

Join All India Mock GATE Classroom Test Series - 2007 conducted by GATE Forum in over 25 cities all over India. Question
Papers including section tests and full tests are designed by IISc alumni according to the latest syllabus. Percentile, All India Rank,
interaction with IISc alumni in our online discussion forums, and more.
visit
www.gateforum.com
Think GATE Think GATE Forum
GATE EC - 1993 www.gateforum.com
Join discussion of this test paper at http://forum.gatementor.com

2.8 Given, V = x cos2 yi$ + x 2e x $j + z sin2 yk and S the surface of a unit cube with one
corner at the origin and edges parallel to the coordinate axes, the value of the
ur
$
integral ∫∫ V .ndS is _________
S

2.9 The differential equation y + y = 0 is subjected to the boundary conditions


y ( 0 ) + 0y ( λ ) = 0 . In order that the equation has non-trivial solution (s), the
genral value of η is _________

2.10 The Laplace transform of the periodic function f ( t ) described by the curve below
i.e.
sin t if ( 2n − 1) π ≤ t ≤ 2nπ ( n = 1, 2, 3, K)
f (t ) = 
0
Otherwise is _________

f(t)

π 2π 3π 4π 5π 6π 7π 8π t
0

SECTION B: PHYSICS
3. In the following questions 3.1 to 3.17 there are some multiple choice
questions and some questions where blanks are to be filled in. Answer
ALL the questions. All multiple choice questions have ONE or MORE
correct answers those suggested. Credit will be given only if every
correct alternative(s), and no incorrect alternative, is selected. Write
only the letters corresponding to the select alternatives in the answer
book. In the fill in the blank type questions, write the answer only in the
answer book.
Useful data: h = 6.63 × 10−34 Jsc = 3 × 108 m / s

3.1. Two particles of masses M1 and M2 ( M1 > M2 ) attract each other with a force
inversely proportional to the square of the distance between them. The particles
are initially at rest and then released. The centre of mass relative to a stationary
observer
(a) moves towards M1 (b) moves towards M2
(c) remains at rest
M1
(d) moves with a speed proportional to
M2
Join All India Mock GATE Classroom Test Series - 2007 conducted by GATE Forum in over 25 cities all over India. Question
Papers including section tests and full tests are designed by IISc alumni according to the latest syllabus. Percentile, All India Rank,
interaction with IISc alumni in our online discussion forums, and more.
visit
www.gateforum.com
Think GATE Think GATE Forum
GATE EC - 1993 www.gateforum.com
Join discussion of this test paper at http://forum.gatementor.com

3.2. The temperature of an ideal gas is held constant while its volume is increased.
The pressure exerted by the gas on the walls of the container decreases because
its molecules
(a) strike the walls with smaller force
(b) strike the walls with lower velocities
(c) strike the walls less frequently
(d) collide with each other more frequently

3.3. Although a laser beam is highly directional, its beam width increase with
propagation. This increase is due to
(a) coherence (b) diffraction (c) polarization (d) interference

3.4. A plane electromagnetic wave of the form


ur
E = yE0 cos 2π 5 × 1014 sec −1 t − 2.5 × 106 m−1 x 
( ) ( )
 
Where E0 is a constant and y is the unit vector along y-direction) represents a
wave propagating along
(a) + x direction (b) + y direction (c) - x direction (d) - y direction

3.5. While you are listening to a programme from a radio, if a near by electric light
bulb is switched on or switched off, you hear a momentary noise in your radio.
This is due to electromagnetic radiation emitted by _____.

3.6. Nuclear fusion reactions required very high temperatures so as to overcome


(a) nuclear forces (b) van der waals forces
(c) coulomb forces (d) gravitational forces

3.7. In radioactive decay, the disintegration rate of the nuclei is:


(a) constant at all times
(b) inversely proportional to half-life of the nuclei
(c) inversely proportional to the number of nuclei at any time
(d) directly proportional to the number of nuclei at any time

3.8. In an hydrogen atom 10.2 eV is given out as radiation when an electron is de-
excited to the ground state. The principal quantum number of the excited state is
___________.

Join All India Mock GATE Classroom Test Series - 2007 conducted by GATE Forum in over 25 cities all over India. Question
Papers including section tests and full tests are designed by IISc alumni according to the latest syllabus. Percentile, All India Rank,
interaction with IISc alumni in our online discussion forums, and more.
visit
www.gateforum.com
Think GATE Think GATE Forum
GATE EC - 1993 www.gateforum.com
Join discussion of this test paper at http://forum.gatementor.com

3.9 Typical current voltage characteristic of a solar cell is given in the following figure
by
A B C D

Current
Voltage

(a) curve A (b) curve B (c) curve C (d) curve D

3.10. Consider a solid sphere and a hollow sphere, both of mass M, radius R and
initially at rest, which start rolling down the same inclined plane without slipping.
At the bottom of the inclined plane, the ratio of speeds Vsolid / Vhollow is:

12 10 25
(a) 1 (b) (c) (d)
7 7 21
[Note: The moment of inertia about any diameter for a solid sphere is (2/5) MR2,
and for a hollow sphere (2/3) MR2].

3.11. An optical fibre consists of a cylindrical dielectric rod of refractive index n1 ,


surrounded by another dielectric of refractive index n2 , where n2 < n1 , as shown
in the following figure. If a ray is incident from air at an angle i to the axis, then
it undergoes total internal reflection at the interface AB if

(a) i ≥ sin−1 n12 − n22 Air


n2
A B
−1
(b) i ≥ sin n1 − n2 n1

i
(c) i ≤ sin−1 n12 − n22

(d) i = sin−1 n1 − n2

3.12. For a uniformly charged sphere of radius R and charge density p, the ratio of
R
magnitude of electric fields at distance and 2R from the centre, i.e.,
2
 R
E r = 
 2
is _______.
E ( r = 2R )

Join All India Mock GATE Classroom Test Series - 2007 conducted by GATE Forum in over 25 cities all over India. Question
Papers including section tests and full tests are designed by IISc alumni according to the latest syllabus. Percentile, All India Rank,
interaction with IISc alumni in our online discussion forums, and more.
visit
www.gateforum.com
Think GATE Think GATE Forum
GATE EC - 1993 www.gateforum.com
Join discussion of this test paper at http://forum.gatementor.com

3.13. A long solenoid of radius R, and having N turns per unit length carries a time
dependent current I ( t ) = I0 cos (ω t ) . The magnitude of induced electric field at a
R
distance readily from the axis of the solenoid is
2
R R
(a) µ0 NI0ω sin (ω t ) (b) µ0 NI0ω cos (ω t )
2 2
R R
(c) µ0 NI0ω sin (ω t ) (d) µ0 NI0ω cos (ω t )
2 2

3.14. In an electron diffraction experiment, planes of a crystal with spacing 1A°


between them yield the first maximum at a Bragg angle of θ=30°. The
momentum of the electrons is _____ J-s/m.

3.15. A conventional unit cell of close packed face centered cubic (FCC) structure made
up of hard spheres has a cube edge of a A°. The radius of the sphere is _____
A°.

3.16. A light beam of frequency 1.2 × 1015 Hz is incident on a metal in a photoelectric


effect experiment. The corresponding maximum kinetic energy of the ejected
photoelectrons from the metal is 6.63 × 10−19 J. The characteristic cut-off
frequency of the metal is _______ Hz.

3.17. Consider the semiconductors A and B. the figure shows variation of In p with I/T,
where p is resistivity and T the temperature, for the two semiconductors. Choose
the correct statements(s).
B

In A
P

1 −1
T
(
K )
(a) the bandgap energy of A is larger than that of B.
(b) the bandgap energy of A is smaller than that of B.
(c) the maximum wavelength of light needed to create an electron hole pair is
larger in A than in B
(d) the maximum wavelength of light needed to create an electron hole pair is
smaller in A than in B

Join All India Mock GATE Classroom Test Series - 2007 conducted by GATE Forum in over 25 cities all over India. Question
Papers including section tests and full tests are designed by IISc alumni according to the latest syllabus. Percentile, All India Rank,
interaction with IISc alumni in our online discussion forums, and more.
visit
www.gateforum.com
Think GATE Think GATE Forum
GATE EC - 1993 www.gateforum.com
Join discussion of this test paper at http://forum.gatementor.com

SECTION F

4. The questions 4.1 to 4.9 below are multiple choice questions carrying 1
mark each. There could one or two or more choices for the answer.
Indicate your choice by writing the appropriate capital letter/letters
against each question number. No credit will be given for writing only
one choice in cases where more than one correct answers are possible.
Answer the questions 4.1 to 4.9 on one page, in the same order in which
they appear.

4.1. A dc circuit shown in figure has a voltage source V, a current source I and several
resistors. A particular resistor R dissipates a power of 4 Watts when V alone is
active. The same resistor R dissipates a power of 9 Watts when I alone is active.
The power dissipated by R when both sources are active will be
(a) 1 W
+ Resistive
(b) 5 W R
V network
(c) 13 W −

(d) 25 W

I
4.2. In the series circuit shown in figure for series resonance, the value of the
coupling coefficient K will be
K
(a) 0.25
(b) 0.5
(c) 0.999 18Ω -j12Ω j2Ω j8Ω
(d) 1.0

4.3. If the secondary winding of the ideal transformer shown in the circuit of the
figure has 40 turns, the number of turns in the primary winding for maximum
power transfer to the 2° resistor will be
Ideal
8Ω Transformer

2Ω
Vg ~

40
turns

(a) 20 (b) 40 (c) 80 (d) 160

Join All India Mock GATE Classroom Test Series - 2007 conducted by GATE Forum in over 25 cities all over India. Question
Papers including section tests and full tests are designed by IISc alumni according to the latest syllabus. Percentile, All India Rank,
interaction with IISc alumni in our online discussion forums, and more.
visit
www.gateforum.com
Think GATE Think GATE Forum
GATE EC - 1993 www.gateforum.com
Join discussion of this test paper at http://forum.gatementor.com

4.4. While starting a dc shunt motor:


(a) reduced armature voltage Vo and reduced field voltage Vf should be applied
and full regulator resistance Rr should be included in the field circuit.
(b) reduced Vo but full Vf should be applied and Rr should be zero.
(c) rated Vo but rated Vf should be applied and Rr should be zero.
(d) rated Vo and rated Vf should be applied and Rr should be maximum.

4.5. A 6 pole 3 phase wound rotor induction machine is driven by another machine at
180 rpm. The rotor of the induction machine is connected to a 50 Hz system. If
the mechanical rotation of the rotor is in the same direction as the rotor winding
flux rotation, then the frequency of the stator voltage will be
(a) 50 Hz (b) 140 Hz (c) 150 Hz (d) 200 Hz

4.6. For the amplifier circuit of figure, the transistor a B of 800. The mid-band voltage
gain V0 V1 of the circuit will be
+15V

200KΩ
470Ω

4.7µF
6.4µF
+
VO
Vi 100KΩ

(a) 0 (b) < 1 (c) = 1 (d) 800

4.7. The wave shape of V0 in figure will be


4.1V 4.1V

10sin314t ~ 10K VO

Join All India Mock GATE Classroom Test Series - 2007 conducted by GATE Forum in over 25 cities all over India. Question
Papers including section tests and full tests are designed by IISc alumni according to the latest syllabus. Percentile, All India Rank,
interaction with IISc alumni in our online discussion forums, and more.
visit
www.gateforum.com
Think GATE Think GATE Forum
GATE EC - 1993 www.gateforum.com
Join discussion of this test paper at http://forum.gatementor.com

5.9
(a)
0

-5.9

(b) 4.1

VO
0

-4.1

(c) 4.1

-4.1

5.9

(d) 0 t (ms)
2 4 6 8 10 12 14 16 18

4.8. For the logic circuit shown in figure, the output Y is equal to

B
Y

(a) ABC (b) A + B + C (c) AB + BC + A + C

4.9. In figure, the ideal moving iron voltmeter M will read


(a) 7.07 V M

(b) 12.24 V
(c) 14.14 V Ideal diode
+ 100µF
(d) 20.0 V ~ 10sin314t

Join All India Mock GATE Classroom Test Series - 2007 conducted by GATE Forum in over 25 cities all over India. Question
Papers including section tests and full tests are designed by IISc alumni according to the latest syllabus. Percentile, All India Rank,
interaction with IISc alumni in our online discussion forums, and more.
visit
www.gateforum.com
Think GATE Think GATE Forum
GATE EC - 1993 www.gateforum.com
Join discussion of this test paper at http://forum.gatementor.com

5. The question 5.1 to 5.8 below carry 2 marks each. Give the complete calculation
necessary for arriving at the results.

5.1. In the circuit of figure, when switch S1 is closed, the ideal ammeter M1 reads 5A.
What will be ideal voltmeter M2 read when S1 is kept open? (The value of E is not
specified). M2
V

4Ω 3Ω

M1
A
6Ω
− +
E 10Ω
5Ω
8Ω
2Ω 3Ω

5.2. For the circuit shown in figure, determine the readings of the two wattmeters.
Phase sequence is RYB.
R

W1 L

400V L L
3Phase
R R
50Hz
supply

Y R=50
L=275.66mH
W2
B

5.3. Figure shows the core and armature of a relay. Flux of 50 mWb is produced in
this circuit. Find the force in Newtons acting on the armature. Neglect fringing.
0.0254cm

Core
Armature

0.635cm 2.54cm

7.62cm
0.635cm
Plan -0.635cm

Join All India Mock GATE Classroom Test Series - 2007 conducted by GATE Forum in over 25 cities all over India. Question
Papers including section tests and full tests are designed by IISc alumni according to the latest syllabus. Percentile, All India Rank,
interaction with IISc alumni in our online discussion forums, and more.
visit
www.gateforum.com
Think GATE Think GATE Forum
GATE EC - 1993 www.gateforum.com
Join discussion of this test paper at http://forum.gatementor.com

5.4. In a 110 volts compound generator, the armature, shunt and series windings are
0.06Ω, 27.5Ω and 0.04Ω respectively. The load consists of 200 lamps each rated
at 55W, 110V. Find the total e.m.f an armature current when the machine is
connected for long shunt operation.

5.5. Two 500 kVA alternators operate in parallel to supply the following loads:
(i) 250 KW at 0.95 p.f. lagging
(ii) 100 KW at 0.85 p.f. leading
One machine is supplying 200 KW at 0.9 p.f. laying. Calculate the p.f. of the
other machine.

5.6. For the ideal op-amp circuit of figure shown, determine the outpour voltage Vo .

100Ω 99Ω

+

100Ω

100Ω
VO
+
4V

5.7. The truth table for the output Y in terms of three inputs A, B and C are given in
table. Draw a logic circuit realization using only NOR gates.

A 0 1 0 1 0 1 0 1
B 0 0 1 1 0 0 1 1
C 0 0 0 0 1 1 1 1
Y 1 1 1 0 1 0 0 0

5.8. M in figure, is a rectifier type 200 V full scale voltmeter having a sensitivity of 10
kΩ/Volt. What will be the reading in M if the source voltage Vo is a symmetrical
square wave of 800 volts peal to peak?

2MΩ

Ideal diode

VS ~ M

Join All India Mock GATE Classroom Test Series - 2007 conducted by GATE Forum in over 25 cities all over India. Question
Papers including section tests and full tests are designed by IISc alumni according to the latest syllabus. Percentile, All India Rank,
interaction with IISc alumni in our online discussion forums, and more.
visit
www.gateforum.com
Think GATE Think GATE Forum
GATE EC - 1993 www.gateforum.com
Join discussion of this test paper at http://forum.gatementor.com

PART II

Read the following instructions carefully:

1. This part contains 16 questions: 6 to 21. Answer all the questions. Question 6
consists of 25 sub-questions each carrying 2 marks. All the sub-questions must
be answered in the sequence in which they appear.
2. In the multiple choice questions, four alternatives A, B, C and D are given, of
which more than one may be correct. Indicate the correct alternative(s) by
writing the corresponding letter (s) in CAPITALS. Marks will be given only if all
the correct alternatives are written. For the “Fill in the blanks” type of questions
and for questions asking for illustrations, sketches etc. only the answers must be
written against the relevant question number in the answer book.
3. Questions 7.21 carry 5 marks each. Answer to each question should appear at
one place only. Answers must be neatly and legibly written and well separated
from any rough work.
4. Make reasonable assumptions wherever necessary. There is no negative marking.

6.1. A network contains linear resistors and ideal voltage sources. If values of all the
resistors are doubled, then the voltage across each resistor is
(a) halved (b) doubled
(c) increased by four times (d) not changed

6.2. The two electrical sub network N1 and N2 are connected through three resistors
as shown in figure. The voltage across 5 ohm resistor and 1 ohm resistor are
given to be 10 V and 5V, respectively. Then voltage across 15 ohm resistor is
+ 10V −

15Ω 5Ω
N1 N2
1Ω
+ − 5V

(a) - 105 V (b) + 105 V (c) - 15 V (d) + 15 V

6.3. In figure, A1 , A2 and A3 are ideal ammeters? If A1 reads 5A, A2 reads 12A, then
A3 should read.
R
(a) 7 A A1

(b) 12 A A3
C
(c) 13 A A2
(d) 17 A 100sinωt
~

Join All India Mock GATE Classroom Test Series - 2007 conducted by GATE Forum in over 25 cities all over India. Question
Papers including section tests and full tests are designed by IISc alumni according to the latest syllabus. Percentile, All India Rank,
interaction with IISc alumni in our online discussion forums, and more.
visit
www.gateforum.com
Think GATE Think GATE Forum
GATE EC - 1993 www.gateforum.com
Join discussion of this test paper at http://forum.gatementor.com

K
6.4. If τ F ( s ) =  f ( t )  = then lim f ( t ) is given by
( s + 1) ( s 2 + 4 ) t →∞

K
(a) (b) zero (c) 0 < A < 12 (d) 5 < A < 12
4

6.5. If s3 + 3s2 + 4s + A = 0, then all the roots of this equation are in the left half plane
provided that
(a) A > 12 (b) -3 < A < 4 (c) 0 < A < 12 (d) 5 < A < 12

6.6. The built-in potential (Diffusion Potential) in a p-n junction


(a) is equal to the difference in the Fermi level of the two sides, expressed in
volts.
(b) increases with the increase in the doping levels of the two sides
(c) increases with the increase in temperature
(d) is equal to the average of the Fermi levels of the two sides

6.7. α - cut off frequency of a bipolar junction transistor


(a) increase with the increase in base width
(b) increases with the increase in emitter width
(c) increase with increase in the collector width
(d) increase with decrease in the base width

6.8. Negative feedback in amplifiers


(a) improves the signal to noise ratio at the input
(b) improves the signal to noise ratio at the output
(c) does not affect the signal to noise ratio at the output
(d) reduces distortion

6.9. In a multi-stage R-C coupled amplifier the coupling capacitor


(a) limits the low frequency response
(b) limits the high frequency response
(c) does not affect the frequency response
(d) blocks the d.c. component without affecting the frequency response

6.10. The bandwidth of an n-stage tuned amplifier, with each stage having a band
width of B, is given by
1
B B B
(a) (b) (c) B 2 n − 1 (d)
n n 1
2 −1
n

Join All India Mock GATE Classroom Test Series - 2007 conducted by GATE Forum in over 25 cities all over India. Question
Papers including section tests and full tests are designed by IISc alumni according to the latest syllabus. Percentile, All India Rank,
interaction with IISc alumni in our online discussion forums, and more.
visit
www.gateforum.com
Think GATE Think GATE Forum
GATE EC - 1993 www.gateforum.com
Join discussion of this test paper at http://forum.gatementor.com

6.11. In a transistor push-pull amplifier


(a) there is no d.c. present in the output
(b) there is no distortion in the output
(c) there are no even harmonics in the output
(d) there are no odd harmonics in the output

6.12. 2’s complement representation of a 16-bit number (one sign bit and 15
magnitude bits) is FFFI. Its magnitude in decimal representation is
(a) 0 (b) 1 (c) 32, 767 (d) 65, 535

6.13. Boolean expression for the output of XNOR (Equivalent) logic gate with inputs A
and B is:
(a) AB + AB (b) AB + AB (c) ( A + B )( A + B ) (
(d) A + B ) ( A + B)
6.14. Consider the circuit shown in figure (a). If the diode used here has the V-I
characteristic as in figure (b), then the output wave form ν 0 is:

Vi dν
= 300Ω
dI
2V for
2π VO V=0.5V
π V
2V 0.5V

Figure (a) Figure (b)

(a) (b)
2V 2π
0 π

0 π 2π
2V

2V
(c) (d) 2V

0 π 2π
0 π 2π

6.15. A pulse train with a frequency of 1 MHz is counted using a modulo 1024 ripple
counter built with J-K flip flops. For proper operation of the counter, the
maximum permissible propagation delay per flip flop stage is ______ n sec.

Join All India Mock GATE Classroom Test Series - 2007 conducted by GATE Forum in over 25 cities all over India. Question
Papers including section tests and full tests are designed by IISc alumni according to the latest syllabus. Percentile, All India Rank,
interaction with IISc alumni in our online discussion forums, and more.
visit
www.gateforum.com
Think GATE Think GATE Forum
GATE EC - 1993 www.gateforum.com
Join discussion of this test paper at http://forum.gatementor.com

6.16. In a microprocessor, the register which holds the address of the next instruction
to be fetched is:
(a) accumulator (b) program counter
(c) stack pointer (d) instruction register

6.17. In a microcomputer, wait states are used ot


(a) make the processor wait during a DMA operation
(b) make the processor wait during an interrupt processing
(c) make the processor wait during a power shutdown
(d) interface slow peripherals to the processor

6.18. The function shown in figure, can represent a probability density function for A
_____
f(x)

3A

2
A

3 2 1 0 1 2 3

6.19. Which of the following demodulator(s) can be used for demodulating the signal
x ( t ) = 5 (1 + 2 cos 2000π t ) cos 2000π t.

(a) Envelope demodulator (b) square law demodulator


(c) synchronous demodulator (d) None of the above

6.20. A superheterdyne radio receiver with an intermediate frequency of 455 KHz is


tuned to a station operating at 1200 KHz. The associated image frequency is
______ KHz.

6.21 Sketch the waveform (with properly marked axes) at the output of a matched
filter matched for a signal s ( t ) , of duration T, given by

 2
 A for 0 < 1 < T
3
S (t ) = 
6.22 for 2 T ≤ t < T
 3

6.22 Six independent low pass signals of bandwidth 3W, W, W, 2W, 3W and 2W Hz are
to be time division multiplexer on a common channel using PAM. To achieve this,
the minimum transmission bandwidth of the channel should be _______.

Join All India Mock GATE Classroom Test Series - 2007 conducted by GATE Forum in over 25 cities all over India. Question
Papers including section tests and full tests are designed by IISc alumni according to the latest syllabus. Percentile, All India Rank,
interaction with IISc alumni in our online discussion forums, and more.
visit
www.gateforum.com
Think GATE Think GATE Forum
GATE EC - 1993 www.gateforum.com
Join discussion of this test paper at http://forum.gatementor.com

6.23 A material is described by the following electrical parameters as a frequency of


σ
10 GHz, σ = 106 mho/m, µ = µ0 and = 10. The material at this frequency is
σ0
considered to be
 1 −9 
 σ 0 = 36π × 10 F / m 
 
(a) a good conductor
(b) a good dielectric
(c) neither a good conductor, nor a good dielectric
(d) a good magnetic material

6.24. Consider a transmission line of characteristic impedance of 50 ohm. Let it be


terminated at one end by +j50 ohm. The VSWR produced by it in the
transmission line will be
(a) +1 (b) 0 (c) ∞ (d) +j

6.25. A plane waveuris incident normally on a perfect conductor as shown in figure. Here
E xr = Hr′ and P are electric field, magnetic field and Poynting vector respectively,
for the incident wave. The reflected wave should be
(a) E xr = −E x′ Exi
x

(b) Hy = −Hy
ur ur z
(c) P = −P
Pi
(d) E x′ = E x Hiy

7. An infinite grid is built up by connecting resistors in the manner indicated in


figure, where each branch represents one ohm resistor. Calculate the effective
resistance between the nodes A and B.

8. Find the Y-parameters (short circuit admittance parameters) for the network
shown in figure.
1F

1Ω 2Ω 2
1

1′ 2′

Join All India Mock GATE Classroom Test Series - 2007 conducted by GATE Forum in over 25 cities all over India. Question
Papers including section tests and full tests are designed by IISc alumni according to the latest syllabus. Percentile, All India Rank,
interaction with IISc alumni in our online discussion forums, and more.
visit
www.gateforum.com
Think GATE Think GATE Forum
GATE EC - 1993 www.gateforum.com
Join discussion of this test paper at http://forum.gatementor.com

9. Consider the following interconnection of the three LTI systems (figure),


h1 ( t ) , h2 ( t ) and h3 ( t ) are the impulse responses of these three LTI systems with
H1 (ω ) , H2 (ω ) and H3 (ω ) as their respective Fourier transforms. Given that

d  sin ω0t 
h1 ( t ) =
dt  2π t  x(t) + y(t)
h1 ( t ) + h3 ( t )
 −2πω  −
h2 ( t ) = exp  
 ω0 
h3 ( t ) = u ( t ) , and h2 ( t )
ω t 
x ( t ) = sin 2ω0t + cos  0 
 2 
Find the output y ( t ) .

10. The reverse saturation current of the collector base junction ( ICBO ) of a BJT is
found to be 10n A at low collector voltages. The low voltage current amplification
factor (α) is 0.98. Find out the change in collector current with its base open
( ICEO ) when the collector voltage is increased such that α increases by the 1.0%.

11. A JFET with the following parameters is used in a single stage common source
amplifier with a load resistance of 100kΩ. Calculate the high frequency cut off
(upper 3 dB cut off frequency) of the amplifier.
Gm = 2.0mA / V
C gd = 2.0 pF
rd = 100k Ω
C gd = 2.0 pF
C gd = 1.0 pF

12. In the following circuit the capacitance varies as C = KW, where K is a constant
equal to 0.5 Farads/Coulomb and Q, the charge on the capacitor in Coulombs.
Determine the current through the circuit and sketch the voltage waveform
across the capacitor (Vc ) for a step input Vi as shown in figure.

i 3KΩ
+
5V Vi C VC

Vi

0 t

Join All India Mock GATE Classroom Test Series - 2007 conducted by GATE Forum in over 25 cities all over India. Question
Papers including section tests and full tests are designed by IISc alumni according to the latest syllabus. Percentile, All India Rank,
interaction with IISc alumni in our online discussion forums, and more.
visit
www.gateforum.com
Think GATE Think GATE Forum
GATE EC - 1993 www.gateforum.com
Join discussion of this test paper at http://forum.gatementor.com

13. Find the output voltage, V0 in the following circuit (figure), assuming that the op-
amps are ideal.
1KΩ 2KΩ

2KΩ

2KΩ
− 2KΩ
A −
+ B VO
+
1V
2V

14. Signals A, B, C, D and D are available. Using a single 8 to 1 multiplexer and no


other gate, implement the Boolean function f ( A, B, C , D ) = B.C + A.B.D + A.C.D

15. A clocked sequential circuit has three states, A, B and C and one input X. As long
as the input X is 0, the circuit alternates between the states A and B. If the input
X becomes 1 (either in state A or in state B), the circuit goes to state C and
remains in state C as long as X continues to be 1. The circuit returns to state A if
the input becomes 0 once again and from then on repeats its behaviour. Assume
that the state assignments are A = 00, B = 01 and C = 10.
(a) Draw the state diagram of the circuit
(b) Give the state table for the circuit
(c) Draw the circuit using D flip flops

16. A microprocessor has five address lines  A0 − A4  and eight data lines D0 − D7 .
An input device A, an output device B, A ROM and a RAM are memory mapped to
the microprocessor at the addresses as shown in figure. Devices A and B have
four addressable registers each; RAM has 8 bytes and ROM has 16 bytes.

B 1F
1C
ROM 1B
0C
RAM 0B
04
A 03
00

(a) Indicate the address lines to be connected to each device and memory.
(b) Obtain the minimum sum of product expression for the chip select (CS)
function of each device/memory.

Join All India Mock GATE Classroom Test Series - 2007 conducted by GATE Forum in over 25 cities all over India. Question
Papers including section tests and full tests are designed by IISc alumni according to the latest syllabus. Percentile, All India Rank,
interaction with IISc alumni in our online discussion forums, and more.
visit
www.gateforum.com
Think GATE Think GATE Forum
GATE EC - 1993 www.gateforum.com
Join discussion of this test paper at http://forum.gatementor.com

17. It is desired to generate a random signal x(t), with autocorrelation function,


−5 r
Rx (T ) = 5η e , by passing white noise n(t), with power spectral density
η
Sn ( f ) =
watts / Hz, through a LTI system. Obtain an expression for the transfer
2
function H(f) of the LTI system.

18. Obtain an expression for the signal ν 3 ( t ) in figure, for


ν 1 ( t ) = 10 cos (2000π t ) + 4 sin (200π t ) . Assume that ν 2 ( t ) = ν 1 ( t ) + 0.1ν 2
1 (t ) and
that the BPF is an ideal unity gain filter with pass band from 800 Hz to 1200 Hz.

Non-linear
BPF
V1(t) system V2(t) V3(t)

19. A low pass signal m(t) band limited to B Hz is sampled by a periodic rectangular
1
pulse train, Pr ( t ) of period T2 = sec. assuming natural sampling and that the
(3B )
I
pulse amplitude and pulse width are A volts and sec, respectively, obtain
(30B )
expression for the frequency spectrum of the sampled signal m2 ( t ) .

20. Consider an array of two non-directional radiators with spacing d = 0.5λ.


Determine the directions of maximum radiation when the radiators are excited as
shown in figure. Calculate the phase shift required for turning the direction of the
maximum radiators by 90°, keeping the separation, d, unchanged.

I∠0° φ I∠0° φ

d=0.5λ

21. Match the following descriptions with each of the diagrams given in figure. Fields
are near the interface, but on opposite sides of the boundary. Vectors are drawn
to scale.
(a) Medium 1 and medium 2 are dielectrics with ε1 > ε 2
(b) Medium 1 and medium 2 are dielectrics with ε1 < ε 2
(c) Medium 2 is a perfect conductor
(d) Medium 1 is a perfect conductor.
E2
E1 D2
D1 E2
1 E1
1 H1 =0 1
1 E1 1

2 E2 2 2
2
2 H2 =0
(i) (ii)
(iii) (iv) (v)
Join All India Mock GATE Classroom Test Series - 2007 conducted by GATE Forum in over 25 cities all over India. Question
Papers including section tests and full tests are designed by IISc alumni according to the latest syllabus. Percentile, All India Rank,
interaction with IISc alumni in our online discussion forums, and more.
visit
www.gateforum.com
Think GATE Think GATE Forum
GATE IES PSU’S-2021
GATE ACADEMY
Leading inStitUte for eSe/gate/PSU’S

ECE ENGINEERING
NOTED-: These Are Only Sample Notes,
If You Want Complete Full GATE
ACADEMY GATE IES PSU ECE Branch
ALL SUBJECT FULL NOTES BUY Click
below Link Buy

Noted-: Download File Zip format so use zip rar app extract zip
file easily open folder available all subject pdf.
GATE EC - 1998 www.gateforum.com
Join discussion of this test paper at http://forum.gatementor.com

SECTION - A
1. For each of the following (1.1 – 1.40) four alternatives (A, B, C and D) are given,
out of which ONLY ONE is correct. Indicate the correct answer by writing (A, B, C
or D) as appropriate, against the corresponding question number in the answer
book.

1.1 A network has 7 nodes and 5 independent loops. The number of branches in the
network is
(a) 13 (b) 12 (c) 11 (d) 10

0 1 
1.2 The eigen values of the matrix A =   are
1 0
(a) 1,1 (b) -1,-1 (c) j,-j (d) 1,-1

ω
1.3 If f ( t ) = , then the value of lim f ( t )
s + ω2
2 t →∞

(a) cannot be determined (b) is zero


(c) is unity (d) is infinite

1.4 The trigonometric Fourier series of a periodic time function can have only
(a) cosine terms (b) sine terms
(c) cosine and sine terms (d) d.c. and cosine terms

1.5 The nodal method of circuit analysis is based on


(a) KVL and Ohm’s law (b) KCL and Ohm’s law
(c) KCL and KVL (d) KCL, KVL and Ohm’s law

1.6 Superposition theorem is NOT applicable to networks containing


(a) nonlinear elements (b) dependent voltage sources
(c) dependent current sources (d) transformers

1.7 The parallel RLC circuit shown in Fig.1.7 is in resonance. In this cicuit
(a) IR < 1mA
IR IL IC
(b) IR + IL > 1mA
1mA RMS R L
(c) IR + IC < 1mA

(d) IR + IC > 1mA

Join All India Mock GATE Classroom Test Series - 2007 conducted by GATE Forum in over 25 cities all over India. Question
Papers including section tests and full tests are designed by IISc alumni according to the latest syllabus. Percentile, All India Rank,
interaction with IISc alumni in our online discussion forums, and more. For more details,
visit
www.gateforum.com
Think GATE Think GATE Forum
GATE EC - 1998 www.gateforum.com
Join discussion of this test paper at http://forum.gatementor.com

1.8 A periodic signal x(t) of period To is given by

1, t < T1

x (t ) =  To
0, T1 < t <
 2
The d.c. component of x(t) is
T1 T1 2T1 T0
(a) (b) (c) (d)
T0 2T0 T0 T1

1.9 The unit impulse response of a linear time invariant system is the unit step
function u(t). For t>0, the response of the system ot an excitation
e − at u ( t ) , a > 0 will be

1
(a) ae − at (
(b)   1 − e − at
 a
) (
(c) a 1 − e − at ) (d) 1 − e − at

 −1 
0 2 
1.10 The short-circuit admittance matrix of a two-port network is 
1 0 
 2 
The two port network is
(a) non-reciprocal and passive (b) non-reciprocal and active
(c) reciprocal and passive (d) reciprocal and active

1.11 The voltage across the terminals a and b in Fig.1.11 is


(a) 0.5 V 2Ω a 1Ω
(b) 3.0 V
(c) 3.5 V + 2Ω 1H 3A
(d) 4.0 V 1V
-

1.12 The electron and hole concentrations in a intrinsic semiconductor are ni and pi
respectively. When doped with a p-type material, these change to n and p,
respectively. Then
(a) n + p = ni + pi (b) n + ni = p + pi (c) npi = ni p (d) np = ni pi

1.13 If fT of BJT is related to its gm , Cπ and C µ as follows:

(a) fT =
Cπ + C µ
(b) fT =
(
eπ Cπ + C µ )
gm gm
gm gm
(c) fT = (d) fT =
Cπ + C µ (
2π Cπ + C µ )
Join All India Mock GATE Classroom Test Series - 2007 conducted by GATE Forum in over 25 cities all over India. Question
Papers including section tests and full tests are designed by IISc alumni according to the latest syllabus. Percentile, All India Rank,
interaction with IISc alumni in our online discussion forums, and more. For more details,
visit
www.gateforum.com
Think GATE Think GATE Forum
GATE EC - 1998 www.gateforum.com
Join discussion of this test paper at http://forum.gatementor.com

1.14 The static characteristic of an adequately forward biased p-n junction is a straight
line, if the plot is of
(a) log I vs. log V (b) log I vs. V (c) I vs. log V (d) I vs. V

1.15 A long specimen of p-type semiconductor material


(a) is positively charged
(b) is electrically neutral
(c) has an electric field directed along its length
(d) acts as a dipole


1.16 The Z-transform of the time function ∑ δ ( n − k ) is
k =0

(a)
Z −1
(b)
Z
(c)
Z
(d)
( Z − 1)2
Z Z −1 ( Z − 1)2 Z

1.17 The number of roots of s3 + 5s2 + 7s + 3 = 0 in the left half of the s-plane is
(a) zero (b) one (c) two (d) three

1.18 The transfer function of a tachometer is of the form


K K K
(a) Ks (b) (c) (d)
s ( + 1)
s s ( s + 1)

1.19. Consider a unity feedback control system with open-loop transfer function
K
G (s) = . The steady state error of the system due to a unit step input is
s ( s + 1)

1
(a) zero (b) K (c) (d) infinite
K

1.20. The transfer function of a zero-order-hold system is


1 1
(
(a)   1 + e − sT
s
) (
(b)   1 − e − sT
s
)
1 1
(c) 1 −   e − sT (d) 1 +   e − sT
s s

1.21. In the Bode-plot of a unity feedback control system, the value of phase of G(jω)
at the gain cross over frequency is –125°. The phase margin of the system is
(a) -125° (b) -55° (c) 55° (d) 125°

Join All India Mock GATE Classroom Test Series - 2007 conducted by GATE Forum in over 25 cities all over India. Question
Papers including section tests and full tests are designed by IISc alumni according to the latest syllabus. Percentile, All India Rank,
interaction with IISc alumni in our online discussion forums, and more. For more details,
visit
www.gateforum.com
Think GATE Think GATE Forum
GATE EC - 1998 www.gateforum.com
Join discussion of this test paper at http://forum.gatementor.com

1.22. Consider a feedback control system with loop transfer fucntion


K (1 + 0.5s )
G (s) H (s) =
s (1 + s ) (1 + 2s )

The type of the closed loop system is


(a) zero (b) one (c) two (d) three

1 + 3Ts
1.23. The transfer function of a phase lead controller is . The maximum value of
1 + Ts
phase provided by this controller is
(a) 90° (b) 60° (c) 45° (d) 30°

1.24. The Nyquist plot of a phase transfer function g(jω) H(jω) of a system encloses the
(-1,0) point. The gain margin of the system is
(a) less than zero (b) zero
(c) greater than zero (d) infinity

2s 2 + 6 s + 5
1.25 The transfer function of a system is
( s + 1)2 ( s + 2)
The characteristic equation of the system is

(a) 2s2 + 6s + 5 = 0 (b) ( s + 1)2 ( s + 2) = 0


2 2
(c) 2s2 + 6s + 5 + ( s + 1) ( s + 2) = 0 (d) 2s2 + 6s + 5 − ( s + 1) ( s + 2) = 0

n
1.26 In a synchro error detector, the output voltage is proportional to ω ( t )  , where
ω ( t ) is the rotor velocity and n equals
(a) -2 (b) -1 (c) 1 (d) 2

1.27 Two identical FETs, each characterissed by the parameters gm and rd are
connected in parallel. The composite FET is then characterized by the parameters
gm gm r rd
(a) and 2rd (b) and d (c) 2gm and (d) 2gm and 2rd
2 2 2 2

+VCC
1.28 The circuit of Fig.1.28 is an example of
feedback of the following type
VO
(a) current series
(b) current shunt
(c) voltage series Vi ~

(d) voltage shunt


Join All India Mock GATE Classroom Test Series - 2007 conducted by GATE Forum in over 25 cities all over India. Question
Papers including section tests and full tests are designed by IISc alumni according to the latest syllabus. Percentile, All India Rank,
interaction with IISc alumni in our online discussion forums, and more. For more details,
visit
www.gateforum.com
Think GATE Think GATE Forum
GATE EC - 1998 www.gateforum.com
Join discussion of this test paper at http://forum.gatementor.com

1.29 In a differential amplifier, CMRR can be improved by using an increased


(a) emitter resistance (b) collector resistance
(c) power supply voltages (d) source resistance

1.30 From a measurement of the rise time of the output pulse of an amplifier whose
input is a small amplitude square wave, one can estimate the following
parameter of the amplifier:
(a) gain-bandwidth product (b) slow rate
(c) upper 3-dB frequency (d) lower 3-dB frequency

1.31 A distorted sinusoidal has the amplitudes A1 , A2 , A3 K of the fundamental, second


harmonic, third harmonic, K respectively. The total harmonic distortion is

A2 + A3 + K A22 + A32 + K
(a) (b)
A1 A1

A22 + A32 + K  A2 + A32 + K 


(c) (d)  2 
 A1 
A12 + A22 + A32 K  

1.32 The emitter coupled pair of BJT’s gives a linear transfer relation between the
differential output voltage and the differential input voltage Vid only when the
magnitude of Vid is less α times the thermal voltage, where α is
(a) 4 (b) 3 (c) 2 (d) 1

1.33 In a shunt-shunt negative feedback amplifier, as compared to the basic amplifier


(a) both, input and output impedances, decrease
(b) input impedance decreases but output impedance increases
(c) input impedance increases but output impedance decreases
(d) both, input and output impedances increase

1.34 A multistage amplifier has a low-pass response with three real poles at s = -ω1,
-ω2 and ω3. The approximate overall bandwidth B of the amplifier will be given by
1 1 1 1
(a) B = ω1 + ω2 + ω3 (b) = + +
B ω1 ω2 ω3
1
(c) B = (ω1 + ω2 + ω3 ) 3 (d) B = ω12 + ω22 + ω32

Join All India Mock GATE Classroom Test Series - 2007 conducted by GATE Forum in over 25 cities all over India. Question
Papers including section tests and full tests are designed by IISc alumni according to the latest syllabus. Percentile, All India Rank,
interaction with IISc alumni in our online discussion forums, and more. For more details,
visit
www.gateforum.com
Think GATE Think GATE Forum
GATE EC - 1998 www.gateforum.com
Join discussion of this test paper at http://forum.gatementor.com

1.35 A high Q-quartz crystal exhibits series resonance at the frequency ω s and parallel
resonance at the frequency ω p . Then

(a) ω s is very close to, but less than ω p . (b) ω s ω p.

(c) ω s is very close to, but greater than ω p . (d) ω s ω p.

1.36 One input terminal of high gain comparator circuit is connected to ground and a
sinusoidal voltage is applied to the other input. The output of comparator will be
(a) a sinusoid (b) a full rectified sinusoid
(c) a half rectified sinusoid (d) a square wave

1.37 In a series regulated power supply circuit, the voltage gain Av of the ‘pass’
transistor satisfies the condition:
(a) Av  ∞ (b) 1 Av < ∞ (c) Av =1 (d) Av 1

1.38 For full wave rectification, a four diode bridge rectifier is claimed to have the
following advantages over a two diode circuit:
(1) less expensive transformer
(2) smaller size transformer, and
(3) suitability for higher voltage application.
Of these,
(a) only (1) and (2) are true (b) only (1) and (3) are true
(c) only (2) and (3) are true (d) (1), (2) as well as (3) are true

1.39 In the MOSFET amplifier of Fig.1.39, the signal outputs V1 and V2 obey the
relationship
V2
(a) V1 = RD
2
+
V V1
(b) V1 = 2 -
2 +
+ V2
(c) V1 = 2V2 RD -
Vi ~
2
(d) V1 = −2V2 -
-

q
1.40 the units of are
kT
J
(a) V (b) V −1 (c) J (d)
K

Join All India Mock GATE Classroom Test Series - 2007 conducted by GATE Forum in over 25 cities all over India. Question
Papers including section tests and full tests are designed by IISc alumni according to the latest syllabus. Percentile, All India Rank,
interaction with IISc alumni in our online discussion forums, and more. For more details,
visit
www.gateforum.com
Think GATE Think GATE Forum
GATE EC - 1998 www.gateforum.com
Join discussion of this test paper at http://forum.gatementor.com

2. For each of the following (2.1 – 2.40) four alternatives (A, B, C and D) are given,
out of which ONLY ONE is correct. Indicate the correct answer by writing (A, B, C
or D) as appropriate, against the corresponding question number in the answer
book.

2.1 The minimum number of 2-input NAND gates required to implement the Boolean
function Z = A B C , assuming that A, B and C are available, is
(a) two (b) three (c) five (d) six

2.2 The noise margin of a TTL gate is about


(a) 0.2 V (b) 0.4 V (c) 0.6 V (d) 0.8 V

2.3 In Fig.2.3, A = 1 and B =1, the input B is now replaced by a sequence 101010….
the outputs x and y will be
A X
(a) fixed at 0 and 1, respectively
(b) x = 1010 …. while y = 0101 ….
(c) x = 1010 …. and y = 0101 ….
(d) fixed at 1 and 0, respectively

Y
B

2.4 An equivalent 2’s complement representation of the 2’s complement number


1101 is
(a) 110100 (b) 001101 (c) 110111 (d) 111101

2.5 The threshold voltage for each transistor in Fig.2.5, is 2V. For this circuit to work
as an inverter, Vi must take the values
(a) -5 V and 0 V
(b) -5 V and 5 V
(c) -0 V and 3 V
(d) 3 V and 5 V Vi VO

-5V
2.6 An I/O processor control the flow of information between
(a) cache memory and I/O devices (b) main memory and I/O devices
(c) two I/O devices (d) cache and main memories

Join All India Mock GATE Classroom Test Series - 2007 conducted by GATE Forum in over 25 cities all over India. Question
Papers including section tests and full tests are designed by IISc alumni according to the latest syllabus. Percentile, All India Rank,
interaction with IISc alumni in our online discussion forums, and more. For more details,
visit
www.gateforum.com
Think GATE Think GATE Forum
GATE EC - 1998 www.gateforum.com
Join discussion of this test paper at http://forum.gatementor.com

2.7 Two 2’s complement number having sign bits x and y are added and the sign bit
of the result is z. Then, the occurrence of overflow is indicated by the Boolean
function
(a) x y z (b) x y z
(c) x y z + x y z (d) xy + yz + zx

2.8 The advantage of using a dual slope ADC in a digital voltmeter is that
(a) its conversion time is small (b) its accuracy is high
(c) it gives output in BCD format (d) it does not require a comparator

2.9 For the identify AB + AC + BC = AB + AC , the dual form is


(a) ( A + B) ( A + C ) (B + C ) = ( A + B) ( A + C )
(b) ( A + B ) ( A + C )( B + C ) = ( A + B ) ( A + C )
(c) ( A + B) ( A + C ) (B + C ) = ( A + B ) ( A + C )
(d) A B + A C + B C = A B + A C

2.10 An instruction used to set the carry Flag in a computer can be classified as
(a) data transfer (b) arithmetic
(c) logical (d) program control

2.11 Fig.2.11 shows a mod-K


counter, here K is equal to
(a) 1 J Q J Q
(b) 2
K Q 1 K Q
(c) 3
(d) 4
CLK

2.12 The current I through resistance r in the circuit shown in Fig.2.12 is


r

I
R R R 2R
-
R 2R +
2R 2R

0 V 0
−V V V V
(a) (b) (c) (d)
12R 12R 6R 3T
Join All India Mock GATE Classroom Test Series - 2007 conducted by GATE Forum in over 25 cities all over India. Question
Papers including section tests and full tests are designed by IISc alumni according to the latest syllabus. Percentile, All India Rank,
interaction with IISc alumni in our online discussion forums, and more. For more details,
visit
www.gateforum.com
Think GATE Think GATE Forum
GATE EC - 1998 www.gateforum.com
Join discussion of this test paper at http://forum.gatementor.com

2.13 The K-map for a Boolean function is shown in Fig.2.13. The number of essential
prime implicants for this function is
AB
(a) 4 00 01 11 10
CD
(b) 5 00 1 1 0 1
(c) 6
(d) 8 01 0 0 0 1

11 1 0 0 0

1 0 0 1
10

2.14 For small signal a.c. operation, a practical forward biased diode can be modeled
as
(a) a resistance and a capacitance in series
(b) an ideal diode and resistance in parallel
(c) a resistance and an ideal diode in series
(d) a resistance

2.15 The amplitude spectrum of a Gaussian pulse is


(a) uniform (b) a sine function
(c) Gaussian (d) an impulse function

2.16 The ACF of a rectangular pulse of duration T is


(a) a rectangular pulse of duration T
(b) a rectangular pulse of duration 2T
(c) a triangular pulse of duration T
(d) a triangular pulse of duration 2T

2.17 The image channel selectivity of super heterodyne receiver depends upon
(a) IF amplifiers only (b) RF and IF amplifiers only
(c) Preselector, RF and IF amplifiers
(d) Preselector, and RF amplifiers only

2.18 In a PCM system with uniform quantization, increasing the number of bits from 8
to 9 will reduce the quantization noise power by factor of
(a) 9 (b) 8 (c) 4 (d) 2

Join All India Mock GATE Classroom Test Series - 2007 conducted by GATE Forum in over 25 cities all over India. Question
Papers including section tests and full tests are designed by IISc alumni according to the latest syllabus. Percentile, All India Rank,
interaction with IISc alumni in our online discussion forums, and more. For more details,
visit
www.gateforum.com
Think GATE Think GATE Forum
GATE EC - 1998 www.gateforum.com
Join discussion of this test paper at http://forum.gatementor.com

2.19 The Fourier transform of a function x(t) is X(f). The Fourier transform of
dX ( f )
will be
df
dX ( f ) X (f )
(a) (b) j2π fX ( f ) (c) jfX ( f ) (d)
df jf

2.20 Flat top sampling of low pass signals


(a) gives rise to aperture effect (b) implies over sampling
(c) leads to aliasing (d) introduces delay distortion

2.21 A DSB-SC signal is generated using the carrier cos (ωet + θ ) and modulating singal
x(t). The envelope of the DSB-SC signal is
(a) x(t) (b) x (t )

(c) only positive portion of x(t) (d) x(t) cosθ

2.22 Quadrature multiplexing is


(a) the same as FDM (b) the same as TDM
(c) a combination of FDM and TDM
(d) quite different from FDM and TDM

2.23 The Fourier transform of a voltage signal x(t) is X(f). The unit of X ( f ) is

(a) volt (b) volt-sec (c) volt/sec (d) volt2

2.24 Compression in PCM refers to relative compression of


(a) higher signal amplitudes (b) lower signal amplitudes
(c) lower signal frequencies (d) higher signal frequencies

2.25 For a given data rate, the bandwidth Bp of a BPSK signal and the bandwidth B0 of
the OOK signal are related as
B0 B0
(a) Bp = (b) Bp = (c) Bp = B0 (d) Bp = 2B0
4 2

2.26 The spectral density of a real valued random process has


(a) an even symmetry (b) an odd symmetry
(c) a conjugate symmetry (d) no symmetry

2.27 The probability density function of the envelope of narrow band Gaussion noise is
(a) Poisson (b) Gaussian (c) Rayleigh (d) Rician
Join All India Mock GATE Classroom Test Series - 2007 conducted by GATE Forum in over 25 cities all over India. Question
Papers including section tests and full tests are designed by IISc alumni according to the latest syllabus. Percentile, All India Rank,
interaction with IISc alumni in our online discussion forums, and more. For more details,
visit
www.gateforum.com
Think GATE Think GATE Forum
GATE EC - 1998 www.gateforum.com
Join discussion of this test paper at http://forum.gatementor.com

2.28 The intrinsic impedance of copper at high frequencies is


(a) purely resistive (b) purely inductive
(c) complex with a capacitive component
(d) complex with an inductive component

∂D
2.29 The Maxwell equation V × H = J is based on
∂t
(a) Ampere’s law
(b) Gauss’ law
(c) Faraday’s law
(d) Coulomb’s law
λ λ
2.30 All transmission line sections 8 2
shown in Fig.2.30 have
Zin 2R0
characteristic impedance
Ro + j0. The input impedance
Zin equals
2 Ro
(a) Ro (b) Ro
3 2

3 1
(c) Ro (d) 2Ro
2 4

2.31 The time averages Poynting vector, in W/m2, for a wave with
ur ur
E = 24e (
j ωt + β z )
ay V/m in free space is
2.4 ur 2.4 ur 4.8 ur 4.8 ur
(a) − az (b) az (c) az (d) − az
π π π π

2.32 The wavelength of a wave with propagation constant (0.1π+j0.2π)m-1 is


2
(a) m (b) 10 m (c) 20 m (d) 30 m
0.05

2.33 The depth of penetration of wave in a lossy dielectric increases with increasing
(a) conductivity (b) permeability (c) wavelength (d) permittivity

ur ur ur
The polarization of wave with electric field vector E = E0e (
j ωt + β z )
2.34 ax + ay is ( )
(a) linear (b) elliptical
(c) left hand circular (d) right hand circular

Join All India Mock GATE Classroom Test Series - 2007 conducted by GATE Forum in over 25 cities all over India. Question
Papers including section tests and full tests are designed by IISc alumni according to the latest syllabus. Percentile, All India Rank,
interaction with IISc alumni in our online discussion forums, and more. For more details,
visit
www.gateforum.com
Think GATE Think GATE Forum
GATE EC - 1998 www.gateforum.com
Join discussion of this test paper at http://forum.gatementor.com

2.35 The vector H in the far field of an antenna satisfies


uur uur uur uur
(a) ∇.H = 0 and ∇ × H = 0 (b) ∇.H ≠ 0 and ∇ × H ≠ 0
uur uur uur uur
(c) ∇.H = 0 and ∇ × H ≠ 0 (d) ∇.H ≠ 0 and ∇ × H = 0

2.36 The radiation resistance of a circular loop of one turn is 0.01Ω. The radiation
resistance of five turns of such a loop will be
(a) 0.002Ω (b) 0.01Ω (c) 0.05Ω (d) 0.25Ω

2.37 An antenna in free space receives 2µW of power when the incident electric field is
20 mV/m rms. The effective aperture of the antenna is
(a) 0.005 m2 (b) 0.05 m2 (c) 1.885 m2 (d) 3.77 m2

2.38 The maximum usable frequency of an ionospheric layer at 60° incidence and with
8 mHz critical frequency is
16
(a) 16 MHz (b) MHz (c) 8 MHz
3
(d) about 6.93 MHz

ur ur
2.39 A loop is rotating about the y-axis in a magnetic field E = B0 cos (ω t + φ ) ax T . The
voltage in the loop is
(a) zero (b) due to rotation only
(c) due to transformer action only
(d) due to both rotation and transformer action

2.40 The far field of an antenna varies with distance r as


1 1 1 1
(a) (b) 2
(c) 3
(d)
r r r r

3. Determine the frequency of resonance and the resonant impedance of the parallel
circuit shown in Fig.3. What happens when L = CR2?
L R

C R

Join All India Mock GATE Classroom Test Series - 2007 conducted by GATE Forum in over 25 cities all over India. Question
Papers including section tests and full tests are designed by IISc alumni according to the latest syllabus. Percentile, All India Rank,
interaction with IISc alumni in our online discussion forums, and more. For more details,
visit
www.gateforum.com
Think GATE Think GATE Forum
GATE EC - 1998 www.gateforum.com
Join discussion of this test paper at http://forum.gatementor.com

4. A voltage source of internal impedance Rs + jX s supplies power to a load of


impedance RL + jX L in which only RL is variable. Determine the value of RL for
maximum power transfer from the source to the load. Also, find the numerical
value of RL if the source impedance is 3.0Ω (purely resistive) and X1 is 4.0Ω.

5. (a) Draw the transfer characteristic of the circuit of Fig.5, assuming both D1 and
D2 to be ideal.
(b) How would the characteristic change if D2 is ideal, but D1 is non-ideal in that
it has forward resistance of 10Ω and a reverse resistance of infinity?

1KΩ

+
+ D1 D2
Vi 1KΩ
Vo
- 1V
2V

6. Given an irrotational vector field


ur ur ur ur
( ) (
F = k1 xy + k2 z 3 ax + 3x 2 − k3 Z ay + 3xz 2 − y az) ( )
ur
Find V. F at (1,1,-2).

SECTION B
(50 Marks)
Answer any TEN questions. Each question carries 5 marks.

7. The loop transfer function of a single loop control system is given by


100
G (s) H (s) = e − sT
s (1 + 0.01s )

Using the Nyquist criterion, find the condition for the closed loop system to be
stable.

8. The characteristic equation of a feedback control system is


s 4 + 20s3 + 15s2 + 2s + K = 0
(i) Determine the range of K for the system to be stable.
(ii) Can the system be marginally stable? If so, find the required value of K and
the frequency of sustained oscillation.

Join All India Mock GATE Classroom Test Series - 2007 conducted by GATE Forum in over 25 cities all over India. Question
Papers including section tests and full tests are designed by IISc alumni according to the latest syllabus. Percentile, All India Rank,
interaction with IISc alumni in our online discussion forums, and more. For more details,
visit
www.gateforum.com
Think GATE Think GATE Forum
GATE EC - 1998 www.gateforum.com
Join discussion of this test paper at http://forum.gatementor.com

9. Draw a signal flow graph for the following set of algebraic equations:
y2 = ay1 − gy3
y3 = ey2 + cy 4
y 4 = by2 − dy 4

y2 y3
Hence, find the gains 1
and
y y1

10. Consider the system shown in Fig.10. Determine the value of such that the
damping ratio is 0.5. Also obtain the values of the rise time tr and maximum
overshoot Mp in its step response.

R(s) 16 C(s)
+
- s ( s + 0.8 )

(1+as)

11. Determine the input impedance of the circuit of Fig.11 and investigate if it can be
inductive.

R2

-
C
+

Z(s)
R1

12. Find the value of R′ in the circuit of Fig.12 for generating sinusoidal oscillations.
Find the frequency of oscillations.
R′

R
- C R
+

R C

Join All India Mock GATE Classroom Test Series - 2007 conducted by GATE Forum in over 25 cities all over India. Question
Papers including section tests and full tests are designed by IISc alumni according to the latest syllabus. Percentile, All India Rank,
interaction with IISc alumni in our online discussion forums, and more. For more details,
visit
www.gateforum.com
Think GATE Think GATE Forum
GATE EC - 1998 www.gateforum.com
Join discussion of this test paper at http://forum.gatementor.com

13. In the circuit of Fig.13, determine the resistance R0 seen by the output
terminals. Ignore the effects of R1 and R2.
+VCC

R1

1K β=99
rπ=1L

R2 5kΩ

VS ~ RO
1K

+12V
14. The JFET in the circuit of Fig.14 is characterized by
the parameters IDSS = 4MA and Vp = −4V .
2k
Find (a) V0 if Vi = 0, and
(b) Vi if V0 = 0 +
Vo
-
+ 2k
Vi
-
-12V

15. The mod-5 counter shown in Fig.15 counts through states


Q2Q1Q0 = 000, 001, 010, 011 and 100
(a) Will the counter lockout if it happen to be in any one of the unused states?
(b) Find the maximum rate at which the counter will operate satisfactorily.
Assume the propagation delays of flip-flop and AND gate to be tF and t A
respectively.

J2 Q2
J0 Q0 J1 Q1

1 K0 Q0 K1 Q1 K2 Q2

CLK

Join All India Mock GATE Classroom Test Series - 2007 conducted by GATE Forum in over 25 cities all over India. Question
Papers including section tests and full tests are designed by IISc alumni according to the latest syllabus. Percentile, All India Rank,
interaction with IISc alumni in our online discussion forums, and more. For more details,
visit
www.gateforum.com
Think GATE Think GATE Forum
GATE EC - 1998 www.gateforum.com
Join discussion of this test paper at http://forum.gatementor.com

16. For the TTL circuit shown in Fig.16, find the current through the collector of
transistor Q4 when V0 = 0.2V.
Assume VCE ( sat ) = 0.2V , β = 100 and VBE ( sat ) = 0.7V . The α of Q1 in its inverse active
mode is 0.01.
5V

500K
40K 20K

Q2

A Q4
Q1
VD

Q3

12K

17. Write a short assembly language program, without using any arithmetic
instruction, to store hexadecimal 5D in the flag register of 8085 microprocessor.
Data in other registers of the processor must not alter upon executing this
program.

18. Implement a monostable multivibrator using the timer circuit shown in Fig.18.
Also determine an expression for ON time T of the output pulse.
VCC

R
Comparator 1
-
Threshold
+ Output
S Q
R
Comparator 2
Trigger 2
- R Q
+

R
Discharge

GND

19. The pulse rate in a DM system is 56,000 per sec. The input signal is 5
cos(2π1000t) + 2cos(2π2000t) V, with t in sec. Find the minimum value of step
size which will avoid slop overload distortion. What will be the disadvantages of
choosing a value of larger than the minimum?

Join All India Mock GATE Classroom Test Series - 2007 conducted by GATE Forum in over 25 cities all over India. Question
Papers including section tests and full tests are designed by IISc alumni according to the latest syllabus. Percentile, All India Rank,
interaction with IISc alumni in our online discussion forums, and more. For more details,
visit
www.gateforum.com
Think GATE Think GATE Forum
GATE EC - 1998 www.gateforum.com
Join discussion of this test paper at http://forum.gatementor.com

20. An SSB signal is demodulated by using a synchronous demodulator. However,


the locally arranged carrier has a phase error θ. Determine the effect of the error
on demodulation. What will be the effect of this error if the input is DSB-SC in
place of SSB?

21. White noise of two-sided spectral density 2 × 10-6 V2/Hz is applied to a simple R-C
low pass filter whose 3dB cut off frequency is 4 kHz. Find the mean squared
value of the noise output.

T T
22. Consider a rectangular pulse g(t) existing between t= − and t= . Find and
2 2
sketch the pulse obtained by convolving g(t) with itself. The Fourier transform of
g(t) is a sine function. Write down to Fourier transform of the pulse obtained by
the above convolution.

23. A rectangular wave guide with inner dimensions 6 cm × 3 cm has been designed
for a single mode operation. Find the possible frequency range of operations such
that the lowest frequency is 5% above the cut off and the highest frequency is
5% below the cut off of the next higher mode.

ur ur
E = 10e (
j ωt − β z )
24. A plane wave with ay is incident normally on a thick plane
conductor lying in the x-y plane. Its conductivity is 6 × 106 S/m and surface
impedance is 5 × 10-4 ∠45°Ω. Determine the propagation constant and the skin
depth in the conductor.
25. The electric field vector of a wave is given as
ur ur ur
ur i (ω t + 3 x − 4y ) 8ax + 6 ay + 5a z
E = E0e V m.
125
Its frequency is 10 GHz.
(i) Investigate if this wave is a plane wave.
(ii) Determine its propagation constant, and
(iii) Calculate the phase velocity in y-direction.

26. The region between a pair of parallel perfectly conducting planes of infinite extent
in the y- and –z directions is partially filled with a dielectric as shown in Fig.26. A
30 GHz TE10 wave is incident on the air dielectric interface as shown. Find the
VSWR at the interface. x

Air εr=4 5cm

24cm
Join All India Mock GATE Classroom Test Series - 2007 conducted by GATE Forum in over 25 cities all over India. Question
Papers including section tests and full tests are designed by IISc alumni according to the latest syllabus. Percentile, All India Rank,
interaction with IISc alumni in our online discussion forums, and more. For more details,
visit
www.gateforum.com
Think GATE Think GATE Forum
GATE EC - 1999 www.gateforum.com
Join discussion of this test paper at http://forum.gatementor.com

SECTION - A
1. This question consists of TWENTY-FIVE sub-questions (1.1 – 1.25) of ONE mark
each. For each of these sub-questions, four possible alternatives (A,B, C and D)
are given, out of which ONLY ONE is correct. Indicate the correct answers in the
boxes corresponding to the questions only on the FIRST sheet of the answer
book.

1.1 Identify which of the following is NOT a true of the graph shown in Fig.P1.1
a
(a) begh
(b) defg
2 c
(c) adfg 1 3
b
(d) aegh
d e f g

h
4 5

1.2 The z-transform F(z) of the function f ( nT ) = anT is

z z z z
(a) T
(b) T
(c) −T
(d)
z−a z+a z−a z + a−T

1.3 If f ( t )  = F ( s ) , then f ( t − T )  is equal to

F (s) F (s)
(a) esT F ( s ) (b) e− sT F ( s ) (c) sT
(d)
1+e 1 − e− sT

1.4 A 2-port network is shown in Fig.P1.4. the parameter h21 for this network can be
given by
I1 R R I2
+ +

V1 10µF R V2

- -

1 1 3 3
(a) − (b) + (c) − (d) +
2 2 2 2

1.5 The early effect in a bipolar junction transistor is caused by


(a) fast turn-on
(b) fast turn-off
(c) large collector-base reverse bias
(d) large emitter-base forward bias

Join All India Mock GATE Classroom Test Series - 2007 conducted by GATE Forum in over 25 cities all over India. Question
Papers including section tests and full tests are designed by IISc alumni according to the latest syllabus. Percentile, All India Rank,
interaction with IISc alumni in our online discussion forums, and more. For more details,
visit
www.gateforum.com
Think GATE Think GATE Forum
GATE EC - 1999 www.gateforum.com
Join discussion of this test paper at http://forum.gatementor.com

1.6 The first dominant pole encountered in the frequency response of a compensated
op-amp is approximately at
(a) 5 Hz (b) 10 kHz (c) 1 MHz (d) 100 MHz

1.7 Negative feedback in an amplifier


(a) reduces gain
(b) increases frequency and phase distortions
(c) reduces bandwidth
(d) increase noise

1.8 In the cascade amplifier shown in


Q2
Fig.P1.8, if the common-emitter io
Vo
stage(Q1) has a transconductance
gm1 , and the common base stage
RL
(Q2) has a transconductance Q1
Vi
gm2 , then the overall
i 
transconductance g= o  of the
 vi 
cascade amplifier is
gm1 gm2
(a) gm1 (b) gm2 (c) (d)
2 2

1.9 Crossover distortion behaviour is characteristic of


(a) Class A output stage (b) Class B output stage
(c) Class AB output stage (d) Common-base output stage

1.10 The logical expression y = A + AB is equivalent to

(a) y = AB (b) y = AB (c) y = A + B (d) y = A + B

1.11 A Darlington emitter-follower circuit is sometimes used in the output stage of a


TTL gate in order to
(a) increase its IOL (b) reduces its IOH
(c) increases its speed of operation (d) reduce power dissipation

1.12 Commercially available ECL gears use two ground lines and one negative supply
in order to
(a) reduce power dissipation
(b) increase fan-out
(c) reduce loading effect
(d) eliminate the effect of power line glitches or the biasing circuit
Join All India Mock GATE Classroom Test Series - 2007 conducted by GATE Forum in over 25 cities all over India. Question
Papers including section tests and full tests are designed by IISc alumni according to the latest syllabus. Percentile, All India Rank,
interaction with IISc alumni in our online discussion forums, and more. For more details,
visit
www.gateforum.com
Think GATE Think GATE Forum
GATE EC - 1999 www.gateforum.com
Join discussion of this test paper at http://forum.gatementor.com

1.13 The resolution of a 4-bit counting ADC is 0.5 volts. For an analog input of 6.6
volts, the digital output of the ADC will be
(a) 1011 (b) 1101 (c) 1100 (d) 1110

1.14 For a second-order system with the closed-loop transfer function


9
T (s) = 2
s + 4s + 9
The settling time for 2-percent band, in seconds, is
(a) 1.5 (b) 2.0 (c) 3.0 (d) 4.0

1.15 The gain margin (in dB) of a system having the loop transfer function
2
G (s) H (s) = is
s ( s + 1)

(a) 0 (b) 3 (c) 6 (d) ∞

1.16 The system mode described by the state equations


0 1  0 
X =  x +  u
2 −3 1 
y = 1 1 x

is:
(a) controllable and observable
(b) controllable, but not observable
(c) observable, but not controllable
(d) neither controllable nor observable

1.17 The phase margin (in degrees) of a system having the loop transfer function
2 3
G (s) H (s) = is
s ( s + 1)

(a) 45° (b) -30° (c) 60° (d) 30°

1.18 A signal x(t) has a Fourier transform X(ω). If x(t) is a real and odd fucntion of t,
then X(ω) is
(a) a real and even function of ω
(b) a imaginary and odd function of ω
(c) an imaginary and even function of ω
(d) a real and odd function of ω

Join All India Mock GATE Classroom Test Series - 2007 conducted by GATE Forum in over 25 cities all over India. Question
Papers including section tests and full tests are designed by IISc alumni according to the latest syllabus. Percentile, All India Rank,
interaction with IISc alumni in our online discussion forums, and more. For more details,
visit
www.gateforum.com
Think GATE Think GATE Forum
GATE EC - 1999 www.gateforum.com
Join discussion of this test paper at http://forum.gatementor.com

1.19. The input to a channel is a bandpass signal. It is obtained by linearly modulating


a sinusoidal carrier with a single-tone signal. The output of the channel due to
this input is given by
 1 
y (t ) =  (
 cos 100t − 10
 100 
−6
)
cos 106 t − 1.56( )
( ) ( )
The group delay t g and the phase delay t p in seconds, of the channel are

(a) t g = 10−6 , t p = 1.56 (b) t g = 1.56, t p = 10−6

(c) t g = 108 , t p = 1.56 × 10−6 (d) t g = 108 , t p = 1.56

1.20. A modulated signal is given by, s ( t ) = m1 ( t ) cos ( 2π fc t ) + m2 ( t ) sin (2π fc t ) where


the baseband signal m1 ( t ) and m2 ( t ) have bandwidths of 10 kHz and 15 kHz,
respectively. The bandwidth of the modulated signal, in kHz, is
(a) 10 (b) 15 (c) 25 (d) 30

1.21. A modulated signal is given by s ( t ) = e − at cos (ωc + ∆ω ) t  u ( t ) , where a,ωc and


∆ω are positive constants, and ωc ∆ω .
The complex envelope of s(t) is given by
(a) exp ( −at ) exp  j (ωc + ∆ω ) t  u ( t ) (b) exp ( −at ) exp ( j ∆ω t ) u ( t )

(c) exp  j ∆ω t  .u ( t ) (d) exp ( jωc + ∆ω ) t 

1.22. An electric field on a plane is described by its potential V = 20 r −1 + r −2 where r ( )


is the distance from the source. The field is due to
(a) a monopole (b) a dipole
(c) both a monopole and a dipole (d) a quadrupole

1.23. Assuming perfect conductors of a transmission line, pure TEM propagation is NOT
possible in
(a) coaxial cable (b) air-filled cylindrical wave guide
(c) parallel twin-wire line in air
(d) semi-infinite parallel plate wave guide

1.24. Indicate which one of the following will NOT exist in a rectangular resonant
cavity.
(a) TE110 (b) TE011 (c) TM110 (d) TM111

Join All India Mock GATE Classroom Test Series - 2007 conducted by GATE Forum in over 25 cities all over India. Question
Papers including section tests and full tests are designed by IISc alumni according to the latest syllabus. Percentile, All India Rank,
interaction with IISc alumni in our online discussion forums, and more. For more details,
visit
www.gateforum.com
Think GATE Think GATE Forum
GATE EC - 1999 www.gateforum.com
Join discussion of this test paper at http://forum.gatementor.com

1.25 Identify which one of the following will NOT satisfy the wave equation.

(a) 50e (
j ω t −3z )
(b) sin ω (10 z + 5t )  (c) cos y 2 + 5t ( ) (d) sin ( x ) cos ( t )

2. This question consists of TWENTY-FIVE sub-questions (2.1 – 2.25) of ONE mark


each. For each of these sub-questions, four possible alternatives (A, B, C and D)
are given, out of which ONLY ONE is correct. Indicate the correct answers in the
boxes corresponding to the questions only on the SECOND sheet of the answer
book.

2.1 The Fourier series representation of an impulses train denoted by



s (t ) = ∑ d (t − nT ) is given by
n = −∞
0

∞ ∞
1 j2π nt 1 jπ nt
(a)
T0 ∑
n = −∞
exp−
T0
(b)
T0 ∑ exp−
n = −∞
T0
∞ ∞
1 jπ nt 1 j2π nt
(c)
T0 ∑ exp
n = −∞
T0
(d)
T0 ∑ exp
n = −∞
T0

2.2. The Thevenin equivalent voltage VTH appearing between the terminals A and B of
the network shown in Fig.P2.2 is given by
(a) j16(3-j4) 3Ω A
+
(b) j16(3+j4)
j2 -j6 j4 VTH
(c) 16(3+j4) 100∠0°V ~

(d) 16(3-j4) -
B

2.3. The value of R (in ohms) required for maximum power transfer in the network
shown in Fig.P2.3 is
(a) 2 5Ω 4Ω

(b) 4
+ 3A
25V R
(c) 8 -
20Ω

(d) 10

2.4. A Delta-connected network with its Wye-equivalent is shown in Fig.P2.4. The


resistance R1, R2 and R
a 3 (in ohms) are respectively a

5Ω R1
30Ω ⇒

R2 R3

b 15Ω c c
b
Join All India Mock GATE Classroom Test Series - 2007 conducted by GATE Forum in over 25 cities all over India. Question
Papers including section tests and full tests are designed by IISc alumni according to the latest syllabus. Percentile, All India Rank,
interaction with IISc alumni in our online discussion forums, and more. For more details,
visit
www.gateforum.com
Think GATE Think GATE Forum
GATE EC - 1999 www.gateforum.com
Join discussion of this test paper at http://forum.gatementor.com

(a) 1.5, 3 and 9 (b) 3, 9 and 1.5


(c) 9, 3 and 1.5 (d) 3, 1.5 and 9

2.5. An n-channel JEFT has IDSS = 2mA and Vp = −4V. Its transconductance gm (in
milliohm) for an applied gate-to-source voltage VGS of –2V is:
(a) 0.25 (b) 0.5 (c) 0.75 (d) 1.0

2.6. An npn transistor (with C=0.3 pF) has a unity – gain cutoff frequency fT of 400
MHz at a dc bias current Ic = 1mA. The value of its C µ (in pF) is approximately
(VT = 26mV )

(a) 15 (b) 30 (c) 50 (d) 96

2.7. An amplifier has an open-loop gain of 100, an input impedance of 1 kΩ, and an
output impedance of 100Ω. A feedback network with a feedback factor of 0.99 is
connected to the amplifier in a voltage series feedback mode. The new input and
output impedances, respectively, are
(a) 10Ω and 1Ω (b) 10Ω and 10 kΩ
(c) 100Ω and 1Ω (d) 10kΩ and 1 kΩ

2.8. A dc power supply has a no-load voltage of 30V, and a full-load voltage of 25 V
at a full-load current of 1A. Its output resistance and load regulation, respectively
are
(a) 5Ω and 20% (b) 25Ω and 20%
(c) 5Ω and 16.7% (d) 25Ω and 16.7%

2.9. An amplifier is assumed to have a single pole high frequency transfer function.
The rise time of its output response to a step function input is 35 nsec. The upper
–3 dB frequency (in MHz) for the amplifier to a sinusoidal input is approximately
at
(a) 4.55 (b) 10 (c) 20 (d) 28.6

2.10. The minimized form of the logical expression A B C + ABC + ABC + ABC is ( )
(a) A C + BC + AB (b) AC + BC + AB
(c) AC + BC + AB (d) AC + BC + AB

2.11. For a binary half-sub-tractor having two inputs A and B, the correct set of logical
expressions for the outputs D (=A minus B) and X (=borrow) are
(a) D = AB + AB, X = AB (b) D = AB + AB + AB, X = AB

(c) D = AB + AB, X = AB (d) D = AB + AB, X = AB


Join All India Mock GATE Classroom Test Series - 2007 conducted by GATE Forum in over 25 cities all over India. Question
Papers including section tests and full tests are designed by IISc alumni according to the latest syllabus. Percentile, All India Rank,
interaction with IISc alumni in our online discussion forums, and more. For more details,
visit
www.gateforum.com
Think GATE Think GATE Forum
GATE EC - 1999 www.gateforum.com
Join discussion of this test paper at http://forum.gatementor.com

2.12. The ripple counter shown in Fig.P2.12 works as a

Preset Preset Preset


J Q J Q J Q

A B C

‘1’ ‘1’ K ‘1’ K


K Q Q

Clock

(a) mod – 3 up counter (b) mod – 5 up counter


(c) mod – 3 down counter (d) mod – 5 down counter

2.13. If CS = A15 A14 A13 is used as the chip select logic of a 4 K RAM in an 8085
system, then its memory range will be
(a) 3000 H – 3 FFF H (b) 7000 H – 7 FFF H
(c) 5000 H – 5 FFF H and 6000 H – 6 FFF H
(d) 6000 H – 6 FFF H and 7000 H – 7 FFF H

2.14. If the closed loop transfer function T(s) of a unity negative feedback system is
given by
an −1s + an
T (s) = n n −1
s + a1s + K + an −1s + an
then the steady state error for a unit ramp input is
an an an − 2
(a) (b) (c) (d) zero
an −1 an − 2 an − 2

2.15. Consider the points s1 = −3 + j 4 and s2 = −3 − j2 in the s-plane. Then, for a


system with the open loop transfer function
K
G (s) H (s) =
( s + 1) 4
(a) s1 is on the root locus, but not s2
(b) s2 is on the root locus, but not s1
(c) both s1 and s2 are on the root locus
(d) neither s1 nor s2 is on the root locus

Join All India Mock GATE Classroom Test Series - 2007 conducted by GATE Forum in over 25 cities all over India. Question
Papers including section tests and full tests are designed by IISc alumni according to the latest syllabus. Percentile, All India Rank,
interaction with IISc alumni in our online discussion forums, and more. For more details,
visit
www.gateforum.com
Think GATE Think GATE Forum
GATE EC - 1999 www.gateforum.com
Join discussion of this test paper at http://forum.gatementor.com

2.16. For the system described by the state equation


 0 1 0 0
   
x =  0 0 1 x + 0 u
0.5 1 2 1 

If the control signal u is given by u =  −0.5 − 3 − 5 x + ν , then the eigen values of
the closed-loop system will be
(a) 0, -1, -2 (b) 0, -1, -3 (c) -1, -1, -2 (d) 0, -1, -1

2.17. The z-transform of a signal is given by C ( z ) =


(
1z −1 1 − z −4 ) . Its final value is
2
(
4 1 − z −1 )
1
(a) (b) zero (c) 1.0 (d) infinity
4

2.18. The Nyquist sampling frequency (in Hz) of a signal given by


6 × 104 sin c 2 ( 400t ) * 106 sin c 3 (100t ) is

(a) 200 (b) 300 (c) 500 (d) 1000

2.19. The peak-to-peak input to an 8-bit PCM coder is 2 volts. The signal power-to-
quantization noise power ratio (in dB) for an input of 0.5cos (ωmt ) is

(a) 47.8 (b) 49.8 (c) 95.6 (d) 99.6

2.20. The input to a matched filter is given by


10 sin 2π × 106 t
( ) 0 < 1 < 10−4 sec
s (t ) = 
0 otherwise
The peak amplitude of the filter output is
(a) 10 volts (b) 5 volts
(c) 10 millivolts (d) 5 millivolts

2.21. Four independent messages have bandwidths of 100 Hz, 200 Hz and 400 Hz,
respectively. Each is sampled at the Nyquist rate, and the samples are time
division multiplexed (TDM) and transmitted. The transmitted sample rate (in Hz)
is
(a) 1600 (b) 800 (c) 400 (d) 200

2.22. In a twin-wire transmission line in air, the adjacent voltage maxima are at 12.5
cm and 27.5 cm. The operating frequency is
(a) 300 MHz (b) 1 GHz (c) 2 GHz (d) 6.28 GHz

Join All India Mock GATE Classroom Test Series - 2007 conducted by GATE Forum in over 25 cities all over India. Question
Papers including section tests and full tests are designed by IISc alumni according to the latest syllabus. Percentile, All India Rank,
interaction with IISc alumni in our online discussion forums, and more. For more details,
visit
www.gateforum.com
Think GATE Think GATE Forum
GATE EC - 1999 www.gateforum.com
Join discussion of this test paper at http://forum.gatementor.com

2.23. A transmitting antenna radiates 251 W isotropically. A receiving antenna, located


100 m away from the transmitting antenna, has an effective aperture of 500 cm2.
The total received by the antenna is
(a) 10µW (b) 1 µW (c) 20 µW (d) 100 µW

2.24. In air, a lossless transmission line of length 50 cm with L = 10 µH/m, C = 40


pF/m is operated at 25 MHz. Its electrical path length is
(a) 0.5 meters (b) λ meters
π
(c) radians (d) 180 degrees
2

2.25. A plane wave propagating through a medium ε r = 8, ν r = 2, and σ = 0 has its
  z
ur − 
ˆ ( )
electric field given by E = 0.5 Xe  3  sin 108 t − β z V m. The wave impedance, in
ohmsis
(a) 377 (b) 198.5∠180° (c) 182.9∠14° (d) 133.3

SECTION – B
This section consists of TWENTY questions of FIVE marks each. ANY FIFTEEN out of
them have to be answered. If more number of questions are attempted, score off the
answers not be evaluated, else, only the first fifteen unscored answers will be
considered.
3. In the circuit of Fig.P3, the switch ‘S’ has remained open for a long time. The
switch closes instantaneously at t = 0
(a) Find Vo for t ≤ 0 and as t  ∞
(b) Write an expression for Vo as function of time for 0 ≤ t ≤ ∞
(c) Evaluate Vo at t = 25 µsec.
10Ω S 12Ω
+
T=0s
+ 2.5µF Vo 5Ω 2A
25V -

4. For the network shown in Fig.P4, evaluate the current I flowing through the 2Ω
resistor using superposition theorem.
I

2Ω

10∠0°A j8 4Ω 10∠20°A

Join All India Mock GATE Classroom Test Series - 2007 conducted by GATE Forum in over 25 cities all over India. Question
Papers including section tests and full tests are designed by IISc alumni according to the latest syllabus. Percentile, All India Rank,
interaction with IISc alumni in our online discussion forums, and more. For more details,
visit
www.gateforum.com
Think GATE Think GATE Forum
GATE EC - 1999 www.gateforum.com
Join discussion of this test paper at http://forum.gatementor.com

5. A coil with a quality factor (Q) of 10 is put in series with a capacitor C1 of 10 µF,
and the combination is found to draw maximum current when a sinusoidal
voltage of frequency 50 Hz is applied. A second capacitor C2 is now in parallel
with the circuit. What should be the capacitance of C2 for combined circuit to act
purely as a resistance for a sinusoidal excitation at a frequency of 100Hz?
Calculate the rms current drawn by the combined circuit at 100 Hz if the applied
voltage is 100 V (rms).

6. A bipolar junction transistor amplifier circuit shown in Fig.P6. Assume that the
current source Ibias is ideal, and the transistor has vary large b,
V 
rb = 0, and r0 → ∞. Determined the ac small-signal midband voltage gain  o  ,
 Vs 
input resistance ( Ri ) and output resistance ( Ru ) of the circuit. Assume VT = 26mV
VCC

Ibias
0.5mA
C2=∞
VO
RB
RL
100KΩ
1KΩ
R3 RO

C1=∞ 50Ω

Ri
VDD
7. A JFET having µ=50 and rd = 10k Ω is used in a common-source configuration as
shown in Fig.P7. The JFET capacitances are C gs = 5pF , C gd = 2 pF , and Cds = 2 pF .
V 
Determine the ac small signal midband voltage gain  o  and the upper –3dB
 Vs 
frequency of the circuit.

2kΩ

10kΩ RD
Vo

RS

+
VS ~
-

Join All India Mock GATE Classroom Test Series - 2007 conducted by GATE Forum in over 25 cities all over India. Question
Papers including section tests and full tests are designed by IISc alumni according to the latest syllabus. Percentile, All India Rank,
interaction with IISc alumni in our online discussion forums, and more. For more details,
visit
www.gateforum.com
Think GATE Think GATE Forum
GATE EC - 1999 www.gateforum.com
Join discussion of this test paper at http://forum.gatementor.com

8. Neatly sketch and label the dc R R


transfer characteristic Vref=1V
( i.e., Vo ν s. Vin ) of the circuit
D1
D2
shown in Fig.P8, as Vin varies
from –2V to +2V. Assume ideal
R
op-amp, and the diodes have a Vin -
forward voltage of 0.6 V and Vo
+
zero incremental resistance.

9. A transistor LC oscillator circuit is shown in Fig.P9. Assume that the transistor has
very high β (so that you may neglect rd ). Derive an equation governing the
circuit operation, and find the frequency of oscillation. Also, state the gain
condition required for oscillation to start.
VCC

C1
C3=∞
VO
C2 RL
Ibias

VEE

10. In the CMOS inverter circuit shown in Fig.P10, the input Vi makes a transition
from VOL ( = 0V ) to VOH ( = 5V ) . Determine the high-to-low propagation delay time

(t pHL ) when it is driving a capacitive load (CL ) of 20 pF. Device data:


VDD=5V
W  2
NMOS : VTN = 1V ; kN = µnCOX   = 40µ A / V , λ = 0,
L
 n
W 
PMOS : VTP = −1V ; k p = µ pCOX   = 20µ A / V 2 , λ = 0.
 L p
Vi VO
Neglect body effect.
VOH CL

VOL 20pF

Join All India Mock GATE Classroom Test Series - 2007 conducted by GATE Forum in over 25 cities all over India. Question
Papers including section tests and full tests are designed by IISc alumni according to the latest syllabus. Percentile, All India Rank,
interaction with IISc alumni in our online discussion forums, and more. For more details,
visit
www.gateforum.com
Think GATE Think GATE Forum
GATE EC - 1999 www.gateforum.com
Join discussion of this test paper at http://forum.gatementor.com

11. The circuit diagram of a synchronous counter is shown in Fig.P.11. Determine the
sequence of states of the counter assuming that the initial state is ‘000’. Give
your answer in a tabulor form showing the present state
QA ( n) , QB ( n ) , QC ( n) , J − K inputs ( A A B C )
J , K , J , J , K and the next state
QA ( n +1) , QB ( n +1) , QC ( n +1) . From the table, determine the modulus of the counter.

J Q J Q J Q

A B C

‘1’
K ‘1’ K Q K Q
Q

Clock

12. In a certain application, four inputs A, B, C, D (both true and complement forms
available) are fed to logic circuit, producing an output F, which operates a relay.
The relay turns on when F(ABCD) = 1 for the following states of the inputs
(ABCD):’0000’, ‘0010’, ‘0101’, ‘0110’, ‘1101’ and ‘1110’. States ‘1000’ and ‘1001’
do not occur, and for the remaining states, the relay is off. Minimize F with the
help of a Karnaugh map and realize it using a minimum number of 3 – input
NAND gates.

13. An 8085 assembly language program is given below:

MVIC, 03H
LXI H, 2000H
MOV A, M
DRC C
LOOP: INX H
MOV B, M,
CMP B
JNC LOOP2
MOV A, B
LOOP2: DCR C
JNZ LOOP1
STA 2100H
HLT

Join All India Mock GATE Classroom Test Series - 2007 conducted by GATE Forum in over 25 cities all over India. Question
Papers including section tests and full tests are designed by IISc alumni according to the latest syllabus. Percentile, All India Rank,
interaction with IISc alumni in our online discussion forums, and more. For more details,
visit
www.gateforum.com
Think GATE Think GATE Forum
GATE EC - 1999 www.gateforum.com
Join discussion of this test paper at http://forum.gatementor.com

Contents of the memory locations 2000 H to 2002 H are:


2000 : 18 H 2001:10H, 2002:2 BH.

(a) What does the above program do?


(b) At the end of the program, what will be
(i) the contents of the registers A, B, C, H and L?
(ii) the condition of the carry and zero flags?
(iii) the contents of the memory locations 2000 H, 2001 H, 2002 H and
2100H.

14. The loop transfer function of a feedback control system is given by


K ( s + 1)
G (s) H (s) = ,K > 0
s (1 + Ts ) (1 + 2s )

Using Routh-Hurwitz criterion, determine the region of K-T plane in which the
closed-loop system is stable.

15. The asymptotic Bode plot of the minimum phase open-loop transfer function G(s)
H(s) in as shown in Fig.P15. Obtain the transfer function G(s)H(s)

G (s) H (s) -40dB/decade


[dB]
20
-20dB/decade
ω2
0 ω1 ω[rad/s]
1.0 (log scale)
-20

-40dB/decade

16. Consider a feedback system with the open-loop transfer function, given by
K
G (s) H (s) =
s ( 2 s + 1)

Examine the stability of the closed loop system using Nyquist stability theory.

Join All India Mock GATE Classroom Test Series - 2007 conducted by GATE Forum in over 25 cities all over India. Question
Papers including section tests and full tests are designed by IISc alumni according to the latest syllabus. Percentile, All India Rank,
interaction with IISc alumni in our online discussion forums, and more. For more details,
visit
www.gateforum.com
Think GATE Think GATE Forum
GATE EC - 1999 www.gateforum.com
Join discussion of this test paper at http://forum.gatementor.com

17. A baseband signal m(t) modulates a carrier to produce the angle modulated
signal, Ac cos 2π × 108 t + k p m ( t )  , where m(t) is shown in Fig.P17. Determine the
 
value of k p so that the peak-to-peak frequency deviation of the carrier is 100
kHz.

12.5
m(t)
[Volts] 15

10

t(ms)
0 10 15
-5 5
-5
-10

18. Input to a linear delta modulator is a sinusoidal signal whose frequency can vary
from 200 Hz to 4000 Hz. The input is sampled at eight times the Nyquist rate.
The peak amplitude of the sinusoidal signal is 1 volt.
(a) Determine the minimum value of the step size in order to avoid slope
overload when the input signal frequency is 800 Hz.
(b) What is the peak amplitude of the input signal, to just overload the
modulator, when the input signal frequency is 200 Hz?
(c) Is the modulator overloaded when the input signal frequency is 4000 Hz?

19. The power spectral density (PSD) of a noise process is given by


  f − 108 
 10−8  1 +  f < 108
  
SN ( f ) =   108 

 0 f > 108

The noise is passed through a unit-gain ideal bandpass filter, centered at 50 MHz
and having a bandwidth of 2 MHz.
(a) Sketch neatly the PSD of the output noise process.
(b) Determine the output noise power.
(c) Using the band-pass representation for the output noise process, sketch the
PSD of the in-phase and quadrature noise components, and determine their
respective powers.

20. A plane wave in free space with


ur
( )(
E = π 10.0 x ˆ + 11.8y ) (
ˆ .exp j 4π × 108 t − kz , where x
ˆ and y )
ˆ are unit vectors in
the x-y directions, respectively, is incident normally on a semi-infinite block of ice
as shown in Fig.P20.

Join All India Mock GATE Classroom Test Series - 2007 conducted by GATE Forum in over 25 cities all over India. Question
Papers including section tests and full tests are designed by IISc alumni according to the latest syllabus. Percentile, All India Rank,
interaction with IISc alumni in our online discussion forums, and more. For more details,
visit
www.gateforum.com
Think GATE Think GATE Forum
GATE EC - 1999 www.gateforum.com
Join discussion of this test paper at http://forum.gatementor.com

For ice, µ=µ0, σ = 0, and ε = 9ε0 (1 – j0.001).


(a) Calculate the average power density associated with the incident wave.
(b) Calculate the skin depth in ice.
(c) Estimate the average power density at a distance of 5 times the skins depth
in the ice block, measured from the interface.
x
µ,ε,σ
Incident wave
y z

µo,εo
Block of ice

21. A 100 m section of an air-filled rectangular wave-guide operating in the


TE10 mode has a cross-sectional dimension of 1.071 cm × 0.5 cm. Two pulses of
21 GHz and 28 GHz are simultaneously launched at one end of the wave-guide
section. What is the time delay difference between the two pulses at the other
end of the wave-guide?

22. The average power of an omni directional antenna varies as the magnitude of
cosθ, where θ is the azimuthal angle. Calculate the maximum Directive Gain of
the antenna and the angles at which it occurs.

Join All India Mock GATE Classroom Test Series - 2007 conducted by GATE Forum in over 25 cities all over India. Question
Papers including section tests and full tests are designed by IISc alumni according to the latest syllabus. Percentile, All India Rank,
interaction with IISc alumni in our online discussion forums, and more. For more details,
visit
www.gateforum.com
Think GATE Think GATE Forum

You might also like